A Simple Legal Lacrosse Check

Transcription

A Simple Legal Lacrosse Check
A Simple Legal Lacrosse Check
Authors: Amy E. Valasek, MD, MSc; Yvette L. Rooks, MD
Affiliation: Nationwide Children's Hospital and The Ohio State College of Medicine, Division of Pediatric Sports
Medicine, Columbus, Ohio
History: 21 y/o lacrosse midfielder was struck by an opposing player's shoulder at the right side of his sternum and
knocked backward during a game. He walked off the field with his own power and complained of chest pain over the
sternum. Pain resolved within fifteen minutes on the sideline. Three hours after the game he complained of pain with deep
breathing and increased chest discomfort. In the local emergency room, he was evaluated and discharged with a chest
wall contusion. Chest radiograph and ECG were interpreted as normal. Forty eight hours later the team physician
concurred and advised activity as tolerated. His sternal pain decreased throughout the week and he participated in a full
lacrosse game without difficulty. Twelve days after the initial injury he developed dyspnea with exertion and presented to a
second team physician for evaluation in the training room.
Physical Exam: On exam, he was alert, interactive, and breathing comfortably at rest. No pulse oximetry was available.
Vital Signs: HR 66 BP 118/77 RR 12 Trachea was midline. He was tender to palpation over anterior right chest specifically
5th-7th ribs. No deformity or flail chest identified. No obvious bruising or swelling noted. No audible breath sounds were
present throughout all right lung fields. Left lung fields were clear to auscultation. Cardiac examination showed regular
rate and rhythm without audible murmur or rub. Capillary refill was brisk and 2+ radial and femoral pulses. Remainer of
examination was normal.
Differential Diagnosis:
Pneumothorax
Tension Pneumothorax
Hemothorax
Pneumomediastinum
Rib fractures
Test Results: Emergent Chest CT demonstrated a moderate right hemothorax with rib fractures involving right 5th and
6th ribs. No pneumothorax was identified.
Final Diagnosis: Hemothorax with rib fractures
Treatment: Emergently transported to Trauma Center. Chest tube placed in the right pleural cavity immediately drained
3.5 Liters. Repeat CT imaging revealed a residual anterior mediastinal hematoma, moderate pleural effusion, and
compressive atelectasis of RLL. He underwent a right thoracotomy and evacuation of right hemothorax. He was discharged
from hospital on postoperative day four.
Outcome: Began light cardio activity one month after surgery and progessed in activity slowly.
Follow-Up: Returned to full lacrosse participation five months after discharge without difficulty.
Heel Pain in College Basketball Player
Authors: Craig S. Best, DO; April M. Fetzer, DO
Affiliation: Rush University Medical Center Department of Physical Medicine & Rehabilitatioin Chicago, IL
History: A 21-year-old female college basketball player presented with an eight month history of bilateral heel pain
without extension into the mid- or forefoot. The pain is worsened with weight bearing, walking, and running and alleviated
with rest. The pain was rated at 5/10 in severity. Ibuprofen and Tylenol provided little relief. She was previously seen by a
Podiatrist and had custom foot orthotics made, but her heel pain persisted. As such, she sought further evaluation and
management from an orthopedic foot & ankle surgeon and was originally diagnosed with bilateral plantar fasciitis. Her
pain persisted and further imaging revealed normal appearance of the plantar fascia (see below for details). Pregabalin
was initiated and an electrodiagnostic study was ordered, which revealed bilateral lateral plantar neuropathy. On followup, the patient was approximately 60% improved with the pregabalin and further surgical options were to be discussed at
a later date.
Physical Exam: The patient walked with a normal gait. The feet were warm and well-perfused. Tenderness to palpation
noted over the bilateral heels, most notably through the region of the first branch of the lateral plantar nerve. Strength
testing revealed 5/5 bilateral ankle dorsiflexion, plantarflexion, eversion, and inversion as well as great toe extension and
flexion. Light touch sensation is intact throughout the foot and ankle bilaterally. Tinel sign was negative at the saphenous,
sural, superficial and deep fibular (peroneal), and tibial nerves.
Differential Diagnosis:
plantar fasciitis
calcaneal stress fracture
Achilles tendonitis
retrocalcaneal bursitis
neuropathy
tumor
Test Results: Weight-bearing radiographs of bilateral foot and ankle were negative for fracture, dislocation, joint space
narrowing, and heel spurs. Ankle mortise was intact. Magnetic resonance imaging of the right ankle revealed a normal
appearance of the plantar fascia, no acute abnormality seen within the ankle, and a likely chronic injury to the anterior
talofibular ligament. An electrodiagnostic study was performed. Nerve conduction studies revealed absent bilateral lateral
plantar nerve sensory nerve action potentials and electromyography demonstrated increased polyphasic motor unit action
potentials; these findings were consistent with bilateral lateral plantar neuropathy.
Final Diagnosis: Bilateral inferior calcaneal (Baxter's) neuropathy
Treatment: Pregabalin was initiated and provided 60% relief. Surgical management options were discussed with an
orthopedic foot & ankle surgeon and will be revisited at a later date.
Outcome: At this time, the patient has had improvement with the pregabalin and can continue her physical activities as
well as her academic pursuits as a college athlete.
Follow-Up: The patient will follow-up in December and discuss surgical release of the plantar fascia and affected nerve.
Rhabdomyolysis...Not so Fast, My Friend!
Authors: Megan L. Meier, MD; Theodore A. Boehm, MD; Brian R. Coleman, MD
Affiliation: University of Oklahoma Health Sciences Center, Oklahoma City, OK
History: 19yo male NCAA Division 1 football player presents with dark urine. The symptoms started the week football
practice resumed for the fall season. He presented to his trainer with associated muscle soreness, which he attributed to
workouts in the weight room and on the field. He denied any other symptoms such as fatigue, nausea, vomiting, diarrhea,
abdominal pain, other GI complaints, dysuria, or any other urinary changes. The training staff advised aggressive oral
hydration to which the patient complied. However, he continued to have dark urine.
Physical Exam: General: A&O, NAD Skin: no rash, lesions, jaundice or pallor HEENT: normocephalic atraumatic CV: RRR,
no murmurs, rubs, gallops Pulm: CTAB, no wheezes, rales, rhonchi Abdomen: soft, non-tender, no hepatosplenomegaly;
mild flank pain b/l Extremities: no edema, normal pulses
Differential Diagnosis:
1.
2.
3.
4.
5.
6.
7.
Rhabdomyolysis
Urinary Tract Infection
Hepatitis
Nephritis
Hemolysis
Cholestasis
Nephrolithiasis
Test Results:
UA-SG 1.020, pH 5.5, protein 1+, blirubin 1+, blood negative, few granular casts, otherwise normal
CK-34
CMP- electrolytes-normal, Total Bilirubin- 1.0, Alkaline phosphatase-444, AST, 215, ALT- 288
CBC- WBC- 15.4, 34% atypical lymphocytes (total 49), 19.2% monocytes (total 2.9)
Hepatitis panel- negative
Monospot- positive
Epstein Barr Virus IgG- 0.29 (normal)
Epstein Barr Virus IgM- 4.40 (high)
Final Diagnosis: Infectious Mononucleosis
Treatment: Advised continuing to stay well hydrated and advised to rest. Pt. advised to avoid physical activity. He was
removed from practice and weight lifting activities for 3 weeks. Abdominal ultrasound was obtained and showed
splenomegaly of 17cm, no hepatomegaly, renal or other abnormalities. Serial liver function tests (LFT's) were obtained.
Outcome: Weekly LFT's were obtained which normalized around the 2 week mark. Urine cleared within 1 week of
presentation. Splenomegaly resolved at recheck 2 weeks later. Pt. noted some mild fatigue which also improved after the
1st week. No other symptoms developed.
Follow-Up: The athlete was allowed to start non-contact cardiovascular activities 3 weeks after his initial presentation. By
this time, all symptoms had resolved. Liver function tests had normalized and splenomegaly had resolved between weeks
2-3. He was then progressed as tolerated to full contact around 3.5 weeks from onset of symptoms.
Heat Related Illness in a High School Football Player?
Authors: David Kramer DO; Michael Seth Smith, MD, CAQ-SM, PharmD
Affiliation: University of Florida Dep. of Community Health & Family Medicine/UF Dept. of Orthopedics &
Rehabilitation
History: A 17-year-old high school football player, suddenly became dizzy, lethargic, and had a near syncopal episode
during the first week of football practice in Florida. The athletic trainer quickly noticed him with his head down while
holding himself up on one knee. At that time the patient denied any other complaints and stated he was getting over a GI
illness from the prior day. During questioning, he became slower to respond to questions. At that time, EMS was activated,
the athlete was submerged in ice water and cooled for 15 minutes. When EMS arrived the athlete had altered
consciousness and was transported to the hospital immediately. Vital signs on initial exam in ED were BP 142/114, HR 89,
RR 20, O2 Sat 99%, Temp 97.9 F taken orally. EKG showed sinus rhythm, normal axis, nonspecific ST and T wave changes.
Labs showed CK elevated at 423, BUN of 21 and Cr of 1.5, but were otherwise normal. He was hydrated and sent home to
follow up with his team physician. He visited our sports medicine clinic with his father (a nurse) 2 weeks after the event. At
that time he had no complaints and no issues. He denied any previous history of exercise intolerance, asthma, heat related
illnesses, chest pain, dizziness, family history of sudden cardiac death, or syncopal episodes. The patient attributed his
recent episode to a combination of a recent GI illness and to drinking 2 cups of coffee and not eating breakfast the
morning prior to practice. PMH: Concussion (10/18/2013), environmental allergies PSH: Finger surgery 2011, T-tube
placement 2005 PFH: Paternal grandfather died at 40 due to “asthma” Social Hx: No smoking, alcohol use, drug
use, or supplement use. Drinks 2-3 cups of coffee or caffeinated soda per day Medications: None Allergies: none Three
days after office visit, return to play options were discussed with his mother on the phone and she mentioned that this was
his second episode of dizziness with near syncope while participating in athletics during the prior year Plan: Patient
referred to Pediatric Cardiology for further work-up including Echocardiogram
Physical Exam: Vital Signs: BP 128/72, HR 76, O2 Sat 97% Constitutional: Alert. Appears well-developed and well
nourished. No distress. Head: Normocephalic and atraumatic. Cardiovascular: Normal S1S2 with regular rhythm. No
murmur, gallop or friction rub. Intact distal pulses, no edema. Pulmonary/Chest: Unlabored with normal breath sounds.
Neurological: Alert to person, place and time. CN2-12 intact. Skin: Skin is warm and dry with no rash or lesions.
Differential Diagnosis:
Exertional Heat Illness
Exertional sickling
Hypervolemic hyponatremia
Cardiac arrhythmia
Structural cardiac abnormality
Test Results: EKG: sinus arrhythmia (bradycardia), normal axis, normal intervals, no chamber enlargement or
hypertrophy, nonspecific T wave changes (flattened T waves on lead III and V1). TTE Results: Left coronary artery originates
from the right coronary orifice. CTA of Heart: The left main coronary artery originates from the right coronary cusp and
passing between the aorta and pulmonary artery in a malignant course with findings likely representing a focal intramural
segment proximally. There is also an acute angle take off of the left coronary artery with elliptical dimensions
Final Diagnosis: Left anomalous coronary artery arising from right coronary cusp with intramural course
Treatment: 1 Pediatric Cardiologist scheduled patient for repair with pediatric cardiothoracic surgeon. 2 CT Surgery
repaired anomalous artery via median sternotomy, and unroofing of intramural left coronary artery originating from the
right sinus of Valsalva with repair of anomalous origin.
Outcome: Patient discharged home on post op day #4.
Follow-Up: Repeat ECGs each month. Most recent one 11/18/14 showed normal blood flow and absence of paracardial
effusion. Having a full NM Myo Stress Test with treadmill on 12/11/14.
A Rapidly Progressive Life Threatening Complication of a Simple Disease
Authors: Krishna Khanal, MD; Sandhya Basyal, MBBS, MPH; Catalin Buda, MD; Prabhat Bhattarai, PA-C
Affiliation: Sartori Memorial Hospital, Cedar Falls, IA
History: A previously healthy 24 yo Hispanic D1 University football player became ill and presented in the ED of a local
community hospital with high fever (103°F), cough, sore throat, nasal congestion and fatigue since 2 days. He had h/o
loose stool since that day. He was a smoker but had no h/o sick contacts. No past h/o similar illness. No recent h/o travel.
No h/o recent influenza vaccination.
Physical Exam: Patient was on mild distress. His temperature was 100.5°F and SaO2 was 96% in room air. Examination
of the posterior pharyngeal wall showed erythema with no exudates. No cervical lymphadenopathy. His lung sounds were
clear with equal air movement.
Differential Diagnosis:
Viral URI
Influenza
Streptococcal Sore throat
Pneumonia
Infectious mononucleosis
Test Results: Normal CBC, CMP, CXR. Both rapid strep tests and mono spot tests were negative. CPK was slightly
elevated. His influenza A/B and blood culture x 2 were negative during initial ED visits. Patient had to go to the ED of
another community hospital next day morning (within less than 12 hrs of first ED visit) in severe respiratory distress,
hemoptysis and hypoxia His repeat blood tests showed leucopenia, repeat chest X-ray showed B/L multilobar pneumonia.
CT scan of the chest showed findings consistent with B/L pneumonia or pulmonary hemorrhage. His blood culture during
the second visit retrospectively showed growth of MRSA.
Final Diagnosis: Rapidly progressive Acute Influenza like illness leading to life threatening bacteremia secondary to B/L
MRSA multifocal pneumonia.
Treatment: During initial visit patient was managed with iv fluids, antipyretics. Since his X-ray and chest CT was
suggestive pneumonia, he was given first dose of levaquin during the second ED visit. Later therapy was changed to iv
doxycycline and vancomycin per infectious disease recommendation. Patient had to be intubated with mechanical
ventilation in ICU because of his critical condition.
Outcome: Because of the severe condition, patient was transferred to the MICU of the nearest University Medical
Center. Rapid immunochromatographic assay for the qualitative detection of the influenza A and B was performed on the
nasopharyngeal secretions and was positive for influenza B. Antibiotic therapy and respiratory supports were continued.
Patient had to go for the chest tube placement to decompensate the pyothorax. Patient’s illness improved despite rapid
life threatening complications and discharged to home after 3 wks of admission in the hospital in stable condition.
Follow-Up: Patient could not keep follow up with the hospital since he was discharged home and football season was
already over.
Worsening Ankle Injury In a Recreational Soccer Player
Authors: Daniel McCabe, MD
Affiliation: John H. Sroger Hospital of Cook County, Chicago, IL
History: 32 year old soccer player presents to emergency department six days after right ankle injury. While dribbling the
ball his cleat caught in the turf causing ankle pronation and external rotation; he was unable to bear weight at time of
injury. Patient was evaluated at another institution on day of injury and again three days later due to worsening pain and
swelling. Ankle x-rays were done at both encounters and did not reveal a fracture; written findings stated normal mortise.
He was diagnosed with an ankle sprain, given ibuprofen for swelling, provided rigid ankle brace and crutches for comfort.
Patient presented to the emergency department for worsening symptoms; he had been attempting to bear weight over
the past week with increased pain and swelling.
Physical Exam: Examination revealed tenderness to palpation at right mid lateral leg without gross deformity or
crepitus. Right ankle exam revealed diffuse swelling, bruising posterior to medial malleolus without gross deformity, at
medial malleolus there was minimal tenderness present over bony prominence though exquisite tenderness to palpation
was present in adjacent soft tissues, active dorsiflexion and plantarflexion intact, pain elicited with passive dorsiflexion and
eversion, negative talar tilt test.
Differential Diagnosis:
1.
2.
3.
4.
Ankle sprain
Ankle fracture
Syndesmotic injury (High ankle sprain)
Proximal fibula fracture with deltoid ligament injury
Test Results: Ankle anterior-posterior and mortise radiographs: Widening of the ankle mortise is seen medially with no
fractures Tibia/fibula anterior-posterior radiographs: Comminuted fracture is seen of the mid fibula with a butterfly
fragment
Final Diagnosis: Maisonneuve fracture (proximal fibula fracture and deltoid ligament rupture secondary to excessive
external rotation of ankle) with subluxation of syndesmotic ligament
Treatment:
1.
2.
3.
4.
Closed reduction in emergency department with immediate immobilization in short leg splint.
Operative repair <48 hours after presentation: open reduction and internal fixation of syndesmosis.
Immobilization for 8 weeks; non-weight bearing.
Range of motion exercises and leg strengthening began 8 weeks after operative repair.
Syndesmotic screws removed at 10 weeks post-placement.
Outcome: Normal use of ankle after surgical repair and immobilization.
Follow-Up: Return to sports 18 weeks post injury when he had full, painless ROM, and normal strength with running.
I Fell Out of My Bed and I Can't Walk!
Authors: Yaowen Eliot Hu, MD, RMSK; Garry Ho, MD, RMSK; Thomas Howard, MD, RMSK, FACSM
Affiliation: VCU-Fairfax Family Practice/Herndon Family Medicine, Fairfax, VA
History: 12 year-old boy with no significant PMH presented with 1 day of right posterior knee pain He is actively
involved in multiple sports, including football and basketball 1 month ago, he was hit during football and fell onto his
right anterior knee without twisting or an audible pop/snap At that time, the pain was more medial and anterior and
resolved with 1 week of rest. The night prior to evaluation, he fell out of his bed and hyperextended his right knee His
pain was described as “sharp and tight&rdquo; and was localized to the right posterior knee The pain is worse with
flexion, extension, and weight bearing There was no swelling, bruising, numbness, or tingling Over-the-counter ibuprofen
had no effect and he is now requiring crutches for mobility He denied any previous surgeries, fever, or skin rashes. After
an initial diagnosis of hamstring and posterior capsule strain, his R posterior knee pain gradually improved spontaneously
However, over the next 5 days, he started developing intermittent fevers ranging from 102-105F He had associated
diffuse and vague abdominal pain but no other upper respiratory, cardiovascular, pulmonary, urinary, gastrointestinal, skin,
or neurologic symptoms.
Physical Exam: Vital Signs: T 97.6-102.7°F, HR 84-88, RR 14-16, BP 100-108/60-66 He was not in acute distress and
appeared nontoxic. His head, eyes, ears, nose, mouth, throat, neck, cardiovascular, pulmonary, skin, and neurologic exams
were all normal. Abdominal exam revealed mild epigastric tenderness but was otherwise normal. Observation of the right
knee showed no deformity, swelling, rashes, or ecchymosis His gait was antalgic requiring use of crutches. There was
tenderness on palpation of the right posterior knee superior to the joint line over the distal hamstring tendons and distal
femur without warmth. There was no effusion of the knee His range of motion of the knee was limited in flexion and
extension due to pain and guarding There was pain with resisted flexion and extension of the knee. Lachman’s,
McMurray, valgus/varus stress, anterior/posterior drawer, dial test, and patellofemoral tests were all negative.
Differential Diagnosis:










Septic Arthritis
Reiter's Syndrome
Immune Complex Arthropathy
Osteomyelitis
Malignancy/Neoplasm
Lyme's Disease and other Tick-Bourne Disease
Juvenile Rheumatoid Arthritis and other Autoimmune Joint Disease
Brodie's Abscess
Meniscus Tear/Ligamentous Damage
Osteochondral Defect
Test Results: WBC 5.5-9.4 with neutrophils 57-61%. ESR 24, CRP 51.1. Blood cultures grew out methicillin-sensitive
staphylococcus aureus that was pan-sensitive to antibiotics. CBC, CMP, Lyme titers, EBV titers, UA, CXR, rapid strep, throat
culture, and urine cultures were normal. MRI of the right knee showed findings consistent with osteomyelitis of the distal
femoral physis without focal abscess or lesions. 2D Echocardiogram was normal.
Final Diagnosis: Osteomyelitis of the distal femoral physis
Treatment: He was initially treated with IV ceftriaxone and IV vancomycin and the infectious disease physician was
consulted He was switched to IV cefazolin and his repeat blood cultures after antibiotic administration were negative His
symptoms improved quickly after a brief hospital stay and a PICC line was placed. He was discharged with IV cefazolin for
a total of 4 weeks with infectious disease follow-up outpatient.
Outcome: He completed 4 weeks of IV cefazolin followed by 2 more weeks of oral cefuroxime and his symptoms
completely resolved. The PICC line was discontinued and removed after completion of cefazolin Repeat CBC, CMP, and
ESR 6 weeks after initial symptoms were all normal.
Follow-Up: He returned to full activity and sports without issues after completing 6 weeks of antibiotics.
US Evaluation of the Acute Posterior Lateral Corner Injury
Authors: Jesse P. DeLuca, DO; Steven A. Potter, MD; Mary A. Noel MD
Affiliation: Martin Army Family Medicine Residency Martin Army Community Hospital Fort Benning, GA
History: 26yo active duty male who presented with a chief complaint of knee pain Patient was competing in combatives
and was in guard position grasping his own right foot in order to perform a choke hold His competitor forcefully tried to
get up and externally rotated and extendedthe patients tibia while the patient resisted the movement The patient felt a
pop and had immediate pain to his posterior lateral knee He rested for several months but was unable to return to
competition and had difficulty ambulating with twisting motions, athletic movements and could not run.
Physical Exam: Normal appearance of the knee. Tenderness to palpation at the posterior lateral corner. Full range of
active motion of the knee,bilaterally equal. Neurvascularly intact. Negative Lachman, Anterior/Posterior drawer,
Valgus/Varus stress, Mcmurray, Thessaley. Positive Dial at 30 degrees of flexion. Negative at 90 degrees. Pain reproduced
with resisted internal rotation and end passive external rotation.
Differential Diagnosis:
Popliteus Tendon Origin Injury
Posterior Horn Meniscus
LCL Injury
PCL Injury
Bone Contusion
Lateral Gastroc Injury
Test Results:
Negative X ray
WNL MRI with Patellar cartilage thinning
WNL CT scan
US evaluation of the postierior lateral corner popliteus tendon origin at the lateral condyle with a linear hyperechogenicit
fragment suggestive of avulsion
Final Diagnosis: Popliteus tendon injury with supporting ultrasound, arthroscopic and open images.
Treatment: The patient was initially seen by ortho and given diagnosis of LCL injury and initialy made minimal progress
with physical therapy. After ultrasound evaluation he opted and underwent dexprolotherapy injection with 1/2cc 50%
dextrose and 1/2cc of lidocaine to the area which immidiately resolved his symptoms. He however had minimal
improvement at 4 week follow up. Due to time course and career implications I discussed the case with orthopedics with
emphasis on the US findings who performed arthroscopy of the posterior lateral corner confirming diagnosis and then
performed open surgery with autologous reinforcement.
Outcome: At 3 months follow up the patient was performing forward/reverse motions without issue and had begun early
twisting/athletic maneuvers with minimal limitation.
Follow-Up: Continues strengthening in physical therapy with likely full return to duty and sport.
Atraumatic Foot Pain in a Young Male
Authors: Travis Currie, MD; Elizabeth Davis, MD; Harrison Youmans. MD
Affiliation: University of Florida/Orlando Health/Arnold Palmer, Orlando, FL USA
History: Patient is a 6 year old Caucasian male with a 1 month history of left foot pain. His parent's state he has been is
walking with an antalgic gait and refusing to fully weight bear with is left foot. The only preceding event the parents could
recall was the child running around in flat soled shoes on a concrete surface around that time. Child complains of pain on
his arch and the dorsum of his left foot.
Physical Exam: On physical exam the child was found to have an antalgic gait predominantly heel striking with mild
internal rotation of the left foot at the ankle. There was pain on palpation of the navicular bone.
Differential Diagnosis:
Kohler's disease
Navicular fracture
Plantar fasciitis
Test Results: Radiographs of the left foot show sclerosis and flattening of the navicular bone compared to the right.
Repeat in 2 weeks showed further sclerosis and flattening.
Final Diagnosis: Kohler's Disease
Treatment: Pt was symptomatically treated with walking boot for a total of 8 weeks (4 weeks in as of today). Pt was
advised use of NSAIDS and Ice.
Outcome: Pt is progressing quite well until he became non compliant with his boot 2 weeks after. He has now become
compliant with is boot is and is pain free 4 weeks into treatment. (I will have followup as time progresses)
Follow-Up: Pt will remain in boot and slowly come out after 6-8 weeks of treatment. Follow up x-rays may be obtained.
16 Year Old Gymnast with Foot Pain
Authors: Rathna Nuti, MD; Robert Dimeff, MD
Affiliation: University of Texas Southwestern Medical Center, Dallas, TX
History: 16 year old elite gymnast presented with nonspecific right dorsal foot pain for 5 months. Pain began when she
landed on the hard part of the springboard used for vault. Denied hearing or feeling a pop or snap, swelling, discoloration,
deformity, paraesthesia, or weakness. She continued participation that day. Later, treated herself with K tape, electrical
stimulation, ibuprofen 400 mg BID, ice, analgesic creams, and chiropractic manipulation with some improvement. Running,
jumping, and landing worsened pain. Able to perform bars and beam without limitations, but modified vault and floor
routines. Since then, developed mild nonspecific left arch pain. History of mild ankle sprains in the past that resolved with
rehabilitation and usual conservative treatments. Takes NSAIDs, MVI, iron, and vitamin D. Denies medical diseases,
previous surgeries and allergies. Besides primary amenorrhea, ROS otherwise unremarkable. FMH noncontributory. Social
history: Denies drug/alcohol/substance abuse and history of eating disorder. She trains 30 hours per week and expecting
to participate in college gymnastics.
Physical Exam: General: Healthy teenage girl, BMI 19, NAD Lower Extremities: Bilateral hips symmetric, pain-free
ROM. No tenderness on palpation. Negative FADIR, FABER. Weakness, no pain, bilaterally with single leg squatting. Knees
and ankles FROM, no swelling, tenderness, erythema, warmth, or instability. Right foot FROM, no swelling or discoloration.
Mild tenderness on palpation at dorsal side of navicular. No other tenderness. Symmetric strength, no pain with resisted
foot or ankle motions. Pulses and sensations intact.
Differential Diagnosis:
Tarsal navicular stress fracture
Painful accessory navicular
2nd metatarsal stress fracture
Lis Franc joint sprain
Midfoot osteoarthritis/sprain/avulsion fracture
Tibialis anterior/EHL strain/tendonosis
Test Results: X-ray Right Foot 5/9/13: Fragmentation of dorsal proximal aspect of navicular and os trigonum MRI Right
Foot 5/9/13: Increased signal throughout the navicular bone. 5 mm linear area of decreased signal dorsal aspect of
navicular consistent with remote injury. CT Scan Right Foot 5/24/13: 8 x 3 x 4 mm ossification from the proximal dorsal
aspect of the navicular consistent with remote avulsion fracture. 6.5 mm linear radiolucency from the dorsal navicular bone
at donor avulsion site tracking medially with surrounding sclerosis. This is consistent with chronic incomplete obliquely
oriented navicular stress fracture. Vitamin D 25OH level 29
Final Diagnosis: Right Navicular Stress Fracture With Remote Avulsion Fracture With Borderline Vitamin D Deficiency
Treatment: Vitamin D3 5,000 IU daily, Exogen bone stimulator, NWB cast immobilization for 6 weeks followed by
progressive weight bearing and return to full gymnastics within 6 months.
Outcome: Because of college showcase events planned in the next 2-3 months, patient elected to train as tolerated. If
able to continue, would then undergo immobilization after the season. Discussed concerns regarding displacement,
difficulty with surgical treatment, possible long-term disability and failure to return to gymnastics.
Follow-Up: Within 2 weeks of treatment with bone stimulator, ice, and activity modification, patient was pain free. Able
to do beam, bars, and run on tumble track without pain. Had only minimal tenderness of the dorsal navicular bone. At 5
weeks, able to do all activities except vault. At 7 weeks, able to do all activities and completed showcase events. Pain free
with a normal exam. It was elected to proceed with follow up CT scan prior to casting. Repeat CT scan Right Foot
7/24/13: Remodeling of bone fragment and near resolution of fracture line. Because of clinical recovery and radiographic
evidence of healing, elected to allow her to train without the use of analgesics. Continued to use bone stimulator and ice,
and completed the season. Subsequently accepted an athletic scholarship to a Division I university.
Persistent Leg Pain
Authors: Ryan C. Kruse, MD; Jacob L. Sellon, MD
Affiliation: Sports Medicine Center, Department of Physical Medicine and Rehabilitation, Mayo Clinic, Rochester,
MN
History: An 18 year old female high school soccer player presented to the clinic with complaints of bilateral shin pain, right
greater than left, which began spontaneously approximately one month prior. She did not recall any specific inciting event.
She described the pain as a deep, burning, pinching pain, 9/10 in severity, located on the distal anterolateral aspect of her
legs. The pain would develop with running approximately one mile and would not subside until she ceased running. She
also noted a small, tender bulge along her bilateral anterolateral shins, within the region of pain. She denied numbness,
weakness, or foot pain. On initial sports medicine evaluation, leg radiographs and exertional noninvasive arterial studies
(ankle-brachial indices and Doppler ultrasound exam) were normal. Ultrasound revealed small bilateral distal anterior
compartment muscle herniations. In addition, post-exertion compartment pressure testing demonstrated elevated anterior
and lateral compartment pressures bilaterally. She subsequently underwent bilateral anterior/lateral compartment
fasciotomies. These were performed proximal to the muscle herniation sites and did not involve the fascia surrounding the
herniations. The patient continued to have similar lower leg symptoms after her fasciotomy and presented to the sports
medicine clinic again for further evaluation.
Physical Exam: General: Athletic appearing young female in no distress. Skin: Well-healed longitudinal surgical scars in
the proximal anterolateral aspect of bilateral legs. Cardiovascular: Peripheral pulses intact and symmetric in the lower
extremities. Musculoskeletal: Subtle, mildly tender masses at the anterolateral aspect of the lower legs, right more
prominent than left. Gait: Normal gait cadence and stride. Toe and heel walking normal and pain free. Neurologic: Sensation
intact to light touch in all leg/foot dermatomes. Strength normal and symmetric throughout her bilateral lower limbs.
Differential Diagnosis:
1) Recurrent/persistent chronic exertional compartment syndrome
2) Anterior/lateral compartment muscle herniations
3) Medial tibial stress syndrome
4) Peroneal neuropathy
5) Arterial claudication
Test Results:
MRI Legs (post-fasciotomy): Small, anterior compartment fascial defects with small muscle herniations, right more than left
Exertional MSK Ultrasound (post-fasciotomy): Right leg - focal muscle herniation at the distal anterior compartment
immediately lateral to the extensor digitorum longus tendon, which increased in size and became tender post-exertion. Left
leg - resting scan showed no hernia. Post-exercise scan revealed tender extensor digitorum longus muscle herniation in the
same distal anterior compartment location as right leg. Bilateral muscle herniations had mass effect on the superficial
peroneal nerves at the fascial exit.
Compartment Pressure Monitoring (post-fasciotomy): pre-exercise compartment measurements were normal. Post-exercise
anterior and lateral compartment pressures were mildly increased in the left leg. In the more symptomatic right leg,
compartment pressures were normal.
Final Diagnosis: Residual symptomatic bilateral anterior compartment muscle herniations after fasciotomies for chronic
exertional compartment syndrome.
Treatment: The patient underwent distal extension of her previous fasciotomies in order to release the muscle herniations.
During this, she also had bilateral superficial peroneal nerve releases.
Outcome: Post-operatively, the patient had dramatic symptomatic improvement in bilateral legs.
Follow-Up: With her leg pain resolved, she was gradually advanced in a progressive rehabilitation program. She is now
attempting to return to college soccer and is currently training for the upcoming season.
Performance Decline in a Golfer
Authors: Michael R. Baria, MD,MBA; Karen L. Newcomer, MD
Affiliation: Mayo Clinic, Department of Physical Medicine and Rehabilitation, Rochester, MN
History: A 53 year-old woman was evaluated in our clinic for left hand weakness that initially presented as difficulty with
her golf swing. Seven months prior to arrival in our clinic, her primary struggles were with ball control and decreasing teeshot distances.; She had a thorough swing mechanics evaluation, with particular attention to stance and club grip. She was
also advised to avoid playing in "scrambles" because the hurried pace would promote poor mechanics. Despite these
interventions her performance continued to decline. She experienced persistent left 4th and 5th finger paresthesias and
pain. Swing mechanics evaluation and adjustment continued by her swing coach until there was a distinct episode when
her left hand was unable to hold onto the club during the follow through phase of her swing, which prompted her to seek
medical evaluation. At an outside institution her initial workup, which included electrodiagnostic testing, concluded that
her problem was secondary to a left median mononeuropathy at the wrist and a left ulnar mononeuropathy at the elbow.
Both nerves were surgically decompressed. Unfortunately, her hand weakness progressed. Pain also became more
prominent, involving the ulnar aspect of her hand and forearm. A cervical spine MRI and a non-contrast brachial plexus
MRI were reported as normal. Seven months after her initial symptoms, she presented to our clinic for further evaluation.
Upon further discussion with the patient, her past medical history was significant for breast cancer which required two
rounds of therapeutic radiation to her left chest, with the most recent therapy occurring 5 years prior. To her knowledge,
the breast cancer was in full remission.
Physical Exam: Her physical examination revealed marked atrophy of left thenar, hypothenar and hand intrinsic
muscles, as well as significant weakness in all left C8-T1/lower trunk muscles. Reflexes were normal and sensory
examination revealed hyperesthesia over the left medial forearm. Cervical spine range of motion was pain-free and
Spurling's maneuver was negative. Thoracic outlet provocative maneuvers were negative.
Differential Diagnosis:
1.
2.
3.
4.
Lower trunk brachial plexopathy secondary to tumor recurrence versus radiation injury.
C8 radiculopathy
Thoracic outlet syndrome.
Peripheral mononeuropathies
Test Results: On EMG, the left median and ulnar nerve conduction studies demonstrated low amplitude compound
muscle action potentials (CMAP) and sensory nerve action potentials (SNAP). Left median CMAP and SNAP latencies were
prolonged as well. Needle EMG of the left upper limb demonstrated severe neurogenic injury with ongoing denervation in
all C8-T1/lower trunk muscles. The remainder of the needle examination, including the cervical paraspinals, was normal.
Myokymic discharges, a finding specific for radiation injury, were absent. This examination was most consistent with a
severe lower trunk brachial plexopathy with a superimposed median mononeuropathy at the wrist. A superimposed ulnar
neuropathy could not be ruled out. A repeat brachial plexus MRI with contrast demonstrated mild enlargement of the
plexus with increased T2 signal and hazy gadolinium enhancement, most prominent in the medial and posterior cords. PET
scan demonstrated increased FDG uptake throughout the plexus, for which the differential included tumor recurrence
versus chronic inflammation. For a definitive diagnosis, a brachial plexus biopsy was performed.
Final Diagnosis: Left brachial plexopathy secondary to biopsy proven metastatic breast cancer
Treatment: Pending; likely systemic chemotherapy (to avoid further plexus irradiation)
Outcome: To be determined, as the patient is awaiting final decision on systemic chemotherapy.
Follow-Up: She has been unable to return to play. Follow-up will occur after definitive treatment.
Silent Killer
Authors: Adam Colen, DO; Michael Miller, MD
Affiliation: St. Elizabeth Health Care, Edgewood, KY
History: Patient MS is a twenty year old male Frisbee golf athlete that presented to clinic for right shoulder pain. Patient
stated that the pain was constant and worsening over the last 3 weeks. He described the pain as sharp and achy which was
located anterior and superior to his coracoid process. The patient also explained that it improved with overhead activity
and over the last seven to five days was associated with increased right arm swelling. He denied a history of trauma. He
was seen in the emergency department two days before being seen in our clinic with a negative axillary Doppler
ultrasound and chest radiograph.
Physical Exam: Inspection: Diffuse superficial swelling and mottled skin ROM: Full to flexion, abduction, extension,
internal and external rotation bilaterally MMT: 5/5 strength flexion, abduction, extension, internal and external rotation
bilaterally Palpation: Mild increased intensity to superior and inferior palpation coracoid region right N/V: Harsh bruit
heard inferior to clavicle right, 2/4 radial artery bilaterally Provocative tests: Negative Hawkins, Neers, Jobes, O'Brien,
Shear, and negative for anterior or posterior shoulder instability
Differential Diagnosis:








Paget von schroetter disease
Thoracic-outlet syndrome
Paget von schroetter syndrome
Superficial phlebitis
Intramuscular hemorrhage
Occult fracture
Severe superficial bruising
Lymphangitis
Test Results: Doppler Ultrasound: Negative Venogram: Acute to subacute right axillosubclavian thrombosis CXR:
Normal CBC: Normal CMP: Normal INR/PT: Normal
Final Diagnosis: Paget von schroetter disease/Thoracic outlet syndrome
Treatment: Catheter directed thrombolysis with tissue plasminogen activator Three months anticoagulation
Outcome: Resolution of swelling and mottled skin Decrease but not full resolution of pain at three weeks from initial
diagnosis of thrombosis
Follow-Up:
Four weeks of no sport
Five to eight weeks of activity modification using pain as guide
Follow up eight weeks out from initial presentation
I Think I Hurt My Wrist!
Authors: Sonia I Millan P., MD
Affiliation: Halifax Health Sports Medicine Fellowship, Daytona Beach, FL
History: 20-year old college football player who presents with left wrist pain since 1 hour when he fell on his
outstretched LEFT hand during practice. Player reports immediate pain and inability to continue play. He also states that
he has numbness and tingling at the fingers of the LEFT hand. He has iced the wrist.
Physical Exam: GEN: No acute distress, but appears uncomfortable LEFT WRIST: +2 swelling, no erythema,
protuberance at volar aspect of wrist; no injury noted proximal to the wrist; Active range of motion at flexion, extension,
radial and ulnar deviation limited secondary to pain; Passive flexion and extension limited mechanically; Decreased
sensation at dorsal and volar aspects of fingers; +2 radial pulse; capillary refill - 2 secs
Differential Diagnosis:
Lunate dislocation
Scapholunate dissociation
Scaphoid fracture
Lunate fracture
Distal radial fracture
Distal ulnar fracture
Extensor tendon injury
Test Results: Plain films: L wrist with volar dislocation of the lunate
Final Diagnosis: Lunate Dislocation (Mayfield Stage IV, Herzberg Satge IIa)
Treatment:
1. Closed reduction of lunate (after 10 minutes of finger trap traction).
2. Splint placed.
3. 8 days after injury: Open reduction and intercarpal pinning Left scaphoid, lunate and triquetrum; scapholunate
ligament reconstruction using dorsal intercarpal ligament capsulodesis; posterior interosseus neurectomy.
4. Splint placed
Outcome:
1.
2.
3.
4.
Immediate relief of pain with closed reduction.
2 weeks post-op: minimal swelling at L wrist, FROM at digits, neurovascularly intact; sutures were removed, short
arm cast was placed.
6 weeks post-op: minimal swelling at L wrist, FROM at digits, Limited ROM at wrist, neuorvascularly intact; thumb
spica splint placed, range of motion in physical therapy; plain films demonstrate alginment of the carpus
maintained and stable anchor positioning; pins pulled
Follow up in 4-5 weeks
Follow-Up: As of this submission,athlete is 7 weeks post-op, not cleared to play. His expected return to football is
unknown at this time. He will continue with rehab.
Adamanti-What???
Authors: Cody Clinton, DO; Stephen Stache, MD
Affiliation: Thomas Jefferson University Hospital, Philadelphia PA; Rothman Institute Orthopadics, Bryn Mawr,
PA
History: T.S., a 12 year-old female with no significant past medical history, presented to the sports medicine clinic with
complaints of left distal leg pain of approximately six months duration. She described the pain as a persistent deep ache
that she could not attribute to any specific motion or activity. She denied paresthesias or weakness. She denied fevers,
chills, night sweats or night pain. Two weeks prior, T.S. stated she tripped and inverted her ankle, causing an increase in
her pain that led her to make the appointment. X-rays of the ankle and tibia/fibula were completed in the office. The ankle
x-rays were normal. The x-rays of the tibia/fibula showed multiple cystic bubbly appearing lesions in the distal tibia and
fibula with sclerotic borders and bony expansion.
Physical Exam: Vitals: Height 5 feet 5 inches, weight 145 lbs, BMI: 21.4, pulse 72, RR 16 General: Awake, alert and
oriented X3, No apparent distress Lymphatic: no gross adenopathy noted Focused left lower extremity exam: no
echymosis or erythema. + tenderness to palpation over the anterior aspect of the left distal tibia and distal fibula,
approximately 10-12 cm. proximal to the ankle mortise. No tenderness to palpation over the midfoot or forefoot. Full
ankle ROM with pain on eversion, 5/5 lower extremity strength,
talar tilt test
anterior drawer test
squeeze test Neurovascular intact.
Differential Diagnosis:
Osteosarcoma
Chondrosarcoma
Aneurysmal Bone Cysts
Fibrous Dysplasia
Osteofibrous Dysplasia
Test Results: MRI of L lower extremity w/o contrast: 1) aggressive cortically based lesion of the distal tibia measuring
1.5 X 1.9 X 2.7 cm with sclerotic borders and reactive adjacent marrow edema. Differential considerations include
adamantinoma, osteofibrous dysplasia like adamantinoma, or cortical based osteofibrous dysplasia 2) non-aggressive
appearing lesion of distal fibula measuring 1.7 x 1.4 X 4.5 cm. with well-defined borders and some cortical thinning.
Differential considerations include fibroxanthoma, osteofibrous dysplasia, or fibrous dysplasia
Final Diagnosis: Osteofibrous dyspasia vs fibrous dysplasia +/- adamantinoma of tibia
Treatment: Weight bearing fracture boot. Expert consultation with orthopedic oncology.
Outcome: T.S. was referred to an orthopedic oncologist and her case was reviewed at a musculoskeletal radiology
conference. The consensus was made that the imaging results were consistent with osteofibrous dysplasia vs fibrous
dysplasia, and not the more aggressive malignant and rare adamantinoma. They advised obtaining a bone scan and
receiving continued follow up with x-rays. If the lesions worsened, then a biopsy was recommended. The bone scan
showed increased uptake seen within the distal left tibia, and to a lesser extent in the distal left fibula. There were no other
areas of uptake throughout the body.
Follow-Up: T.S. was followed by orthopedic oncology who obtained repeat x-rays that showed gradual filling in of the
bony lesion on the tibia. Her pain and discomfort in her left lower extremity completely subsided within a few months of
her presenting to the clinic. As she was deemed not to have any risk for pathologic fracture, she was allowed to participate
in all activities without restrictions. The patient is to be followed every 6 months with continued repeat x-rays and careful
monitoring. This case illustrates the diagnostic uncertainty that can occur when differentiating the extremely rare and
potentially fatal malignant bone tumor known as an adamantinoma, from more benign bone tumors such as osteofibrous
dysplasia and fibrous dysplasia. All three tumors share similar characteristics as they all can typically occur in the tibial
diaphysis and ipsilateral fibula, are cortically based, and can have aggressive features. Thus, it is paramount to get an
accurate diagnosis, as AD can be fatal and requires surgical resection, while the other bengin tumors can regress naturally.
An Unusual Hematuria in College Football Player
Authors: Shawn Skarpnes, MD; Brian Harrell, MD
Affiliation: PGY-2 Resident at Baton Rouge General Family Medicine Residency Program in Baton Rouge, LA
History: An 18 year-old otherwise healthy college football wide receiver noticed frank blood in his urine 1 day following an
evening practice in which he sustained blunt trauma to his right torso from the helmet of a defensive player. The patient
reported moderate localized right flank pain immediately following the impact; however, it was the blood-tinged urine that
alerted the athlete to seek further evaluation in the emergency room.
Physical Exam: ER Vital signs: BP 142/77 HR 60 RR 18 Temp 97.9° Sat 100% on RA GEN: Alert and oriented young African
American male of athletic build in mild distress due to right flank pain. HEENT: Moist mucous membranes with clear
oropharynx. RESP: Non-labored breathing with lungs clear-to-auscultation bilaterally. CVS: Heart rate and rhythm regular
with palpable distal pulses. GI: Abdomen soft, non-distended, non-tender, with normal bowel sounds. No
hepatosplenomegaly. MSK: Right costovertebral angle tenderness to palpation. Rib cage and spine non-tender to
palpation without deformity. Motor function intact with FROM. GU: Normal external genitalia, no meatal erosions or
lesions, no urethral blood or discharge. SKIN: Warm, dry, intact, no edema, rashes, or bruising.
Differential Diagnosis:
Nephrolithiasis
Pyelonephritis
Rhabdomyolysis
Sickle Cell Crisis
Exercise Induced Hematuria
Penile Trauma
Acute Tubular Necrosis
Glomerular Disease
Vasculitis
Renal Contusion
Infarction
Renal Cell CA
Polycystic Kidney Disease
Tuberculosis
Schistomiasis
Test Results: Lab Studies CBC: Normal; CMP: Mild Hypokalemia 3.3 and Total Bilirubin mild elevation 1.3; PT: Mild
elevation of 15.5, INR 1.2, PTT 30; Macroscopic UA: red-tinged, 3+ blood, 2+ protein, +1 urobilinogen, trace leukocyte
esterase, no nitrites, no glucose; Microscopic UA: &gt;150 RBCs, 0 WBCs, few bacteria & epithelial cells; Urine Culture: No
growth.
Imaging:
1. CT Abdo/Pelvis without Contrast on day of admission: Mild right hydronephrosis with subtle right peripelvic fat stranding. No
ureteral calculus. No spleen, liver, pancreatic, GB or right kidney abnormality.
2.
CT Abdo/Pelvis with Contrast 1 day later: Subtle decreased enhancement at mid pole of right kidney consistent with renal
contusion with adjacent soft tissue stranding. Otherwise unremarkable.
Final Diagnosis: Hematuria secondary to blunt force genital urinary trauma with grade 1 right renal contusion.
Treatment: IV fluid hydration & PO Norco with urine output and Hb/Hct monitoring. Gross hematuria resolved prior to
discharge. Patient was discharged after 24hr hospital stay with instructions to follow up with Urology prior to sport
participation. Avoid NSAIDs or antiplatelet/anticoagulants and take Tylenol PRN pain.
Outcome: The athlete was seen by Urology within 3 days of discharge with instructions detailing no workouts of any type
for the first 2 weeks post injury Then if urine is clear, he may begin modified workouts (i.e. weight lifting and cycling but no
running or physical contact) over the following 4 weeks Afterwards, the patient may be cleared to resume sport activity.
Follow-Up: He successfully completed a 6 week stepwise progression without complication and rejoined his team for the
remainder of the season.
Chronic Fatigue & Joint Pain - When 3 Wrongs Make the Diagnosis
Authors: Jennifer Gayagoy, MD; Tran, Q; Kernan, M; McCaul, J; Tucker, J
Affiliation: Department of Family Medicine, St. Joseph's Hospital Health Center, Syracuse, NY
History: 19 year old caucasian male on crew team who presented to the University Sports Medicine clinic multiple times
over a course of 3 weeks. His initial complaint was sore throat without cough for 3 days associated with aches and chills
but no fever. Two days later he noted dysphagia, headaches and poor sleep. With treatment the patient's dysphagia
improved but the fatigue and aches persisted and he subsequently complained of dysuria, denied abdominal pain and
penile discharge. Urinary symptoms persisted, but before antibiotics were completed the patient returned complaining of
bilateral eye redness and discharge. Despite treatment with ophthalmic drops his ocular symptoms worsened and his joint
pain had become more pronounced at the bilateral knees, lower back and shoulders. Dysuria had resolved.
Physical Exam: Initial physical examination was consistent with viral URI. At follow up the patient was febrile (T 101.8F)
and oropharyngeal exudates with tender cervical lymphadenopathy noted. When he returned with dysuria abdominal
exam was benign and no costovertebral angle tenderness elicited. Subsequent physical examination revealed bilateral
conjunctival injection, blephardema and clear ocular discharge. Joint exam was negative for effusions and erythema with
mild tenderness to palpation.
Differential Diagnosis: The differential diagnosis had transitioned along with the patients symptoms; and included viral
syndrome, Strep throat, infectious mononucleosis, UTI, prostatitis, conjunctivitis, allergic reaction to medication, overtraining and reactive arthritis.
Test Results: Strep, Culture and Monospot were negative and CBC unremarkable. Urinalysis was remarkable for pyuria
and trace blood. Urine culture and urine GC/Chlamydia were also negative. ESR and CRP were elevated and HLA-B27 was
negative.
Final Diagnosis: The patient was evaluated by a Rheumatologist who agreed that this was indeed a case of Reiter's
syndrome.
Treatment: The patient's treatment course consisted of supportive care for viral URI, Penicillin for presume Strep throat,
Keflex for presumed UTI; and a prolonged course of Bactrim DS when urinary symptoms failed to improve. Gentamicin eye
drops were initially prescribed for his conjunctivitis but when his symptoms worsened, Ophthalmology advised switching
to Tobramycin. Due to the persistent joint pain Doxycycline, Prednisone and Naprosyn were added. Rheumatology
subsequently started the patient on immunosuppressant therapy.
Outcome: Reactive Arthritis, formerly known as Reiter's syndrome, has been classically described as a clinical triad
consisting of conjunctivitis, urethritis and arthritis after a bacterial infection involving the urogenital or gastrointestinal
system. Rarely does the disease present after a nasopharyngeal infection or even as the complete triad itself, guiding the
change in the title of the disease to include all non-specific symptoms associated with a post-infectious arthropathy. This
uncommon diagnosis can easily be overlooked at a Primary Care and/or Sports Medicine Center where athletes often
present with viral symptoms and acute or chronic joint pain that can easily be attributed to training This case not only
describes an athlete who presented with the rarely reported complete clinical triad of Reactive Arthritis but it also
highlights the importance of repeat follow up with an athlete until complete resolution of symptoms to allow for
appropriate diagnosis and management before allowing safe participation and return to play.
Follow-Up: Upon return the patient stated he felt much better and was back at baseline He was advised a graded return
to activity as tolerated over 3-4 days. The patient had no further ocular, urinary or joint complaints. He was cleared for full
activity and advised to follow up at the University Clinic and the Rheumatologist at any time.
An Unusual Situation Where Stress Radiographs are Superior to MRI
Authors: Thomas Mulroy, MD; Nathan Holmes, MD; Steven Frey, MD
Affiliation: Inspira Medical Center at Woodbury, Woodbury, New Jersey
History: An otherwise healthy, skeletally immature, 16 year old male whom had closing physis, presented after having a
right knee injury during a high school football game. He was unsure of the mechanism of the injury but thought he was
struck on the outside of the knee while his foot was planted. He noted pain over the medial and anterior aspect of his
knee and reported a sensation of instability. He had reported swelling and inability to bear weight. He denied numbness
or tingling.
Physical Exam: General: AAO x 3 in NAD HEENT: NC/AT, PERRL CV: S1 and S2 heard, RRR Lungs: CTA B/L Abd: soft,
NT, ND Right knee: large effusion noted. There was laxity with valgus stress at 30 degrees. There was no discernible
endpoint with Lachman's test. Posteior drawer testing was negative. There was tenderness over the medial and lateral
femoral condyles. No tenderness of the medial or lateral joint lines. Pulses were intact and capillary refill is less than 2
seconds.Range of motion was from 20-60 degrees.
Differential Diagnosis:




ACL/MCL tear
Salter Harris fracture
Medial Retinacular tear/MPFL detachment
Patella dislocation
Test Results: Right knee xray: Tiny ossific densities near the intercondylar notch of questionable significance and a
supra patellar effusion. Additionally, not described in the radiological report there was faint linear lucency appreciable
within the central epiphysis. MRI right knee: Torn ACL, partially torn PCL, macerated medial patellar retinaculum medially,
multiple intrarticular loose bodies which were likely soft tissues/ligamentous in etilogy, medial femoral condyle osseous
contusion, grade II injury medial head gastrocnemius musculature and an anatomic variant cleft within the intercondylar
region of the femoral condyles with a psuedo-joint. There was no edema nor increased signal appreciable at the site of the
cleft on MRI making fracture unlikely. The MCL did appear intact. Follow up Stress Xray right knee: Widening of the
medial femoral epiphysis consistent with a Salter Harris type 3 fracture.
Final Diagnosis: Salter Harris type 3 fracture of the distal medial femoral condyle, ACL tear, Medial Retinacular MPFL
detachment femoral orgin, transient dislocation of the patella, right knee loose bodies
Treatment: The patient was treated surgically in a staged fashion. He underwent ORIF of his medial femoral condyle
fracture and repair of medial retinaculum tear initially. The patient was followed every 2 weeks after the initial surgery. He
was initially put in a brace in locked extension and was started in physical therapy to minimize his overall stiffness. Weight
bearing was restricted to touchdown for 6 weeks. After 1 month he was allowed range of motion from 0 to 40 degrees
and was increased to 70 degrees 1 week after that. He had a repeat X ray that showed a healed medial condyle fracture.
At 9 weeks post surgery, the patient was scheduled for ACL reconstruction. Intraoperatively, a meniscal tear was noted
which was additionally repaired.
Outcome: The patient has been doing well post-operatively and has been continuing his physical therapy for
strengthening.
Follow-Up: The patient is continuing with physical therapy at this current time and will be following up with the
Orthopedic surgeon consistently prior to returning to activity.
Thigh Pain in a Young Skier
Authors: Terra Blatnik, MD; Michelle Schweiger, MD; Ryan Goodwin, MD
Affiliation: Cleveland Clinic Children's, Cleveland, OH
History: 7 year old active female skier presents with right thigh pain. Pain started approximately one week prior to office
visit while she was skiing. She had fallen a few times while skiing but none that was memorable for an injury to the thigh.
She typically skis one day every weekend. Pain is worst after walking a lot. Parents have noticed a limp. Past medical
history includes short stature and subsequently diagnosed growth hormone deficiency. At the time of diagnosis she was
placed on Somatropin (exogenous growth hormone). She is currently on Somatropin 1.1 mg subQ six days per week
(given in thighs). Family history is significant for short stature in her mother. Social history is non-contributory. Review of
systems is negative for fevers, leg swelling or erythema of the leg. Her injection sites have looked normal. On this initial
visit, her exam was relatively benign except for tenderness to palpation over the right mid-thigh. Hip x-rays (AP/frog leg)
were normal. She was diagnosed with a quadriceps muscle strain and given instructions for supportive measures (ice,
gentle stretching, and gradual return to activity). Pain improved for 2-3 months and she resumed normal activities. She
returned to clinic 4 months later with one week of worsening pain in the right thigh. Parents report significant limping and
unusual gait while walking over the prior weekend. Pain is reported as 3/10 and is not waking her at night.
Physical Exam: Height is 3ft 8 in, weight 48lbs. Well appearing girl. Hip/knee exam is significant for full and pain free
range of motion at the hip and knee. Strength is slightly reduced for knee extension but is otherwise normal. She has
tenderness to palpation over the mid-quad. There are no skin changes, swelling or erythema of the thigh. Negative FABER
and FADIR. Patient has slight limp on exam today but video shown by parents reveals a significant antalgic gait.
Differential Diagnosis:






Right quadriceps muscle strain
femoral stress fracture
local irritation from growth hormone injections
osteomyelitis from growth hormone injections
Legg-Calve-Perthes
slipped capital femoral epiphysis.
Test Results: AP/Frog leg pelvis/right hip: negative (done at initial visit) MRI right hip: Avascular necrosis of the right
proximal femoral epiphysis with small reactive right hip effusion.
Final Diagnosis: Legg-Calve-Perthes potentially related to growth hormone treatment.
Treatment: Findings were discussed with both endocrinology and orthopedics. Her dose of Somatropin was decreased
to 0.6mg SQ six days per week because of the potential connection of growth hormone use and Legg-Calve-Perthes. She
was seen by orthopedics and placed on naproxen twice daily. Follow-up 2 weeks later revealed minimal improvement so
she was started in physical therapy.
Outcome: Follow-up with orthopedics was done two months after initiation of physical therapy. At this time, repeat x-ray
showed progression of her disease with a break in Shenton's line. Due to this change in clinical course, it was decided that
she should undergo a right adductor tenotomy with arthrogram and placement of bilateral Petrie casts.
Follow-Up: The patient has recently had her casts removed and has started into physical therapy and aquatherapy. She
will be seen again in one month for repeat x-rays.
My Foot and Ankle Hurt, Again!
Authors: Velyn Lisa Wu, MD
Affiliation: Lynchburg Family Medicine Residency Lynchburg, VA
History: A 57 yo post-menopausal Caucasian female presents for a second opinion regarding her new onset right foot
and ankle pain. 4 weeks ago she developed right lateral ankle and foot pain that occurred mainly with activity and
subsided after rest. Pain is now present when simply standing. The ankle hurts more than the foot. She endorses moderate
swelling; she denies bruising, bleeding, skin lesions and toe numbness. She denies injury around onset of pain, increased
or new physical activity to her daily routine. She was placed in a walking boot a week ago and this has helped her pain
some when she uses it. 3 weeks ago, she saw a PA and was diagnosed with Achilles tendonitis and received an Achilles
tendon sheath steroid injection which resulted in no relief. She then saw a doctor in the same clinic and was offered an
ankle joint injection which she refused. Her feet/ankle problems started about 1.5 years ago with right 4-5th metatarsal
foot pain that was diagnosed as metatarsalgia and treated with corticosteroid injection and multiple PO steroid packs. She
also sustained a traumatic right 4th middle phalanx fracture about 1 year ago which was treated conservatively and had
healed appropriately. Normally she is able to ambulate pain free and walk for exercise effortlessly.
Physical Exam: Gait - limps, favoring right side. No assistive device, not wearing boot as previously instructed. Right
foot/ankle - No visible deformity. Moderate swelling is present to lateral ankle, no foot swelling. No erythema, no skin
lesions. Active range of motion is full with no pain in all planes of movement. Passive range of motion is full with no pain
on forced ankle inversion, eversion or dorsiflexion. Strength is 5/5 on ankle plantar/dorsiflexion, inversion/eversion, toe
extension/flexion Palpation revealed no effusion. Significant tenderness over anterolateral ankle and 4-5th metatarsal
shaft. Maximally over lateral talus. Negative squeeze test, negative anterior drawer test. One leg stand on right leg is
mildly painful.
Differential Diagnosis:








Metatarsalgia with referred pain to ankle
Fracture
Overuse injury
Osteoarthritis
Osteoporosis/osteopenia
Tumor
Ligamentous injury
Extensor tendonitis
Test Results: Xray ankle: no fracture, no tumor, no malalignment, no foreign body. Triple Phase Bone Scan(obtained
instead of MRI due to copay): Increase flow and tracer accumulation to right ankle. Scout view also showed increased
uptake to left knee. Dexa Scan: osteoporosis in lumbar spine and left femoral neck. Osteopenia in left trochanter and
total hip.
Final Diagnosis: Osteoporotic insufficiency fracture - clinically localized to right talus
Treatment: Non weight bearing with walking boot, knee walker at work, Passive ROM ankle exercises with progression
to light weight as tolerated, Work modification to desk duty, IV Bisphosphonate, Calcium/vitamin D, Stop frequent use of
IM and PO steroids
Outcome: Patient was seen monthly over the next 4 months, initially experienced severe worsening of pain and swelling
due to noncompliance with walking boot. She experienced marked improvement a few weeks after her IV bisphosphonate
treatment. Patient was gradually progressed to weight bearing status as her pain resolved. She was also referred to PT
once her pain improved. She was able to ambulate at home painlessly without walking boot but not during shopping.
Follow-Up: Gradual return to full weight bearing as tolerated. Out of boot unless painful. Continue to conduct trials of
ambulation with daily activities. Follow-up monthly until pain free and ambulating normally.
Successful Management Following Failed Endoscopic Release
Authors: Adam M. Pourcho, DO; Mederic M. Hall, MD
Affiliation: Pourcho - Swedish Sports Spine and Musculoskeletal, Seattle, WA Hall - Iowa Sports Medicine, Des
Moines, IA
History: A 47 year old male presented to the sports medicine clinic in February (2014) with a complaint of recalcitrant
right plantar fasciopathy status post endoscopic release in January (2013). His pain started insidiously 2 years prior to his
surgical release. Initial management included a night splint, orthotics, physical therapy, and cortisone injections. These
interventions provided modest short term relief. In January, 2013 he underwent an endoscopic plantar fascia release. After
1-2 months of relief, his symptoms return to pre-operative levels, although now involving a slightly greater region of the
plantar medial heel and arch. On presentation, he had pain with prolonged standing that was worse with the first few
steps in the morning. The pain was located over the posteromedial heel anddistally over the mid-medial arch. The patient
had a Foot and Ankle Hind Foot score (FAS-HF) of 68/100 on initial presentation.
Physical Exam: On physical examination he had severe pain with palpation at the origin of the plantar fascia on his
medial calcaneus and 2 cm distal to the origin. There was limited dorsiflexion with some reproduction of pain. He had a
negative Tinel's sign over his tarsal tunnel and abductor canal.
Differential Diagnosis:





Recalcitrant plantar fasciopathy
Baxter's neuropathy
Flexor digitorum brevis tendinopathy
Tarsal tunnel syndrome
Chronic exertional compartment syndrome of the medial foot compartment
Test Results: Standing X-rays of his foot demonstrated pes planus, a large calcaneal plantar heel spur and small plantar
punctate calcifications just distal to the calcaneal tubercle. On sonographic examination, patient was found to have diffuse
plantar fascia thickening with a large hypoechoic region in the mid-substance of the plantar fascia approximately 2 cm
from the calcaneus, at the site of presumed endoscopic release. Within this area were several punctate calcifications and
there was reproduction of symptoms with increased transducer pressure in this location. The first branch of the lateral
plantar nerve (Baxter's nerve) was without abnormality.
Final Diagnosis: Recalcitrant plantar fasciopathy status- post endoscopic plantar fascia release
Treatment: We proceeded with sonographically guided percutaneous fasciotomy utilizing the TX-1 device (Tenex
Health&reg;) in April (2014). A sonographically guided tibial nerve block was performed for anesthesia. Using orthogonal
sonographic views, the area of plantar fasciosis was debrided. Total energy time was 2 minutes and 51 seconds. The
patient was placed in a walking boot for 5 days.
Outcome: Plantar fasciopathy is common, affecting approximately 2 million patients a year with up to a 10% lifetime
incidence. Risk factors such as obesity, age, prolonged walking or standing, excessive pes cavus or pes planus, and heel
cord tightness often contribute to the disease. Conservative management with activity modification, weight loss, orthotics,
stretching and strengthening, night splints, oral and topical anti-inflammatory medications and injections are often firstline treatment. When these interventions fail, surgical treatment is often offered. The presented patient had failed
conservative and surgical management and had developed what appeared to be excess scar tissue at the site of surgical
release. We successfully performed a sonographically guided percutaneous fasciotomy with the TX-1 device and
demonstrated excellent results. To our knowledge, this patient represents the first submitted case using the TX-1 device
for failed endoscopic release of recalcitrant plantar fasciopathy and represents a potential future avenue for management
of patients who have failed conservative and surgical management of chronic plantar fasciopathy.
Follow-Up: At 8 weeks follow-up his FAS-HF score had improved to 87/100 and had resumed most activities. At 6
months, he had returned to running and reported that he was very satisfied with his outcome and would recommend the
procedure.
A Baseball Player With Sudden Onset of Dyspnea While Showering
Authors: Rizwan Ahmad, MD, Nicole M. Protzman, MS, Fidel D. Bautista, MD
Affiliation: Coordinated Health, Bethlehem, PA
History: In September of 2014, a 21 year old senior varsity baseball player at Lehigh University developed dyspnea,
pleuritic chest pain, and voice alterations after practice while showering in the locker room. He had no history of asthma,
recent infections, or blunt trauma. He was subsequently taken to the emergency room for evaluation. A physical
examination and imaging were performed.
Physical Exam: Upon physical examination, the patient demonstrated findings consistent with subcutaneous
emphysema, such as crepitus on the anterior chest wall. His auscultatory findings were clear bilaterally along the anterior
and posterior chest wall.
Differential Diagnosis:










Afebrile pneumonia syndrome
Asthma
Bronchitis
Dissecting thoracic aneurysm
Medial pneumothorax
Myocarditis
Paratracheal air cyst
Pericarditis
Pneumopericardium
Spontaneous pneumothorax
Test Results: Posterioranterior and lateral x-rays were ordered and demonstrated evidence of air surrounding the heart
and mediastinum. There were minimal biapical pneumothoraces without evidence of pleural effusions. Subcutaneous
emphysema was noted bilaterally in the chest wall and supraclavicular regions. He was treated conservatively and
observed overnight. The next day he was discharged and instructed to follow-up with a cardiothoracic surgeon.
Final Diagnosis: Considering the clinical presentation and radiographic findings, the patient was diagnosed with
spontaneous pneumomediastinum.
Treatment: The patient followed up with the cardiothoracic surgeon one week later. Per the cardiothoracic surgeon’s
evaluation, the patient was cleared to return to play with the exclusion of strenuous weightlifting over the following three
weeks.
Outcome: Following discharge, the patient’s symptoms resolved. At the one week follow-up, the patient had no
residual symptoms nor any complaints.
Follow-Up: The patient has returned to activity and is preparing for the upcoming 2015 baseball season.
Surgical Stinger
Authors: Seth Schmoll, DO; Michael DaRosa, DO
Affiliation: Community Health Family Medicine Residency
History: 16 year old, previously healthy male, called off field in 4th quarter for injury check during varsity football game after
complaining of left arm numbness. Four plays prior he sustained a hit while being blocked, causing a quick R-sided bending and
rotation motion to upper torso. Pt got up immediately after the play, shook out neck and returned to defensive huddle and continued
to play at high level without any major contact. After being walked over to sideline he reported L-arm numbness had resolved. Did
complain of moderate L-neck/shoulder pain and new numbness over L-upper quadrant and L-lateral rib cage. Denied any weakness,
dizziness, headache, chest pain, shortness of breath or abdominal pain.
Physical Exam: Gen: AOX3, NAD, sitting comfortably on exam table. HEENT: Normocephalic, atraumatic. Neck: No midline tenderness.
Mild tenderness to palpation over L-paraspinal muscles, ROM full, end ROM exacerbated L-paraspinal pain. Upper Extremity muscle
strength/ sensation normal and equal bilaterally. Negative Spurling test. Pulm: LCTA B/L, no wheezes, rhonchi, or rales CV: RRR no
murmur, rub or gallop L-Shoulder: TTP over L-trapezius muscles. Nl ROM, muscle strength and sensation. Negative Neers/Hawkins,
Cross Arm and AC load, O'Brien's test, Speeds and Yergasons. Thoracic Spine/Chest Wall: Thoracic spine non-TTP. Anterior and lateral
Rib compression produced LUQ abdominal pain. Gross thoracic ROM caused LUQ abdominal pain Abdomen: Normal inspection
without ecchymosis. BS X4. Non-distended. TTP over LUQ. Abdomen rigid w/guarding and rebound tenderness present. No flank
tenderness to percussion. Skin: no rash, erythema or ecchymosis
Differential Diagnosis:
















Stinger Injury/Brachial Plexus Injury
Cervical Spinal Cord Injury/Fracture
Cervical Disc Herniation
Cervical Sprain/Strain
AC joint sprain
Clavicle Fracture
Shoulder Dislocation
Shoulder Sprain/Strain
Rib Fracture
Pneumothorax
Hemothorax
Concussion
Intracranial bleed
Stomach injury
Kidney injury
Splenic injury
Test Results: Initial ED Vitals: BP 128/64, P-71, R-21, SPO2 99%, T-36.78C Initial Labs/radiology in ED: CBC- WBC 7.47, RBC 4.5,
Hgb 13, Hct 39.7, MCV 88.3, MCH 28.9, MCHC 32.7, RDW 13.6, Platelet 189 BMP: Glucose 108, BUN 20, Cr 1.5, Na 140, K+ 4, Cl 103,
CO2 28.3, Ca 9.9 Hepatic panel: Albumin 5.1, tot prot 6.8, alk phos 190, AST 28, ALT 13, tot Bili 0.1, Direct Bili 0.3 Lipase: 24 CT
Head/Brain W/O contrast - Impression: No evidence of acute intracranial process. No evidence of intracranial hemorrhage CT Spine
Cervical W/O contrast - Impression: Straitening of the normal cervical lordosis. No evidence of acute fracture or traumatic
malalignment CT Abdomen and Pelvis W Contrast - Impression: Shattered lower pole spleen w/evidence of subcapsular hematoma.
Moderate free fluid in abdomen, bilateral paracolic gutters and large amount of fluid seen within the pelvis. There is high density fluid
seen in the lower pole of the spleen in the area of multiple lacerations or shattered spleen indicative of active bleeding which would
indicate vascular involvement. ***Pt was transferred to Level 1 trauma center**
CBC: Wbc 7.3, RBC 3.47, Hgb 10.1, hct 30.3, MCV 87, MCH 29.2, MCHC 33.4 RDW 13.3 platelet 148 Pt: 12.9, INR- 1.19, PTT 26.0, Xray
Left Shoulder 3 view: normal
Final Diagnosis: Grade 4 Splenic Laceration w/extravasation and blush on CT. Acute Anemia Secondary to blood loss. Cervical sprain
Stinger: resolved
Treatment: Emergent Coil embolization of lower pole of spleen w/selective catheterization and diagnostic angiography of celiac artery
and splenic artery including upper and lower pole segmental arteries.
Outcome: Pt discharged in stable condition after 5-day hospitalization w/instructions to resume gentle activities such as walking and
seated activities. Restricted from any physical activities exposing to repeat trauma to abdominal area until follow up.
Follow-Up: Ultrasound 5-weeks post injury - Stable appearance of inferior pole splenic laceration w/infarct from embolization
procedure and no evidence of complication. Recommended return to non-contact practice for 2-weeks then return to full contact
practice and game. Pt returned to full contact game 6-weeks post injury without any complaints/complications.
Knee Injury in Division 1 Football Player
Authors: Anna R. King, DO, MPH; Ramin R. Tabaddor, MD
Affiliation: Maine Medical Center, Portland, ME
History: A 20-year-old collegiate football player presented after sustaining an injury to his right leg. His right leg was
planted, knee slightly flexed, and he was struck forcefully from the left side. He was unable to ambulate. Swelling was
evident 20 minutes later. He had no history of prior injury to the knee and no significant medical history.
Physical Exam: 5 days post-injury, inspection revealed ecchymosis along the medial aspect of the knee and visible
patella alta. ROM was limited due to patient guarding. Palpation revealed 3+ effusion and a defect of the patella tendon.
He was unable to perform a straight leg raise and was still unable to ambulate. Grade 3 MCL insufficiency was present. He
was stable to varus stress. Lachman and pivot shift tests were limited secondary to pain and swelling.
Differential Diagnosis:







Patellar tendon rupture
MCL tear
meniscal injury
ACL tear
tibial plateau fracture
femoral condyle fracture
patella fracture
Test Results: MRI 4 days post-injury: full thickness tear of the proximal ACL, full thickness tear of the distal tibial
insertion of the superficial MCL with 2.5cm of retraction, full thickness rupture of the patella attachment of the patellar
tendon with 2cm of retraction, and a posterolateral corner meniscal tear.
Final Diagnosis: Simultaneous rupture of patellar tendon, ACL and MCL.
Treatment: He was treated in a staged manner with acute primary repair of the patellar tendon followed by a 3-month
rehabilitation period during which serial increases in flexion while in the knee immobilizer were permitted. Physical therapy
was begun at 2 weeks post-op. At 2 months, patient had 4/5 quadriceps strength, normal patellar tracking and no MCL
instability to lateral stress. At 3 months, he returned to the operating room for ACL reconstruction and intraoperative
assessment of the MCL and menisci. The MCL was treated conservatively given its stability with 30 degrees valgus load,
and the medial and lateral menisci were found to be normal upon probing. He was placed on a standard postop ACL
protocol.
Outcome: At 6 months, he had negative Lachman's, anterior drawer, and pivot shift tests. He had no effusion and only
mild quad atrophy. He was able to straight leg raise and was ambulating well.
Follow-Up: He returned to sport 12 months after ACL reconstruction.
Stiff Leg in a 16 y/o Football Player
Authors: Stephan Esser, MD, USPTA; Kevin Murphy, MD
Affiliation: Southeast Orthopedic Associates Ponte Vedra Beach, FL
History: 16 y/o male High School Football player presented to an outpatient SM Clinic for evaluation of R-knee pain
and altered knee range of motion. He reported that two weeks prior he was playing Football and was tackled by opponent
who struck full force of helmet into front of his thigh. He had immediate pain and swelling but able to walk. He was
removed from play by coach then iced and rested his right leg for the next two days. Then slowly increased walking but
noted still had a lot of pain and very limited range of motion. He went to see PC physician about one week later since not
improving. Xrays of the knee and femur showed no pathology and the PC physician ordered an MRI which was denied by
insurance. Patient then presented to an Outpatient SM Clinic where he was evaluated. In view of the loss of knee range of
motion and persistent pain, an MRI of the knee was ordered to rule out intra-articular sources of dysfunction. When this
proved unrevealing, Xrays of the femur were repeated and a MSK US was completed. In light of the extensive bony
changes noted in the anterior quad, an MRI of the femur was ordered.
Physical Exam: Gen: Healthy appearing, muscular AA male in NAD, no swelling in the right quad or effusion in the right
knee Gait: Antalgic on the right, minimal flexion of the right knee during all phases of gait ROM: Right knee with 0
degrees of extension and 30 degrees of both active and passive flexion Neuro: 5/5 strength in the distal L4-L5 myotomes
and intact sensation to LT in the L2-S1 dermatomes, 2+ patellar and achilles reflexes bilaterally Palpation: No pain with
palpation of the knee, however mild-moderate pain with palpation over the anterior mid-quadriceps, with minimal fullness
Differential Diagnosis:



Myositis Ossificans of the Quads
Posterior Meniscal Tear
Intra-articualr loose body
Test Results: Early Xrays: Unremarkable right knee and femur Xrays MRI Knee: Unremarkable Ultrasound: Changes
in the bony contour of the proximal, mid-femur, without evidence of muscle disruption or hematoma Repeat Xrays Right
Femur: Extensive intra-muscular sclerotic changes anterior to the femur in the region of the vastus intermedius MRI
Femur: Extensive sclerotic changes throughout the vastus intermedius with small heterogenous foci of increased signal
Final Diagnosis: Myositis Ossificans of the right Vastus Intermedius
Treatment: Oral Anti-inflammatories and Ice Extensive Manual Therapy: Without Benefit Mobilization under
Anesthesia: Pending Surgical Resection: Pending
Outcome: The patient had no improvement with extensive manual physical therapy including use of a passive motion
device and various splinting techniques. In view of his severe range of motion limitations, we will be trialing progressive
mobilization while under anesthesia. If this fails to provide benefit we will advance to surgical interventions once the
myositis has plateaud and is no longer metabolically active on bone scan or MRI.
Follow-Up: The patient has been unable to participate in sports due to his severe limitations in knee flexion. He has
been encouraged to maintain range of motion and physical activity as tolerated. He is being actively followed in clinic at
this time. PLEASE NOTE: We believe this is a very important case since it is the result of inadequate early care and could
likely have been prevented. We also have compelling images including Xray/US and MRI. The patient is in the midst of
treatment and by the end of the year he will have likely completed all his treatment and we will have the full case with
regards to outcomes etc. We are submitting now with incomplete results due to the time constraints. We hope you will
give us serious consideration.
Return to Play After Seizure
Authors: Edward Qiao, DO; Corey Dean, MD
Affiliation: St. Joseph Mercy Hospital, Ann Arbor, MI
History: 15 year old male with history of well controlled asthma who presents with 3 episodes of loss of consciousness
within the past year. There is no family history of premature sudden death or seizure. Each episode lasts 1-2 minutes and
occurs with rest or activity, but are not associated with trauma or head injury. Patient experiences a dull frontal headache
beforehand. Sometimes the episodes are associated with "whole body shaking". During one of these episodes, he was
incontinent of urine. Patient feels confused for about 5 minutes after each episode.
Physical Exam: Height 6'4". Weight 225. Blood pressure was 140/82. Cardiology exam is regular rate and regular
rhythm without murmurs appreciated. Extremities are well perfused and no edema are seen. Pulmonary exam reveals
lungs that are clear to auscultation bilaterally without wheezes, rhonchi or crackles. Neurologic exam reveals that patient is
alert and oriented x 4, cranial nerves II to XII are intact, visual fields and extra ocular muscles are intact without
papilledema, no dysmetria or dysdiadochokinesia, sensation intact to light touch and strength is five out of five in all four
extremities. Reflexes are 2+ and symmetric at the biceps, triceps, knees, and ankles. Plantar responses are flexor. Patient
has a nonataxic gait.
Differential Diagnosis:





Seizure (generalized, complex partial, simple partial, absence)
vasovagal syncope
underlying structural heart disease
breath holding spell
exercise induced syncope
Test Results: LABS: Sodium 139 mEq/L, Potassium 4.2 mEq/L, Chloride 107 mEq/L, Carbon Dioxide 27 mEq/L, Glucose
141 mg/dL, BUN 8 mg/dL, Creatinine .68 mg/dL, Calcium 9.8 mg/dL, Glucose 141 mg/dL, Alkaline Phosphatase 234 IU/L,
ALT 39 IU/L, AST 29 IU/L, Bilirubin .2 mg/dL. Thyroid Stimulating Hormone .74 White blood cell count 5.9 thou/mcl,
Hemoglobin 13.3 gm/dL, Hematocrit 39.2%, Mean Corpuscular Volume 86 Fl Toxicology negative for alcohol,
amphetamine, barbiturates, benzodiazepines, cocaine, opiates, phencyclidine, tetrahydrocannabinoids IMAGING: EKG
showed normal sinus rhythm with QTc 405 and without Q waves or ST-T wave changes. J point elevation noted diffusely.
Chest Xray showed no acute process. MRI brain w/o contrast showed no acute process. Outpatient EEG-epileptic activity
ECHO negative for any structural heart disease.
Final Diagnosis: Generalized (tonic-clonic) seizure
Treatment: 500mg BID Keppra
Outcome: Cardiology did not believe his "syncopal" episodes were cardiac related. Outpatient EEG showed seizure and
patient was placed on 500mg BID Keppra by his neurologist. Patient has not had recurrence of symptoms since being
started on Keppra.
Follow-Up: Patient will be re-evaluated for basketball 3 months from the date of last seizure. He will be restricted from
archery, riflery, motorized sports, swimming, diving, rowing, and contact sports including football for 6 to 12 months from
date of last seizure.
Chronic Foot Pain in a Rower
Authors: Nicholas J. Crognale, DO; Maheep Vikram, MD
Affiliation: St. Lukes University Hospital and Health Network, Bethlehem, PA
History: The patient is a 37 year old male presenting with a one year history of continuous left foot and ankle pain. He
describes the pain as a sharp and throbbing sensation. It is located mostly on the dorsal and lateral aspect of the left foot.
He states that the pain is non-radiating and is reproduced with standing for long periods of time and also while kayaking
in the Bahamas. He also notes episodic periods of generalized swelling of the foot. He denies any numbness or tingling
sensation in the foot or ankle region. He denies any warmth to the skin area. He denies any symptoms of motor or
sensory deficits.
Physical Exam: Musculoskeletal: There is swelling noted on the lateral aspect of the left ankle. There is noted pinpoint
tenderness at the anterior lateral gutter. There is full range of motion with flexion and extension, inversion and eversion.
Squeeze test, talar tilt and thompson test are all negative
Differential Diagnosis:




Ankle sprain
Bursitis
Osteochondritis dissecans
Tarsi Tunnel Syndrome
Test Results: MRI left ankle without contrast: There is a small retrocalcaneal bursitis. There is a 2.6 x 0.8 x 1.2 cm
ganglion cyst emanating from the sinus tarsi and extending laterally. Articular surfaces all in tact. Normal ligamentous
structures. Normal tendon structures.
Final Diagnosis: Sinus Tarsi Syndrome
Treatment: Aspiration of 2 ml of clear fluid from the sinus tarsi tunnel ganglion cyst with injection of 1 ml lidocaine 1%
and 1 ml of betametasone 6mg/ml.
Outcome: The patient return to the office 2 weeks following the procedure and reports complete resolution of the
sharp, throbbing sensation in the dorsal foot and lateral ankle region.
Follow-Up: The patient was followed up at 4 months and reports complete resolution of his pain. He is able to stand
for long periods on his feet with no reproduction of his pain. He has returned to kayaking with no restrictions and reports
no further incidences of pain re-occurrence.
Painful Lumps in a National-Level Shooter
Authors: Robyn Knutson Bueling, MD, MS
Affiliation: Sanford Orthopedics and Sports Medicine, Fargo, ND
History: A national junior-level shooting athlete returned to clinic for "painful lumps" on her lower legs. She has
previously been seen and treated successfully with physical therapy for shin splints. This seems different. No known injury.
No fever. "Regular high school kid" illnesses recently. No swollen joints or other locations that are painful. No previous
similar bumps. No skin rashes. No family history of similar. She does lie down with her legs in a figure-of-four position
when she shoots or is standing on her feet for long periods of time on concrete. She can't think of a piece of equipment or
anything that may have been pushing on or irritating her lower legs.
Physical Exam:




Her entire physical exam, including joint exam is entirely normal with the exception of slightly reddened, tender,
mobile soft-tissue lumps on the bilateral lower extremities.
1.5cm mass near the medial proximal right tibia
3cm mass overlying the distal right tibia.
Series of four smaller masses, all about 1-2cm on the left lower leg just lateral to the proximal tibia.
Differential Diagnosis:





Fibroma
Lipoma
Shin splints
Malignancy
Rheumatologic disease
Test Results:
Bilateral tibia/fibula Xrays are normal.
Ultrasound examination reveals that the masses are not fluid filled and do not arise from the bone.
MRI reveals mild-to-moderate periosteal reactions on the bilatearl anteromedial tibial diaphyseal periostia consistent with
shin splints and early stress reaction. No abnormalities are noted in the soft tissues at the location of her palpable painful
lumps.
CBC, CMP, ESR, RF, HLA B27, TSH are normal.
ANA screen is elevated at 2.29 (0-0.99) and reflex Centromere antibody is high at 5.0 (&lt;1.0)
Final Diagnosis: Erythema Nodosum.
Treatment: She was seen by rheumatology, where positive ANA and anti-centromere were not felt to represent lupus
or scleroderma due to lack of exam findings. It was noted that she had a recent streptococcal pharyngitis, but no ASO was
obtained. The painful lumps were felt to be erythema nodusum as a sequelae of strep throat. She was treated with
Naproxen 500mg BID for 3 weeks until symptoms resolved on their own.
Outcome: Symptoms resolved after about 2.5 weeks and have not returned.
Follow-Up: She continued successful participation in her shooting sports without limitations and continued treatment
of her shin splints on an ongoing basis.
Persistent Posterior Leg Pain in Marathon Runner
Authors: Stephanie Giammittorio, DO; Carly Day, MD
Affiliation: Cleveland Clinic
History: A 68-year-old marathon runner presented with complaints of bilateral, right greater than left, knee and posterior
leg pain for 4 years. It started while training to do 4 marathons in a 12-week period, however he denied any specific injury.
The pain was up to 7/10, penetrating, and located in the posterior thigh and anterior knee with occasional vague distal
radiation. His right leg felt weak when pushing off during running. He was symptom free at rest and only had pain while
running. He denied any back pain, numbness, tingling, or bowel and bladder changes. Previously other physicians
diagnosed him with patellar tendinitis, patellofemoral pain, and hamstring tendonitis. He did 2 courses of PT with gait
analysis, which only gave him minimal relief. He tried a neoprene sleeve for his thigh and multiple patellofemoral knee
braces which did not help. Use of anti-inflammatory gel was unsuccessful. He modified his activity and tried cross training
but continued to have pain. Previous MRI of his hamstrings showed proximal tendinitis bilaterally. He has run 32
marathons in the past. He has been running 40-50 miles per week with a 25 mile long run.
Physical Exam: Examination of the lumbar spine was notable for no tenderness to palpation, mild decrease with both
flexion and extension with mild back pain at end range lumbar flexion. His lower extremity strength was intact. There was
no tenderness over the ischial tuberosity or mid belly hamstring. His hip range of motion was 45° of external rotation and
20° of internal rotation bilaterally without pain. There was no tenderness throughout bilateral knees. Knee range of motion
was notable for full extension and 140 degrees of flexion bilaterally. Distal sensation and pulses were normal. His foot
exam was notable for mild pes planus with normal hindfoot alignment.
Differential Diagnosis:






Hamstring tendinitis
Hamstring tear
Femoral stress fracture
Patellar tendinitis
Patellofemoral pain
Lumbar radiculopathy
Test Results: Knee xrays showed mild degenerative changes and thickened patellar tendon Lumbar xrays revealed
severe lower lumbar apophyseal DJD with grade 1 L4-5 listhesis and degenerative L4-5 disc disease Lumbar MRI showed
L4-L5 prominent facet degenerative changes with ligamentous hypertrophy, grade I degenerative anterolisthesis and
uncovering of disc material causing severe central canal stenosis with moderate right and mild left foraminal stenosis.
Final Diagnosis: Lumbar foraminal stenosis and L4/5 anterolisthesis causing radicular leg pain with running.
Treatment: L5/S1 intralaminar epidural steroid injection
Outcome: The patient felt 95% better after the injection
Follow-Up: The patient was able to return to running 40 miles per week with a long run of up to 24 miles with very mild
pain. He did not have any pain at rest or with daily activities. He has resumed marathon training.
Sport Focused Treatment to Optimize Return to Play in a Baseball Pitcher: A Case Report
Authors: Ashley D. Zapf, MD; Kevin R. Vincent, MD; Giorgio Zeppieri, Jr, MPT; Jason L. Zaremski, MD
Affiliation: Schwab Rehabilitation Hospital, Chicago, IL
History: A 14 year old, left handed pitcher with no prior injury to his throwing arm presented to the sports medicine
clinic with an acute onset of left medial elbow pain. The pain was described as a sharp sensation which started after
throwing several pitches. The patient denied experiencing a &#8220;pop,&#8221; and he had no associated neurological
symptoms.
Physical Exam: Initial physical examination revealed tenderness to palpation of the left medial epicondyle. Valgus
maneuver of the left elbow elicited mild pain as well as laxity with a solid endpoint. The rest of the examination was
normal.
Differential Diagnosis:







Medial Epicondylitis
Ulnar Collateral Ligament Strain
Ulnar Collateral Ligament Disruption
Traction Apophysitis
Medial Epicondylar Fragmentation
Ulnar Neuritis
Medial Epicondyle Fracture
Test Results: Initial radiographs of the left elbow revealed a smooth chronic avulsive osseous injury at the medial
epicondyle at the origin of the Ulnar Collateral Ligament (UCL). Given the patient's sport, he was made non-weight bearing
to the left upper extremity for 6 weeks to allow for osseous bridging. At his 6 week follow-up appointment, repeat
radiographs were obtained which revealed an unchanged osseous lesion. On physical examination, there was no
tenderness to palpation of the left medial epicondyle; however, there was a mildly positive valgus maneuver compared to
the contralateral side. An MRI with Arthrogram was ordered to assess for stability of the UCL. The MRI Arthrogram
demonstrated a completely intact UCL with a chronic bony avulsion injury at the origin of the UCL.
Final Diagnosis: Ulnar Collateral Ligament Variant Injury
Treatment: In order to develop a successful treatment plan, it was necessary to interpret the MRI Arthrogram results in
terms of how they would affect the patient's ability to keep up with the demands of his sport. Given that he was a baseball
pitcher, the decision was made to treat the patient as if he had a partial UCL tear. After the initial 6 weeks of non-weight
bearing to the left upper extremity, he started a 6 week non-throwing physical therapy program. During that time, he was
also allowed to begin swinging. He followed-up in the sports medicine clinic at the 12 week post-injury mark. He was
asymptomatic, and therefore, was transitioned to a 12 week throwing program.
Outcome: The patient successfully completed a progressive 12 week throwing program, and was able to return to full,
unrestricted activity prior to the start of his high school baseball season.
Follow-Up: In the case of this athlete, it was imperative to understand that pitching creates a valgus torque, and allowing
this motion too early in the recovery process could have created malunion or non-union of the osseous lesion. Thus,
understanding sport specific demands on different joints can help tailor treatment and rehabilitation programs to allow
athletes to successfully return to play without set-backs.
Kneed for Speed
Authors: Nikki Castel, BS; Justin Young, MD
Affiliation: University of Hawaii John A. Burns School of Medicine, Honolulu, HI
History: 23 year-old female with PMH significant for asthma and IBS presented with left medial knee pain for 2 weeks.
She was running in an area that is very hilly, training for a half marathon, and felt the pain at the 4-mile point. She had
been running every day and her longest run was 17 miles. Due to the pain, she started walking. Over the next 2 weeks, the
pain got progressively worse so she came to see a sports medicine doctor. Pain is 5/10 at rest and 10/10 with running.
Pain is aggravated with activity and alleviated with cold compress and rest. She reportedly has regular menstrual periods;
her LMP was 3 weeks ago. She has no history of fractures. Remainder of ROS is negative.
Physical Exam: T 37.2, HR 96, BP 89/59, Ht 5'2", Wt 97lb, BMI 17.7 General: Healthy pleasant female in no acute
distress Left knee:
Full ROM with flexion and extension to 135/-5°
No effusion
Strength intact
Negative patellar compression test for pain or crepitus
Negative Clarke’s inhibition test, no patellar instability
No ligamentous laxity to varus or valgus stress testing at 0 and 130°
Negative Lachman's test. Negative anterior and posterior drawer tests
Pain to palpation over medial knee, specifically over the joint line
McMurray's testing produces pain but no crepitus
No patellar tendon tenderness, pes anserine bursa tenderness, or distal IT band tenderness.
Differential Diagnosis:






Medial meniscus tear
Medial collateral ligament sprain or tear
Medial patellofemoral ligament sprain or tear
Pes anserine bursitis
Medial plica syndrome
Stress fracture
Test Results: X-ray of left knee on presentation is normal MRI of left knee two weeks after initial presentation shows:
Medial tibial condyle fracture, most compatible with a stress fracture. Surrounding marrow edema in the medial tibial
condyle extending into the midline. X-ray of left knee three weeks after initial presentation shows: Non-displaced fracture
in the posterior, medial aspect of the proximal tibial metaphysis, compatible with stress fracture seen on recent MRI.
Final Diagnosis: Stress fracture of tibial plateau
Treatment:
Pain management with acetaminophen.
Over a period of 8 weeks, transition from non-weight bearing on crutches to limited activity.
Pain-free activity is the main criteria used to advance her activity. Assess at 2-week intervals.
Outcome:
Pain was well managed with acetaminophen. She was off the analgesic medication after 2 weeks
After week 1, she started walking with limited weight bearing. Over the next three weeks, she progressively increased her
weight bearing
At week 4, patient was able to fully bear weight, pain-free.
From week 4-8, patient did low-impact cardiovascular activity.
Follow-Up:
Patient was able to begin running at the end of treatment.
Patient was relocated to a different state for schooling.
A Surprising Cause of Chronic Neck Pain in a Collegiate Rugby Athlete
Authors: Kathryn L Gloyer, MD
Affiliation: Penn State Primary Care Sports Medicine, State College, PA
History: 22yo otherwise healthy male collegiate rugby athlete presents for evaluation for 5 months of ongoing right-sided neck pain.
Reports an initial acute hyperextension and rotation injury during a rugby game 5 months ago. Had significant pain at that time and saw
his PC provider and xrays were obtained. Report was read as normal. He reports the initial severe pain decreased over time and now has
mild persistent right-sided neck pain with extending the neck. There was never any radiation of the pain, associated numbness, tingling
or weakness of the upper extremities. He has been compliant with physical therapy with his athletic trainer since the injury without
improvement. He has continued to play full contact rugby for the last 5 months at prop position. He reports the pain as 4/10 with rugby
activities and 0/10 with rest from rugby activities.
Physical Exam: Patient is seated comfortably, in no apparent distress. Focused exam of his C-spine reveals full range of motion with
flexion, extension, lateral rotation, and side bending, however, he has reproducible right-sided neck pain with extension and rotation to
the right. His C-spine is nontender to palpation and his paraspinal muscles are nontender to palpation. Spurling's test is negative
bilaterally. He has no pain with active or resisted shoulder shrugging or resisted trapezoid elevation. Pain is not reproducible with range
of motion of the right shoulder. Strength is 5/5 in bilateral upper extremities. He is neurovascularly intact in bilateral upper extremities.
Differential Diagnosis:

Cervical strain

cervical sprain

myofascial pain

cervical spondylosis

cervical discogenic pain

cervical facet syndrome

fracture

stress fracture
Test Results: Cervical spine Xray: Fracture at the base of spinous process C2 with the margins of the fracture showing developing
sclerosis which may indicate a developing nonunion. Slight grade 1 subluxation C2 on C3 with all remaining components of the cervical
spine: unremarkable. Review of C-spine xrays from 5 months prior-Acute fracture of C2 consistent with Hangman's fracture. MRI
findings: 1. Old ununited C2 Hangman's fracture. 2. 2 mm of anterior subluxation of C2 on C3. 3. No disc herniations identified. 4. No
evidence of spinal or foraminal stenosis. MRI c-spine in flexion and extension: 1. No evidence of induced instability comparing neutral
flexion or extension sagittal views. 2. Facet hypertrophy at C2-3 possibly related to chronic ununited hangman's fracture/posterior
element defects at C2. 3. Bilateral neural foramen stenosis at C2-3, probably similar comparing neutral flexion and extension views,
subtle foraminal narrowing/stenosis also at C3-4 less pronounced appearing on the left on neck extension and neutral views than with
neck flexion. Bone scan of C spine: Symmetric foci of moderate radiotracer uptake corresponding to the nonunited C2 fractures shown
on prior CT and MRI.
Final Diagnosis: Old ununited C2 hangman's fracture without instability
Treatment:
Miami J Collar immobilization
Discontinuation of strenuous physical activity and all rugby activities
Surgery was initially planned for stabilization. However, after consultation with several neurosurgeons, patient decided on conservative
management with cervical collar
Outcome: Follow up studies Repeat bone scan in 6 months shows
Persistent increasing activity at the C2 level of the cervical spine
intensity of activity is perhaps slightly diminished from the prior exam with no evidence for new, interval, or progressive focus of
increased activity. Repeat cervical XR at 7 months shows:
The C2 fracture line is no longer visualized.
There is 1.7 mm of anterior subluxation of C2 on C3 in extension. This increases to 3 mm in flexion. This is consistent with physiologic
movement. Outcome: Bony union of C2 fracture
Follow-Up: The cervical collar was discontinued. The patient has returned to exercise, but no sports. The patient was advised to not
return to rugby.
A Rare Confluence of Diagnoses in a Division I Collegiate Football Player
Authors: Michael R. Tiso, MD; John J. Leddy, MD
Affiliation: University at Buffalo Sports Medicine Buffalo, NY
History: A 20-year-old African American male college football player presented to the training room with four days of
nausea, abdominal pain and loose stools. The athlete's symptoms started gradually over a few days with associated
weakness, fatigue and possible fevers. He was initially treated with a course of Ciprofloxacin for three days but had
continued worsening of his abdominal pain and diarrhea, which started to become mixed with blood. He was anorexic and
losing weight. The athlete had no recent sick contacts and his only travel was from his home in Florida a few weeks prior.
He reported no other past medical history or medications. He had two Hepatitis A immunizations within the past 5 years
and was up to date on all other vaccinations. He reported no abdominal trauma.
Physical Exam: Vitals: Temperature: 100.8°. Blood pressure: 114/76. Pulse: 86 General: Appeared fatigued and chilled;
HEENT: Normal sclerae, no pharyngeal erythema or tonsillar exudates Cardiovascular: Regular rate and rhythm, no
murmur, 2+ radial pulses Pulmonary: Lungs clear bilaterally without rhonchi, rales or wheezes. Abdominal: Bowel sounds
were present, mild left upper quadrant tenderness to palpation without rebound or guarding, nondistended. No palpable
organomegaly. Skin: No rash or open wounds
Differential Diagnosis:




Viral gastroenteritis
Infectious colitis
Inflammatory bowel disease
Acute viral syndrome
Test Results: Notable initial labs: AST 266, ALT 343, Alk phos 129, WBC 16.6, Hgb 14 Initial Imaging: RUQ Ultrasound:
non-obstructive thrombus in left portal vein. CT abdomen: splenomegaly (14cm) and non-obstructive thrombus in right
and left portal veins. Splenic laceration with subcapsular and perisplenic hemorrhage. Follow up labs: AST 61, ALT 73,
Total bilirubin 0.5, Direct Bili 0.1, Alk Phos 120 INR 1.2, WBC 18.4, Hep A IgM reactive, Hep B Surface Ab reactive, Hep B
Core Ab non-reactive, Hep C Ab non-reactive, HIV Nonreactive, EBV IgM negative, IgG positive, CMV IgM and IgG positive,
Cryptococcus stool antigen, giardia stool antigen, C. difficile stool toxin all negative, Ceruloplasmin 48 (18-36), Ferritin 510
(20-341), ESR 42 (0-22), Colonoscopy: Entire colonic and rectal mucosa inflamed with erythema, granularity and exudate.
Final Diagnosis:
1.
Acute Hepatitis A despite history of appropriate immunization (occurring in less than 5% of immunized
individuals).
2. Hepatitis A-associated initial presentation of ulcerative colitis.
Treatment: Mesalamine and prednisone were prescribed and the athlete was held out of contact sports.
Outcome: The player had resolution of his bloody diarrhea and regained the 20 pounds he had lost over the past six
weeks. A repeat abdominal CT scan six weeks after diagnosis showed normal splenic size with resolution of both the
laceration and the portal venous thrombosis. He was cleared for full participation and started working on returning to
playing condition.
Follow-Up: The athlete has returned to full game play after a period of acclimatizing to contact. He continues on daily
mesalamine, is finishing a six-week prednisone taper and remains free of abdominal pain and diarrhea.
A New Rib Technique? Watch Out Lineman!
Authors: Courtney Kandler, BS; Gale Prentiss, MD; Justin Young, MD;
Affiliation: University of Hawaii-John A Burns School of Medicine, Honolulu, HI
History: A 19-year-old male college football defensive lineman complains of shoulder pain after working out. He was
preparing for spring camp by doing strenuous weight training. Pain is maximally experienced with overhead lifting. He
took naproxen and this did not help his pain. Ice and electrical stimulation also provided little relief. It has worsened over
the past two weeks. He localizes the pain over the left posterior shoulder. No direct trauma to the region, nor any
weakness, numbness, or tingling reported. Patient reports history of left sided stingers years ago, but states this feels
completely different.
Physical Exam: Shoulder: Full range of motion in flexion, extension, abduction, adduction, but with full forward flexion
and abduction there is pain over the posterior shoulder. There is no tenderness to palpation over the clavicle,
acromioclavicular joint, glenohumeral joint, or sternoclavicular joint. There is tenderness to palpation over the posterior
shoulder specifically the superior-medial border of the scapula and the distal trapezius. Strength is 5/5 in abduction,
external rotation, and internal rotation. Hawkin's and Neer's tests are negative. Speed's test is negative. Roos test is
negative. 2+ radial pulses bilaterally. Neck: Full range of motion in flexion, extension and side bending. Spurling's test is
negative. Deep tendinous reflex of triceps, biceps, and brachioradialis are 2+ and symmetric. Sensation to light touch is
intact. Palpation over thoracic and cervical spine elicits no pain. Back: Smooth scapulothoracic motion
Differential Diagnosis:








Rhomboid/peri-scapular muscle strain
subscapular bursitis
scapular dyskinesis
scapular fracture
clavicle fracture
C5-C6 disk herniation
myofascial pain syndrome
first rib fracture
Test Results: Radiographs of the left shoulder showed a normal glenohumeral joint. There is widening of the AC joint.
There is a non-displaced fracture of the first rib. MRI of the chest subsequently obtained showed edema in the left first rib
surrounding a non-displaced fracture. Edema was also noted in intercostal muscles between left first and second ribs, and
in the left anterior, middle, and posterior scalene muscles. No obvious asymmetry of the brachial plexus was noted.
Final Diagnosis: Non-displaced fracture of the left first rib.
Treatment: Patient was managed conservatively with acetaminophen and tramadol. He was held out of weight lifting
and football for an additional four weeks until pain improved.
Outcome: Patient was able to return to football pain free in four months.
Follow-Up: Patient was able to return to light activity in six weeks, and weightlifting at eight weeks. However, he still
continued to have shoulder pain. He had an orthopedic consultation and was told to continue non-operative
management. Patient was able to return to football within four months of initial presentation. He currently has no pain.
Ataxia and Sensory Changes in a Football Player after an Episode of Acute Head and Neck Pain
Authors: James C. Presley, MD; Stephen J. Wisniewski, MD; Edward R. Laskowski, MD
Affiliation: Mayo Clinic Department of Sports Medicine, Rochester, MN
History: An 18-year-old college football quarterback with a past history of migraine headaches was weight lifting and
performing a shoulder press lift as part of his strength-training program. After the last repetition, he moved to set the bar
on the rack behind his head while laterally rotating and extending his neck to the right. He felt the sudden onset of sharp,
right-sided neck pain with radiation to the shoulder. Within minutes, he developed vision changes including "wavy" vision
in the bilateral visual fields and blurred peripheral vision consistent with his previous migraine auras. He went on to
develop a severe bifrontal migraine headache. He retired for the night and on awaking the next morning noted significant
gait instability with falling to the right and sensory changes on the left side of his body involving the left leg, trunk and
arm. He then presented to the local emergency department.
Physical Exam: Neurological exam in the ER was significant for decreased pinprick sensation in the left lower extremity
and trunk up to the axilla. Notably, his balance deficit had resolved by the time of ER presentation and there was no facial
weakness, dysarthria, or dysphagia. General medical examination was unremarkable.
Differential Diagnosis:
1) Atypical Migraine headache
2) Ischemic Lateral medullary stroke secondary to vertebral artery dissection, coagulopathy, or other embolic
etiology
3) Cervical spinal cord injury
Test Results: Brain MRI scan showed several punctate areas on diffusion weighted imaging in the bilateral occipital
lobes concerning for ischemic stroke. Other pertinent findings included an incidental Chiari malformation and no evidence
of stroke in the brainstem, specifically the lateral medullary area. MRA of the head and neck was negative for cervical
artery dissection. Further testing including ECG and transthoracic and transesophageal echocardiograms showed no
evidence of arrhythmia or anatomic abnormality, including no patent foramen ovale. MRI of the cervical and thoracic
spine were performed and showed no evidence of spinal cord abnormality. Repeat brain MRI obtained 17 days after the
initial event showed resolution of previous occipital infarcts.
Final Diagnosis: Probable lateral medullary ischemic stroke secondary to vertebral artery dissection
Treatment: Aspirin was recommended for anticoagulation for 3 months. Patient was advised to refrain from contact
sports and head or neck manipulation for at least 3 months. Consider refraining from football indefinitely as this activity
would likely increase the risk of recurrence.
Outcome: Sensory symptoms persisted for 4 weeks and gradually resolved There have been no recurrent symptoms
and no further migraine headaches. Repeat MRA was performed 9 months after initial incident with no evidence of
vascular dissection.
Follow-Up: Risk of recurrence was discussed with the patient, family, and athletic advisors. Concern that return to sport
may increase risk of recurrence resulted in university officials not clearing the patient for return to sport. A second opinion
was sought from another neurologist but the patient has not been cleared for return to play by university officials.
Just an Average Ankle Sprain?
Authors: Benjamin Burkam, MD; D. Buck, MD; T. Quinn, MD
Affiliation: Kettering Sports Medicine Fellowship, Wright State Department of Emergency Medicine, Kettering,
OH
History: S.R. is a 16-year-old male high school football player with history of recurrent right inversion ankle sprains, who
presents to discuss re-injury of right ankle. He reports a recent inversion ankle sprain during football practice, followed a
week later by an instability episode causing a fall at school. Patient states he has sprained his right ankle every football
season for the past 4-5 years. He has attempted rehabilitation with his school athletic trainer and tried using a lace up
ankle brace unsuccessfully. He denies previous history of fracture, other notable joint injuries, or pertinent past medical
history.
Physical Exam: Inspection of the right ankle reveals moderate swelling on the lateral ankle. He is tender to palpation
over calcaneofibular (CF) and anterior talofibular (ATF) ligaments. Range of Motion is normal for dorsiflexion, plantar
flexion, inversion, and eversion. Muscle strength testing is 5/5 with dorsiflexion, plantar flexion, anterior tibial, posterior
tibial, gastrosoleus, and peroneal muscle. Anterior drawer test is positive. Talar tilt test is positive. There is no erythema,
laceration, or ecchymosis notable. He is neurovascularly intact and able to bear weight, but antalgic gait is notable.
Differential Diagnosis:






ATFL
CFL or anterior inferior talofibular ligament (AITFL) sprain/rupture
syndesmotic injury
distal fibula fracture
osteochondral defect of talus
osteosarcoma
Test Results: X-ray Ankle: Findings - No acute fracture, bony abnormality, or malalignment noted. Normal talar dome
alignment and spacing. MRI Ankle: Findings - High-grade sprain anterior talofibular ligament with a full-thickness tear.
Intermediate to high-grade sprain calcaneofibular ligament. Peroneal tenosynovitis without tear. Bone marrow edema
seen within the cuboid, consistent with bone contusion.
Final Diagnosis: Chronic ankle instability with ATFL rupture, CF sprain, cuboid contusion.
Treatment: Non-operative treatment options were discussed with patient and father, but due to chronicity of injury and
MRI pathology, surgical repair was chosen. The open Brostrom Procedure for ATFL reconstruction with Arthrex Internal
Brace Augmentation was completed.
Outcome: Successful ATFL reconstruction
Follow-Up: Patient currently completing rehabilitation. He reports significant improvement in stability and strength of
ankle.
Calf Pain in a Cheerleader
Authors: Kelly Chain, MD; Rachel Apple, MD
Affiliation: Vanderbilt University, Nashville, TN
History: A 16 year old cheerleader with no significant past medical history presents to sports medicine clinic due to two
years of right calf pain. She reports that approximately 2 years prior she had a fall while cheerleading and felt a "pull" in
her right calf at that time but was able to continue cheering. A few days later while tumbling, her calf pain acutely
increased, prompting her to present to her local emergency department where she was diagnosed with a calf strain. She
initially treated the injury with ice, heat, NSAIDs, a sleeve, and physical therapy, but she continued to have pain and
progressive loss of motion in her right calf such that she was unable to continue to participate in cheerleading. At time of
presentation, she complained of pain with walking and running as well as tightness in her calf in the morning.
Physical Exam: Examination of the right calf revealed no deformity, swelling, or bruising but did demonstrate decreased
bulk and tone as well as a palpable mass in the soleus muscle, which was firm and tender to palpation. She had full
strength but decreased dorsiflexion at the right ankle. Gait and neurovascular examinations were normal.
Differential Diagnosis:







Gastrocnemius muscle strain/tear
Deep Vein Thrombosis
Myositis Ossificans
Soft Tissue Mass
Chronic Exertional Compartment Syndrome
Popliteal Artery Entrapment Syndrome
Peroneal Nerve Injury
Test Results: CT Scan of the Right Lower Extremity without contrast showed a calcification within the soleus measuring
approximately 4.5 x 1.2 x 7.1 cm (transverse, AP, and CC). The calcification is more dense peripherally and lucent centrally
likely representing a myositis ossificans. A Nuclear Medicine Bone Scan showed increased activity within the medial aspect
of the right calf, consistent with the diagnosis of myositis ossificans.
Final Diagnosis: Right Calf Myositis Ossificans
Treatment: After her presentation to clinic and initial CT scan, a nuclear medicine bone scan was performed to
determine if the heterotopic ossification in her calf continued to be metabolically active or was a mature bony lesion. The
bone scan demonstrated that her ossification continued to be metabolically active two years after the initial injury. Since
her injury, she has been inconsistent with her treatment regimen and has not been compliant with the prescribed physical
therapy. Given the metabolic activity of the ossification and because of the high risk of reoccurrence after surgery, it is not
recommend at this point in her recovery. Her return to play has been limited by her pain symptoms, range of motion, and
strength. She has been unable to increase activity even gradually.
Outcome: At three months from her initial visit she continues to have pain after prolonged walking and jogging on the
treadmill. She is able to tolerate the stationary bike. She continues to be non-compliant with physical therapy and NSAIDs.
Follow-Up: During her treatment with conservative measures she will be followed at regular intervals to see if her pain,
motion, or strength improves. If she does not improve, a repeat bone scan can be considered in 6 to 12 months to
reexamine the metabolic activity of the myositis ossificans. She will also be referred to an orthopedic oncologic surgeon to
determine her eligibility as a surgical resection candidate. Other treatment possibilities that are available if she does not
improve and is ineligible for surgery include bisphosphonate therapy or extracorporeal shock wave therapy.
Leg Pain That Won't Quit
Authors: Broderick C. Howard, DO; Stephen M. Simons, MD
Affiliation: Saint Joseph Regional Medical Center 5215 Holy Cross Parkway Mishawaka, IN, USA
History: A fifteen-year-old female high school soccer player presented with a one-year history of intermittent bilateral
calf pain. The pain routinely occurred about five minutes into a run. The pain was occasionally accompanied by foot
paresthesias. The discomfort was most severe along the lateral calves. Symptoms generally resolved 20-30 minutes after
running cessation. The patient denied increasing her training intensity or starting a new training program. She denied a
specific injury or inciting event. X-rays were unremarkable. An initial MRI demonstrated bilateral tibial stress fractures. She
completed an appropriate rest period. Continuing to play soccer, she re-presented two years later, complaining that her
symptoms never resolved.
Physical Exam: Vitals: Temperature 98°, BP 108/60, Pulse 80, Respiratory rate 18 Constitutional: No acute distress
Lower Legs: Left: Inspection: No swelling Palpation – Tenderness along posteriomedial cortex of the tibia and
tenderness to the anterior cortex. Range of motion – Normal Strength: 5/5 dorsi and plantar flexion Toe raise exercise –
Exacerbated pain after a few minutes Right: Inspection – No swelling Palpation – Tenderness along posteriomedial
cortex of the tibia and tenderness to the anterior cortex Range of motion – Normal Strength – 5/5 dorsi and plantar
flexion Toe raise exercise – Exacerbated pain after a few minutes
Differential Diagnosis:
1.
2.
3.
4.
5.
Compartment syndrome
Popliteal artery entrapment syndrome
Tibial stress fracture
Shin splints
Muscle tear
6. Complex regional pain syndrome
Test Results:
1.
2.
Lower leg x-ray: Negative
MRI upper tibia/fibula/calves without contrast: Proximal tibia stress reaction bilaterally
3. Intra-compartment limb measurement: Elevated anterior, lateral and deep posterior compartment pressures
bilaterally
Final Diagnosis: Bilateral lower extremity compartment syndrome
Treatment: 1. Bilateral lower extremity 4 compartment fasciotomy 2. Physical therapy
3. Rest
Outcome: Five months after the surgery, she denied any improvement other than minimal relief from paresthesias. A
repeat MRI only revealed mild periostitis. Popliteal Artery Entrapment was considered. Bilateral arteriography with and
without plantarflexion stress revealed truncation of the popliteal arteries with plantar flexion stress. Vascular Surgery
requested dynamic MR arteriography to further delineate the associated anatomy. This test FAILED to demonstrate
popliteal artery entrapment. Surgical consideration was cancelled.
Follow-Up: Patient has since stopped playing soccer and continues to have pain despite proper rest and physical
therapy. Patient's clinical symptoms are consistent with vascular compromise, however, conflicting vascular studies render
surgical intervention risky. The technical challenge is to determine which test is errant; a false positive stress arteriography
or a false negative MR arteriogram. This is, as yet, unresolved.
Persistent Hip Pain Not Always From The Hip
Authors: Karen Frye, DO; Michael Reeder, DO
Affiliation: St. Mary's Sports Medicine Fellowship, Grand Junction, CO/Rocky Mountain Orthopaedics
Associates, Grand Junction, CO
History: 65yo active male presented to our clinic with worsening R-low back and buttocks pain, R-anterior medial hip pain and pain
radiating down his anterior thigh to knee. Patient is very active, works out regularly, participates in recreational ice climbing and is
involved w/search and rescue. has history of DJD of lumbar spine and hip for which he has been seen for the past 5-7 years. Denied any
recent trauma, falls or injury. Pain worsened over past 2 months. Pain worse w/hip flexion and external rotation, worse after sitting for
prolonged time and w/rising from sitting to standing. Patient able to do elliptical without pain, but hurt afterwards. Denied any
numbness or tingling in his lower extremities, no bowel or bladder dysfunction, no saddle anesthesia, no fever or chills, and no weight
loss.
Physical Exam: Gen: Pleasant 65yo male, alert and oriented, no acute distress. Back: Forward flexes to 3 inches from the floor.
Extension and side bending are easy without pain. Some TTP over right SI joint and right gluteus maximus. Negative straight leg raise.
Hip: No tenderness over greater trochanter. Tenderness in right medial groin with flexion, external & internal rotation of the hip. + Pain
with FABER test. Limited ROM with figure of four bilaterally, R&gt;L. Some discomfort with Thomas test in extension on right. Negative
log roll. Normal pulses, no masses. Knee: No effusion, normal ROM, negative Lachman's and posterior drawer. No pain or laxity with
varus or valgus strain
NV: Walks with non-antalgic gait, able to heel/toe walk without difficulty. Strength 5/5 bilaterally. DTR's 2/4
without clonus. Sensation intact and equal bilaterally.
Differential Diagnosis:
1.
2.
3.
4.
5.
6.
7.
8.
9.
10.
11.
Osteoarthritis of femoroacetabular joint
Hip labral pathology
Hip flexor tendinopathy
Chronic gluteal tendinopathy
Lumbar spine radicular symptoms/Nerve root irritation
SI dysfunction/Sacroiliitis
Stress fracture (femur, pubic rami, acetabulum)
Avascular necrosis of femoral head
Athletic pubalgia
Malignancy
Infection
Test Results:

AP Pelvis standing Xray:
No obvious fracture or dislocation. Mild joint space narrowing. No bony lesions.

L spine AP & lateral Xray:
No obvious fractures or dislocations. Severe disk space narrowing at L5-S1 and less so at L4-L5. No spondylolysis or
spondylolisthesis

2nd visit: Intra-articular hip injection under fluoroscopic guidance:
No improvement in pain

3rd visit: RI L spine:
Central disk bulge L4-5 & possible annular tear. L5-S1 central disk bulge
Lumbar facet joint arthritis most significant at L4-5, L5-S1

4th visit Transforaminal epidural injection of Right L4-5, L5-S1 under fluoroscopic guidance.
Had 50-70% relief of pain for almost 2 months

2 months later patient had return of right low back and right anterior groin pain. ROS newly positive for hematuria. No fever or
chills, no testicular pain, no GI/GU symptoms, no weight loss, no decreased appetite.

MRI pelvis:
Aggressive tumor epicentered in the right acetabulum and right superior ramus with pelvic adenopathy NM whole body
bone scan:
No additional areas of tracer uptake.
Focal abnormal radiotracer uptake at right acetabulum and superior pubic ramus

PSA: 78 (done after MRI results). Previously 3.8 in 2012
Final Diagnosis: Adenocarcinoma of the Prostate with metastasis to right hip and pelvis (T3b N1M1; stage IV)
biopsy showed pathologic fracture through the anterior column of acetabulum
Scan from CT guided
Treatment: Prostate biopsy consistent with adenocarcinoma of the prostate. Hip biopsy: anterior column and inferior ramus PSA
positive Radiation/Oncology recommends palliative radiation to the hip and prostate.
Outcome: Currently getting treated with Depo-Lupron and following with Oncology and Urology
Follow-Up: Will be determined after treatment
A 40-Year-Old Male with Persistent Left Elbow Pain
Authors: Samuel F. Thompson, BS; Aaron D. Gray, MD
Affiliation: University of Missouri-Columbia, Department of Family and Community Medicine, Columbia, MO
History: A 40-year-old male presented with an eight-year history of left elbow pain. Three years prior to presentation
the patient underwent a radiograph at an outside hospital that revealed a left elbow lesion, but at that time the patient did
not have insurance and could not afford a follow-up visit. At presentation, the patient's elbow pain was described as
persistent, sharp and 8/10 on a pain scale. The pain was exacerbated with pronation of the wrist and raising the left arm
above the head. The pain radiated down the left forearm. The pain was partially relieved by wrist extension stretches. The
patient also reported frequent left hand spasms with numbness and paresthesias in the third through fifth digits.
Occasionally the spasms also occurred in his first and second fingers. There was no history of trauma or injury. No fevers,
chills, weight loss, or night sweats. ROS: 10-point review of systems was negative except for mild fatigue and mild chest
pain. Soc. Hx: Recreational athlete. Currently employed as a truck driver. Former excavator for a construction company.
Married with three children. No tobacco or alcohol use. Fam. Hx: Father and grandparents with heart disease and prostate
cancer. No family history of rheumatoid arthritis or systemic lupus erythematous. Surg. Hx: Bilateral carpal tunnel repair
(2008). Meds: Ibuprofen
Physical Exam: Right elbow: Nontender, full range of motion, full strength, normal stability, normal sensation in right
hand. Left elbow: No erythema, swelling or increased warmth. No visible mass. Severe tenderness to palpation of proximal
olecranon, no tenderness over the medial and lateral epicondyles. Full pronation and supination without pain. Full strength
(5/5) with resistance to pronation and supination of the hand, as well as with flexion and extension of the wrist. Normal
pulses and no lymphedema. Moderate atrophy of the left forearm muscles in comparison to the right. Normal sensation to
light touch in left hand. Reflexes normal. Shooting sensation radiating into the fourth and fifth fingers with Cubital tunnel
and Tinel's sign. Shoulders: No abnormalities on bilateral shoulder exam. Neck: Full range of motion and negative
Spurling's.
Differential Diagnosis:






Ulnar nerve entrapment
Triceps tendinosis
Olecranon impingement
Olecranon stress fracture
Chondromyxoid fibroma
Noncalcified enchondroma
Test Results: Radiograph of left elbow: A 3.5 x 2.1 x 1.6 cm lesion was present in the proximal ulnar metadiaphysis. No
acute fracture or dislocation. MRI of the left elbow: Lesion displayed homogenous enhancement with contrast. Cortical
bone thinning and scalloping was present. Mild thickening and hyperintensity of the ulnar nerve just proximal to the
cubital tunnel. CT-guided biopsy: Pathology consistent with chondromyxoid fibroma.
Final Diagnosis: Chondromyxoid fibroma
Treatment: Curetting and allografting of the lesion.
Outcome: Successful surgical removal of the lesion.
Follow-Up: The patient was placed in a long-arm splint for two weeks following surgery. He then transitioned to the
use of a left arm shoulder sling with instructions to avoid any weight bearing activity. He also began a four-week course of
physical therapy. At a six-week follow-up appointment, the patient was found to be non-compliant with the use of the
sling and he reported continued use of his left arm to complete job requirements (operating heavy machinery, installing
septic tanks, etc). At latest follow-up (five-month follow-up appointment), range of motion at the elbow was
approximately 0-130° without significant discomfort. Radiograph of the elbow revealed slight resorption of bone graft
material but no acute abnormalities. Against recommendations, he reported lifting 40-50 pounds without significant pain.
Sensation was intact to light touch, with only mild tenderness to palpation at the corticotomy site. Recommendations were
made to discontinue all heavy lifting.
Left Lower Extremity Injury
Authors: Gracia Pierre-Pierre, GPP, MD
Affiliation: University of Texas Health Science Center/Trinity Mother Frances, Tyler, TX
History: Sixteen-year old male presented to clinic reporting pain in left lower leg with difficulty walking and decreased
sensation in the lateral left lower leg from proximal fibula down to his foot. Patients reports that 2 days prior to visit, he
sustained an injury when running the ball in football practice. He twisted his ankle while cutting. He experienced
immediate severe pain and fell to the ground. He was not able to bear weight and hopped off the field. Four hours later
he went to the ER for evaluation; X-rays were obtained and were negative for fractures. Patient was provided crutches and
prescribed naproxen. Patient reports constant numbness and weakness of the left lower leg as well as intermittent,
intense, non-radiating sharp/burning pain in the proximal 1/3 lateral aspect of the leg, which is increased by moving the
leg or trying to walk. Rest, as well as cryotherapy and naproxen provided some relief. He denies any previous issue with
the leg. ROS: Otherwise negative
Physical Exam: Gen: Unable to bear weight L knee: Decreased flexion due to pain at proximal 1/3 of lateral lower leg.
Otherwise unremarkable. Inspection: Localized mild swelling of the proximal 1/3 of lateral aspect of the lower leg.
Palpation: Pain on palpation of lateral proximal 1/3 of the lower leg. Lateral aspect of ankle non tender. ROM: Active
ankle DF and reat toe extension to neutral. Inversion 5-10° with pain. Pain free full PF. Eversion to 5° with pain. Full passive
ROM except for limited passive inversion due to pain in fibular head area Strength: Ankle DF, great toe extension and
eversion 2/5. Inversion and PF 5/5 Special test: Negative anterior drawer and talar tilt. Unable to assess fibular head
stability due to pain Neurovascular: Numbness of anterolateral aspect of lower leg, dorsal aspect of the foot, 1st web
space. Normal patellar and Achilles tendons reflex. Intact vascular status. Positive Tinel's test at the fibular head.
Differential Diagnosis:





Traumatic common peroneal nerve lesion
Entrapment neuropathy due to soft tissue swelling
Proximal fibular fracture
Fibular head instability
Acute compartment syndrome
Test Results: Previous X-ray reviewed and unremarkable.
Final Diagnosis: Common peroneal nerve injury
Treatment: Initially hold out of all activities and NWB on crutches and boot. Regular use of ice and NSAIDs prn pain.
Gentle rehab guided by degree of nerve function. Reevaluation every 2-3 weeks initially. Will consider EMG/NCS if
symptoms persist more than 6 weeks without much improvement and would consider MRI if results are concerning for
entrapment
Outcome: At 7 weeks, his limp significantly improved. There was persistent tenderness of palpation around the fibular
head. There was full active DF and eversion of the ankle without pain with 4/5 resisted strength as well as full great toe
extension with 3/5 resisted strength. There was some residual paresthesia on both dorsum of the foot and 1st web space.
At this point we recommend continuing to progress rehab based on degree of nerve function. Since improving, will
monitor his response over the next 3-4 weeks.
Follow-Up: At 11 weeks, there is full strength and full pain free active ROM with slight deficits with
proprioception/balance and small area of dyesthesia on dorsum of great toe only. Will recommend rehab emphasizing
strength, proprioception and will begin a gradual activity progression program that must be completed successfully
without issues before he can be cleared.
Knees, Nerves and Stones After a Hike
Authors: Abimbolu Makinde, MD; Adam Pourcho, DO
Affiliation: Swedish Spine, Sports and Musculoskeletal Medicine, Seattle, WA
History: A 45 year old female presented to the sports medicine clinic in September 2014 complaining of dull and achy
left lateral knee pain for 3 weeks which worsened after a day of hiking and subsequent foam rolling one week prior. After
attempting to use a foam roller over her lateral knee her pain worsened and she developed mild dorsiflexion weakness.
On initial evaluation her pain was 4/10 at rest and 10/10 with movement. She had pain with weight bearing and was
ambulating with crutches.
Physical Exam: On physical examination there were no gross deformities of her knee. She had pain at end range of
terminal knee extension and full flexion at the lateral knee proximal to the fibular head. She had exquisite tenderness to
light and deep palpation proximal to the fibular head and posterior to the biceps femoris tendon insertion. She had a
positive Tinel's over the common peroneal nerve. Lachman's, valgus stress, varus stress, McMurray's and dial tests were
all negative. She had mild decreased sensation over the dorsum of the foot and first web space and 4+/5 strength with
dorsiflexion. The remainder of her knee and ankle examination was without abnormality.
Differential Diagnosis:








Common peroneal nerve palsy secondary to compression from foam rolling
Popliteus, peroneal or biceps femoris tendinopathy
Proximal tibio-fibular joint dysfunction
Lateral tibial plateau or proximal fibular stress fracture/stress reaction
Distal Iliotibial band friction syndrome
Lateral meniscal tear
Maisonneuve fracture
Posterior-lateral corner injury
Test Results: X-ray of the knee revealed calcific density proximal to the fibular head without fracture. Complete lateral
knee musculoskeletal ultrasound revealed a calcific mass proximal to the fibular head and posterior to the biceps femoris
insertion, measuring 1.15cm in the coronal plane and 0.44 cm in the axial plane. There was extensive neovascularization
and hyperemia surrounding the calcium deposit. The common peroneal nerve passed just 0.2cm posterior to the calcific
mass. The patient reported increased pain and numbness over the dorsum of the foot with light and deep transducer
pressure over the nerve at this point. The remainder of the lateral knee examination was without abnormality. MRI of the
knee demonstrated the same calcific density with reactive bone marrow edema of the proximal fibula.
Final Diagnosis: Atypical hydroxyapatite deposition disease (HADD) with superimposed peroneal neuropraxia
Treatment: A sonographically guided calcific lavage of the calcium deposit was performed. A moderate amount of
calcium hydroxyapatite was aspirated and sent to the lab for analysis. Following lavage the area was fenestrated to break
up remaining calcium and injected with 1 cc of 6mg/ml Betamethasone.
Outcome: Our patient experienced a transient block of the common peroneal nerve following procedure and was
without persistent symptoms for four hours. Her pain was reduced by 60% in the immediate post-procedure period.
Follow-Up: At two week follow-up our patient demonstrated 100% resolution of her pain and peroneal nerve symptoms
with full return to activity and work. Hydroxyapatite deposition disease (HADD) involves the deposition of calcium within
tendons or peri-articular structures. The most common areas involved are the supraspinatus and infraspintatus.
Percutaneous lavage (Barbotage procedure) involves sonographically guided needle irrigation and aspiration of calcific
hydroxyapatite deposits, and is well described in the treatment of calcific tendonopathy of the rotator cuff. This case
represents an atypical calcific deposition within the periarticular tissues of the proximal tibio-fibular joint, with
superimposed peroneal nerve neuropraxia successfully treated with sonographically guided calcific lavage. Calcific lavage
has been shown to be safe and effective in patients with calcium hydroxyapatite deposits and can be considered in
treatment of patients with calcium deposits in atypical locations.
Left Forearm Pain and Paresthesias in a High School Fast-Pitch Softball Player
Authors: Augustus P. Evangelista, MD, MBA; Ryan T. Hubbard, MD; Cara C. Prideaux, MD
Affiliation: Department of Physical Medicine and Rehabilitation, Mayo Clinic, Rochester, MN
History: A 16-year-old left-hand dominant female fast-pitch softball player with history of a R-medial epicondyle fracture
8yrs ago presented with one month history of gradual onset L-medial forearm pain. Complained of pain localized to
medial forearm just distal to medial epicondyle w/radiation to the wrist and occasional paresthesias in L-third, fourth and
fifth digits. During pitching, she throws the ball using underhand motion with a windmill-type movement and slaps her
ulnar arm against her hip at release point. She said her pain was worse at the release point of her pitch with the greatest
pain occurring when she attempted to throw curve balls or with any rotation of wrist. She denied any trauma or inciting
event associated with onset of pain, night pain, pain at rest or neck pain. Prior treatment included icing her left forearm
following every game. She had not stopped pitching. She pitches almost year-round, including a travel team in the
summer, and is committed to a collegiate scholarship for pitching.
Physical Exam: GENERAL: Well developed, well nourished. LEFT ELBOW: Normal appearance and ROM. Non-tender
over olecranon, medial/lateral epicondyles, and distal biceps tendon. Tenderness over medial forearm 3 inches distal to
medial epicondyle, extending distally but not including wrist. SPECIAL TESTS OF LEFT ELBOW: Negative varus and valgus
stress. No evidence of subluxing ulnar nerve. Negative Tinel's over cubital tunnel. Positive Tinel's over medial forearm with
tingling produced in ulnar digits. Prolonged elbow flexion reproduced paresthesias in ulnar digits. LEFT UPPER
EXTREMITY STRENGTH: Normal strength. Mild pain with resisted pronation and supination. LEFT UPPER EXTREMITY
SENSATION: Normal sensation.
Differential Diagnosis:







Flexor carpi ulnaris strain/tear
Ulnar neuropathy
Medial collateral ligament injury
Medial epicondylosis
Stress fracture
Cervical (C8) radiculopathy
Bone tumor
Test Results: Left elbow radiographs: No visible joint effusion, fracture or osseous abnormality. Left upper extremity
MRI: Longitudinal, undisplaced stress fractures of the proximal and distal ulna.
Final Diagnosis: Longitudinal, undisplaced stress fractures of the proximal and distal left ulna, likely related to extensive
pitching and overuse as well as her specific pitching technique.
Treatment: Relative rest, avoiding most activities with L-arm for 6 weeks and then a physical therapy program. Rule
out an underlying disorder that could increase her risk for fracture, such as Female Athlete Triad. Discuss concept of
overuse and the importance of proper pitching mechanics.
Outcome: She did not have any risk factors for Female Athlete Triad. Normal menstrual cycles and caloric intake. She
avoided all activities with her L-arm for 6 weeks and then followed up in Sports Medicine Clinic. She noted complete
resolution of all symptoms at 5-6 weeks. She then initiated physical therapy focusing on upper body flexibility and
strength, as well as gluteal and core strength. Pitching mechanics were addressed to avoid significant contact between the
L-ulnar forearm and ipsilateral hip at release point.
Follow-Up: After 6 weeks of therapy, she was able to demonstrate appropriate pitching mechanics. Advised she could
progress to pitching w/maximal speed and intensity, and to gradually introduce curveball pitches over a 4 to 6 week
period as tolerated. We advised her to take appropriate breaks from pitching (e.g. not pitching more than 2 or 3 days in a
row, watching pitch counts, avoiding playing for multiple teams with overlapping seasons, and not playing year-round).
She was told to remove herself from a game at the onset of arm fatigue as that is when her pitching mechanics are most
likely to suffer, thus putting her at increased risk for injury. We suggested rotating to other positions besides pitcher if
necessary.
Splitting At the Seam
Authors: Jonathan P. Bonnet, MD; Jeff Bytomski, DO
Affiliation: Duke University, Durham, NC
History: 48 year old male pathologist with 10 days of left foot pain and swelling. Located proximal dorsal mid foot
anteriorly. No known mechanism of injury or inciting event. No changes to training program, or shoes; trail running 50
miles per week. No prior history of injury. Right foot without pain. Medical history non-contributory, no prior surgeries.
Physical Exam: Gen: NAD with normal respiration. Right Foot: No swelling, deformities, or tenderness to palpation. Full
range of motion at the ankle. 5/5 strength testing without pain. Sensation intact. 2+ dorsalis pedis pulse. Left Foot: trace
amount of swelling on the dorsal mid foot. Tenderness to palpation over the navicular. Full range of motion ankle
eversion, inversion dorsiflexion and plantar flexion. 5/5 strength testing. Mild pain with resisted dorsiflexion. Sensation
intact. 2+ dorsalis pedis pulses.
Differential Diagnosis:
Contusion
Ligament Sprain
Navicular Fracture
Stress Fracture
Tendinitis
Tendon Tear
Test Results: X-ray: No acute fracture-dislocation. Joint spaces well preserved.
MRI: Intermediate to high signal on
the anterior tibialis tendon as it passes over the midfoot compatible with split tear. Associated dorsal foot superficial soft
tissue edema.
Final Diagnosis: Anterior Tibialis Tendon Split Tear
Treatment: Rest, ice, compression, elevation. Put in post-op boot for 8 weeks. NSAIDS every 8 hours for 14 days. Physical
therapy for 3 weeks to work on range of motion, dynamic and eccentric stretching and pool running.
Outcome: Repeat MRI six months post injury showed resolution of linear high signal within anterior tibialis tendon. Mild
tendinosis present at distal anterior tibialis tendon.
Follow-Up: NSAIDs provided minimal relief. Pain decreased with rest over 8 week period in the boot. Patient gradually
returned to running utilizing the Galloway run-walk method. Mileage was increased by 5% per week with progression up
to 13 miles with only an occasional dull ache. Significance of Case: Anterior tibilias tendon ruptures are commonly
reported in the literature, however, tendinosis and split tears of the anterior tibialis tendon in isolation are rare. This case
seeks to explore the existing literature that currently exists and offers insight into the recovery process for such an injury
Silent Shortness of Breath Following Shoulder Replacement
Authors: Tanjuihsien Marx, MD; Jeffrey B. Roberts, MD
Affiliation: Bon Secours St. Francis Primary Care Sports Medicine Fellowship
History: 64 y/o male consulted by Orthopedic Surgery for management of asymptomatic hypertension 4 hours s/p Right
Total Shoulder Athroplasty, Biceps Tenodesis and Axillary nerve release. Additionally complains of mild, progressively
worsening painless shortness of breath. Denies fever, chills, cough, chest pain, tightness, palpitations or pleurisy. Denies
abdominal pain, nausea or vomiting. No change with position or pleuritic pain. No history of other cardiac or lung disease.
No history of smoking.
Physical Exam: VS: BP 141/81, HR 88, T- 97.6, RR 18. SpO2 94% on RA Gen: A&O x 3, no respiratory distress. CV: RRR,
nml S1 S2, no murmurs, bruits, rubs or gallops. Distal pulses 2+/=. Chest: CTAB, no wheezing on left. Absent sounds on
anterior/posterior apex and bases on right. Increased dullness to percussion on right posterior base and middle lung
fields. Abdomen: Soft, non-tender, benign. Extremities: Warm, non-tender. No swelling of shoulders or calfs. Surgical
site with bandages in place. Right UE sling with subclavian nerve block via pain pump in place. Neuro: non-focal on nonsurgical extremities. Right arm with decreased sensatoin to light touch and decreased grip strength.
Differential Diagnosis:






Atelectasis
tension pneumothorax
diaphragm paralysis
PE
MI
pneumonia
Test Results: Portable Shoulder radiograph immediately post-op: S/P RTSA, 6 ribs seen. CXR PA& Lateral Inspiratory/expiratory film - New elevation of the right hemidiaphragm without movement on inspiration. Atelectasis at
the right lung base.
Final Diagnosis: Right-sided Diaphragmatic Paralysis due to phrenic nerve involvement s/p Interscalene nerve block and
pain pump.
Treatment: Discontinue Pain pump, continuos pulse oximetry, incentive spirometry and serial examination.
Outcome: Patient with complete recovery and resolution of sx within 4 hours.
Follow-Up: Patient allowed back to activity under regular movement restrictions per Ortho and PT. No medical
restrictions.
How a Seemingly Minimal Physical Exam Finding Unveiled a More Serious Condition
Authors: Joshua Rothenberg, DO; Kimberly Ross, MD, MBA; Alexander Harrington, MBA; Alberto J. Panero, DO
Affiliation: Department of Physical Medicine and Rehabilitation, University of Miami Miller School of Medicine,
Miami, FL
History: Cerebral contusion following a concussion is a severely debilitating and possibly life-threatening condition that
is not readily uncovered in American football players. We describe the case of a 16-year-old high school male, with a sixyear history of playing tackle football and no prior reported concussions, who was involved in a helmet-to-helmet headon collision during a high school football game. He had a few seconds of loss of consciousness after the collision and then
was able to walk off the field. He presented to a university-based concussion clinic five days post helmet-to-helmet
collision with symptoms of nighttime headaches, dizziness, and difficulties with concentration and word finding.
Physical Exam: Field tests had been performed on the sideline and found the patient to be disoriented and confused.
Neurocognitive evaluation demonstrated decreased visual and verbal memory, including mildly impaired short-term
memory and decreased psychomotor speeds of processing, complex attention, and executive functions. Physical
examination revealed unusual neurologic signs that prompted further investigation. Lower extremity deep tendon reflex
testing revealed hyperreflexia with pathologic spread to bilateral knees and Achilles tendons. Provocative testing exposed
vestibular signs, which worsened when he turned his eyes and head to the left.
Differential Diagnosis: When evaluating a patient such as this one, it is important to keep in mind all possible
diagnoses that can have overlapping features or similar presentations. These include concussion, depression, headache
disorder, seizure disorder, cervical spinal cord injury, and more severe traumatic brain injuries such as a subdural
hematoma, epidural hematoma, cerebral contusion, or intracerebral hemorrhage.
Test Results: The patient's focal neurologic signs prompted obtention of brain and cervical spine imaging, which
demonstrated a right inferior temporal lobe contusion around the sylvian fissure as well as multiple cervical disc
herniations.
Final Diagnosis: These imaging findings revealed that a brain contusion complicated the clinical diagnosis of concussion
in our patient.
Treatment: Management included physical and cognitive rest until his acute symptoms resolved. He completed a
stepwise gradual return to play protocol detailed in the latest Consensus Statement on Concussion and Sport. The patient
was counseled not to participate in contact sports for the rest of the season and was referred to neurosurgery for further
evaluation.
Outcome: Patient's symptoms of headache and dizziness persisted, although slightly less severe, at his follow up
appointment four weeks post injury. However, as of his follow up 4 months post injury, patient was asymptomatic.
Follow-Up: Four months after the injury, he had no residual symptoms. He continues to have lower extremity
hyperreflexia and occasional neck spasms since the injury occurred. He has returned to school and is doing well
academically and socially. He currently participates in water polo, running, and lifting weights without any difficulty.
Follow-up cervical MRIs demonstrated some cervical congenital stenosis, but no large disc herniations and no spinal cord
entrapment, with normal CSF signal surrounding cord. Although there is no absolute contraindication to returning to
football, the patient still remains at a slightly higher risk of neurological deficit if he sustains another head injury. The
neurosurgeon encouraged the patient to continue with water polo rather than football. This case emphasizes the
importance of a thorough neurologic examination. Failure to recognize these critical signs could have resulted in
premature or unsafe return to play, ultimately exposing the athlete to the risk of additional trauma. Concussion is primarily
diagnosed clinically and therefore imaging is not routinely obtained. Thus, fastidious upper motor neuron testing should
be a fundamental component of the standard neurologic examination when evaluating players suspected of having a
concussion.
Sudden Headache After Snatch Exercise
Authors: Jacques Courseault, MD; Vincent Shaw, Jr., MD; Brian Harrell, MD
Affiliation: Baton Rouge General Sports Medicine Fellowship Program, Baton Rouge, LA
History: A 22-year old college football player was performing the snatch exercise when he developed an acute onset of
right-sided headache. He described a 10/10 sharp, shooting right-sided head pain that radiated from the right base of the
skull to the right eyebrow. The pain was associated with a pressure sensation and blurry vision in the right eye and
photophobia. ROS: No balance difficulty, weakness or sensory deficits in the extremities. Other systems reviewed were
negative. PMH: Sickle cell trait
Physical Exam: HEENT: No cranial deformity, swelling or bruising. Whisper audible, PERRLA. Neuro: Upon palpation of
the right occiput, there was an immediate reproduction and worsening of reported symptoms with impaired visual acuity
in the right eye. Difficulty with one leg stance and tandem stance. Cranial nerves otherwise intact. Strength intact.
Sensation to light touch, pinprick and proprioception intact. Finger to nose intact. DTR's were normal. Negative Romberg.
Gait was slow, but otherwise normal.
Differential Diagnosis:















TIA
CVA
CNS or peripheral lesion such as occipital neuralgia
intracranial mass
muscle strain
acute glaucoma
tension headache
cluster headache
cervical fracture
cervical disc herniation
cervical facet syndrome
Chiari malformation
atypical migraine
atypical seizure
conversion disorder
Test Results: Normal head and neck x-ray. Normal CT and MRI of head. Slightly elevated CPK and creatinine.
Final Diagnosis: Occipital Neuralgia. Occipital neuralgia is a condition in which the lesser or greater occipital nerves are
injured or inflamed as they course from the spinal cord through the scalp. The nerves run deep to the obliquus capitis
inferior, semispinalis capit
Treatment: Right occipital nerve block
Outcome: The athlete reported immediate improvement in pain, light sensitivity and blurry vision.
Follow-Up: He returned to limited practice for 2 days and full contact thereafter. Symptoms were resolved at one week
follow up.
Grappling With Pain - Lower Leg Pain in a Wrestler
Authors: Ramon Julian Pesigan, MD; Moneef Hauter, MD; Wazim Buksh, MD; Dean Padavan, MD
Affiliation: Atlantic Sport Health, Morristown, NJ
History: A 17 year old high school wrestler presents to the clinic for evaluation of right shin and knee pain that he
experienced 2 days prior at wrestling practice. According to him, he had been placed in a compromising position by a
much heavier opponent and felt a pop coming from the outside portion of his proximal lower leg associated with pain. He
denied any initial swelling or discoloration at the time and was able to put weight on his leg, but was unable to continue
practicing. Over the next few days he iced his leg, took anti-inflammatory medication, and refrained from further practice.
The school's athletic trainer expressed concern that he could not put full weight on his right leg, and placed him on
crutches pending further evaluation. She also instituted additional treatment modalities, which allowed him to return to
exercise on a stationary bike with tolerable discomfort. He denies prior injuries to his lower extremities other than an MCL
sprain the previous year which was treated conservatively. He denies a history of fractures, fever, chills, unintentional
weight loss, night pain or other symptoms on his review of systems. Additionally, his past medical and family history are
non-contributory.
Physical Exam: BP: 100/70; HR- 60bpm General exam revealed a well-appearing, well-nourished male, with pain rated
at 7/10 with weight bearing. There was no noted swelling, bruising or deformity of the affected leg. Active and passive
range of motion were full in the knee and ankle joints. However, slight discomfort was noted with ankle eversion and
circumduction. Mild weakness was also evident on resisted ankle dorsiflexion and eversion. Ligamentous laxity in either
joint was absent, and neurovascular status was intact and symmetric bilaterally. His gait had a flattened heel strike and
decreased toe-off on the affected side, with compensatory decreased stride length on the contralateral side. A radiograph
of the affected leg showed a lucency in the fibular neck that did not appear aggressive. Further imaging with an MRI was
ordered due to persistence of pain and to characterise the cyst further
Differential Diagnosis:
1.
2.
3.
4.
Pathologic Fracture of the Proximal Fibula
LCL Sprain
Bony Stress Reaction Secondary to Trauma
Lateral Meniscus Injury
Test Results: Right Tibia/Fibula XRay: Non aggressive lucent lesion in the fibular neck, differentials include non
ossifying fibroma, bone cyst, fibrous dysplasia. MRI recommended for further evaluation Right Lower Extremity MRI: 1.
Cystic lesion within the proximal fibula, with characteristics suggestive of benign bone cyst. 2. Non specific edematous
change seen around proximal fibula which may be related to contusion from recent injury. No discrete fracture identified
within fibular head or involving cystic lesion. A second read from an MSK radiologist revealed that the lesion had
compromised the cortex of the proximal fibula.
Final Diagnosis: Pathologic fracture of Proximal Fibula secondary to Cystic Lesion
Treatment: Patient underwent surgery for removal of the pathologic lesion along with the proximal fibula, with
subsequent bone grafting.
Outcome: Pathology revealed an aneurysmal bone cyst with fragile bone surrounding it. Patient suffered right peroneal
nerve injury as a result of the surgery, with subsequent EMG findings consistent with peroneal neuropathy. Orthotic use
was discussed to help with great toe and ankle dorsiflexion, but the patient opted out.
Follow-Up: He subsequently returned to full physical activity 10 months post-injury with a 3-6 month follow up
electrodiagnostic study to track peroneal nerve function.
Stepping Foot Injury in a Rugby Player
Authors: Joann Y. Chang, MD; Abigail S. Fletcher, MD
Affiliation: Long Beach Memorial Family Medicine, Long Beach, CA
History: A 36-year-old female club rugby player (5' 3" and 210 lb) presented with left foot pain after stepping on
another player's foot during a game. She reports striking her heel on the ground and her forefoot landing on another
player's foot when she heard and felt a "crunch". This was followed by immediate pain in her foot. She hobbled to the
sideline but stayed out the rest of the game. She did not note any immediate swelling or hematoma, but she iced
immediately and went to urgent care. There, plain films were reportedly negative and she was given crutches and a CAM
boot with Tylenol #3 for pain. She has a previous left-sided foot injury from her foot getting stepped on in a game 7
months prior. No audible pops or cracks were heard and she was able to continue playing, but she complained of a
persistent, dull midfoot ache ever since. She also has a distant history of a grade 3 left ankle sprain.
Physical Exam: There was mild edema of the midfoot and medial longitudinal arch. Hematoma was noted over the 2nd
phalanx. Pes planus was present bilaterally with noticeable arch loss in her affected foot. She was tender along the dorsal
surface of her midfoot. The piano key test was negative. Her toes and ankles had full range of motion and strength,
though she did have pain with toe extension. She was unable to bear weight without pain.
Differential Diagnosis:





Midfoot sprain
lisfranc ligament tear
metatarsal fracture
navicular fracture
medial cuneiform fracture
Test Results: Non-weightbearing plain films from urgent care were negative for fractures and showed no widening of
the Lisfranc joint. After 2 weeks and persistent pain in her midfoot, an MRI was ordered and she was referred to Foot and
Ankle Surgery. The non-weightbearing MRI showed a high-grade partial tear of the Lisfranc ligament without widening of
the Lisfranc joint. The MRI also showed mild peroneal tenosynovial fluid distal to the cuboid groove.
Final Diagnosis: Grade III Lisfranc ligament tear
Treatment: The athlete was offered surgical fixation or conservative treatment with 6-8 weeks of non-weight bearing in a
CAM boot. She opted for conservative treatment.
Outcome: At the time of this writing, the athlete has completed 3 weeks of non-weight bearing treatment. After 6-8
weeks, she will have weight-bearing plain films to see if she is able to start weight-bearing or if surgical fixation will be
required.
Follow-Up: Pending 6-8 weeks of non-weight bearing followed by physical therapy versus surgical fixation.
The Dive! The Catch! The Chest Pain?
Authors: Matt Chatfield, MD; Byron Moran, MD; Anish Potty, MD; Tyag Patel; Eric Coris, MD
Affiliation: University of South Florida/Morton Plant Mease Hospital, Tampa, FL
History: A 21 yo African-American male collegiate football wide-receiver exited the game after making a spectacular
diving catch. He was able to leave the field unassisted, but was clutching his chest and had to sit down quickly to rest. He
immediately reported feeling a sharp pain in the left parasternal region of his anterior chest and felt associated shortness
of breath. It was quickly decided that a more thorough examination was needed and the player was able to jog to the
training room. Upon arrival to the training room he reported continuation of a sharp, moderate-severe, nonradiating pain
along the left side of the sternum, and no change in his dyspnea. He now also complained of a feeling of hoarseness,
although this was not audibly apparent to others. His jersey and equipment were removed to allow better evaluation lifting his arms above his head caused increased discomfort. The patient denied any past medical history and took no
medications.
Physical Exam: On physical exam vitals signs showed normal respiratory rate, mild tachycardia, and an oxygen
saturation of 99% on room air. He showed apparent discomfort, but was in no acute distress, was alert and oriented and
able to answer all questions appropriately with confluent sentences. HEENT was unremarkable. Lungs were clear to
auscultation bilaterally in all lung fields. Visual inspection of his chest showed no ecchymosis or skin lesions, but did show
a mild asymmetry and prominence of the left 2nd and 3rd costosternal joints that were tender to touch with mild crepitus.
There was no other apparent chest wall point tenderness. Spring test was negative. He experienced a slight increase in
pain with deep inspiration that was pleuritic in nature. Cardiovascular exam yielded normal S1 and S2, regular rate and
rhythm, and no murmurs, rubs, or gallops. Abdomen was soft, nontender, and nondistended with normal active bowel
sounds.
Differential Diagnosis:









Rib fracture
costochondral
subluxation/dislocation
pneumothorax
sternoclavicular dislocation
clavicular fracture
hemopericardium
pulmonary contusion
pectoralis major tear
Test Results: PA and lateral chest x-rays were obtained at the stadium within minutes of the injury and were normal
appearing, showing no signs of pneumothorax, pleural effusion, fracture or other bony abnormality. Noncontrast chest
CT was ordered showing a nondisplaced right first rib fracture with a small right-sided pneumothorax,
pneumomediastinum and subcutaneous emphysema. CT angiogram of the chest was ordered and was negative for any
acute vascular injury.
Final Diagnosis: Nondisplaced right first rib fracture with a small right-sided pneumothorax, pneumomediastinum and
subcutaneous emphysema.
Treatment: Initially, close observation with serial vitals and exams. Restriction from contact and football activities for 6
weeks. Retrun-to-play protocol completed before full clearance.
Outcome: Full Return to play in 6 weeks without complication.
Follow-Up: A follow-up CT scan was done two weeks later which showed a stable right first rib fracture with possible
subluxation of the left first costochondral joint and resolution of the pneumothorax and subcutaneous emphysema. The
player was held from participation for six weeks from his injury date. He did have an additional CT scan prior to release for
participation in full contact practice and games, which was unchanged from previous. He was able to return to play
without any return of symptoms or apparent change in performance.
Pump Up The Forearms
Authors: Bradley Kuske, DO; Amanda Weiss-Kelly, MD
Affiliation: Division of Pediatric Sports Medicine, Rainbow Babies and Children's Hospital/University Hospitals
Case Medical Center, Cleveland, OH
History: This is the case of a 15yo male quad (4-wheeler) racer who races on the semi-professional circuit. He presented
initially to an orthopedics clinic for evaluation of a few weeks of bilateral forearm pain while riding. He describes pain with
exertion, known in the motocross and quad racing world as "pump arm". Within a few laps of riding his quad, he
experiences bilateral hand numbness and weakness. He has never experienced a similar sensation prior to a traumatic jolt
that he sustained to both wrists/forearms when he landed hard on the front tires of his quad while racing a few months
prior. He had no significant pain or deformity following that episode and it wasn't until a few months later that his forearm
symptoms presented. He has been utilizing over the counter wrist braces and taking NSAIDs with no relief. Plain films on
initial evaluation were negative. An EMG was negative. His carpal tunnels and radial sensory nerves were injected with
depomedrol by an orthopedic surgeon. On follow up four weeks later, an MRI of the left forearm was also negative.</P
We initially evaluated him on referral from orthopedics in our primary care sports clinic three months after the onset of his
symptoms. At the time, he reported immediate onset of numbness, pain, and weakness within the first lap of riding in a
recent race. He had similar symptoms while pushing the lawnmower in his father's landscaping business. His pain typically
lasts for a few hours after activity, and gradually resolves.
Physical Exam: Healthy WD/WN male, 6ft tall weighing 214lbs. Stable vital signs. Neuro exam revealed normal sensation
to light touch throughout the involved extremities and radial pulses were symmetric with brisk cap refill. His forearms and
elbows had normal appearance at rest and were nontender to palpation. His flexor muscle mass was tight with palpation
bilaterally. Range of motion and strength of the elbows was full and pain free. On examination of the wrists, he was noted
to have Tinel's signs that were equivocal at the carpal tunnel and negative at cubital tunnel. Thoracic outlet testing was
negative.
Differential Diagnosis:




Chronic Exertional Compartment Syndrome
Ulnar Neuritis
Carpal Tunnel Syndrome
Scapholunate ligament sprain
Test Results: Plain films of bilateral wrists, EMG, and MRI of the left forearm were negative. Compartment pressure
testing was performed for both forearms. Pre- and Post-exercise pressure testing: Right volar: Pre 6 mm Hg, post 40 mm
Hg (normal pre 1-19 mm Hg, post 0-19mm Hg) Right dorsal: Pre 8 mm Hg, post 16 mm Hg (normal pre 2-27 mm Hg, post
2-24 mm Hg) Right lateral: Pre 23 mm Hg, post 47 mm Hg (normals not determined) Left volar: Pre 10 mm Hg, post 44
mm Hg (normal pre 1-19 mm Hg, post 0-19 mm Hg) Left dorsal: Pre 10 mm Hg, post 11 mm Hg (normal pre 2-27 mm
Hg, post 2-24 mm Hg) Left lateral: Pre 6 mm Hg, post 15 mm Hg (normals not determined) Grip strength, wrist
extension, and wrist flexion exercises were preformed for 45 minutes prior to elective cessation due to discomfort in the
flexor masses of each forearm. Pulses were intact following the procedure.
Final Diagnosis: Chronic Exertional Compartment Syndrome of the forearms
Treatment: Elective compartment and carpal tunnel releases.
Outcome: The patient was referred back to his orthopedic surgeon where he was taken for bilateral carpal tunnel release
and bilateral compartment release (at the time of this submission, he was in the post-operative recovery phase with a plan
to perform physical therapy once his surgical sites fully heal).
Follow-Up: pending surgical recovery
A Blue Arm
Authors: Kelly Wilkinson, MD; Scot Scheffel, MD
Affiliation: Family Medicine Residency of Idaho, Boise, ID
History: 15 year-old male right-handed rodeo steer roper was injured on 9/11/14 when he fell on his right shoulder
while getting tackled in a high school football game. He was seen the same day in a local emergency room where he was
diagnosed with a right midshaft clavicle fracture with 1.8 cm overlap and no significant angulation. He was treated with a
sling. He was seen in clinic for follow up on 9/15/14. He complained of mid-clavicle pain and motion at his fracture site.
He also complained of occasional numbness/tingling in his right fourth and fifth digits and sometimes trouble sleeping at
night. He had prominent mid-clavicle pain and swelling. His distal neurovascular and strength exam was normal Repeat
radiographs were taken. He was transitioned to a figure-of-eight splint and pain medicines were adjusted. He was seen
in clinic for routine follow up on 9/28/2014. He reported his pain had improved and was continuing to wear the figure-ofeight splint. Radiographs were taken and showed no significant change. He was seen again in clinic for routine follow up
on 10/12/2014. He complained of bluish discoloration of his arm for the past few weeks that seemed to come and go. He
reported mildly increased pain in his right clavicle after he had a minor fall playing dodgeball the week prior. He denied
arm pain, numbness, tingling, paresthesias, swelling or weakness.
Physical Exam: On exam his entire right hand and arm appeared ruddy to violacious in color compared to his left arm
up to the shoulder level. He had no arm or hand swelling. He had 5/5 strength and no sensory deficits of the median,
radial, or ulnar nerves. His brachial and radial pulses were strong and present. Allen test was normal. His capillary was less
than 3 seconds and equal to his left hand.
Differential Diagnosis:






Subclavian deep venous thrombosis or compression
Subclavian artery thrombosis or compression
Thoracic outlet syndrome
Cervical radiculitis
Brachial plexus injury
Complex regional pain syndrome
Test Results: Radiograph: A radiograph was done and demonstrated no significant change in fracture displacement and
evidence of bony callous formation. Upper extremity Doppler ultrasound: no evidence of upper extremity deep venous
thrombosis or superficial venous thrombosis. There was notable complete extravascular obstruction of subclavian venous
flow with contralateral head rotation Flows were not obstructed with neutral head position. An arterial study was not
performed.
Final Diagnosis: Subclavian vein dynamic extravascular compression secondary to midshaft clavicle fracture.
Treatment: 10/17/2014: ORIF midshaft clavicle fracture
Outcome: Successful ORIF midshaft clavicle fracture reduction with clavicle plate. Intraoperative findings included
significant bony callous formation requiring debridement prior to plate fixation. Fascia underlying fracture appeared intact
intraoperatively.
Follow-Up: Patient recovered well from surgery and had complete resolution of his arm discoloration. He had no
further problems with arm pain, swelling, numbness or weakness. Anticipated return to contact sports and rodeo activities
after approximately three months.
Shoulder Pain: A Zebra with Increasing Incidence
Authors: B. Grant Marshall, MD; Stephan Esser MD, USPTA
Affiliation: St. Vincent's Family Medicine Center and St. Vincent's Hospital, Jacksonville, FL
History: A 47-year-old Caucasian female without significant past medical history presented to a sports medicine clinic
with the chief complaint of six weeks of worsening right arm pain and swelling in the setting of increased upper extremity
training. Her pain worsened with movement of her extremity, and she did not note any alleviating factors. She did not
have a prior history of similar symptoms, and nor did she report additional associated symptoms. The only medication that
the patient was on at the time was an oral contraceptive medication.
Physical Exam: This patient's exam was significant for diffuse right upper extremity non-pitting edema, increased
vascular congestion and pain with active and passive range of motion of the shoulder. The remainder of her muscular and
neurological examination of the right upper extremity was normal including strength, sensation, reflexes and rotator
cuff/labrum testing. In addition to the right upper extremity findings, the rest of her exam was normal.
Differential Diagnosis:
Upper extremity deep venous thrombosis
superior vena cava syndrome
lymphedema
heart failure
neoplasm
anabolic steroid use
rotator cuff injury
Test Results: The patient was sent for an emergent upper extremity duplex, and was diagnosed with an upper extremity
deep vein thrombosis in the subclavian vein.
Final Diagnosis: Upper extremity deep venous thrombosis in the subclavian vein
Treatment: Hematology was consulted and she was started on Warfarin and an enoxaparin bridge. She was also
encouraged to discontinue her oral contraceptive medications.
Outcome: Her symptoms improved with anticoagulation therapy. She did not have a recurrence of deep venous
thrombosis.
Follow-Up: A few weeks after initiation of anticoagulation therapy she was able to resume her normal exercise and
strength training routines.
Elbow Pain in a Baseball Athlete
Authors: Nathan Granneman, MD; Robert Hosey, MD; Darren Johnson, MD
Affiliation: Orthopaedic Surgery and Sports Medicine/University of Kentucky, Lexington, KY
History: In September 2012, a 12 year-old male baseball player presented to the sports medicine clinic with a 1 week
history of medial sided elbow pain that increased with throwing activities. Initial exam was positive for tenderness over the
medial epicondyle, lack of 10° on extension of the elbow, and pain with resisted wrist flexion/valgus stress to the elbow. Xray showed widening/fragmentation of the medial epiphysis. Patient was diagnosed with little league elbow and advised
to refrain from all throwing activities for 3 months. Physical therapy was initiated at 1 month for scapular stabilization and
rotator cuff/core strengthening. At 3 month follow-up visit patient was asymptomatic, repeat imaging showed interval
healing of his fragmented growth plate, and physical exam was normal. A throwing program was initiated and patient was
released from the clinic. In July 2014, patient returned at age 14 with a 2 week history of gradual onset medial/posterior
elbow pain that increased with throwing activity. No pain was present at rest or while hitting. Prior to onset, patient had
pitched for three teams over the previous several months. His pitch count was approximately 70 per game, 2-3 games per
week. The physical exam was positive for tenderness overlying the olecranon process. Right elbow X-ray was obtained
with results being normal. He was diagnosed with olecranon apophysitis and advised to refrain from all throwing activities
for 2 weeks. Physical therapy was initiated focusing on rotator cuff strengthening and scapular stabilization with
resumption of the throwing program. Patient was seen 8 weeks later. He had followed recommendations and was
tolerating his throwing program. Two weeks prior to this visit he was able to pitch one inning with no elbow pain.
However, during the following game he had sudden onset of posterior elbow pain during the first 10 pitches. He also had
associated numbness in the medial side of his elbow and developed elbow swelling within the first several minutes.
Physical Exam: Elbow joint effusion present with no overlying ecchymosis seen. Direct TTP overlying olecranon process.
AROM 20-120 elbow flexion. Reproduction of pain on resisted elbow extension and passive elbow pronation. Grossly
stable to varus/valgus stress at 30°. Distal neurovascular exam was intact.
Differential Diagnosis:
1.
2.
3.
4.
5.
Olecranon apophysitis
Olecranon fracture
Medial epicondyle avulsion fracture
OCD lesion of capitellum
UCL injury
Test Results: Elbow Radiographs: Mildly displaced intra-articular fracture of olecranon
Final Diagnosis: Right Olecranon fracture
Treatment: Patient had open reduction with internal fixation of the right olecranon. A 6.5 mm screw measuring 60 mm
was inserted after reduction of fracture. Post-op, he was placed in elbow brace locked at 90° for 2 weeks. At 2 week postop appointment, the sutures were removed and the brace was unlocked with patient remaining in brace at all times. At 6
week post-op appointment, the brace was removed and patient initiated PT with no concurrent sporting activity. There is
no plan for removal of hardware unless he becomes symptomatic.
Outcome: Patient tolerated the procedure well and had full ROM of elbow by 3 months post-op visit.
Follow-Up: Current plan is for patient to begin throwing program at 4 months with release back to competitive play at
approximately 6 months post-op.
Strange Lower Leg Pain in 18 Year Old Football Player
Authors: Britney Else DO; T. Jeffery Emel, MD; Thomas Allen, DO, MPH; Lamont Cavanaugh MD
Affiliation: Center for Exercise and Sports Medicine, Department of Family Medicine, University of Oklahoma
School of Community Medicine, Tulsa, OK | Eastern Oklahoma Orthopedic Center, Tulsa OK
History: Development of right lower leg pain for three weeks without direct blow. Heavy radiating pain down lower leg
with running and walking. Pain is 8/10 and continuous. After one week of rest continued to have pain with activity. Denies
foot pain, back pain, numbness and tingling.
Physical Exam: Extremeties: exquisitely tender to palpation over the course of the peroneal nerve below the fibular
head. Pain to palpation down the shaft of the fibula and in the muscles over the lateral aspect of the lower leg. Peroneal
nerve strength is intact. No knee or lower back pain.
Differential Diagnosis:




Peroneal nerve injury
Fibular fracture
Fibular head dysfunction
Peroneal muscle strain
Test Results: X-rays negative. MRI revealed full-thickness discontinuity of the origin of the peroneous longus muscle at
the proximal aspect of the lower leg. Distal retraction of the tendon fibers of 15-20mm.
Final Diagnosis: Peroneous longus tear at the site of orgin
Treatment: Initial treatment included calf sleeve and ankle brace. Because of no improvement, PRP injection under
ultrasound guidance preformed.
Outcome: 72 hours following PRP injection, significant improvement of pain. Physical therapy initiated nine days after
injection with good results.
Follow-Up: Four weeks after initial presentation was cleared to return to play for football.
Quarterback With Throwing-Induced Abdominal Pain
Authors: Marisa Formica, MD; Brian Harrell, MD; Vincent Shaw, MD
Affiliation: Baton Rouge General Medical Center - Sports Medicine Fellowship program Baton Rouge, LA
History: A 19-year old college football quarterback was throwing a pass in a game when he felt a tear in his lower
abdominal region with almost immediate, subsequent LLQ abdominal pain. Prior to this injury he complained of having
arm pain in his throwing arm and was compensating by twisting and turning his body to throw. In the days following the
incident, he received E-stim and U/S in the training room, but symptoms progressively worsened over a three week
timespan. Symptoms were aggravated by throwing, practicing, lifting weights, performing crunches, and planting feet to
throw. He denied any alleviating symptoms and had not been taking any medication. He denied any weight loss, chest
pain, shortness of breath, nausea, vomiting, fever, chills, or changes in urinary/bowel habits.
Physical Exam: Vital signs: Within Normal Limits HEENT: Normocephalic, PERRLA, nares and throat clear, no neck mass
or thyromegaly Lungs: CTAB Heart: RRR Abdomenn: Bowel sounds normal, soft, tenderness in LLQ with guarding and
palpable mass approximately 3cm in diameter, no rebound tenderness, negative McBurney's and Murphyr's sign
Vascular: No edema, +PP Genitourinary: No urethral discharge, no testicular mass, no inguinal hernias noted Lymphatic:
No lymphadenopathy Skin: No rashes or bruises, no skin rash or lesion Musculoskeletal: Gait normal, no joint
tenderness Neuro: Alert and oriented, no acute distress
Differential Diagnosis:
Athletic pubalgia
abdominal wall abscess
abdominal wall hematoma
psoas abscess
retroperitoneal hemorrhage
ischemic colitis
inflammatory bowel disease
incarcerated hernia
prostatitis
seminal vesiculitis
ureterolithiasis
urinary tract infection
Test Results: Diagnostic Ultrasound was performed which revealed a left sided distal rectus abdominus hematoma,
approximately 3.1 x 4cm in transverse plane and larger in sagittal plane. An MRI was performed which confirmed a 5cm
hematoma in the inferior aspect of the left rectus muscle.
Final Diagnosis: Rectus sheath hematoma
Treatment: Rest, electromyostimulation, ultrasound therapy
Outcome: The athlete was compliant with rest and training room treatments. After three weeks from the time of initial
ultrasound imaging, the athlete was symptom free and a repeat ultrasound was performed which revealed a resolving
hematoma of approximately 1.2 x 1.5cm.
Follow-Up: The athlete returned to activity 6 weeks after injury and follow up imaging revealed resolution of the
hematoma.
When Football Stadiums Attack
Authors: Emily Dixon, DO
Affiliation: TriHealth Orthopedic and Spine Institute Cincinnati, OH
History: While covering a high school football game, the student section of the home team rail collapsed while being
filmed by a local news station. Approximately 20 students fell about 4-5 feet through the railing onto concrete or steps.
The railing did not fall completely to the ground, but was instead suspended by the stairs going up the student section.
Immediately, myself the PCSM fellow and the Orthopedic physician responded and were able to triage the injuries.
Several medical personnel from the stands and the on field EMTs assisted in care.
Physical Exam: 1. Two female students hit their heads on the concrete steps. Both were disoriented and slightly altered
with palpable hematomas on the occiput. They did not have parents/guardians present and were immediately sent to the
hospital by squad. 2. One male student presented with a Right wrist injury, with distal radius tenderness and mild Colles
deformity and was transferred to the hospital. 3. About 8-12 students presented with superficial abrasions and one deep
laceration.
Differential Diagnosis:



Distal Radius Fracture
Abrasions/Lacerations
Concussion/TBI
Test Results: 3 students were transferred to the local hospital per EMTs: One for a presumed distal radius fracture, and
two for head injuries; presumed concussions. Several students who had guardians present were treated for superficial
abrasions that were irrigated and bandaged. One student had about a 4-5 cm laceration down the front of his leg that
was sutured in the training room with parental consent by the PCSM fellow.
Final Diagnosis: Mini Mass Casual Event: Football Stadium Rail Collapse 1. 2 concussions 2. Right distal radius fracture
3. Multiple lacerations and abrasions: 8 students
Treatment: The majority of the students were easily treated on the sideline with supplies from our medical kits and the
paramedics. One student's laceration was sutured and monitored by the PCSM fellow at the school, with suture removal
one week later. All lacerations and abrasions were given verbal instructions to obtain tetanus booster. This was followed
up by written instructions via email to parents from the school. Both concussions were treated symptomatically. They were
not student athletes and were treated by their PCPs. The distal radius fracture had a closed reduction, and was treated in a
cast for 6 weeks without complication.
Outcome: Always be prepared for a mass causality event. Mass causality action plan was reviewed. New signs, and
monitoring of the stadium closely by school staff and security.
Follow-Up: Sporting events were canceled at the stadium, where both the HS and Jr. high play, for a week until it could
be assessed by the engineers and the rail replaced. The next home game was able to be played there 2 weeks later
without incident.
3% Chance
Authors: Christian Basque, MD
Affiliation: Geisinger Health Systems, Sports Medicine/Orthopaedics, Wilkes-Barre, PA
History: 13 year old male football player was running at practice. Patient states he felt a pop and then a sudden pain in
the right groin. Patient fell to the ground was unable to walk he had to be carried off of the field. When pain did not
resolve patient was seen at pediatric urgent care the following day. He was given crutches, prescribed NSAIDs and referred
to sports medicine.
Physical Exam: Patient seen and examined in office, he walked with a limp and had a slight antalgic gait. Patient was
point tender in the right groin. He denied any tenderness over the trochantic bursa or ASIS. He had full ROM with forward
flexion, abduction, adduction, internal and external rotation. His strength was 5/5 except for forward flexion strength was
3/5, his quadriceps strength was only 3/5. Pain was a limiting factor during testing.
Differential Diagnosis:





Slipped Capital Femoral Epiphysis
Legg- Calve-Perthes Disease
Hip Pointer
Sports hernia
Avuslion fracture
Test Results: X-ray was completed of the pelvis and right hip with comparison views of the left hip.
There was
widening of the right lesser trochanter in comparison to the left trochanter, the hip joint showed normal joint spacing with
no abnormality visualized.
Final Diagnosis: Avulsion fracture of the lesser trochanter
Treatment: Conservative non-weight bearing crutches for 2 weeks.
F/U in 1 week
Outcome: Patient was seen in athletic training room 6 weeks out from injury he had no pain on exam and was cleared to
resume activity
Follow-Up: No return to play until 6 weeks
Chronic Hip Pain in an Adolescent Female Athlete
Authors: Nathan Falk, MD; Geoff McLeod, DO
Affiliation: Offutt AFB/University of Nebraska Medical Center Family Medicine Residency Omaha, NE
History: A 15-year-old female athlete presents with severe abdominal cramping, lumbar and left hip pain preceding and
during menstruation. Initial conservative management with OCP medications and NSAIDs partially alleviated menstrual
symptoms, but remained ineffective for her chronic lumbar and hip pain. Patient reported new complaints of midline back
pain with proximal extension along the spinal column several weeks following initial presentation with exacerbation of
symptoms dependent on position. Despite failure of conservative measures NSAID therapy was continued with the
addition of Physical Therapy (PT) and a Gynecology consultation. Gynecology was unable to improve symptoms through
adjustment of her OCP. After several weeks of treatment, patient was reassured and encouraged to continue PT despite
lack of improvement. Four months after initial presentation, Sports Medicine was consulted after patient's pain became
debilitating and was having difficulty sitting for greater than ten minutes due to unrelenting left leg and hip pain
described as "an achy feeling". Due to the failure of therapy, escalating pain and persistence of symptoms our patient had
stopped swimming competitively and performing in her school flag corp in order to minimize her symptoms.
Physical Exam: Legs appeared symmetric without length discrepancy. No tenderness to palpation over lumbar spinous
processes or paraspinous muscles, no ecchymosis or signs of trauma. Range of motion in the left hip was within normal
limits on external rotation. Internal rotation of hip at 90° of flexion was limited and produced lateral and anterior hip pain.
Straight leg test was positive at 30° on left leg. Poor range of motion to flexion and extension of left hamstring and hip
flexors, with popliteal angle 35° despite 5 weeks of PT. No reproducible pain or instability with posterior compression or
direct palpation of femoral head. No sensory deficits. Leg strength was 5/5 bilaterally to hip, knee and ankle flexion and
extension.
Differential Diagnosis:







toxic synovitis
slipped capital femoral epiphysis
Legg-Calve-Perthes
osteomyelitis
sarcoma
eosinophilic granuloma
leukemia
Test Results: XR pelvis: negative for osseous deformity/lesion. MRI left hip: bone marrow edema in the
intertrochanteric area without pathologic fracture. Negative for abscess formation. CT left hip: moderate cortical and
minor medullary destruction without cortical reaction in the intertrochanteric area. Negative for abscess formation
Final Diagnosis: Ewing sarcoma of left femoral neck and head
Treatment: Patient underwent six rounds of chemotherapy protocol AEWS0031 and included vincristine, doxorubicin,
cyclophosphamide, ifosphamide and etoposide. Following completion of chemotherapy resection of the proximal half of
her femur with placement of mega prosthesis performed.
Outcome: Orthopedic surgery continued to follow post-resection. Patient suffered pain within the lower left extremity for
months following surgery. Patient was referred to Pain Management four months following femoral resection due to failed
conservative pain management. Oncology determined patient was in remission five months following surgical resection.
Patient is currently managed by pain management, physical therapy and quarterly appointments with Oncology.
Follow-Up: Due to continued pain, our patient has made slow progress attaining her previous level of daily living and
has not been able to return to sports. She continues to struggle with lower extremity and lumbar pain with radicular
symptoms in her left lower extremity; however, these symptoms have improved significantly since completion of
chemotherapy and femoral resection. Due to side effects associated with her chemotherapy and a protracted course of
rehabilitation she withdrew from school to cope with her disease, returning after one year, and graduating a year behind
her previous classmates.
A Foot Sprain or Something More?
Authors: Sara Gould, MD, MPH; Ashley Reed Estes, MD
Affiliation: Children's of Alabama, University of Alabama at Birmingham, Birmingham, AL
History: 13 year old female presented for evaluation of chronic right foot pain. The patient reported that the pain had
been present for over 2 years. She reported minor, remote trauma, such as pain after kicking a ball. She denied recent
injury to the foot and ankle. She stated that the pain was worse after playing soccer and with plantar flexion. She reported
improvement in the pain with rest. She is a very competitive soccer player.
Physical Exam: Right foot: There are no skin changes. The foot is non-tender. She is able to plantar flex to 75° on the
on right, versus 95° on the left. She has limited hindfoot inversion and eversion on the right side. Her stability exam is
significant for a negative talar tilt test and negative drawer test. Negative posterior compression test. The foot is
neurovascularly intact.
Differential Diagnosis:





Hypermobile flat foot
Sinus tarsi syndrome
Tarsal tunnel syndrome
Tarsal coalition
Foot sprain
Test Results: Right foot radiographs were obtained. The AP lateral and oblique images demonstrated a
calcaneonavicular coalition.
Final Diagnosis: Calcaneonavicular coalition
Treatment: The initial treatment plan was for conservative management. The patient was given a prescription for
physical therapy. She continued to have pain, so an orthosis was prescribed.
Outcome: The patient continued to have pain. She had another injury while playing soccer and was diagnosed with a
non-displaced navicular fracture. She was scheduled for a calcalneonavicular coalition takedown.
Follow-Up: The patient was able to return to soccer 6 weeks after the surgery and has not had recurrent injury since that
time.
Swimmer with Shortness of Breath
Authors: Dena Florczyk, MD
Affiliation: University of California - Los Angeles, Los Angeles, CA
History: A 21 year-old Caucasian female NCAA Division I swimmer with a history of ADHD and anxiety presented to the training room
for evaluation of one day of shortness of breath. Upon awakening she felt short of breath with deep inspiration, and during her morning
swim practice felt increasingly dyspneic with exertion. She had associated mild dizziness, but no syncope or pre-syncope. She denied
fevers, chills, nasal congestion, cough or recent illnesses. She denied palpitations, chest pain, leg pain or leg swelling, or history of
pulmonary issues such as asthma. She denied taking any new medications, recent injuries or recent travel. She did endorse increased
stress with schoolwork and swimming. She has no known allergies. Her only medications were an oral contraceptive pill (Apri) which
she has been on for one year, and Adderall which she last used during final exams two months previously. She had a wisdom tooth
extraction in 2008 and her only hospitalization was in 2011 for a panic attack. She has no significant family history. She is a junior in
college and is sexually active. She admitted to rare use of alcohol, but denied using tobacco use, illicit drugs, or performance enhancing
drugs.
Physical Exam: VS: Temp 98°F, HR 80, BP 108/62, RR 12, pulse oximetry 97% on RA General: WNWD, comfortable appearing and
speaking in full sentences. HEENT: NCAT, EOMI, Oropharynx without any erythema or exudate, nares/TM/external auditory canals WNL.
Neck: supple, no LAD or TM, no carotid bruits, no JVD. Cardiovascular: RRR, no m/r/g, no precordial heave. Lungs: CTAB, no w/r/c.
Abdomen: Normal BS, soft, NTND, no HSM. Extremities: WWP, no C/C/E; no palpable cords, negative homan's sign; 2+ radial/DP/PT
pulses. Peak Flows pre-beta agonist: 290, 300, 350 Peak Flows post-beta agonist: 350, 380, 400
Differential Diagnosis:

Anxiety/stress

ADHD or stimulant medication side effect

Asthma/exercise induced bronchoconstriction

Pulmonary embolism

Arrhythmia

Structural heart disease

Upper respiratory infection

Atypical pneumonia
Test Results: Outpatient: EKG: NSR rate of 60, normal axis, normal intervals, no ST changes Chest XR PA/Lateral: normal Labs: D-dimer
elevated at 2781; normal CBC, electrolytes, ferritin, liver and kidney function. Emergency Room: V/Q Scan: High probability for acute
pulmonary embolism with complete perfusion abnormality of the right lung and two large sub-segmental perfusion abnormalities in the
left lung. Pregnancy test negative, PT/INR/PTT normal Hospital Admission:
Venous US Doppler Bilateral Legs: No DVT
Chest CT Angiography: Multiple acute pulmonary emboli extending from right main pulmonary artery to RUL, RML, RLL and smaller
emboli to left posterior segmental and lingular pulmonary arteries. LLL wedge shaped infarction. Diffuse peribronchial thickening.
Echocardiography: Normal
Thrombophilic Workup:
Negative Factor V Leiden, DRVVT, ANA, anti-cardiolipin antibodies, lupus anticoagulant antibodies, beta-2-GP-1 antibodies,
prothrombrin G20210.
Normal Antithrombin III, APC resistance ratio, and homocysteine. Factor VIII was initially 248% but decreased to 185% when rechecked.
On warfarin Protein C antigen was 46% and protein S total was 51%.
Final Diagnosis: Sub-massive pulmonary embolism due to oral contraceptive pills without underlying thrombophilia.
Treatment: She was hospitalized and started on a heparin drip and warfarin. She remained stable and was discharged after 48 hours
with an enoxaparin bridge. Her OCP was discontinued and she was encouraged to discontinue her Adderall.
Outcome: She completed three months of anticoagulation at which time her D-dimer remained normal, thus her anticoagulation was
discontinued.
Follow-Up: Three weeks post-diagnosis she was asymptomatic and was cleared to gradually begin all non-contact activity. Two months
post-diagnosis she was at full training volume and by four months post-diagnosis was competing. She will require prophylactic
anticoagulation during pregnancy and the post-partum period, as well as with any lower extremity orthopedic surgeries or periods of
immobilizations.
Chest Wall Pain in College Recreational Runner After Moving Furniture
Authors: Jane S. Chung, MD
Affiliation: NYU Hospital for Joint Diseases, NYU Langone Medical Center, New York, NY
History: A 20 year old female college student presents with left sided chest wall pain after helping her brother move, carrying heavy
boxes and furniture 2 weeks ago. She was evalauted in the local ER where radiograph and ultrasound of chest was unremarkable. She
was given instructions to follow-up in sports medicine clinic for musculoskeletal chest pain. Patient also admits to smoking "weed" for
the first time few days prior and reports feeling like a "panic attack" with heart palpitations, shortness of breath and chest tightness,
which she did not reveal in ER as her mother was with her at the time. Reports symptoms of left sided chest tightness and shortness of
breath when she went for a run yesterday, which is new. Symptoms were somewhat relieved with massaging the chest. Also reports a
"bump" underneath her skin on left neck.
Physical Exam: General: Well appearing, healthy female Inspection: 2x3cm left supraclavicular node, no erythema or ecchymosis over
pectoralis or left chest wall Palpation: + tenderness of left pectoralis major muscle at clavicular insertion and left lateral and anterior
ribs 2,3 and 4. Minimal "soreness" over left supraclavicular node, mobile Auscultation: Cardiovascular</em - regular rate and rhythm,
no murmurs, rubs or gallops. Normal S1 and S2. Respiratory - clear bilaterally, no wheezes, crackles, rales ROM: Full pain free ROM or
cervical spine, bilateral elbows and shoulders Strength: Pain with resisted left subscapularis lift off testing-5/5, all other strength testing
5/5 as well Neuro: 2+ deep tendon reflexes of bilateral upper extremities, sensation intact Extremities: capillary refill &lt;2 seconds,
warm and well perfused
Differential Diagnosis:
Costochondritis
Pectoralis muscle strain
Tietze's syndrome
Spontaneous strenoclavicular subluxation
Slipping rib
Sternalis syndrome
Xiphoidalgia
Stress or insufficiency fracture
Fibromyalgia
Rheumatoid arthritis
Angina
Coronary artery disease
Coronary vasospasm
Pericarditis
Pulmonary hypertension
Pleural effusion
Neoplasms/Mediastinal mass
Test Results: Radiographs done in ER: CXR, 2 views SC joints and ultrasound chest were all negative for fracture, dislocation,
pneumothorax, effusion. Electrocardiogram and echocardiogram done by cardiology: normal function In clinic, MRI left upper
extremity without IV contrast was ordered: no evidence of pectoralis muscle tear or abnormality, conglomerate of lymph nodes
increased in size in left supraclavicular region and prominence of soft tissues in the anterior mediastinum. Findings nonspecific and
possibilities include: 1. lymphoproliferative disease 2. infection or inflammatory disease Further evalaution with CT chest with contrast
recommended. CT chest with IV contrast: anterior mediastinal mass measuring 3.5 x 2.1 cm and additional anterior mediastinal lymph
node 1.7 x 1.8 cm adjacent to left subclavian, left supraclavicular lymph nodes, findings suggest lymphoma or thymic neoplasm with
metastatic disease
Final Diagnosis: Lymphoma
Treatment: Patient referred to hematology/oncology for consultation the following day. She underwent biopsy to mediastinal mass.
Outcome: Patient started chemotherapy 2 weeks later, currently still undergoing treatment. She did not return back to school for fall
semester.
Follow-Up: Once patient is cleared by her oncologist for light exercise, she may start light exercise.
Chest Pain in a Division I Basketball Player
Authors: Christopher Jenks, MD; Jonathan Schultz, MD; Margaret Gibson, MD
Affiliation: UMKC Sports Medicine Fellowship, Truman Medical Center Lakewood, Kansas City, MO
History: A previously healthy 18 year old African American Division I basketball player presented to his athletic trainer
secondary to substernal chest pain that he was experiencing between 800 meter dashes in a conditioning drill during the
first week of practice. He stated that the pain was worse "with each beat" of his heart, but did not radiate. The pain
reportedly lasted for two minutes at a time and seemed to improve when he would resume running. At the time he denied
any associated nausea, vomiting, diaphoresis, dyspnea, pre-syncope or syncope. He denied any change in the pain with
taking deep breaths, with pushing on his chest, or with lifting weights. He stated that these symptoms had not occurred
before and that he did not believe that it was related to what he had eaten earlier that day. He denied any recent illness,
fevers or chills. The prior week the patient had been found to have an elevated blood pressure during his PPE with systolic
in the 140s and diastolic in the upper 80s. His blood pressure following his symptoms remained elevated in the 150s over
90s. He denied any prior history of murmur or elevated blood pressures. The athlete's father was diagnosed with
hypertension in his early 20's. There was no family history of coronary artery disease or sudden death.
Physical Exam: VS: HR=72 R=16 BP= 152/72 and 161/80 on recheck Cardiac: normal rate, regular rhythm, no murmur
with sitting, standing, squatting, no gallop, 2+ pulses in bilateral upper extremities and lower extremities. Pulmonary:
lungs clear to auscultation bilaterally, no wheezes, rales or rhonchi. GI: soft, non-tender, non-distended, no masses
palpated. Remainder of exam normal.
Differential Diagnosis:
Atypical Chest Pain secondary to:










Hypertrophic Cardiomyopathy
Dilated Cardiomyopathy
Arrhythmogenic Right Ventricular
Cardiomyopathy
Dissecting Aortic Aneurysm
Malignant Hypertension
Coronary Artery Disease
GERD
Musculoskeletal Chest Pain
Basic deconditioned athlete with sudden increased activity.
Test Results: EKG: Sinus rhythm with diffuse deep and symmetric T wave inversion in the inferior and anterolateral
leads. Echocardiogram: Concentric left ventricular hypertrophy. Cardiac MRI: Mild concentric left ventricular hypertrophy
(wall thickness of 1.4cm) with no delayed gadolinium enhancement to suggest hypertrophic cardiomyopathy.
Final Diagnosis: Athlete's Heart with superimposed changes secondary to hypertension.
Treatment: Amlodipine 2.5mg PO Daily
Outcome: The athlete has had no further complications of continued chest pain and blood pressure has been well
controlled on low dose of amlodipine.
Follow-Up: The athlete was allowed to return without restriction following cardiac MRI. Amlodipine has been titrated for
adequate blood pressure control. We will continue to monitor him closely on an annual basis with EKG and
Echocardiogram. He may require a repeat cardiac MRI in the future.
Exertional Leg Pain in a Marathon Runner
Authors: Kavita Deshpande, MD; John Herb Stevenson, MD
Affiliation: University of Massachusetts, Worcester, MA
History: This is a 52 year old female marathon walker who presented with left anterior shin pain worsening over the
past year. Initially the patient thought she had shin splints and tried to alleviate the pain with home exercises and
stretching. The pain gradually worsened over time preventing her from walking, running and doing her daily activities. The
patient mentioned that the pain is worse with weight bearing exercises; and occasionally is associated with numbness or
tingling to the dorsum of her foot. ALLERGIES: NKDA PMH: Right bunion surgery, c-section MEDICATIONS: MVI,
Calcium, Vitamin D, Vitamin B12 FAMILY HISTORY: noncontributory SOCIAL HISTORY: smokes cigarettes occasionally,
minimal alcohol use REVIEW OF SYSTEMS: full 12 system ROS negative except as noted above
Physical Exam: General: alert and oriented x3 HEENT: NC/AT, EOMI, PEERL Respiratory: normal respiratory effort
CVS: S1 S2 heard, RRR, palpable pulses MSK: normal gait Left leg: Tenderness to palpation over the proximal tibia-fibular
joint, fullness noted of the left leg anterior compartment strength: 4/5; no difficulty with plantar flexion, dorsiflexion,
inversion, eversion of foot Neuro: sensation intact in lower extremities bilaterally with no focal deficits or paresthesias.
Differential Diagnosis:






Exertional Compartment Syndrome
Stress Fracture
Muscle Strain
Soft tissue mass
Ganglion cyst
Popliteal artery entrapment
Test Results: Left leg XRAY: negative for stress fracture, degenerative changes, or other bony abnormalities. 7/2014
MRI showed a low T1 signal focus extending longitudinally consistent with ganglion cyst 15.8x2.1x2.1 in size.
Final Diagnosis: Left lower leg tibia-fibular joint ganglion cyst involving the anterior compartment
Treatment: After review of the findings and the treatment option were discussed, cyst aspiration was performed. Highly
viscous cyst fluid was aspirated under ultrasound and the cyst was injected with corticosteroid. A compression sleeve was
applied. The patient returned four weeks later with recurrence of fluid accumulation and a second aspiration and steroid
injection was performed.
Outcome: The patient was referred to Orthopedic Surgery after the second fluid accumulation for possible cyst excision.
Conservative treatment verses surgical exicison were discussed. Patient opted for conservative treatment as her symptoms
had improved.
Follow-Up: There is discussion about attempting another aspiration with attempt at sclerosing the cyst stalk at the
tibia-fibular joint and injecting a corticosteroid under ultrasound guidance. At the moment, patient is continuing to
exercise, speed walking and cross training as tolerated.
Hip Pain in a College Student
Authors: Ryan Siegel, DO; James Borchers, MD; Kendra McCamey, MD
Affiliation: The Ohio State University Wexner Medical Center, Department of Sports Medicine
History: A 21 year-old female presented with progressively worsening left hip pain with running over the previous 8
weeks without history of injury or trauma. She had been seen by two separate physicians who diagnosed her with a
muscle strain and inflammation, respectively. At the time, an x-ray of her left hip was reported to her as negative (neither a
copy of the images nor the report were available for review). Despite rest, NSAIDs and modified activity (non-weightbearing), her anterior hip pain persisted. Upon presentation, she had an inability to bear weight, loss of motion and hip
weakness. Pain was rated 6/10 and sharp. Movement or weight-bearing activity exacerbated her symptoms. She denied
any back pain, loss of sensation or numbness/tingling. She has no previous history of injury. Prior to having pain, she was
doing interval training and running 12 miles per week.
Physical Exam: VS: HR 60, BP 110/64, Ht 5'5", Wt 51 kg/113 lbs U: Well-developed, well-appearing female, no apparent
distress. Alert & oriented x 3. MSK: Left hip without deformity or ecchymosis. (+) tenderness over the anterior aspect of
the hip, Full active and passive ROM with pain in all directions. Strength was 4/5 in abduction, adduction, and flexion. (+)
FABER test, (-) Ober test (+) significant pain with one-legged hop and balance testing. Neuro/Vascular: Normal pulses
and sensation with +2/4 reflexes in bilateral patella/achilles.
Differential Diagnosis:







Femoral neck fracture
femoral head avascular necrosis
femoroacetabular impingement (FAI) syndrome
hip tendonitis and bursitis
hip pointer
snapping hip syndrome
osteitis pubis
Test Results: The original x-ray was reported to the patient as negative 6 weeks ago at the orthopedics office (a copy of
the images nor the report were available for review). An x-ray obtained by our office revealed sclerosis in the inferior
femoral neck concerning for a stress injury/fracture. A follow-up MRI was then obtained which showed an incomplete
compression sided stress fracture (~33%) of the medial femoral neck with surrounding edema.
Final Diagnosis: Left femoral neck stress fracture, compression side.
Treatment: The patient used crutches with toe touch for 2 weeks while waiting for the MRI, and her pain never
improved. Given the severity of the consequences if the fracture progresses to a complete transcervical fracture including
nonunion, varus malunion, osteonecrosis and subsequent arthritic changes, the patient underwent percutaneous screw
fixation, even though her stress fracture line was less than 50% of the width of the femoral neck, which is typically treated
conservatively.
Outcome: The patient had no significant post-operative complications and is currently undergoing a post-operative
course.
Follow-Up: Post-surgically, the patient was placed on crutches with toe-touch weight-bearing for a total of four weeks.
She is currently experiencing mild pain and is in the process of weaning from the crutches. She is able to ride a stationary
bike, but is still refraining from impact activity including running. She has not yet been cleared for full activity.
Posterolateral Knee Swelling in a Distance Runner
Authors: Katherine Nanos, MD; Cara Prideaux, MD
Affiliation: Mayo Clinic, Rochester, MN
History: An otherwise healthy 43 year old marathon runner presented to our Sports Medicine Center for a second
opinion of a painful posterolateral right knee swelling. He first noted the swelling after completing a marathon two years
prior and he sought evaluation in a local orthopedic clinic. The initial diagnosis included a Baker's cyst and distal
hamstring (biceps femoris) tendonitis. Treatment included rest, compression, and aspiration with corticosteroid injection of
the presumed cyst. The procedure note stated that ten milliliters of dark red blood was aspirated from the lesion. At his
one month follow up, the patient reported no change in his symptoms. However, he was still running marathons so no
additional treatments were recommended. It was at this point that he sought evaluation in our Sports Medicine Center.
He reported mild pain (2-3/10) after longer runs and after prolonged sitting in a car. He noted that the fullness and
swelling at the lateral knee was variable and tended to be better with activity and worse with inactivity. He denied any
other symptoms.
Physical Exam: General: Healthy-appearing individual in no apparent distress. Skin: No superficial skin changes.
Vessels: Peripheral pulses intact in the lower extremity. Musculoskeletal: Inspection revealed a 9 cm fluid collection at the
posterolateral aspect of the right knee around the distal hamstring tendon, which was mildly tender to palpation. No
tenderness otherwise and no joint effusion. Joint ROM: The patient displayed more prominent genu recurvatum on the
right lower extremity as compared to the left. He had painless full knee flexion bilaterally. Special tests: No significant
knee joint effusion. No pain or laxity on varus and valgus stress. Negative Lachman, posterior drawer, and McMurray tests.
Gait: Non-antalgic gait. Neurologic: Strength and sensation were intact throughout bilateral lower extremities.
Differential Diagnosis:
1.
2.
3.
4.
5.
6.
7.
8.
9.
Popliteal cyst (typically medial)
Posterolateral corner injury
Vascular malformation
Meniscal cyst
Iliotibial bursitis
Proximal tibiofibular joint cyst
Synovial chondromatosis
Popliteal vein varix
Hemangioma
Test Results: 1. MRI: Small popliteus muscle strain. Posterolateral corner otherwise negative. Prominent vascular and/or
lymphatic structures with fluid-fluid and/or blood-fluid levels extending from the level of the distal femoral diaphysis to
the level of the joint line and lying deep and superficial to the distal biceps femoris, measuring 9.4 x 3.8 x 4.6 cm. There is
no intraarticular component, no bony invasion and no dilated superficial venous structures. 2. Ultrasound: Venous
malformation at the lateral right knee with connection to the popliteal vein.
Final Diagnosis: Right lateral knee venous malformation.
Treatment: The patient was seen and evaluated by a Vascular Interventional Radiologist and ultimately underwent right
thigh slow flow vascular malformation percutaneous sclerotherapy. Post-procedure, he was instructed to wear a 20-30
mmHg compressive garment for 5 days.
Outcome: The patient will undergo a second sclerotherapy treatment, to optimize outcome.
Patients often require
more than one sclerotherapy session to effectively treat vascular malformations. 80% of patients experience relief from
sclerotherapy. Those that do not may attempt treatment with oral or topical pain medicines.
Follow-Up: After the initial 5 days post-procedure, the patient was instructed to slowly reintegrate back into long
distance running and should continue to wear compression hose daily when upright and active. When faced with a
similar scenario, it is important, as a physician, to consider a range of potential diagnoses in order to select the
appropriate treatment.
Increased Unilateral Vascularity
Authors: Adrian Tabares, MD
Affiliation: UC San Diego Primary Care Sports Medicine La Jolla Family and Sports Medicine, San Diego, CA
History: A 21 year-old right hand dominant male without significant past medical history presents with increased
swelling and vascularity in the right forearm, shoulder and chest. He note that he has always had more definition the right
arm compared to the left and first noticed this while he was in early middle school. More recently, he noticed "new veins"
in the shoulder. He has been working with a strength coach and denies any problems with weight training. He is a catcher
on a college baseball team and also complains of pain in the forearm and shoulder with increased right upper extremity
activity. Pain symptoms first started 4 months ago during baseball season. He denies trauma but has been playing more
games this past season. He endorses a reduction in velocity and distance when throwing the ball. No previous shoulder
problems. No neck or contralateral shoulder symptoms. No surgical history. He has not tried anything for his symptoms
but notes improvement with rest.
Physical Exam: BP 131/74, Pulse 79, Temp 96.7F, Ht 5'10", Wt 178 lb, BMI 25.54
Gen: NAD, comfortable HEENT:
atraumatic, EOMI, MMM Skin: Intact, no rashes, erythema or wounds MSK: Neck, Right Upper Ext., Shoulder No
atrophy or gross deformity No bony or muscular TTP Neck FROM, no TTP, negative Spurling's test Right Shoulder: full
AROM Left Shoulder: full AROM - 5/5 deltoid, supraspinatous, triceps, biceps, internal and external rotation strength
bilaterally Sensation intact in C4-C8 distribution No vascular compromise, cyanosis, 2+ radial pulses bilat Increased
prominence of superficial veins on right chest Positive Roos test after 2 min Negative Adson's test Positive Allen's test
Negative Impingement tests, Speed's, Yergason's, Apprehension and Obrien's tests
Differential Diagnosis:












Spontaneus Upper Ext. DVT (Paget-Schroetter disease)
Brachial plexus injury (Burner/Stinger)
Rotator cuff tear
subacromial impingement
AC Joint Injury
Cervical Disc Radiculopathy
Clavicle injury
Pancoast Tumor
Cubital tunnel syndrome
Thoracic Outlet Syndrome
Carpal Tunnel
Adhesive capsulitis
Test Results: Chest X-ray: Unremarkable, no evidence of acute disease or cervical ribs MRI/MRA and venogram of the
neck, chest and subclavian arteries and veins: Stenosis of the right subclavian vein near its insertion on the right jugular
vein elicited by arm raising. Vein measures 13 mm with arms down and 6 mm with arms raised. No complete occlusion or
thrombus. Remaining structures including right subclavian artery are normal.
Final Diagnosis: Venous Thoracic Outlet Syndrome
Treatment: Conservative management with physical therapy and referral to neurosurgical colleagues. Neurosurgeons
recommended 81 mg Aspirin daily and surgical decompression after baseball season ended or if symptoms worsen.
Outcome: The patient's symptoms improved with physical therapy and rest.
Follow-Up: He plans to participate in his last season of college baseball and return for follow up should his symptoms
return or at the end of baseball season, whichever comes first. At that time he will consider surgical decompression.
An Unusual Neck Mass in an 11 Year Old Female Volleyball Player
Authors: Brent Nathan; Jeffrey M Mjaanes, MD
Affiliation: Midwest Orthopaedics at Rush/Rush University Medical Center, Chicago, IL
History: This athlete complained of right-sided neck pain after serving repeatedly in volleyball practice. She felt she
"pulled a muscle" and had intermittent soreness for the next week or so. Approximately 2 weeks after the initial injury, her
mother went to massage the area and felt a small bump. Over the next two to three weeks, the area increased in size and
swelling and became visibly noticeable. Overall, the neck mass does not bother her unless she turns her head to the left or
reaches her arm over her head. She has had no numbness, tingling or weakness in her arm or hand. She has no
constitutional symptoms.
Physical Exam: Vital signs are within normal limits. The patient is well developed, well appearing and afebrile. Palpation
reveals a non-tender, homogenous, well-circumscribed mass measuring 4 cm x 4 cm in the right supraclavicular region.
There is no regional lymphadenopathy. Overlying skin is neither indurated nor erythematous. Range of motion is normal
in all directions but tightness is felt on lateral flexion. Shoulder and upper extremity exam reveals normal strength and
sensation.
Differential Diagnosis:









Congenital-brachial cleft cyst
Infective/Inflammatory-reactive
lymphadenopathy (viral, bacterial)
lymphadenitis
Neoplastic-lipoma
lymphoma/leukemia
Vascular-hematoma
aneurysm
Metabolic-myositis ossificans
Test Results: MRI with and without contrast of the cervical region was obtained and revealed a clearly demarcated cystic
mass. The fluid consistency is suggestive of a hematoma or an abscess. The surrounding tissue is clearly intact with no
infiltration of the mass or inflammation. Furthermore, the adjacent scalene muscle exhibits the appearance of an
intramuscular strain. There is no regional lymphadenopathy. The remainder of the exam was normal.
Final Diagnosis: Scalene Muscle Strain with hematoma/seroma formation
Treatment: Watchful waiting was recommended with close follow up including weekly measurements both manually and
with ultrasound. Furthermore, the patient was instructed to avoid activities that could lead to contact with the afflicted
area or require extensive use of the arm, shoulder or neck.
Outcome: Upon return visit 2 week later, the fullness in the right supraclavicular fossa had decreased significantly with
associated improvement in the tightness described with neck movements.
Follow-Up: At this time, the athlete has not yet returned to activity but is expected to return to full participation without
limitation.
Not Your Typical Groin Strain
Authors: Aaron Watters, MD; Troy Smurawa, MD
Affiliation: Akron Children's Hospital, Akron, OH
History: 12 yo boy who was referred to the sports medicine clinic by his primary care physician for right sided groin
pain. His pain started two years ago during his baseball season. It gradually worsened over the season. He denied any
acute injuries or trauma. He was seen by his pediatrician and was diagnosed with a groin strain. He was able to continue to
participate in baseball without any limitations. Over the next two years he had intermittent groin pain associated with
activity. Two months prior to presentation, the groin pain became persistent despite not playing in any organized sports at
that time. His parents did not recall any recent injuries. The pain is described at dull and achy to sharp. The pain is rated at
5/10 and is worsened with prolong running. He has tried using an ace wrap and NSAIDs with minimal relief. He has been
able to walk without a limp or antalgic gait. His parents have noted swelling on the inside of his groin.
Physical Exam: Vitals: BP 104/60 Ht: 160 cm Wt: 46.18 kg General: Well appearing in no acute distress
Cardiovascular: Pulses symmetric with regular rate and rhythm Respiratory: Non-labored respirations Gait: slight limp
Inspection: Noticeable right proximal medial thigh prominence without erythema or skin breakdown. Palpation:
Tenderness over anterior hip. Palpable tender mass versus firm swelling over anterior thigh extending into the medial
thigh/groin. ROM: Decrease flexion, internal rotation and external rotation of right hip Strength: Abduction: 5/5,
Adduction: 4/5, Flexion: 4/5 Special Tests: FABER, FADIR, Thomas and Ober were all negative
Differential Diagnosis:
Chronic Adductor Strain
Slipped Capital Femoral Epiphysis
Femoral Acetabular Impingement (FAI)
Legg- Calves-Perthes
Transient Synovitis
Septic arthritis
Juvenile Idiopathic Arthritis
Sports Hernia
Inguinal Hernia
Osteitis Pubis
Primary Bone tumor
Osteomyelitis
Test Results: Pelvis AP and Frog Leg: Mixed lytic and sclerotic lesion in the superior pubic ramus with expansion
extending into the inferior pubic ramus There is cortical disruption on the superior aspect of the superior pubic ramus.
MRI Pelvis: Right soft tissue mass extending from the pelvis into the right upper medial thigh Bony involvement of the
right superior and inferior ischiopubic rami.
Final Diagnosis: Ewing's Sarcoma
Treatment: He was admitted to the pediatric oncology service for further workup of his pelvic mass An incisional
biopsy of his pelvic mass revealed small round blue cell malignancy consistent with Ewing’s Sarcoma Metastatic workup
to date includes a bone scan, chest CT, and whole body PET scan without evidence of distant metastasis He will undergo
six cycles, 11 weeks, of induction chemotherapy followed by local control either with radiation therapy or surgical
resection
Outcome: Still undergoing induction chemotherapy
Follow-Up: He has not returned to sports and is currenlty being followed by pediatric Hem/Onc.
An Unexpected Ankle Sprain
Authors: Julian Perez, MD; Sean Colio, MD; Adam Pourcho, MD
Affiliation: Swedish Sports Medicine/Primary Care Fellowship, Seattle, WA
History: 40 y/o female marathoner presents with acute left ankle inversion sprain 3 days before a planned &frac12; marathon. After
exam and US evaluation, she was advised to RICE and brace her ankle and not continue running or participate in the &frac12; marathon.
Patient returned 8 days later after participating in the &frac12; marathon. She mostly jogged and walked, having her ankle ace wrapped
"tightly". She reported worse pain, swelling, and bruising over the lateral ankle. However, over the anterior ankle joint she had a new
lump. No new sensory or motor symptoms in the foot. Past Medical/Surgical/Family History: Hypothyroidism, Anterior tarsal tunnel
release and dorsal midfoot ganglionectomy of affected foot. ROS: Negative Social History: never used tobacco Allergies: NKDA
Medications: Ibuprofen PRN, levothyroxine
Physical Exam: 1st Exam: Soft tissue swelling and tenderness around distal fibula. Skin intact, no bruising. Mild tenderness over
anterior tib-fib joint. Strength: 5-/5 plantarflexion, dorsiflexion, inversion and eversion limited by pain. Sensation: LT and PP symmetric
to other ankle. Pulses symmetric in both ankles. Pain and mild laxity on talar tilt and anterior drawer testing. Negative squeeze test of
syndesmosis, negative calcaneal thump, negative TAPS test. 2nd Exam: Increased soft tissue swelling and new bruising of lateral ankle.
Increased swelling over the anterior tibiotalar joint and a new painful lump over the TA tendon.
Differential Diagnosis:








Tibiotalar effusion
Ankle fracture
OCD lesion of talus
Anterior ankle tendon tear/rupture
Lateral ankle sprain
Cyst
Hematoma
Aneurysm
Test Results: 1st US: Acute ATFL grade 2 sprain. CFL grade 1 sprain, mild peroneal tenosynovitis, abnormal medial talar dome near
anterior tibialis tendon w/ normal tendon at that level, no joint effusion, AITFL and syndesmotic fibers intact, remainder of ankle normal.
XR: Anterior soft tissue swelling, otherwise normal 2nd US: Stable grade 2 ATFL sprain, stable CFL grade 1 sprain, split anterior tibialis
tendon overlying the tibiotalar joint with tenosynovitis. Dynamic testing shows longitudinal split tendon crossing over clockwise within
the sheath during dorsiflexion and reverses counterclockwise with plantarflexion. Split doesn't appear to be a rupture and the tendon's
integrity indicates it may be chronic. MRI: 1. 1 cm bone bruise posterior medial talus with minimal cortical impaction. 2.
Musculotendinous strain of the Achilles 3. Mild increased fluid in the anterior tibialis tendon sheath with normal TA tendon. 4. Grade 2
sprain ATFL and grade 1 sprain CFL.
Final Diagnosis:
Ankle inversion sprain
Medial talar bone contusion
Medial gastrocnemius musculotendinous junction strain
TA tenosynovitis, chronic longitudinal split tendon, possibly anatomic variant.
Treatment: 1st visit: RICE, weight bearing as tolerated and ankle stabilization lace-up orthosis. Avoid high impact exercises, especially
running. 2nd visit: Immobilize in CAM walker, stop activity, continue RICE, X-rays followed by MRI to confirm integrity of TA and talus
Outcome: 3 weeks in CAM walker to rest talus, ligaments and TA tendon. Swelling resolved. Transitioned into lace up ankle brace and
started rehab.
Follow-Up: After 4 weeks of immobilization and bracing she started wearing hiking boots and walked on flat terrain progressing to 5
miles without pain. Then she began easy 1-3 mile jobs on a treadmill without pain. Two months post injury, she started ankle
strengthening exercises. She still feels weak in the anterior ankle with a non-painful shifting sensation around the TA. She is still
progressing in her return to running program.
With Great Power Comes Great Pain
Authors: Andrew Bryant, BS; Wade Rankin, DO
Affiliation: University of Kentucky, Lexington, KY
History: A 55 y/o right hand dominant white male presented after a 1 day h/o pain to his right wrist after using a power
drill. Pt states that the power drill got hung up and forcefully supinated his wrist. Pt described something "moved toward
the ulnar side of the dorsum of his wrist" and he felt like it did move back. Patient continued with pain to the dorsum of
his hand and wrist in the midline of his wrist. Patient denied any numbness, tingling or weakness of his hand.
Physical Exam: Well appearing, healthy male with minimal swelling to both his dorsal and volar wrist without deformity.
Tenderness to palpation noted to radial styloid, first dorsal compartment, scapholunate interval and ulnar styloid.
Restricted active wrist extension to 80° and active flexion to 70° Positive ulnar grind test and Watson's test with negative
Phalen's, Tinel's of the wrist, and Finkelstein's test.' His neurologic exam and distal pulses were intact.
Differential Diagnosis:













Colles' Fracture
TFCC Tear
Carpal Fracture (Scaphoid/Lunate)
Tendinopathy (Extensor Carpi Ulnaris)
De Quervain's Tenosynovitis
Neuropathy (Ulnar, Median)
Kienbock's Disease
Scapholunate Ligament Tear or Instability
Lunotriquetral Instability
Dorsal Ganglion Cyst
Dorsal Intercalated Segment Instability (DISI)
Volar Intercalated Segment Instability (VISI)
Ulnar Impaction/Impingement Syndrome
Test Results: Xray Wrist - widening of the scapholunate interval consistent with ligamentous injury. Scapholunate angle
is increased without fracture. Noncontrast MRI Wrist - full thickness tear of the scapholunate ligament, with dorsal tilt of
the lunate with volar tilt of the scaphoid, findings compatible with developing dorsal intercalated segmental instability.
Split tear of the extensor carpi ulnaris tendon at the level of the ulnar styloid level, degenerative tearing of TFCC and small
ganglion cyst present at level of the dorsal aspects of the capitate and hamate.
Final Diagnosis:
Scapholunate ligament tear with developing dorsal intercalated segmental instability
Extensor Carpi Ulnaris tendon tear
Degenerative tearing of TFCC
Dorsal ganglion cyst
Treatment: 1. Splinting, NSAIDS, Occupational therapy-did not resolve pain 2. Orthopaedic hand for scapholunate
ligament reconstruction 3. Finger exercises, ice, cast placement 4. Currently, occasional minor pain with movement and
additional reduced grip strength. Able to return to daily activities, with exception of manipulation-intensive activities such
as hammering/kayaking.
Outcome: Currently, occasional minor pain with movement and additional reduced grip strength.
Follow-Up: Able to return to daily activities, with exception of manipulation-intensive activities such as
hammering/kayaking.
Routine Shoulder Tackle in a NCAA Division I Athlete
Authors: Patricia A. Bornhorst, DO; Matthew Mcelroy, DO
Affiliation: Geisinger Health System, Geisinger Orthopedics & Sports Medicine, Danville, PA
History: A 21 year old male NCAA Division One collegiate football player presented with right shoulder pain after performing a routine
tackle during a regular season game Immediate sideline examination was limited only to active shoulder abduction to 30 degrees
Further examination revealed the inability to actively or passively forward flex or rotate the right arm He was immobilized in a sling and
followed up with orthopedics 2 days later With clinical findings and witnessed mechanism of injury a decision to observe for 48 hours
followed Imaging was obtained at the two day follow up appt. after only slight improvement in forward flexion to 90°.
Physical Exam: Office follow-up revealed equivalent active and passive range of motion to 60° forward flexion, 30° shoulder abduction
and zero degrees abduction and external rotation with little improvement from sideline exam The proximal biceps tendon was tender
to palpate with a positive Speed's test No tenderness with palpation or crepitus at the acromioclavicular joint Both Hawkin's and
Neer's tests showed Grade III shoulder impingement Subscapularis motion was intact There was mild scapular dyskinesia with limited
motion however medial winging was not evident Neurovascular function was intact. The shoulder was too acute at the initial office visit
to evaluate for labral injury or instability.
Differential Diagnosis:
Anterior Shoulder Contusion
Labral tear
Rotator Interval tear
Glenohumeral dislocation
Glenoid fracture
Reverse Bankart Lesion
Scapular fracture
Proximal Humeral fracture
Suprascapular nerve injury
Test Results: MRI: Moderate Joint effusion Recent dislocation with Hill-Sachs and small bony Bankart lesion Anteroinferior labral tear
appearing to extend into the posterior labrum Incompletely imaged fracture of scapular body with approximately 1.5cm displacement
Fracture extends into base of coracoid process without extension into the acromion or glenoid Mild acromioclavicular osteoarthritis
with mild diffuse rotator cuff tendinopathy. Xray: Oblique displaced fracture through the neck and blade of the scapula with 1.7cm
displacement greatest inferior. CT scan with 3D reconstruction showed a comminuted posteriorly displaced fracture of the right scapular
neck extending 4.5cm from the inferior glenoid cavity into the lateral scapular body. The distal fragment on Scapular Y view is angulated
23° with posterior translation of the distal tip 2.5cm The glenoid shows a 1.5cm posterior displacement on axial CT Glenopolar angle is
37°. Both the acromioclavicular and glenohumeral joints are intact.
Final Diagnosis: Comminuted displaced oblique fracture of the right scapular neck extending into the lateral scapular body closest to
the A2.3 Glenoid neck fracture type but not completely described by any of the variations of the AO/OTA classification of scapular
fractur
Treatment: Issues regarding insurance coverage for operative care lead the patient to seek another opinion regarding operative versus
non-operative management. With improvement in range of motion three weeks post injury he opted for conservative management with
initial immobilization followed by gentle range of motion after two weeks, progressing to strengthening exercises in week eight after
follow up imaging.
Outcome: The patient elected on his own to incorporate weight training into his rehab program in weeks 3-4 without knowledge of
medical staff On evaluation 6 weeks post injury he denied pain A bony protrusion was palpable in the posterior lateral shoulder in the
location of the distal fracture fragment Active range of motion in abduction, forward flexion and internal rotation were intact With
shoulder abduction and elbow flexion however, external rotation was limited to 80 degrees.
Follow-Up: Function and strength are not at the athlete's baseline and currently not cleared to return to activity A follow up CT of the
right shoulder is pending at time of case submission.
Full Court Pressure
Authors: Carolyn Kienstra, MD; M. Braun, MD; A. Hergenroder, MD; A. Riley, MD; H. Justino, MD
Affiliation: Baylor College of Medicine, Houston, TX
History: A 16 year old male was seen at school for a sports physical and referred to his pediatrician for blood pressure readings in the
180's/90's. He followed up with his pediatrician two days later. Blood pressure readings remained elevated, so he was sent to the ER. He
was an otherwise healthy high school basketball player. Review of systems was negative for headaches, blurry vision, fever, shortness of
breath, chest pain, urinary symptoms, exercise intolerance, or palpitations. He was taking naproxen PRN for recent knee pain. His father
was diagnosed with hypertension at 46 years of age, and his grandmother has hypertension complicated by a stroke in her 50's. He
denied drug use.
Physical Exam: Vital signs: Temperature 36.6°C, Heart rate 63, Respiratory rate 18, Height: 193cm, Weight: 79.5kg Four extremity
blood pressure: -RUE 196/80 mmHg -LUE 191/89 mmHg -RLE 196/120 mmHg -LLE 192/116 mmHg General: well nourished, no
distress HEENT: Pupils equal and reactive, throat without exudate Neck: no JVD Cardiovascular: regular rate, no murmurs Pulmonary:
clear breath sounds bilaterally Abdomen: soft, bowel sounds present, no masses, + abdominal bruit Neurological: alert, CN 2-12 intact,
normal tone Skin: warm, no erythema or edema, no rashes
Differential Diagnosis:








Essential hypertension
Aortic coarctation
Mid Aortic Syndrome
Pheochromocytoma
Drug use
Hyperthyroid
Collagen vascular disease
Renal parenchymal disease
Test Results: He was admitted to the hospital and started on nifedipine. Laboratory Evaluation: Sodium 142, Potassium 4.5, Chloride
101, Carbon Dioxide 28, Glucose 77, BUN 16, Creatinine 0.73, Calcium 9.7, Magnesium 2.1, Phosphorus 5.1 TSH, Free T4, and UA within
normal limits; urine drug screen negative Repeat four extremity blood pressure after starting nifedipine: -RUE 177/73mmHg -RLE
152/91mmHg -LUE 167/85mmHg -LLE 152/92mmHg Chest x-ray: normal Renal ultrasound: no structural abnormalities, normal
parenchyma Renal duplex: aorta caliber change from 1.5cm to 0.77cm below the level of the origin of the superior mesenteric artery
CT angiogram: 5.1cm segment of abdominal aortic narrowing from the origin of the superior mesenteric artery to the inferior
mesenteric artery origin involving bilateral renal artery ostia with severe stenosis of the proximal renal arteries ECG: Normal sinus
rhythm with sinus arrhythmia and left ventricular hypertrophy ECHO: Moderate to severe concentric left ventricular hypertrophy.
Normal biventricular systolic function.
Final Diagnosis: Mid Aortic Syndrome
Treatment: He underwent bilateral renal artery angioplasties in the hospital and was discharged on aspirin, transdermal clonidine, and
long-acting nifedipine.
Outcome: One month later, he continued to have difficult to control hypertension, so abdominal aorta and bilateral renal artery
angioplasties with stenting were performed resulting in improved blood pressure control. He was started on lovenox after the
procedure.
Follow-Up: He was anxious to begin playing basketball, so he was seen by Sports Medicine for athletic clearance. He had lost about
ten pounds since diagnosis, so he was given a walk to jog program and allowed to begin basketball with no contact given his
anticoagulation. He underwent treadmill cardiovascular exercise stress testing according to the Bruce protocol. Results were significant
for a resting blood pressure of 142/70mmHg and pulse of 67 beats per minute which increased to 238/58mmHg and 166 beats per
minute with maximum effort at 13 minutes 30 seconds. He remained asymptomatic during the test, and ECG revealed no evidence of
arrhythmias or ischemia. His maximum systolic blood pressure remained below 250mmHg, the ACSM and American Heart Association
cutoff to stop testing. He was cleared to gradually progress back to participation in basketball without restriction.
An Uncommon Cause of Low Back Pain in an Athlete
Authors: Katherine Guran, MD; Tallat Rizk, MD
Affiliation: University of Toledo College of Medicine, Toledo, OH
History: A 56 year-old male cyclist presented to the Physical Medicine and Rehabilitation clinic with significant pain in
his right groin radiating to his lower back and right hip. For the past 20 years, he has ridden a bicycle for two hours per
day, with no recent change in activity or injury. He began to develop his pain about two months prior to presenting and it
was getting gradually worse. He was initially seen by his primary care physician who ordered a lumbar MRI, which showed
mild disc disease. He then received an epidural injection by Pain Management, which did not provide relief. He was then
seen by Orthopedics who ordered a hip MRI, which showed mild osteoarthritis. Accordingly, he underwent an intraarticular hip injection, which also provided no relief. Despite these treatments he continued to have increasing lower back
pain, as well as groin and right hip pain, which affected his ability to ride, walk, or sleep comfortably.
Physical Exam: Vital signs were within normal limits. Patient is a well-developed, well-nourished male. He had mild
tenderness to palpation of lumbar paraspinal muscles especially around L5. He had no tenderness to palpation of his
bilateral SI joints. He had moderate tenderness to palpation over his symphysis pubis. Neurovascular exam was normal.
Reflexes were equal and symmetric. Strength was 5/5 throughout bilateral lower extremities. Straight leg test was negative
bilaterally. Patrick’s test was positive on the right, negative on the left.
Differential Diagnosis:
(1)
(2)
(3)
(4)
(5)
(6)
(7)
(8)
Lumbar Radiculopathy
Hip Osteoarthritis
Hip Avascular Necrosis
Osteitis Pubis
Groin Muscle Strain
Greater Trochanteric Bursitis
Pubic Stress Fracture
Inguinal Hernia
Test Results: His primary care physician ordered an MRI of the lumbar spine that showed he had mild disc degeneration
at L4-L5 and L5-S1. He was referred to Orthopedics, where he received an MRI of bilateral hips, which showed mild
osteoarthritis of the right hip. Upon being seen in the Physical Medicine and Rehabilitation clinic, a bone scan was
ordered, which showed a high intake area around his symphysis pubis.
Final Diagnosis: Osteitis Pubis
Treatment: After his lumbar MRI, he received two epidural steroid injections by Pain Management. After his hip MRI, he
received a fluoroscopy-guided injection of his right hip. Finally, after the bone scan, he received an in-office steroid
injection to the symphysis pubis.
Outcome: He had no relief after the lumbar epidural and intra-articular hip injections. After the steroid injection to his
symphysis pubis he reported 100% relief of pain at one week.
Follow-Up: He had continued pain relief at three- and six-month follow-up visits. He was able to return to cycling
without further pain. Osteitis Pubis is a relatively common cause of groin pain in running and kicking athletes (soccer,
hockey, rugby, running, etc.). It is seen most commonly in men ages 30 to 50 years. An imbalance of the hip adductors and
the rectus abdominis, as well as repetitive microtrauma during activity, have been shown to cause the pubic symphysis to
become unstable leading to inflammation and Osteitis Pubis. This generally results in pain in the pubic symphysis that may
radiate to the groin, thigh, or abdomen. As in this case, although uncommon, pain may also radiate to the lower back or
hip. This unusual presentation may delay treatment and keep athletes from their desired sport longer than necessary.
Treatment includes rest, oral anti-inflammatory or steroid medications, steroid injections, or possibly surgery if there is no
improvement with conservative treatment options. Therefore, in any patient presenting with a history of possible repetitive
microtrauma and back or hip pain the physician should include Osteitis Pubis in their differential diagnosis to ensure
correct treatment.
Patient with Left Thigh Pain
Authors: Kamalpreet Buttar, MD; Jeffrey Roberts, MD
Affiliation: VCU - Bon Secours St. Francis Primary Care Sports Medicine Fellowship, Midlothian, VA
History: 53 year old male who presented to the office with left thigh pain on and off for past two years with recent
exacerbation for the past two weeks and right knee pain, unrelated. Pain is sharp, in lateral thigh, and intermittently burns
as well. Has been gradually worsening. When he bears any kind of weight the pain is increased. Pain also keeps him up at
night. No acute injury. No smoking history. No numbness and tingling. Denies any recent weight loss, fevers, chills, chest
pain, or SOB. No relief after taking flexeril and percocet for the pain. Family History of arthritis.
Physical Exam: Sensations intact in upper and lower extremities bilaterally. Peripheral pulses intact. Mild tenderness to
palpation of the left femur, especially on the left thigh, especially on the lateral side. No swelling or redness noted. Full
ROM of bilateral knees on flexion and extension. Tender to palpation on medial and lateral joint lines of the knee. Full
dorsi and plantar flexion as well as strength of bilateral ankles. Good quadricep and hamstring strength bilaterally. ROM of
left hip is 60° flexion, 40° abduction. Diminshed internal rotation and external rotation of left hip with pain in the lateral
aspect of the hip. Good ROM in right hip with slightly decreased internal rotation.
Differential Diagnosis:








Osteoarthrtis of the left hip
Hip Fracture
Adductor Muscle or tendon strain
Hip Sprain
Hernia
Groin strain
Left Lateral femoral cutaneous nerve entrapment
Paget's disease
Test Results: Imaging done in the form of X-Ray of the left femur and was found to have cortical thickening extending
two thirds the length of the left femur, beginning at the femoral head, alongside some trabecular thickening. This finding
was considered compatible with Paget's disease. There was also an incomplete fracture along the lateral margin of the
midshaft of the diaphysis.
Final Diagnosis: Paget's disease of the left femur
Treatment: Corticosteroid injection for knee OA. Placed in a non weight bearing brace to prevent fracture progress of
left femur. Referred to orthopedic oncology and surgery (open reduction with internal fixation alongside osteotomy) was
recommended due to the potential of an impending pathologic fracture secondary to anterior lateral bowing and micro
fracture seen on plain films. Patient was sent home with pain medication, crutches and told to take off from work.
Outcome: About a month later the patient was taken to the operating room and had intramedullary nail with proximal
and distal interlocking screws placed in the left femur with osteotomy. Patient tolerated the procedure well, spent one
week post operatively in the hospital, and was discharged to skilled nursing facility for rehabilitation.
Follow-Up: He followed up to the orthopedic clinic 5 days after discharge from the hospital and then consequently was
seen at monthly intervals with good progress and healing. At the time of this submission the patient had now been
ambulating using a cane and having much less pain with no other signs or symptoms of Paget's disease.
Hip Check - A Case of Hip Pain in an Adolescent Ice Hockey Player
Authors: Kaleigh Suhs, DO; John Nickless, MD; S. Chandran, MD
Affiliation: Advocate Christ Medical Center-Family Medicine Residency, Hometown, IL
History: A 14-year-old male hockey player was checked into the boards and fell "awkwardly" hyper-extending and
abducting his right hip as he landed on the ice. The patient had severe pain after the injury. The pain slightly improved
over the course of a week, however he continued to have pain with ambulation as well as pain with long strides while
skating.
Physical Exam:
- Gait was antalgic with patient favoring left leg during stance phase
- Right leg had weakness with hip flexion
- Pain with resisted right hip flexion
- Painless passive range of motion with flexion, internal, and external rotation of right hip
- Non tender over palpation of right ASIS
- Negative straight leg raise
Differential Diagnosis:
1.
2.
3.
Iliopsoas muscle strain vs. tendon sprain vs. illiopsoas tendon rupture
Femur fracture
Hip labral tear
Test Results: MRI Right Hip: tendon tear of iliopsoas off of the lesser trochanter
Final Diagnosis: Right iliopsoas tendon rupture
Treatment: Patient was instructed to restrain from skating for 3 weeks as well as lower body weight lifting, and to limit
off-ice activities in which he hyper-extended his right leg. He was instructed to follow up in clinic in 1 month.
Outcome: Patient was able to return to skating in 1 month and participate in hockey try-outs.
Follow-Up: He is currently playing for his high school junior varsity hockey team and is instructed to follow up on an as
needed basis.
Recurring Headaches in a 30 Year-Old Football Player
Authors: Scott J. Curtis, DO
Affiliation: Atlantic Health System/Morristown Medical Center, Morristown, NJ
History: A 30-year-old semi-professional football player presented to a large community hospital after he developed
three episodes of vomiting, severe headaches and confusion. The patient stated that he has been having recurring
headaches over the past 2 months; however over the past 7-10 days his headaches have become more intense and have
been associated with 1-2 episodes of vomiting daily. The patient suffered what he described as a particularly painful
helmet-to-helmet collision two months prior during his last game of the season. He did not lose consciousness. He
continued to play the rest of the game, and did not seek medical attention until the following day when he was diagnosed
with a concussion. The patient was advised to refrain from physical activity and comply with a return to play protocol. He
did not seek further medical follow-up over the next two months despite intermittent headaches, which were minimallycontrolled by over-the-counter non-steroidal anti-inflammatory drugs. He has a past medical history of one prior
concussion, which occurred approximately one year earlier. He has had surgery to repair a tibia/fibular fracture when he
was 18 years old. The patient denied any other medications, allergies or significant family history. He denied drug or
tobacco use but admitted to drinking 6-10 alcoholic beverages per week. The patient worked as a physical education
teacher.
Physical Exam: Vitals on admission showed a blood pressure of 126/73, pulse 93, respiratory rate 20, temperature 36.6°,
and he was saturating 96% on room air. His cardiovascular, pulmonary and abdominal exams were unremarkable. His
neurological exam at the time of presentation showed a Glasgow Coma Scale of 15. Cranial Nerves 2-12 were grossly
intact. He was awake, alert and oriented x3. His speech was clear and fluent. His tongue was midline. His smile was
symmetric. He moved all extremities. His strength was greater than 4+/5. His sensory and proprioception exam appeared
to be intact. He was able to identify right versus left and identify objects and their uses.
Differential Diagnosis:




Traumatic Brain Injury
Post Concussive Syndrome
Intracranial Hemorrhage (specifically Subdural Hematoma)
Complex Migraine Headaches
Test Results: The patient was taken for a computed tomography of the head in the Emergency Department which
showed a large right isodense subdural hematoma causing mass effect on the right cerebral hemisphere and lateral
ventricle.
Final Diagnosis: Subdural Hematoma
Treatment: The patient was taken emergently to the operating room where he underwent a right-sided burr hole with
drainage of the subdural hematoma.
Outcome: The patient was monitored in the neuro-intensive care unit post-operatively where he continued to improve.
The drain was taken out and he was discharged three days later in stable condition.
Follow-Up: The patient retired from semi-professional football. He was cleared to resume non-contact exercises after he
followed up with his primary care physician one month after discharge. He was symptom free at that time.
This Case Will Make Your Skin Split
Authors: Jacob Erickson, DO; Andrew Peterson, MD
Affiliation: University of Iowa Hospitals and Clinics, Iowa City, IA
History: 16-year old male high-school wrestler had been followed in the Sports Medicine Clinic for chronic pre-patellar
bursitis, recurrent Morel-Lavallee lesions and recurrent MRSA cellulitis. He presented with full thickness thigh laceration
sustained while pivoting on his knee during a wrestling match (a remarkably trivial trauma). The wound measured 15x5 cm
and exposed the fascia of the quadriceps muscles. This impressive injury, combined with his medical history, made our
team suspicious for a connective tissue disorder.
Past Medical History: Anorexia nervosa (in remission); Recurrent bilateral knee Morel-Lavallee lesions; Recurrent left knee
MRSA cellulitis and rare MRSA skin abscesses; Right pre-patellar bursitis status post bursectomy; Easy bruising
Hypothyroidism; Similar injury to right hand with 2 cm split of 3rd interdigital webspace with trivial trauma; Hypogonadism
(Tanner I level sex hormones at age 16); Iron deficiency anemia
Family History: Older brother with elastic skin and history of similar skin problems, otherwise unremarkable
Physical Exam: General: 16-year old boy that appears that of a 14-year old. He has a small frame, but has prominent
musculature and is strikingly lean. He has bruises around both eyes with swollen cheeks.
Skin: 15 cm laceration oriented horizontally arcoss his left anterior thigh. 20 stitches present.
Musculoskeletal: No effusion about left knee. Knee flexion limited to 20°. Has full extension. Strength through limited
range of motion is 5/5 in knee flexion/extension.
Differential Diagnosis:







Chronic Morel-Lavallee lesion leading to skin laceration
Connective tissue disorder
Infection
Malnutrition
Complication related to HGH or testosterone
Feak accident
Simply a result of skin tension from having virtually no subcutaneous fat
Test Results: Dermatology consultation and skin biopsy confirmed the diagnosis of an Ehlers Danlos subtype-Dermatosparaxis.
Final Diagnosis: Dermatosparaxis
Treatment: Wound was closed. Sutures removed at 14 days. Gentle range of motion exercises for his knee began at one
week from initial injury. Light aerobic and light resistance training was incorporated two weeks out. He had consultations
with cardiology, ophthalmology, endocrinology and genetics.
Outcome: He healed uneventfully and gradually resumed full activities.
Follow-Up: Three weeks after his initial injury and one week after progressing through aerobic and resistance training,
he returned to wrestling practice. He will continue to be followed closely in the coming months.
Pronounced Winged Scapula Without History of Trauma or Overuse
Authors: William Emanuele, DO; Amanda Weiss Kelly, MD
Affiliation: Rainbow Babies and Children's Hospital, Cleveland, OH
History: A 23-year-old caucasian male with no significant medical history presented to the sports medicine clinic with
complaints of 3 days of right shoulder pain followed by a protrusion noted in his back with movement of his arms. He
stated that approximately four weeks prior to onset of symptoms he was diagnosed with pharyngitis that was treated with
antibiotics. Although cultures were obtained at an outside facility the patient was not aware of the results. He stated that
forward flexing the arm while in the chair was most concerning to him as he could feel his shoulder blade rub up against
the chair. The patient also endorsed weakness and limited range of motion noted with activities such as ultimate frisbee
and overhead movements. There was no history of trauma or recent surgery.
Physical Exam: Sensation to light touch was intact. Reflexes 2+ and equal bilaterally at biceps, triceps, brachioradialis
and achilles. Cervical range of motion was intact and the spinous processes and paraspinal musculature was non tender to
palpation. Spurling's maneuver was negative bilaterally. Right shoulder exam was significant for scapular winging on the
right. ROM in both flexion and abduction were limited on the right. Muscle strength was 4+ throughout except for 4- in
push off. Signs of impingement, labral pathology or biceps tendon pathology were not present. Left shoulder exam was
within normal limits.
Differential Diagnosis:





Long thoracic nerve palsy
Spinal accessory nerve palsy
Dorsal scapular nerve palsy
C7 Radiculopathy
Toxin Exposure
Test Results: AP/Y and Outlet view xrays of the bilateral shoulders were within normal limits with no signs of mass,
fracture or dislocation.
Final Diagnosis: Long thoracic nerve palsy secondary to post viral nerve inflammation.
Treatment: A conservative approach was initiated geared at working on strengthening the serratus anterior musculature
and supporting structures as this approach has worked in previous cases. This was carried out by having the patient attend
twice weekly formal physical therapy sessions working on scapular stabilization.
Outcome: After three and a half months of biweekly physical therapy the patient has noted significant improvements in
strength and states he no longer experiences pain. Follow up exam at that time showed persistent scapular winging
although significantly improved range of motion and strength from initial presentation. There were no other significant
findings on exam
Follow-Up: The patient is noting improvements in functionality and is now able to participate in sports activity he
previously was having trouble with and is happy with his current status. He is to follow up only if his symptoms return.
Irish Break Dancing: When Lucky Charms are not so Magically Delicious
Authors: Calvin Spellmon, Jr, MD; Christopher McGrew, MD
Affiliation: University of New Mexico, Albuquerque, NM
History: A 6 year old previously healthy female who presents with a c/o right knee/leg pain x 6 weeks. Per mom, she was
at a birthday party in a bouncer when she landed wrong on a jump. She immediately started crying and complaining of
pain in her right lower ext. Over the next several weeks she would vaguely complain of pain off and on. Of note, she is an
Irish dancer and parents explain that since the bouncer accident, she is typically able to make it through practice, but c/o
significant pain the following day. She attends practice 3 times per week, with each session lasting 60-90 minutes each. As
her pain became more severe and frequent, parents scaled back her practice to 2 days per week, then eventually one
session per week without any relief. Parents explain that when she is hurting, she limps and only resting will relieve her
pain. Mom admits that she received some new dance shoes 1 week after the initial injury that aggravates her pain, but no
other aggravating factors. Mom and Dad admitted to swelling with the initial injury but none since, and they deny any
redness, deformity, warmth, numbness/tingling, bruising.
Physical Exam: VS: All VSS Wt: 21.3 kg Ht: 122.8 cm BMI: 14.12 Gen: WD/WN; NAD; polite and energetic HEENT:
NC/AT; no lymphadenopathy CVS: Nl s1/s2 no m/g/r rrr Lungs: CTAB Skin: Warm; clear; cap refill - 2 secs Neuro: CN
II-XII intact M/S: Leg/Knee/Ankle Exam: Inspection: No swelling/edema; no redness, bruising or deformities in her
knees, legs or ankles Palpation: No crepitus; Vague TTP over the posterior aspect over leg over the calf muscle, along the
distal tibia, and just lateral and medial to the distal tibia all on her right. Vague tenderness over her distal fibula. No
tenderness in knees or ankles. ROM - Normal hip, knee and ankle Strength: 5/5 strength bilaterally all muscle groups in
the lower extremity Neurovascular: 2+ pulses in her lower ext bilaterally, sensation intact throughout. Special Test: Knee
exam- normal exam for all special testing. Ankle exam- completely normal including: ant drawer; talar tilt; external
rotation, and squeeze test. Compression test- positive for vague pain throughout her tibia and fibula as described above.
Functional: Can walk, hop, jump and perform her Irish Step Dancing in the clinic without obvious deficits.
Differential Diagnosis:









Growing Pains
Leukemia
Bone Tumor
Fracture
Calf Strain
Osgood-Schlatter Disease
Legg-Calve-Perthes Disease
JIA
Complex Regional Pain Syndrome
Test Results: X-RAYS Right Knee: Normal Right Tib/Fib: Periosteal new bone is noted in the distal fibular diaphysis.
Alignment and Mineralization is normal Right Ankle: Normal
Final Diagnosis: Non displaced stress fracture of the distal fibular diaphysis.
Treatment: Conservative Management PRICE Therapy Rest from dancing activity, and any activity that causes pain
Acetaminophen as needed with food, prn
Outcome: Patient made a full recovery with conservative treatment and rest.
Follow-Up: Patient followed up in 4 weeks with some improvement, but she had continued pain over the healing stress
fracture. Dad admitted she was still very active in PE including basketball. We suggested again no physical activity that
caused pain to that injury, and still no dancing. We suggested a stationary bike and swimming. At her 8-week f/u she was
pain and symptom free, and was allowed to return to full activity without any restrictions. She demonstrated her Irish
Dancing in the office to express her thanks.
Subacute Atraumatic Shoulder Pain in an Active Adult
Authors: Christopher Miller, MD; Grant Jones, MD; James Borchers, MD, MPH
Affiliation: Division of Sports Medicine, Department of Family Medicine, The Ohio State University Wexner
Medical Center, Columbus OH
History: A 42 year old male recreational weight lifter was evaluated for subacute left shoulder pain. The patient described
an acute onset of left arm weakness and dysesthesias about six weeks prior to his initial presentation to our office Shortly
after that, he developed severe left shoulder pain. The pain was limited to the posterior and lateral shoulder and was
persistent, affecting both daily activities and sleep The pain did not improve with non-steroidal anti-inflammatory drugs,
ice, or with decreasing duration and intensity of upper body strength training The pain was worsening with time The
dysesthesias were primarily in the area of the posterior shoulder and in the area overlying the triceps He also complained
of some numbness in the first and second fingers No obvious exacerbating factors for the dysesthesias were identified.
Physical Exam: VS: BP 120/70, HR 95, RR 14, SpO2 98% RA GEN: Well-developed, no apparent distress, answers
questions appropriately. NECK: No tenderness in the cervical spine, full range of motion, Spurling test negative bilaterally.
UPPER EXTREMITIES: Right shoulder exam demonstrates no deformity, no tenderness, full passive and active range of
motion and full strength. Left shoulder exam demonstrates atrophy of the infraspinatus and tenderness to palpation of the
posterior shoulder. No atrophy of the supraspinatus or deltoid. Passive range of motion full in all three planes, but patient
is unable to actively externally rotate the left shoulder. Normal strength with the exception of external rotation. Neer,
Hawkins, cross-arm, and drop-arm tests negative bilaterally. Sensation grossly normal in bilateral arms and hands with the
exception of decreased sensation in the area overlying the left triceps.
Differential Diagnosis:



Suprascapular nerve compression or entrapment at the spinoglenoid notch
Neuralgic amyotrophy (Parsonage-Turner Syndrome)
Suprascapular nerve injury secondary to remote trauma
Test Results: Electrodiagnostic studies of the left upper extremity demonstrated normal spontaneous activity in the left
supraspinatus muscle and abnormal spontaneous activity in the left infraspinatus muscle. Conduction velocities were
reduced across the carpal tunnel on the left. No other abnormalities. Radiographs of the left shoulder, including four
views, demonstrated no osseous abnormality. Magnetic resonance imaging of the left shoulder, without intravenous
contrast, demonstrated a posterior labral tear with an associated paralabral cyst tracking into the spinoglenoid notch.
Final Diagnosis: Entrapment neuropathy of the suprascapular nerve at the spinoglenoid notch secondary to paralabral
cyst; Carpal tunnel syndrome
Treatment: The patient was scheduled for surgery to repair the labrum, excise the paralabral cyst, and resolve the
entrapment neuropathy.
Outcome: As of six weeks after surgery, the patient was progressing appropriately. We are hopeful for continued return
of strength in the infraspinatus and teres minor.
Follow-Up: The patient will undergo post-surgical rehabilitation with the aim of working toward full range of motion and
full strength in the affected shoulder. Return to recreational weight training will occur gradually in the course of his
rehabilitation.
Anterior Left Shoulder Deformity in an Obstacle Race Athlete
Authors: James P. Toldi, DO; James J. Guerra, MD
Affiliation: Florida State University Family Medicine Residency Program, Lee Memorial Hospital, Fort Myers, FL
History: A 35-year-old Caucasian male athlete presented with acute left shoulder pain and weakness 8 days after
participating in an obstacle run. During the run he was crossing an obstacle, which required the athletes to "walk" across
parallel bars using their arms at full extension, and felt a deep "thud". He described the sensation as if somebody kicked
him deep in the joint. He was immediately evaluated by the medic on site for a possible gleno-humeral joint dislocation
and/or vascular compromise, both of which were negative. The athlete did mention that he believed there was asymmetry
in his anterior chest that was not appreciated by the medic. He was then able to complete the race, but noted pain with
arm swinging and weakness with any pushing or climbing motions. Swelling in the left axilla, arm, and chest wall was
almost immediate, and by the initial evening left shoulder ROM was severely limited. Extensive bruising over the proximal
anteriomedial aspect of his left arm developed 36-48 hours after the incident.
Physical Exam: On presentation, there was ecchymosis and edema over the proximal anteriomedial aspect of the left
arm and tenderness to palpation in the bicipital groove and anterior axillary fold regions. There was an obvious visual
asymmetry of the anterior chest musculature that was more prominent with muscle contraction. Left shoulder movements
were limited by pain, but there did not appear to be any shoulder or joint instability or weakness of the rotator cuff
muscles. Apprehension, Empty Can, Speed's, and O'Brien's tests were all grossly negative. Weakness was noted when
comparing shoulder adduction to the contralateral side. A palpable defect was noted in the anterior axillary fold on the
left and was further accentuated by abducting the arm to 90°.
Differential Diagnosis:






Long head of biceps tendon subluxation
Shoulder dislocation
Proximal Humerus fracture
Rotator Cuff tear
Pectoralis Major rupture
Labrum Tear
Test Results: A plain film Xray was performed in the office that did not reveal any bony abnormalities or fractures. An
ultrasound was also performed in the office which was inconclusive, but did show significant edema. An MRI was then
ordered for definitive diagnosis and showed a rupture of the pectoralis major tendon at the humeral insertion with medial
retraction of 3.8 cm. The MRI also showed slight superior subluxation of the long head of the biceps tendon and
prominent fascial edema along the left humerus related to the acute injury. The rotator cuff and glenoid labrum were
intact and there was no significant glenohumeral joint effusion.
Final Diagnosis: Complete rupture of the left pectoralis major tendon at the level of the humeral insertion.
Treatment: Given the patient's young age and desire to continue with high functional demand activities, open surgical
repair of the pectoralis major rupture was performed.
Outcome: The left pectoralis major was successfully reattached to its anatomic insertion on the anterior humerus using
specifically designed suture anchors (Pec Endobuttons, Arthrex, Inc.). Postoperatively, the left anterior axillary fold was
recreated.
Follow-Up: His left upper arm was placed in an immobilizer sling for 6 weeks. Gentle passive range of motion exercises
were started at the 6-week mark and progressed to full active range of motion with isometric strengthening exercises.
Isotonic exercises were started around the 2 month point, with an anticipation of returning to the gym around 4 months.
He will then be allowed to gradually build up to his pre-injury status over the next 6-8 weeks. He is expected to regain full
range of motion and nearly approximate, if not fully reach, his pre-injury capacity.
Right Upper Extremity Pain and Weakness Following Tumor Resection
Authors: Eliana Cardozo, DO; Michael Stubblefield, MD
Affiliation: New York Presbyterian - Hospital of Columbia and Cornell, Department of Rehabilitation and
Regenerative Medicine, New York, NY
History: A 65 year old man with a recent past medical history of right upper lobe Pancoast tumor (consistent with
squamous cell carcinoma) diagnosed nine months ago presents complaining of right shoulder and arm weakness and pain
which began shortly after his resection. His tumor extended from his right apical lung to the adjacent ribs, T2 vertebral
body, right T1,T2 and T3 neural foramens. He was treated with chemotherapy and intensity modulated radiotherapy to the
right upper lobe mass. He then underwent resection of the mass four months ago with posterior fixation at C7-T6,
hemilaminectomy and complete facetectomy at T1-T2, partial facetectomy at T3-4, rhizotomy of T2-T4, dissection of the
T1 nerve root to the junction of the lower trunk of the brachial plexus at C8 and posterolateral thoracotomy, resection of
the tumor and partial T2 vertebral body resection. He reports beginning to feel right posterior shoulder pain, radiating to
his proximal arm about 2 weeks after surgery. This pain was accompanied with weakness of the shoulder and hand. He
also endorsed decreases in his range of motion at the shoulder and right hand paresthesias. His pain was described as
achy and sharp, worsened with movement of his shoulder and improved with opioids prescribed post surgically. He
denied neck pain, bowel or bladder changes. He had been referred to local physical therapy by his surgeon but unable to
fully participate due to pain.
Physical Exam: Physical examination was notable for bilaterally protracted shoulders, tenderness to palpation over the
right medial scapula and subacromial space. The patient had decreased right shoulder range of motion with abduction
and flexion to 45°, external and internal rotation to 20°, along with pain. His strength was within normal limits except for
slightly decreased strength in his right hand intrinsic musculature. Reflexes were diminished in his right upper extremity,
normal otherwise. Sensation was intact except for decreased sensation over incision sites. Impingement signs were
positive in the right shoulder.
Differential Diagnosis:





Lower cervical radiculopathy
Rotator cuff injury
Adhesive capsulitis
Radiation induced myelopathy and/or radiculopathy
Brachial plexopathy
Test Results: MRI of the right shoulder showed tendinosis and partial-thickness articular surface tears of the
supraspinatus and infraspinatus tendons. Surveillance MRI total spine showed post surgical changes, no myelopathy and
stable tumor involvement of the T2 and T3 neural foramen, bilaterally.
Final Diagnosis: The patient presented with a main complaint of weakness and pain of his right shoulder. Given his
extensive history with neurologically involved tumor burden as well as risk of radiation associated injury to neural
structures there was significant concern
Treatment: The patient received a right subacromial bursa corticosteroid injection with 1 ml of 40 mg/ml triamcinolone
and 3 ml of 0.25% bupivacaine. He subsequently underwent physical therapy to focus on shoulder range of motion,
scapular stabilization, strengthening and home exercise program.
Outcome: The patient's pain significantly improved after injection, he was able to participate in physical therapy and
improve his range of motion.
Follow-Up: The patient's right shoulder pain and range of motion continue to improve, he is currently still undergoing
physical therapy.
45 Year Old Male Marathon Runner With Groin Pain
Authors: Kyle Yost, DO; Kara Vormittag, MD
Affiliation: Advocate Lutheran General Hospital, Park Ridge, IL
History: 45 year old male with history of testosterone deficiency who presented with left hip and groin pain for one
month that started while on a marathon training run Patient jumped awkwardly early in the run, but denied pain being
associated with this motion He felt pain at mile ten, and was unable to continue running. Patient presented to his PMD 11
days after initial injury with persistent pain, which ranged in severity from two to eight out of ten on the pain scale He
localized his pain to the lateral hip with radiation to the groin His symptoms were exacerbated by walking and with
weight bearing activity The pain was sharp at times The patient was initially referred to physical therapy, but prior to PT
evaluation he was sent to Sports Medicine due to the severity and persistence of his symptoms.
Physical Exam: Gait: Antalgic Mild tenderness to palpation over the left side of the groin and left greater trochanter
Full painless external and internal rotation bilaterally of the hip join; Neurovascularly intact distally Left leg 102cm from
umbilicus to lateral malleolus Right leg 101 cm from umbilicus to lateral malleolus
Differential Diagnosis:





Stress fracture
femoroacetabular impingement
trochanteric bursitis
gluteus medius strain
adductor muscle strain
Test Results: X-Ray Left Hip: Consistent with Femoroacetabular Impingement MRI Left Hip: Non-displaced fracture of
superior pubic ramus extending into left anterior medial acetabulum without intra-articular extension Non-displaced
healing fracture of inferior pubic ramus Grade 1 obturator externus and gluteal medius tendinosis DEXA: PA Spine T
Score -1.6 and Z score -1.4; Left Femoral Neck T score -1.5 and total hip T score -.7, Z score -.8 and -.4 respectively Right
Femoral Neck T score -1.3 and total hip T score -.4, Z score -1.3 and -.4 respectively Left Forearm T score -2.3 and Z score
-2.0; Ultradistal radial T score -2.1 and z score -1.6
Final Diagnosis: Stress fracture of superior and inferior pubic rami with superior pubic ramus extending to ipsilateral
acetabulum possibly secondarily to low testosterone
Treatment: 8/26/14: Patient seen by Sports Medicine MRI and Physical Therapy ordered at that time; 8/27/14: MRI
results obtained and discussed with patient. Given extent of findings and involvement of acetabulum, the patient was
referred to orthopedic hip specialist; 8/28/14: Patient seen by orthopaedic doctor-No high resolution CT needed as the
stress reaction did not extend into the articular surface. Patient advised to limit activity to pain free movements; 8/29/14:
Patient to have dexa scan for BMD; 9/9/14: Patient referred to endocrinologist for metabolic workup with past medical
history of low testosterone and subsequently abnormal dexa scan 10/1/14: Patient seen by orthopaedic doctor, WBAT
and avoid high-impact activities for 6 weeks, will revaluate for possible repeat of MRI of hip if still having pain
Outcome: 10/1 X-Rays show left superior and inferior pubic ramus fracture healed with CAM femoroacetabular
impingement
Follow-Up: Await follow-up with Sports Medicine, Orthopedics and Endocrinology
Dizziness and Muscle Cramping in a 13 Year Old Basketball Player
Authors: Megan Long, MD; Noe'D. Romo, MD
Affiliation: Albert Einstein College of Medicine-Jacobi
History: Patient Presentation: A 13-year-old boy presents to Pediatric Emergency Department with dizziness and muscle
cramp after playing basketball. History: After 90 minutes of basketball, patient was walking and started to sprint. He then
experienced a feeling of dizziness, chest discomfort, and heart-racing with a sudden "Charlie horse" type pain of both
lower legs anteriorly. He fell to the ground, landing on palms and anterior knees. He was then unable to stand, with 7/10
pain, so EMS was called. No knee instability, clicking, locking, or buckling. No loss of consciousness. No trauma other than
that with the ground. No prior fractures. No history of any connective tissue or bone disorder. No previous shortness of
breath, heart-racing, or loss of consciousness associated with running or sport.
Physical Exam: Vital signs within normal limits. There was bilateral knee swelling and tenderness to firm mass below the
knees. Motor and sensation were intact to bilateral lower extremities, capillary refill brisk, and patient had strong dorsalis
pedis pulses. Patient was unable to straight leg raise either leg and unable to bear weight.
Differential Diagnosis:





Calf muscle spasm
Medial tibial stress syndrome
Fracture
Non-accidental unreported trauma
Cardiac arrhythmia
Test Results: Initial Treatments and Outcomes: Patient received Valium and Morphine for presumed muscle spasm vs.
medial tibial stress syndrome. Labs were drawn, including CK which was 840. Patient received three IV fluid boluses
without improvement. His exam remained unchanged, and he was unable to ambulate, so Orthopaedics was consulted
and radiographs were done. Orthopaedics diagnosed medial tibial stress syndrome.
X-ray: Salter-Harris II fracture of right posterior tibia. Left proximal fibular fracture and possible Salter-Harris II fracture of
posterior tibia.
CT: Right comminuted proximal tibial Salter Harris II fracture, no plateau involvement. Left comminuted proximal tibial
Salter Harris II fracture.
Final Diagnosis: Bilateral proximal tibia displaced epiphyseal fracture
Treatment: Patient was admitted as an inpatient and underwent ORIF of left and right proximal tibial epiphysis without
operative complications. Patient developed intermittent fever on POD #2, initially attributed to atelectasis and postoperative inflammation, but when fevers continued to POD #4, a full work-up was done.
Outcome: CBC was within normal limits, CRP 134, and blood culture, chest x-ray, and viral panel were negative. Work-up
for cause of bone pathology revealed hypocalcemia (8.0) and Vitamin D deficiency (6.0); no osteopenia was noted on
previous radiographs. EKG, echocardiogram, and holter monitoring were all normal. CT bilateral lower extremities showed
no DVT.
Follow-Up: Treatment initiated for vitamin D deficiency with 50,000 units ergocalciferol weekly for an 8-week course,
although patient's vitamin D deficiency was not sufficient to cause pathologic fractures. Patient was discharged home on
POD #10 after no fever for 24 hours and CRP trending downward (122). No source of fever identified with all cultures
negative and no signs of wound infection. Fever was attributed to cytokine release from bone manipulation of bilateral
tibial fractures. Sutures removed in Ortho Clinic 4 weeks post-operatively, knee immobilization continued. Bledsoe brace
and range of motion with weight bearing as tolerated in addition to physical therapy started 5 weeks post-operatively.
Phantom Menace
Authors: Kamyar Nabegh, MD
Affiliation: St. Joseph's Hospital Family Medicine Residency PGY-2, Syracuse NY
History: We present a case of a 20 year old, previously healthy female lacrosse player who suffered from a mysterious
case of AKI while running the mile during daytime training. Her constitution of symptoms began with a general,
nonspecific feeling of weakness which progressed to dizziness, feeling light headed and nauseous. She then began
vomiting and eventually syncopated. She was monitored on the field sidelines by the team doctor for approximately 2
hours before being sent to the local hospital for a complete workup including an echocardiogram, EKG and pertinent labs.
The labs included in her cardiac workup were all normal. She was sent home but returned to the ER the next day with
refractory nausea, vomiting, weakness and headache. Lab tests in the ED showed acute kidney injury with a creatinine level
elevated to over 6 at the time of admission. Her creatinine continued to rise, and by the next morning had reached to over
8. The patient was dialyzed and monitored when her blood pressure elevated and she started complaining of a more
severe headache, blurry vision and became mildly altered. Further evaluation revealed our patient to have Posterior
Reversible Encephalopathy Syndrome (PRES). Her blood pressure was controlled and her symptoms eventually resolved.
Our patient may have indulged in weight loss drugs, under hydration, or even chronic NSAID use, and these may have
contributed to her initial AKI 2/2 to ATN. Collegiate athletes have an enormous amount of pressure on them to perform at
a high level in this era of commercialized college sports. Female athletes, in particular, have a rather unique dynamic of
stressors placed upon them in the collegiate sport dynamic. This project will discuss the pathophysiology of AKI caused
by ATN 2/2 to toxic drugs, and her resulting PRES syndrome. The details of the case will be discussed along with these
topics. Furthermore, a discussion will ensue about how to prevent such disease entities from occurring, and what
directives can be completed from a primary care standpoint to avoid potential serious consequences of not screening
athletes properly for the above risk factors
Physical Exam: On the field during initial syncopal episode: GEN: NAD, Pale, age appropriate female, slow to respond
to questions. HEENT: PERRLA, EOMI, NC/AT. CVS: s1s2, tachycardic, no MRG appreciated. LUNGS: CTA BL, no WRR
appreciated. ABD: No tenderness to palpation. EXTR: No edema in extremities. Neuro: Follows commands sluggishly,
responds to verbal cues, diminished power in the lower and upper extremities bilaterally, no sensory defecits.
Differential Diagnosis:


AKI 2/2 to dehydration
AKI 2/2 to ATN caused by NSAIDS or weight loss medications
Test Results: Vital signs at the time of initial syncopal episode: T-99.7 BP-140/84 HR-102 RR-22.
Labs in the emergency room during second visit:
BMP: Na-141 K-4.9 Cl-104 Bicarb-21 BUN-30 Creatinine-6.4.
CBC: WBC-8.3 HGB-14 HCT-43 PLT-320. Liver Enzymes: WNL. Troponin level: WNL.
MRI after admission: Edema in the occipital and parietal lobes bilaterally.
Final Diagnosis: Posterior Reversible Encephalopathy Syndrome, also known as Reversible Posterior
Leukoencephalopathy Syndrome.
Treatment: Blood pressure control and supportive treatment.
Outcome: Our patient made a full recovery within a week.
Follow-Up: She was able to return to full activity within 4 weeks with no neurological defecits.
Traumatic Shoulder Pain in a High School Football Player
Authors: Alan A. Zakaria, DO, MS
Affiliation: William Beaumont Health System Royal Oak, Michigan University of Michigan Health System Ann
Arbor, Michigan
History: A 16 year old male high school football player presents to the office three weeks after injuring his right shoulder
He fell and hit the ground with his shoulder after being tackled in a football game He was taken to the emergency room
and had negative x-rays of his shoulder and was placed in an arm sling and told to follow up with a sports medicine
physician He complains of pain over his anterior right shoulder and decreased range of motion but denies any swelling
other than over his right clavicle medially He denies any catching or locking in his shoulder
Physical Exam: General: NAD HEENT: NC/AT Musculoskeletal: Normal range of motion in all planes of motion
except for abduction of his right shoulder, + Speeds test on the right, + OBrien's test on the right, + sulcus sign bilaterally,
+ scapular dyskinesis on the right, + swelling over distal right clavicle with no tenderness to palpation, + tenderness to
palpation over right coracoid process, 4/5 strength with resisted abduction, forward flexion and external rotation of his
right shoulder, normal shoulder exam otherwise
Differential Diagnosis:
Coracoid process fracture
Glenoid Labrum tear
Short Head of the Biceps
tear/tendinopathy
Anterior dislocation of sternoclavicular joint
Distal clavicle fracture
Test Results: Right shoulder and right clavicle x-ray in ER: No acute fracture or dislocation evident Right clavicle x-ray
(performed as an outpatient 3 weeks later): No acute bony abnormality CT sternoclavicular joint without IV contrast
(performed 5 weeks after injury): Displaced healing intra-articular fracture of the medial end of the right clavicle with the
anterior fragment dislocated anteriorly from the manubrium. The smaller posterior fragment remains articulated.
Final Diagnosis: Anterior right sternoclavicular dislocation with distal clavicle fracture
Treatment: Patient was in a sling for four weeks and then started range of motion exercises with minimal to no pain.
Patient progressed to physical therapy with strengthening, range of motion, and functional rehabilitation. Eight weeks
after his injury, the patient exhibited full strength and range of motion in his right shoulder.
Outcome: After eight weeks, patient was able to return to non-contact activity. He was able to throw a football and catch
a football with no difficulty.
Follow-Up: He is to follow up at twelve weeks post-injury to re-assess for his ability to participate in full contact activities
Left Hip Pain in a High School Football Player
Authors: Sally Hinman, MD, PhD
Affiliation: University of Florida, Gainesville, FL
History: An 18-year-old previously healthy male high school running back presented to the emergency department via
ambulance after sustaining a left-sided leg tackle in his football game. Patient reported a valgus directed force at the
lateral aspect of the femur, feeling a pop, an inability to ambulate, and severe left leg pain without paresthesia. He was
evaluated in the emergency department by orthopedic surgery and discharged home after a radiograph of his femur was
read as normal without dislocation or fracture.
Physical Exam:
Vitals: Blood Pressure 143/64, Heart Rate 71, Respiratory Rate 18, Height 1.702 m, Weight 88.5 kg
Constitutional: He appears well-developed, well-nourished, and in no distress.
Head: Normocephalic and atraumatic.
Eyes: Conjunctivae and external ocular movements are normal. Pupils are equal, round and reactive to light.
Neck: Normal range of motion.
Cardiovascular: Normal rate.
Pulmonary/Chest: Normal effort and breath sounds.
Abdominal: Soft, non-distended, non-tender. Bowel sounds are normal.
Musculoskeletal: Normal lower extremity range of motion. He is tender to palpation of left leg, hamstring and left upper
leg. He exhibits no edema.
Neurological: He is alert and oriented to person, place, and time.
Skin: Skin is warm and dry.
Differential Diagnosis:
1.
2.
3.
4.
5.
6.
7.
Bone contusion
Hamstring strain
Hip apophyseal avulsion fracture
Hip apophysitis
Slipped capital femoral epiphysis
Femoral stress fracture
Greater trochanter fracture
Test Results: AP/Lateral Radiograph of Left Femur: Initial read was normal without fracture or dislocation. Over-read
indicated non-displaced fracture of the greater trochanter.
Final Diagnosis: Non-Displaced, Left Greater Trochanter Fracture
Treatment: The patient was instructed to be non-weight-bearing on crutches for six weeks.
Outcome: The patient was seen in sports medicine clinic for emergency department follow-up. He progressed to toe
touch weight-bearing at his two week follow-up appointment after the radiograph showed callous formation and healing,
non-displaced greater trochanter fracture.
Follow-Up: The patient will start physical therapy after six weeks of non-weight-bearing assuming his fracture continues
to heal.
An Unusual Cause of Headache and Fatigue in a Division 1 Collegiate Hockey Player
Authors: Ali Y. Makki, MD; Rajiv Jain, MD; John Leddy, MD; Koki Takano, ATC
Affiliation: UBMD Orthopaedics and Sports Medicine, University at Buffalo, Buffalo, NY
History: 21 year old male college hockey left winger who presented to the Canisius College athletic training room with complaints of
recurrent headache and generalized fatigue. He developed a headache two weeks prior after getting checked in the chest. A concussion
was ruled out by physical examination, SCAT 3, ImPACT and Buffalo treadmill testing. His headache improved several days after the
injury but then returned with repeat non-head contact during play. He was then diagnosed with cervical and postural dysfunction for
which he attended formal neck and vestibular PT. He had a history of similar episodes and fatigue starting in late childhood. Evaluations
for anemia, iron deficiency and other causes of fatigue had been normal. The headache episodes were associated with lightheadedness,
"spotty vision", fatigue, decreased energy, parasthesias and extreme generalized weakness. Alcohol consumption exacerbated the
episodes. Initially his symptoms were consistently related to exertion but then became sporadic and unrelated to hockey. Sometimes he
could hardly get through a game yet play "the best game of his life" in the next game. He would sometimes sleep 12-14 hours a day
during hockey season. He eventually had to leave the team because of these symptoms. On review of systems, he gave a history of
multiple reported concussions. He also described numerous photosensitive skin lesions consisting of pus-filled welts that scarred after
draining and healing. These had been diagnosed and treated as acne.
Physical Exam: Vitals: Normal. General Appearance: Alert and oriented, normal appearing 21 yo male. Appropriate mood. Ht: 6' 2" Wt:
201 lbs. BMI: 25.8
Pertinent positives and negatives: No adenopathy.
Abdomen: No hepatosplenomegaly.
Neurological: Alert and oriented x3. No focal neurological deficit. CN II-XII normal. Strength 5/5 in all extremities. Sensation intact. Deep
tendon reflexes 2+ in all extremities. Tandem gait normal.
Skin: Multiple scattered dark scarred lesions predominantly at previous injury sites including the left outer thigh and left forearm.
Differential Diagnosis:
1.
2.
3.
4.
5.
6.
Post-traumatic headache
Migraine headache
Post-concussion syndrome
Obstructive sleep apnea
Anemia
Hypothyroidism
Test Results: Initial Labs: Na 139; K 5.1; Chloride 103; Ca 10.0; Glucose 91; Creatinine 0.89; BUN 13; Alkaline Phosphatase 62; AST 17;
ALT 17; Total Protein 7.2; WBC 3.9; Hemoglobin 4.95; Hematocrit 44.4; Platelets 172; MCV 89.7; MCHC 34.5; TSH 1.67; Total T4 6.0
Nocturnal Polysomnogram: No obstructive sleep apnea. All stages of sleep normal. No abnormal leg movements.
MRI Brain with and without contrast: No acute intracranial process. Single nonspecific T2 weighted focus in left subcortical white matter
sometimes seen with history of migraines. Grandmother diagnosed with a rare form of porphyria eight months after initial patient
encounter so patient was also tested.
Genetic Testing: Positive missense mutation, R168H, in one protoporphyrinogen oxidase (PPOX) allele that established the diagnosis of
Variegate Porphyria. Mutation positive in all three siblings, his mother and all four of his maternal aunts.
Final Diagnosis: Variegate Porphyria
Treatment: Avoidance of specific triggers, including alcohol and medications such as sulfonamides, antifungals anticonvulsants and
barbiturates. Advised to avoid extreme exertion given exacerbation of symptoms.
Outcome: The patient is doing very well and avoids triggers such as alcohol and intense physical stress from contact sports. He has not
required any hospitalizations for acute porphyria attacks.
Follow-Up: The patient stopped playing competitive hockey about 6 months ago because of headaches and excessive fatigue, months
prior to his diagnosis. He will be attending a national porphyria conference with his family to learn more about the condition.
Bilateral Shoulder Pain in a High School Football Player
Authors: Brian David Kurose, MD; Michael K. Fong, MD
Affiliation: Kaiser Permanente Sports Medicine, Los Angeles, CA
History: A 17-year old senior right-hand dominant high school football defensive end was referred to Sports Medicine
after sustaining a non-traumatic left shoulder contusion during a game late in season. He was evaluated by the sideline
physician whose exam was notable for only AC-joint tenderness. He then revealed having bilateral shoulder soreness and
instability throughout his four years of high school football as result of unreported injuries including subluxing his left
shoulder 6 different times without a complete dislocation during his senior season.
Physical Exam: Notable for bilateral anterior slouched shoulders in resting position. Reported discomfort with left
shoulder apprehension testing only and pain when the shoulder was abducted in external rotation. No tenderness over
the AC-joint, bicipital groove or greater tuberosity at the rotator cuff insertion. Full range of motion without scapular
winging; normal rotator cuff muscle strength. Neurovascularly intact.
Differential Diagnosis: 1. Shoulder subluxation 2. Intra-articular pathology 3. Fracture (humeral head)
Test Results:
1. Bilateral shoulder X-Rays: Normal
2. MSK shoulder ultrasound:

Left shoulder folding labrum with external rotation. Rotator cuff muscles intact
3. Left shoulder MRI Arthrogram:



Chronic posterior Bankart fracture with minimal displacement and associated labral tear
Several intra-articular loose bodies
Subacute-chronic Hill-Sachs lesion of humeral head
4. Left shoulder CT (better assess reverse Bankart lesion):

Chronic mildly displaced fracture of the posterior glenoid rim extending from superior to inferior with maximal
fracture gap of approximately 5 mm
5. MRI Right Shoulder:

Chronic posterior Bankart Fracture with associated posterior labral tear extending to the superior labrum
Final Diagnosis: Left shoulder reverse Bankart Fracture with associated labral tear and Hill-Sachs lesion Right shoulder
reverse Bankart Fracture with associated posterior labral tear
Treatment: Offered left shoulder arthroscopy with loose body removal and posterior bony Bankart
repair/capsulorrhaphy
Outcome: Patient declined surgical intervention as symptoms not bothersome with daily activities and completion of
football career Follow-up as needed. To date not seen in 9 months
Follow-Up: Completed football season Competed in senior season basketball without problem
Weight Loss in a Collegiate Rower
Authors: Erin Hammer, MD; Kathleen Carr, MD
Affiliation: University of Wisconsin, Madison, WI
History: An 18 year-old collegiate lightweight rower with a history of generalized anxiety disorder and depression presented with
nausea, vomiting and weight loss. Her symptoms began one year prior with diarrhea associated with stressful swim competitions. Her
family physician diagnosed her with irritable bowel syndrome, but her symptoms eventually progressed to early satiety with vomiting
after most meals. Within six months of starting college, she had lost over 30 pounds. She denied dysphagia but endorsed fatigue, cold
intolerance and constipation. Her medications include citalopram 20 mg daily and alprazolam as needed.
Physical Exam: Blood Pressure 105/75, Heart Rate 48, Respiratory Rate 16, Oxygen saturation 98% Height: 1.753 m (5&amp;' 9")
Weight: 104.2 pounds BMI: 15.4 kg/m2 General: Alert, oriented HEENT: Moist mucous membranes, normal dentition Chest: Clear to
auscultation Cardiovascular: Bradycardia, normal rhythm with no murmurs, rubs or gallops Abdomen: Scaphoid abdomen, bowel
sounds active, no tenderness or masses Extremities: No edema. Lanugo noted on bilateral forearms.
Differential Diagnosis:






Eating disorder
Gastroparesis
gastrointestinal obstruction
eosinophilic gastroenteritis
gastroesophageal reflux
variegate porphyria
Test Results: Laboratory: Sodium 139 mmol/L, Potassium 2.5 mmol/L, Chloride 95 mmol/L, Glucose 105 mg/dL, BUN 21 mg/dL,
Creatinine 0.95 mg/dL, Calcium 9.7 mg/dL, Magnesium 2.0 mg/dL EKG: Sinus bradycardia, U-waves noted in precordial leads, QTc 551
ms Imaging: KUB: Bowel gas pattern may represent ileus or partial small obstruction.Upper GI with small bowel follow-through:
Decreased transit of contrast in the supine position at the level of the third portion of the duodenum, which resolved on left lateral
decubitus positioning is consistent with superior mesenteric artery syndrome.MRA abdomen: Abnormally decreased superior mesenteric
artery angle compatible with superior mesenteric artery syndrome. Associated focal effacement of the duodenum and narrowing of the
crossing left renal vein without thrombosis or asymmetric renal enhancement.
Final Diagnosis: Superior mesenteric artery (SMA) syndrome due to anorexia nervosa
Treatment: In the hospital setting, the patient's electrolytes were repleted. She was monitored with cardiac telemetry until her
conduction abnormalities resolved. Emesis of the majority of her meals continued with modest relief with anti-emetic medication. The
consulting gastroenterologist thought the patient's dramatic weight loss was due to anorexia and subsequent loss of the SMA fat pad
resulting in SMA syndrome. A naso-jejunal tube was attempted, but the patient did not tolerate it. Interventional radiology then placed
a gastro-jejunal tube and tube feeds were started. She was monitored for refeeding syndrome. She gained 10 pounds over the 18 days
of her admission. Psychiatry was consulted and recommended continued outpatient therapy for anorexia but the patient continued to
deny she had an eating disorder.
Outcome: The patient was discharged to her family home with continued tube feeds. She decided to withdraw from college for the
spring semester and sought outpatient nutritional and psychiatric care.
Follow-Up: The patient returned to college the next fall and presented for a pre-participation physical for crew. She denied nausea,
vomiting or dietary restrictions. She reported improved mood. Her weight was 118 pounds. She agreed to achieve and maintain a
weight of 127 pounds and to have regular visits with a psychologist, nutritionist and team physician. She gained 7 pounds over the next
two weeks but urinalysis revealed that her urine was dilute. While she had not been permitted to train with the crew team, she had been
training for a half marathon and lifting weights. She was advised that water loading was not an appropriate method to increase her
weight and she was discouraged from training for the half marathon. Over the next month, she maintained her weight at 125 pounds
and was permitted to gradually return to training for crew.
Muscle Wasting in a College-Aged Weight Lifter
Authors: Kendra McCamey, MD
Affiliation: The Ohio State University, Columbus, OH
History: 22 year old male weight lifter presents with a 6 week history of right shoulder weakness. The weakness started
suddenly 30 minutes into an upper body work-out. He didn't have pain, numbness, tingling or feel a pop. Patient stopped
lifting. For the next 2 days, his right arm felt "dead", and he couldn't elevate his arm. After a few days, his shoulder felt a
little stronger, but still felt like it wasn't working like it should He saw his PCP and was started in physical therapy. After 3
weeks of PT, the patient noticed a visible loss in tone of his right posterior deltoid and felt like he could not activate his
posterior deltoid. Patient still feels these symptoms Patient states that he had a similar episode last year in right bicep.
One day when lifting he noticed his right bicep was weaker. Soon after that he noticed it wouldn't fire and was smaller.
This wasn't as severe as current problem and resolved with rest. Denies recent illness or recent immunizations. No neck
pain, back pain or shoulder pain. No fever, chills, or rashes. No joint pain. No other neurologic symptoms. No significant
personal or family history. Only medication is fish oil. Denies supplement use, illicit drugs and smoking.
Physical Exam: Neck/Spine: normal Shoulders: On inspection, visible muscle wasting of right deltoid compared to left.
Rest of muscles are very well developed. ROM: Full active ROM with FF and abduction bilaterally, however with right is
recruiting many accessory muscles with movement When minimizing accessory muscle recruitment, only can actively
abduct to 100 Normal passive motion Posterior internal rotation to L5 bilaterally Mild left side winging and significant
right side winging with motion. Palpation: No tenderness anywhere in neck, upper back, shoulder or arm. Strength: FF:
5/5 left, 4+/5 right; external rotation 5-/5 left, 4/5 right; internal rotation 5/5 bilaterally Deltoid strength is 3/5 with arm
directly at side on right, 5/5 left When arm passively abducted above 90 on right, patient is unable to keep arm in this
position without assistance. Normal strength with elbow flexion/extension, wrist flexion/extension and intrinsic muscles of
fingers. Subjectively feels weaker with elbow flexion and extension. 2+ radial and ulnar pulses. Special tests: Negative
Roos costoclavicular bracing, Wright and Adson test. Negative spurling. Normal sensation to light touch bilateral upper
extremities. Normal bicep/tricep reflexes. Normal LE exam.
Differential Diagnosis:
Peripheral neuropathy
Cervical neuropathy
Brachial plexopathy
Trauma
Space occupying lesion
Test Results: Xrays shoulder/cspine:negative, no cervical rib EMG: Diffuse sensory motor demyelinating
polyneuropathy Superimposed bilateral ulnar neuropathy at the elbows. Can not exclude bilateral medial nerve
entrapments. Findings are consistent with a possible hereditary sensory motor neuropathy with susceptibility to
entrapment neuropathies.
Final Diagnosis: Hereditary neuropathy with pressure palsies (HNPP)
Treatment: Continue PT. Patient was referred to neurology. Patient's father was present at the neurology visit. A brief
history provided by the patient's father suggested that his father actually has the disorder as well and simply was unaware
of it. His paternal cousin also had an incident that would be consistent with what could be seen in HNPP. Neurology
recommended to avoid situations that could trigger a focal nerve palsy. Also recommended genetic confirmation. Patient
agreed but never followed through on this.
Outcome: Patient finished PT and regained strength and muscle tone in his posterior bicep. Was able to return to full
activity without problems.
Follow-Up: Return to full activity. Avoid situations that could trigger a focal nerve palsy.
Bilateral Medial Plantar Foot Pain in a Female Runner
Authors: Daniel Lueders, MD; Cara Prideaux, MD
Affiliation: Mayo Clinic, Rochester, MN
History: A 28-year-old female with no significant past medical history presented to Sports Medicine clinic for evaluation of bilateral foot
pain of five years duration. Pain was rated as 7/10 at worst and was worse with running. No known injury or initial inciting event While
training for a 10k race she experienced progressive worsening of chronic bilateral plantar foot pain, which would begin at the lateral
calcaneus and spread across the plantar foot in an anteromedial distribution. Pain would begin reliably 1 mile into a run, worsen with
continued running, and subside within several hours upon stopping running. Numbness and tingling was noted in toes two through five
No weakness, swelling, color changes, or changes in temperature in the feet. Treatment prior to presentation had consisted of trialing
multiple different shoes including a minimalist shoe, treatment for plantar fasciitis with icing and stretching, and ibuprofen, without
significant benefit. Resting from running for a period of three weeks provided improvement in rest pain but symptoms worsened upon
returning to running.
Physical Exam: Inspection: No gross abnormalities or deformities Palpation: Mild right plantar metatarsal and intermetatarsal region
tenderness, none on left. No plantar fascia tenderness. Joint ROM: Full, pain-free range of motion throughout. Special Tests: No pain
with stretching the plantar aspect of feet or metatarsal squeeze. Repeated toe raises produced mild pain. Gait: Normal non-antalgic
gait. Sensation: Sensation intact to light touch throughout the feet. Negative Tinel's sign over posterior malleoli. Strength: Normal
throughout bilateral feet
Differential Diagnosis:
Tarsal tunnel syndrome
Nerve entrapment
Exertional compartment syndrome in the feet
Plantar fasciitis
Tendinopathy
Test Results: MRI Right Foot: Mild thickening and T2 hyperintensity involving the medial plantar nerve (MPN) medial to the crossing
of the flexor hallucis longus (FHL) and flexor digitorum longus (FDL) tendons at the "knot of Henry" suggestive of MPN entrapment. No
stress reaction or stress fracture. Benign localized bone marrow edema within the lateral aspect of the first metatarsal head of uncertain
etiology and clinical significance. Slightly increased bursal fluid between the first and second and second and third metatarsal heads
suggesting possible intermetatarsal bursitis. Moderate soft tissue edema within the sinus Tarsi. MRI Left Foot: Slight thickening and T2
hyperintensity involving the MPN lateral to the knot of Henry concerning for possible MPN entrapment Increased fluid about the
peroneus longus tendon at the level of the midfoot and proximal forefoot suggesting tenosynovitis. Soft tissue edema within the sinus
Tarsi could suggest Sinus Tarsi syndrome Tiny focus of edema within the inferior calcaneal tuberosity, but unlikely to reflect significant
plantar fasciitis. Small fluid collection deep to the distal anterior tibialis tendon, possibly representing a ganglion cyst or a small bursal
fluid collection.
Final Diagnosis: Bilateral medial plantar nerve entrapment about the knot of Henry, "Jogger's Foot"
Treatment: Sonographically guided bilateral Knot of Henry peritendinous/medial plantar perineural anesthetic and corticosteroid
injections
Outcome: Initial anesthesia in the distribution of the medial plantar nerve bilaterally. Incomplete long-term improvement of pain.
Follow-Up: The patient was lost to follow up because of superseding medical developments. Advanced imaging, pain location in the
medial aspect of the arch, and initial anesthesia provided strong support for bilateral medial plantar nerve entrapment at the Knot of
Henry but, curiously, corticosteroid injections into these regions provided no complete long-term symptom improvement. The patient is
hopeful to return to Sports Medicine clinic in the future for further diagnostic evaluation and management, including a formal running
analysis, correction of underlying biomechanical errors, and a physical therapy program.
Left Anterior Shin Pain in a Recreational Runner
Authors: James Crownover, MD; Rebecca Morgan, MD
Affiliation: University of Tennessee Medical Center - Knoxville, Department of Family Medicine, Knoxville, TN
History: A 20 year old female recreational runner presented with the chief complaint of left medial shin pain The pain
had been present for 3 years without an inciting event and it was intermittently exacerbated with periods of more
consistent running The pain was located anteriorly around the proximal third of her lower leg and typically had improved
with decreased amounts of weight bearing activity; however she was now experiencing pain despite not running much for
several months She denies nocturnal pain.
Physical Exam: There was no swelling, bruising or erythema She ambulates without an antalgic gait pattern She had
exquisite tenderness to palpation over the anterior tibia 2-3 cm distal to the tibial tuberosity without any soft tissue pain
surrounding the area There was not any tenderness to percussion of the heel. Range of motion was normal about the
knee and ankle.
Differential Diagnosis:
1.
2.
3.
4.
5.
Medial tibial stress syndrome
Tibial stress fracture
Bone or soft tissue mass
Myositis Ossificans
Tibialis anterior strain (chronic)
Test Results:
1.
2.
3.
Tibial fibular xrays demonstrated a 1cm lucency with slight periosteal elevation in the anterior proximal tibia
evident on the lateral view.
Lower leg MRI demonstrated a smoothly marginated 13 x 7 x 4mm isointense T1, homogenous bright T2 intensely
enhancing soft tissue nodule along the anterolateral surface of mid-tibia. There was suggestion of an enhancing
hyperintense tail proximal and distal to this soft tissue nodule. There was an associated sharply defined focal area
of partial thickness lytic erosion of the underlying cortex measuring 11 x 5 x 2mm.
CT scan of the lower leg demonstrated a 12 x 6 x 4mm ill-defined and slightly expansile osteolytic lesion in the
anterior cortex of the proximal tibial diaphysis with cortical breakthrough and extraosseous extension into the
anterior soft tissues. There was no periosteal reaction or pathologic fracture identified.
Final Diagnosis:
1.
2.
3.
4.
Adamantinoma
Osteoid osteoma
Osteosarcoma
Non-Ossifying Fibroma
Treatment: She is currently managing her pain with relative rest, ice and ibuprofen as needed
Outcome: The patient was referred to a musculoskeletal oncologist for a biopsy and potential resection of the lesion.
Follow-Up: Her biopsy will occur soon and the management strategy will depend upon the specific pathology results. In
the meantime she is continuing to avoid high impact activities including running, and managing her symptoms with
conservative measures. Follow-up will occur after the results are obtained.
Would Someone Give Me a Hand...With This Hand?
Authors: Timothy Thomsen, MD; Christopher P. Hogrefe, MD
Affiliation: University of Iowa Hospitals and Clinics, Department of Emergency Medicine, Iowa City, IA
History: An 82 year-old right-handed female presented to the Emergency Department (ED) with a chief complaint of left wrist pain
and swelling that began after tripping on a curb 90 minutes prior to arrival. She described falling forward onto her outstretched left
hand. Her sharp pain was moderate, localized to her lateral wrist, and was exacerbated with movement. Shortly after her arrival in the ED
she developed a burning/numbing sensation in her thumb, index, and middle fingers. She denied any other injuries, including loss of
consciousness and/or head injury. She does not take any blood thinners or aspirin. Past Medical/Surgical History: Urinary Incontinence
Hypothyroidism Dyslipidemia Bladder surgery Social History: No tobacco, alcohol, or drugs. Family History: No family history of
osteoporosis or bleeding disorders.
Physical Exam: Temp: 36.9°C; BP: 184/73; HR: 73; RR: 14; SpO2: 97%; Ht: 5'4"; Wt: 81 kg General: Awake, alert, interactive HEENT:
Atraumatic; PERRL Neck: Her ROM is full and non-painful; no worsening paresthesias or pain with Spurling's test. Cardiac: RRR; no
murmurs. Lungs: Clear bilaterally. Musculoskeletal (Left upper extremity): Wrist and hand are markedly swollen and dorsal wrist
appears deformed. The fingers are sausage-like with a subtle purple hue. The wrist is very tender, particularly along the radial border.
The distal forearm and dorsum of left hand are tense. Mid- and proximal forearm are soft and non-tender. Unable to palpate radial
pulse; capillary refill is ~2-3 seconds. Wrist ROM is deferred secondary to pain. She is able to abduct her fingers and make an OK sign.
She has difficulty with thumb abduction/extension. She endorses decreased sensitivity to light touch on palmar surface of thumb, index,
long, and ring fingers. Joint laxity was not assessed secondary to pain.
Differential Diagnosis:
Perilunate dislocation
Lunate dislocation
Distal radius fracture/dislocation
Metacarpal fracture
Carpal bone fracture
Scaphoid dislocation/fracture
Median nerve contusion
Acute carpal tunnel syndrome
Anomalous lumbricals
Hematoma formation
Acute compartment syndrome
Traumatic aneurysm
Median artery dissection/thrombosis/rupture
Arteriovenous malformation
Test Results: WBC: 9.6 Hb/HCT 10.0/31 Platelets: 228 Na: 139 K: 4.1 Cl: 111 CO2: 21 BUN: 17 Creatinine: 0.76
Left wrist xray: Intra-articular radial styloid fracture with 6 mm of displacement.
INR: 1.0 PTT: 23
Final Diagnosis: Acute carpal tunnel syndrome secondary to an intra-articular radial styloid fracture resulting in a radial artery
transection.
Treatment: The patient was noted to have enlarging ecchymosis on her volar wrist as her workup progressed. An emergent ED
Orthopaedics consult was obtained due to the paresthesias, weakness, and expanding ecchymosis. She went to the operating room
emergently with the hand surgeon where she had release of the carpal tunnel and distal antebrachial cutaneous fascia. She had
extensive bulging hematoma in the carpal canal and the volar forearm, which was evacuated. Further exploration revealed an intact,
uninjured median nerve and a transected radial artery which was ligated after collateral ulnar flow was confirmed. ORIF of the radial
styloid with two K wires was performed and a thumb spica splint applied.
Outcome: Post-operatively the patient complained of lightheadedness. Her hemoglobin recheck was 8.0 and she received 1 unit of
packed red blood cells. Her numbness in the thumb, index, long, and ring fingers resolved shortly after surgery. She was discharged
home on post-operative day one.
Follow-Up: Her recovery was unremarkable and uncomplicated. She regained full function of her left hand.
Unilateral Chest Wall Deformity and Brachydactly in a Collegiate Football Player
Authors: Bryan Hess, DO; Sara Neal, MD; Timothy Draper, DO; Karl Fields, MD
Affiliation: Moses Cone Family Medicine/Sports Medicine, Greensboro, NC
History: During pre-participation physical for freshman athletes at a local college, an 18 y/o freshman football defensive
back was examined. He had a severe right unilateral chest wall deformity with asymmetry along with evidence of
correction of a brachysyndactly of the right hand. Otherwise, the patient was a healthy male with no known previous past
medical history. He denied any previous injury to his chest, surgical treatment to his chest or weakness on this right side.
Physical Exam: General: Well appearing, well-nourished male in no acute distress. INSPECTION: Shoulders: Bilateral
shoulder muscle bulk and tone appeared symmetrical. No effusions or scars. Chest wall: Obvious muscular atrophy or
defect of the right chest wall musculature. Back: no deformity or scoliosis Upper Extremity: Measurement from greater
tuberosity of the humerus to the ulnar styloid 54 cm bilaterally. Hands: right hand smaller than left with scarring on
palmar surface. Feet: Bilaterally same size. PALPATION: Shoulders: No tenderness to palpation over the AC and SC joints.
Chest Wall: No palpable rib abnormalities anteriorly. RANGE of MOTION: Shoulders with Full ROM B/L in all Planes
Finger ROM full and normal bilaterally. MUSCLE STRENGTH: Internal rotation, external rotation, forward flexion, and
abduction of the shoulder/arm was bilaterally equal. Grip Strength equal between hands CV: RRR, PMI at the midclavicular line in the 5th left IC space. No murmurs, normal S1 S2.
Differential Diagnosis:
1.
2.
3.
4.
5.
Amastia
Pectus Excavatum
Hemi-hypertrophy (Hemi-hyperplasia)
Poland syndrome
Ankylosing Spondylitis
Test Results: None
Final Diagnosis: Poland syndrome a rare, sporadic congenital defect resulting in a syndrome that combines aplasia or
hypoplasia of the sternocostal portion of the pectoralis major muscle with at least one of the following: syndactyly,
brachydactyly, absence of ribs, chest
Treatment: The patient had been diagnosed with Poland syndrome at birth. He had undergone a Z-plasty of his right
hand as an infant to correct a syndactly He now has full grip strength and hand mobility although his right hand is visibly
smaller than his left. The lack of the pectoralis major muscle could predispose the patient to risk of rib trauma, especially
during contact sports such as football Surgical intervention is usually reserved to correct significant chest wall hypoplasia
from rib agenesis or to correct a strength deficit. The patient had no rib agenesis and no evidence of dextrocardia. His
muscle strength in arm adduction and internal rotation was equivalent on gross exam, indicating compensation of
secondary musculature, including pectoralis minor, latissimus dorsi, and serratus anterior No treatment was indicated.
Outcome: After an extensive literature search, specific guidelines for clearance to play and considerations in athletes with
Poland Syndrome could not be found. Poland Syndrome is rare and there is a slight male predilection of 2:1. The incidence
ranges from 1 in 30,000 to 1 in 100,000, and the etiology is unknown but speculated to be secondary to vascular
developmental anomalies during the 6th week of gestation, with hypoplasia of the subclavian artery. As well, the right
side is affected more often than the left, however, the syndrome causes more of an aesthetic concern than an actual
weakness of the affected muscles. This is believed to be secondary to surrounding musculature compensating for the lack
of strength from the pectoralis major.
Follow-Up: The patient was cleared to participate in collegiate football without further testing and did not experience
any isolated right upper extremity injuries the season Specific follow up to include Biodex pectoralis major muscle testing
comparison, ultrasound visualization of the right chest wall, and Chest X-ray for rib abnormalities.
Acute Posterior Ankle Pain in a Preprofessional Male Dancer
Authors: Rebecca Martinie, MD; Joseph Chorley, MD
Affiliation: Baylor College of Medicine, Houston TX
History: Patient is an 18 year old preprofessional male dancer presenting with acute onset of left ankle pain. The patient
had left posterior ankle pain following a hyper-plantarflexion injury while doing exercises on a bosu ball. Immediately
after injury he could bear weight but had a shuffling antalgic gait. He was placed in an ankle brace that limited
plantarflexion. He limped for 4 days and now is able to walk but has pain if he pushes off his toe. Patient complains of
tenderness and edema along the posterior ankle. He denies previous injury to this ankle He has no neurovascular
complaints. He is concerned as plans to attend dance school in the near future.
Physical Exam: Vital signs are within normal limits. Patient is well developed and well nourished. Left ankle: Mild
edema posterior to medial malleolus, without ecchymosis. Skin and neurovasculature is intact. Negative tinel of tarsal
tunnel. Pain with palpation of medial malleolus and posterior ankle, medial more than lateral. Posterior ankle pain at
terminal plantar flexion active and passive. Resisted range of motion of the ankle and great toe are normal. Patient is
able to bear weight but has pain with two feet toe raises. All special tests for the ankle are normal.
Differential Diagnosis:
Posterior impingement syndrome
Flexor hallucis longus (FHL) stenosing tenosynovitis Achilles tendon partial rupture
Osteochondritis dissecans of talus Posterior tibial tendon tear
Tarsal tunnel syndrome
Bursitis - retrocalcaneal vs pericalcaneal
Fracture/Stress fracture
Peroneal tendon subluxation
Test Results: XR Left Ankle: No Fracture. Well corticated defect in the posterior aspect of the talus, likely an os
trigonum. MRI Left Ankle: 1. Posterior impingement syndrome with subtalar and tibiotalar joint effusion/synovitis and a
stieda process of the talus with incomplete bony union and adjacent stress-related marrow changes. 2. Stress-related
marrow changes in the talus and second metatarsal. 3. Periligamentous stress-related marrow changes of the distal fibula
and subtalar calcaneus.
Final Diagnosis: Acute on chronic posterior impingement syndrome of left ankle.
Treatment: He was placed in a boot (WBAT) and instructed to wear it continuously except for rehab and bathing. He
was also instructed to ice at least daily, take NSAIDs, continue bi-weekly physical therapy, and follow-up in two weeks.
Outcome: After two weeks, he was able to walk in boot without pain, thus transitioned to wearing boot only at rest.
Patient was instructed to wear ankle brace with exercises, continue PT and icing. He was also started on weekly
ionophoresis with dexamethasone with some improvement. After six weeks, he was non-tender and had improved range
of motion. He was noted to have a non-painful click at posterior lateral ankle with passive plantar flexion. He had
continued pain with toe raise and expressed experiencing locking when performing demi pointe. He was continued on
current management and allowed to work on range of motion as long as non-painful. Four weeks later he had minimal
improvement in range of motion and worsening of pain. Repeat MRI showed almost complete resolution of previous
inflammatory changes. Given continued tenderness, the decision was made to proceed with ultrasound guided injection of
the FHL peritenon with corticosteroid and anesthetic. After injection patient reported markedly improved pain but
continued popping, although less prominent.
Follow-Up: Patient is currently doing dance exercises, PT once a week and working towards returning to dance classes.
He will follow-up again in 2 months.
Utility of Point-of-Care Ultrasound in the Assessment of Thoracic Cage Trauma in a Collegiate
Wide Receiver
Authors: Joshua Sole, MD; Jason Mogonye, MD; David Gable, MS, ATC, LAT; Michele Kirk, MD
Affiliation: John Peter Smith Health Network, Orthopedic and Sports Medicine Center, Arlington, TX; Texas
Christian University, Fort Worth, TX
History: 20 year-old male Division 1A wide receiver and hurdler was evaluated at football practice after landing on a football during
wide receiver drills. The athlete endorsed severe left upper quadrant pain and shortness of breath immediately following the injury. Pain
was primarily located around the left lower rib cage and rated 9/10. The athlete denied history of previous abdominal or thoracic injury.
The injury was reported to the head football ATC who promptly consulted team physicians to evaluate further.
Physical Exam: On palpation, there was acute translation of the left costal margin, with what felt to be crepitance and reduction of
misaligned ribs. This resulted in severe pain. Bowel sounds were normoactive. Abdomen was soft, non-tender, and non-distended
without guarding, organomegaly, or ecchymosis. Sensation in the thoracic dermatomes was preserved. Initial vitals showed elevation in
SBP and DBP to 149/90 was attributed to pain. The athlete's other vital signs remained within normal limits.
Differential Diagnosis:








rib contusion
rib fracture
costochondral displacement (subluxation/dislocation)
blunt abdominal injury
spleen injury (hematoma/laceration)
slipping rib syndrome
intercostal muscle strain/tear
pneumothorax
Test Results: We initially performed bedside diagnostic ultrasonography of the thoracic cage and abdomen. Complete displacement of
the anteriomedial margins of ribs 9-10 was observed in long-axis, concerning for rib fracture. This correlated with the region of
maximum tenderness to sonopalpation. The normal sonographic appearance of the intercostal muscles and neurovascular bundle was
preserved (long and short axis). Limited FAST scan of the abdomen was benign for obvious abdominal trauma or bleed, including
splenic injury. A complete, dedicated rib series was obtained including oblique radiographs of ribs 9-10. There was no evidence of
osseous and/or cardiopulmonary abnormality apparent on radiographs. Advanced imaging was not deemed clinically necessary and
was not pursued.
Final Diagnosis: Costal cartilage fractures of left 9th and 10th ribs
Treatment: Protection, rest, and ice were recommended. NSAIDs, Tylenol, and Tramadol were prescribed as needed for analgesia. The
athlete returned the following day to our Athletic Training Room for reevaluation with ATC and physician staff. The athlete reported
1/10 subjective pain and was not requiring oral analgesia for pain management. There was continued, significant tenderness to
palpation over the costal margin. Rib fractures were still apparent on ultrasound.
Outcome: Following a graduated return to play rehabilitation and monitoring program with athletic training staff and team physicians
supervising, the athlete returned to full practice and game participation without related symptoms. Serial ultrasonography was pursued
to monitor for complete resolution and interval healing of the costal cartilage rib fractures.
Follow-Up: The athlete went through a graduated return to activity comprised of treatment modalities in the training room and serial
evaluations until pain was significantly decreased. Subsequently, he began conditioning, plyometric, and running activities with the
training staff for one week. The athlete was gradually integrated into non-contact position specific drills during practice. At 2 weeks
post-injury the athlete was pain free with all activities and team physicians and training staff released him to full contact with rib
protective gear. Follow-up ultrasound imaging was utilized to monitor for resolution and healing of the costal cartilage rib fractures. He
continued to participate in practice and game competition without related symptoms.
Abdominal Injury - Soccer
Authors: Jennifer Payne, MD; Mark Lavallee, MD
Affiliation: York Hospital, York, PA
History: A 19-year-old male Division III collegiate soccer goalie was involved in play near his goal. In the chaos, his own
teammate elbowed him in the anterior left abdomen. The patient dropped to the ground, vomiting and writhing in pain.
No other history was able to be obtained due to patient status. Records revealed no medications or other PMH. Social
EtOH use.
Physical Exam: Patient was writhing on the ground vomiting, appearing in considerable pain. Heart and lungs were
clear. Vital signs stable. Significant abdominal pain continued. No bony tenderness on spinal or rib palpation. BP 119/72,
HR 60, T 97°F, RR 18, O2 sat 100% on arrival to Gettysburg Hospital.
Differential Diagnosis:
1.
2.
3.
4.
5.
6.
7.
Splenic injury
Diaphragm injury
Rib contusion
Bowel rupture
Ureteral disruption
Appendiceal rupture
Renal contusion
Test Results: Gettysburg Hospital: WBC 9.4 81%PMN, HB 15.4, HCT 42.7, plt 225 INR 1.1, Cr 1.4, BUN 16, Na 136, K 3.9
Cl 102, CO2 24, glc 114 LFTs wnl, lipase 22, Ca 10.2.
CT scan abdomen with contrast displayed delayed contrast extravasation with leaking of urine in area of left ureteropelvic
junction or proximal ureter. No obstruction, free air or stranding of mesenteric fat.
Final Diagnosis: Rupture of ureter at ureteropelvic junction.
Treatment: Stabilization and transport from field via EMS. Patient was taken to Gettysburg Hospital where preliminary
testing was completed; it was determined he needed to be transferred to York Hospital; whence it was determined given
the diagnosis the patient should be transferred to Johns Hopkins for definitive surgery by a surgeon with experience in
this rare condition. Johns Hopkins urology (JHU) was able to pass a nephroureteral stent through the affected area from
the outside; patient did not need a urostomy bag but rather had to change a dressing once a day and flush stent twice a
day from external port; followed weekly at JHU.
Outcome: Patient able to be discharged with stent on 10/1. Patient developed fever and dysuria with 2+ LE on 10/21,
diagnosed with URI but remote possibility of pyelonephritis and placed on ciprofloxacin; symptoms resolved by 10/27.
Awaiting possible stent removal mid-November.
Follow-Up: Clearance and return to play pending removal of stent and urology approval.
A Case of Insidious Calf Pain
Authors: Maria V. Karipidis Pouria, MD; Mark Mirabelli, MD;
Affiliation: University of Rochester Medical Center Rochester, NY
History: 28 year old female with history of insidious onset of bilateral calf pain temporarily related to exercise/activity
that had been chronic for several years and increased in intensity and severity. Her usual exercise routine consisted of
biking for 1/2 hour and leg presses, denied overexertion. She noticed a diffuse, dull, achy pain with hardening in calves
which occurred after exercising, most notable in the evenings after exercises, and extended for a period of 2-3 weeks.
Lingering calf pain woke patient up at night after she has been active and only affected her calves, was not related to
swelling of the muscles, red inflamed joints, or any rashes. Denied any actual cramping of the muscle or dark cola colored
urine associated with these episodes. Patient did not experience any specific weakness: she could go up and down a
flight of stairs, get into and out of chairs/bed, reach her arms over her head. No fixed numbness or tingling, changes in
voice, double vision, ptosis or shortness of breath with activities. No recent changes in medications associated with her
symptoms. Nothing alleviated her symptoms besides curtailing her exercise program and standing for long periods and
exercise exacerbated her symptoms.
Physical Exam: Patient noted to have normal vital signs, no acute distress. Heent: NC/AT, PERRL, EOMI, Optic disk
margins clear, spontaneous venous pulsations present. Visual fields full. No thyromegaly, oropharynx clear. CVS: S1, S2+,
RRR, no M/R/G. Resp: CTAB Msk: Back: Full ROM, neg straight leg raise, no vertebral body tenderness. No tension or
prominence in paraspinal musclesExtremities: All 4 extremities without edema, clubbing or cyanosis Gait: Normal and
non antalgic, able to toe walk without difficulty. Hip and Pelvis and Knees b/l: No swelling, erythema, ecchymoses or
deformity. No palpable effusion or warmth. Non tender to palpation. Full strength. Calves: no deformities, mildly
underdeveloped on palpation, no firmness or tenderness. Neurovascular: 2+ DP pulses in both feet, capillary refill &lt;3
seconds, sensation intact to light touch, 5/5 strength throughout lower extremities.
Differential Diagnosis:
1.
2.
3.
4.
5.
6.
Chronic exertional compartment syndrome
Vascular insufficiency
Popliteal artery entrapment
Myofascial pain
Inflammatory myopathy
Deep venous reflux
Test Results: EMG/NCS normal. CK ranged from normal to a high of 499, with most recent normal. Thyroid studies
normal. ANA positive Muscle MRI showed patchy areas of hyperintensity in vastus lateralis and gastrocnemius of
unclear significance. Muscle biopsy showed variability in fiber size and type 1 predominance of unclear significance. CPT2
testing normal. Vascular ABI studies normal. A progressive treadmill run lasted 5 minutes. The patient did have
progressive symptoms of tightness, pain, but not tingling and weakness and on exam did have decreased strength after
exertion with dorsiflexion, eversion, but not inversion or plantarflexion. Calf circumference was measured pre and post
exercise and was found to increase in the bilateral legs. MRA of lower extremities (not done under plantar flexion
protocol) revealed no significant vascular or other abnormality identified in the visualized portions of the bilateral lower
extremities. There was no evidence of popliteal artery entrapment syndrome. Compartment pressure testing remarkable
for elevated pressures at lateral compartment which was not consistent with where the patient was complaining of pain.
Angiogram of bilateral lower extremities with stress views revealed bilateral popliteal artery entrapment syndrome of the
behind the knee popliteal artery.
Final Diagnosis: Bilateral popliteal artery entrapment.
Treatment: Bilateral popliteal artery release and resection of the medial head of the gastrocnemius muscle.
Outcome: Patient symptomatically improved and reported improvement in pain with increased levels of activity.
Follow-Up: Activity as tolerated and was to follow up with vascular surgery as an outpatient.
Chest Pain: Not a Typical Day at the Pool
Authors: Justin Mark J Young, MD
Affiliation: University of Hawaii John A. Burns School of Medicine Department of Family Medicine &
Community Health, Division of Sports Medicine, Honolulu, HI
History: 22 year-old female competitive swimmer presented with chest pain 1 day duration after a strenuous swim
practice. She has never had this pain before. On exam, the pain was reproducible over the anterior chest wall. She was
diagnosed with costochondritis, given NSAIDs, and relative rest, and the pain improved within the week. However, she
came back in 2 weeks saying that the pain reoccured with minimal exertion while doing light yoga. Again the pain was
over the right anterior mid chest. It was worse with inspiration. She had no past medical history of any health problems.
She was on no medications. There was no weight loss, cough, fever or chills. Family history was unremarkable.
Physical Exam: Vital signs: 98.2°F, 108/70 (76), 5'8", 162 pounds, BMI 24 General: No apparent distress. Healthy
appearing young female HEENT: Normocephalic and atraumatic. Chest: Lungs are clear to auscultation and percussion.
Heart is regular and there are no rubs, gallops or murmurs. There is tenderness to palpation over the 3rd and 4th right rib
space, mid-clavicular line. Abdomen: soft and non-tender, normal active bowel sounds
Differential Diagnosis:
Costochondritis
Chest wall strain
Rib Fracture
Pneumonia
Pneumothorax
Test Results: Chest and rib x-rays were done. The ribs appeared normal. However, there was a 4 cm mass over the
anterior mediastinum. Subsequent labs were ordered. CBC did reveal an elevated white count of 14.0. Differential
showed neutrophil predominance at 83%, lymphocytes 10%, monocytes 6%, basophils and eosinophils &lt;1%.
Hemoglobin 12.4, Hematocrit 37.3, platelets 321,000. Sedimentation rate 30 (0-20 is normal). Chemistries are normal:
glucose 102, Sodium 138, Potassium 4.2, Chloride 99, CO2 29, AST 17, ALT8, calcium 9.5, Alkaline phosphatase 83, total
bilirubin 0.4, total protein 8.1, albumin 4.6 CT Scan of the chest: Large heterogeneous mass in the anterior mediastinum
measuring 7.3-cm transverse x 6.7-cm anterior-posterior x 7.3-cm craniocaudal.
Final Diagnosis: Primary mediastinal diffuse B cell large cell Non-Hodgkin's lymphoma
Treatment: The patient was sent to cardiothoracic surgery for biopsy of the mass. It was thought she most likely had
thymoma, but this could also be a malignancy. The mass was resected. Surgical pathology revealed that she actually had
Non-Hodgkin's Lymphoma. She was concurrently seen by oncology who started chemotherapy.
Outcome: The patient was diagnosed with a primary mediastinal diffuse B cell large cell Non-Hodgkin's lymphoma. She
is currently undergoing treatment with R-CHOP and radiation therapy.
Follow-Up: Currently undergoing treatment and doing well.
The Hamstring Strain That Wasn't
Authors: Nilesh Shah, MD
Affiliation: Summa Health System Akron, OH
History: 54 y/o male runner complains of left hamstring pain since a 1/2 marathon race 3 months earlier. 2-3 miles into
the race he started with pain in his hamstring, but was able to finish the race. He feels some fullness or swelling to the
hamstring - but wonders if he is just scrutinizing it more. He feels he had some ecchymosis posteriorly after the half
marathon, but not significant. Previously diagnosed with a hamstring strain and started into physical therapy. Overall he
is better with physical therapy and can run 7-8 miles without significant discomfort. The pain gets to be a 2-3/10 at its
worst and does not affect his daily life. No lower back pain, radiculitis, or worsening pain with coughing, sneezing, or
valsalva. Denies fevers, chills, night sweats, or unexplained weightloss.
Physical Exam: Vital signs: BP: 116/75 HR: 60 RR: 10
General: NAD with normal mood and affect Spine: Gait and
heel and toe walking were normal. Normal ROM lumbar spine and ER of both hips with mild IR loss bilaterally. Normal
strength bilateral lower extremities. Gluteus medius strength was fair bilaterally. Negative seated SLR 90-0° bilaterally
Normal lower extremity reflexes bilaterally Normal gross sensory exam bilaterally. Negative clonus and downgoing toes
with Babinski testing. Extremity: Resisted knee flexion without pain. Tight hamstrings and very tight quadriceps more on
the left than right. Significant crepitation to the knee on the left. The rest of the left knee exam was negative. Pes planus
feet bilaterally with mild pronation. No obvious redness, warmth or swelling noted. There may be some swelling the midmuscle belly and on distally. Palpation to the mid muscle belly of the hamstring and distal medially causes him some
discomfort with no palpable defect.
Differential Diagnosis:
Hamstring strain
Hamsting tendinopathy
Osteoarthritis of the hip and/ or knee
FAI/Labral tear
Femoral Stress Fracture
Lumbar etiology - radiculitis/ stenosis
Infection
Vascular - claudication
Tumor
Test Results: X-rays of the femur and knee demonstrated the hip joints being concentric without sign of crossover/ FAI.
There was mild degenerative change at the pubic symphysis. The knee films demonstrated mild degenerative changes to
the tibiofemoral joints with severe, bone-on-bone, degenerative changes to the patellofemoral joints. Continued with
formalized physical therapy and added more core stab, glut medius strength, and eccentric exercises to the hamstring.
Final Diagnosis: Hamstring strain
Treatment: The patient continued with lowgrade pain and returned for follow-up about 6 weeks later. Still same pain
and able to run. Continues to describe "swelling" to the hamstring area. Re-examination: No change in previous physical
examination. There may be subtle fullness to the hamstring in the mid muscle belly on distally as compared to previous.
MRI of the thigh ordered and revealed: Large 18.0 x 4.3 x 6.8 cm intramuscular fat-containing lesion within the mid and
distal left thigh in the expected region of a markedly atrophic/distally absent abductor magnus and longus muscle.
Outcome: Orthopedic oncologist performed marginal resection demonstrating well-differentiated liposarcoma.
Final
pathology: low-grade lipomatous tumor. The patient was followed yearly for two years watching for local recurrence.
Follow-Up: He continues to run at this point with no further issues. He is to follow-up only of he feels a recurrent
mass.
An Elite Gymnast with a Surprising In-Office Ultrasound Finding
Authors: Drew Glover, MD; Jon Schultz, MD
Affiliation: University of Missouri-Kansas City Sports Medicine Fellowship, Kansas City, MO
History: An 18 year old female elite gymnast presented to our Sports Medicine Center three weeks after sustaining a left
ankle injury while "landing short" during a skill at a competition. The athlete stated she had left lateral ankle pain that is
"nagging" and persistent thought she has been training and competing through the pain. She initially had some lateral
ankle swelling which was mild to moderate and she denied any popping sensation. She participated in a competition in
Chicago, IL one week prior to presentation and noted increasing pain with landing and jumping during her skills. She also
noted a feeling of "movement" over her lateral ankle worsened with pushing off from the ground to jump. The athlete
noted she has a competition in ten days in which she would like to compete and again in one month to make the United
States Worlds. She denied taking any medications and denied prior ankle sprains, instability, or surgeries during her
career.
Physical Exam: She was in no acute distress.She walked in unaided, without a brace or crutches without an antalgic gait.
Inspection revealed mild swelling over the ATFL but no erythema or ecchymosis. Palpation revealed mild tenderness over
the ATFL and peroneal muscle with no malleolar tenderness, deltoid tenderness or CFL tenderness. Active range of motion
was full with 50 degrees of plantar flexion and 20 degrees of dorsiflexion bilaterally. Strength testing revealed 5/5 in
plantar flexion, dorsiflexion, pronation and supination. Resisted plantar flexion, dorsiflexion, supination and pronation
produced no increased pain. The Anterior drawer testing revealed mild laxity as compared to the contralateral side. Talar
tilt was positive. Squeeze test, External rotation test, and Calcaneal squeeze test were negative.
Differential Diagnosis:






Ankle sprain of ATFL
Peroneal tendonitis/tendinosis
superior peroneal retinaculum injury with subluxating peroneal tendon
sinus tarsi syndrome
os peroneum
lateral malleolar fracture
Test Results: 3 view radiograph of the left ankle showed closed growth plates, no joint effusion, no evidence of fracture,
no soft tissue swelling or other bony abnormality. Limited Diagnostic In-office Ultrasound of the left ankle with use of a
high-frequency probe showed no evidence of fluid within the peroneus longus, brevis or tertius tendon sheaths. A shallow
peroneal groove was noted, and dynamic testing consisting of weight-bearing plantar flexion demonstrated near
subluxation of the peroneus brevis tendon at the lateral malleolus. In addition a small hyperechoic density directly inferior
to the distal end of the lateral malleolus was visualized.
Final Diagnosis: Type III Superior Peroneal Retinaculum Injury
Treatment: Because the athlete desired to compete in her elite level competitions within the next month we decided to
try a period of immobilization in a CAM walker boot when she was not competing or training. She was prescribed an ankle
home exercise program as well as continuation of her formal physical therapy
Outcome: At the time of submission the athlete had made the United States Worlds gymnastic team and was competing
in China. We will follow up with her once she returns to the United States.
Follow-Up: The athlete did return to activity in time for her upcoming competitions and will follow-up after she returns
to the United States.
Knee Pain with a Kick
Authors: Jason Holinbeck, MD; Tim Palomera, MD; Eliot Young, MD; David Schmidt, MD
Affiliation: Christus Santa Rosa Primary Care Sports Medicine Fellowship, San Antonio, TX
History: 17 yo male football player presenting with anterolateral left knee pain. He did not have any specific injury and
has had no prior knee problems. His pain has been gradually worsening over the past few months and is worst when
planting to kick the soccer ball and descending stairs. He has no instability, patellar subluxation or locking episodes and
has been able to continue playing despite his pain. He did notice some swelling that developed a few hours after his last
game. On exaxination,. he had diffuse tenderness over the patella as well as the lateral joint line Plain radiographs were
normal, and an MRI ordered to evaluate for a lateral meniscus tear revealed marrow edema at the anterior/inferior patella
consistent with a bony contusion. The patient returned to clinic three months later after relative rest, physical therapy
and NSAIDS had failed to completely improve his symptoms. 3T cartiogram ordered to evaluate the bone contusion /
possible chondral defect revealed extensive bone marrow edema throughout the patella. The patient was diagnosed with
a patellar stress reaction and instructed to not participate in football and a patellar stabilizing brace was added to his
treatment regimen. Approximately one month later, the patient returned to clinic with symptoms occurring constantly,
including at rest. His family was questioning why his bruise was taking so long to heal. History revealed that the patient
had stopped football but began playing soccer. In fact, the patient recently went to kick a ball when another player
blocked it and he has had difficulty walking since then.
Physical Exam: Mod effusion, no bruising. Range of motion limited to 10-100°, Strength testing limited by pain but able
to extend his leg. Neg lachman, neg posterior drawer. Stable to varus/valgus stress. Guarded McMurray's laterally. Distal
sensation, pulse intact. Exquisite peripatellar tenderness.'
Differential Diagnosis:











Patellofemoral pain syndrome
meniscus tear
discoid lateral meniscus
osteochondritis dissecans
patellar tendonitis
bony contusion
patellar fracture
fat pad impingement
avascular necrosis
neoplasm
infection
Test Results: 5/14 Xray: Neg for fracture 5/14 MRI: Marrow edema at the anterior / inferior patella consistent with a
bony contusion. 8/14 3T MRI: Extensive bone marrow edema throughout the patella. 9/14 Xray: Mildly displaced
longitudinal stress fracture of the patella. 10/14 Post op Xray: Fracture line still visibile
Final Diagnosis: Longitudinal stress fracture of the patella
Treatment: Nonweight bearing until surgery evaluation. Left knee ORIF/scope/ bonegraft surgery due to displacement
and nature of fracture.
Outcome: One month post op, pt reporting tolerating rehab exercises with only mild pain.
Follow-Up: Limit flexion for the first six weeks.
Heart Palpitations in a 19 Year-Old Collegiate Soccer Player
Authors: Megan Ferderber, MD; Jeanne Doperak, DO
Affiliation: UPMC St. Margaret Family Medicine Residency Program, Pittsburgh, PA
History: The patient is a 19 yo male Division III collegiate soccer player who presents to the training room with two days of palpitations
Symptoms began two days prior to admission during a soccer game, when at half time he chugged ice cold water and noted
palpitations He played the remainder of the game and two days later when symptoms persisted he brought this to the attention of the
athletic trainer at soccer practice This prompted his presentation to the emergency room for further evaluation. Review of systems
performed and positive only for palpitations and occasional chest discomfort He denied chest pain, shortness of breath, and recent
illness He was not on any medications nor used illegal substances, has no allergies, no concerning family cardiac history, and no social
concerns Of note, his past medical history was significant for a perimembranous ventricular septal defect (VSD), aortic insufficiency and
right ventricular outflow tract (RVOT) obstruction which was surgically corrected in 2006 with no subsequent complications.
Physical Exam: Vital Signs: HR 130, BP 110/80, Resp 14, Temp 98.1°F General: No acute distress CV: regular rate, irregular rhythm,
2/6 SEM LLSB. No peripheral edema. Respiratory: CTAB/L, no w/r/r, no increased wob. HEENT, GI, MSK, Derm, Neuro, Psych: all within
normal limits
Differential Diagnosis:
Pulmonary embolism
paroxysmal supraventricular tachycardia
atrial tachycardia
myocardial ischemia/infarction
heart failure
hyperthyroidism
anxiety
anemia
substance abuse
Test Results: Complete blood count and basic metabolic panel - within normal limits Chest x-ray - Cardiac size upper limit of normal.
Normal pulmonary vascularity. No effusion. Electrocardiogram (EKG) - Rate 106 bpm. Atrial flutter with right bundle branch block
(RBBB).
Final Diagnosis: Atrial Flutter
Treatment: Patient was transferred to a tertiary care center where he was admitted overnight and anticoagulated with heparin The
following morning the patient had a transesophageal echocardiogram (TEE) to assess for thrombus formation prior to cardioversion
The TEE was indicative of the following: moderately depressed biventricular systolic function, mildly dilated RV, no residual RVOT
obstruction, trivial residual VSD, and trace aortic valve insufficiency along with trace tricuspid, mitral and pulmonary valve insufficiency
After successful synchronized cardioversion, the TEE confirmed lingering depressed biventricular systolic function but improved.
Outcome: The soccer player was easily cardioverted back to normal sinus rhythm He was placed on 12.5 mg Metoprolol tartrate BID
and Aspirin 162 mg daily upon discharge until his outpatient follow up.
Follow-Up: The patient followed up with cardiology two weeks following cardioversion At that time a transthoracic echocardiogram
(TTE) was performed confirming known and pre-existing findings status post his cardiac surgery in 2006, and reaffirmed the recent
finding of low normal biventricular systolic function, citing the LV ejection fraction to be approximately 50% An EKG was performed as
well, indicative of sinus bradycardia with marked sinus arrhythmia and RBBB. Given the low normal systolic function and recent atrial
flutter, it was recommended that he should refrain from competitive sports until he had gone two to three months without atrial flutter
with or without drug therapy. In the interim he was allowed to perform low impact aerobic exercise and limited isometric sports.
Lisinopril 5 mg daily was added to his medication regimen due to reduced systolic output. A follow up exercise study was arranged in
two months to assess him as a candidate for return to full competitive play, and an EKG, TTE and 24 hour Holter monitor were arranged
in 6 months for routine follow up.
17 Year Old Female with Right Ankle Pain
Authors: Rachel A. Coel, MD, PhD
Affiliation: Medical Director and Staff Physician, Queen's Center for Sports Medicine, Honolulu, HI
History: A 17 year old female hula dancer presents to sports medicine clinic with a chief complaint of right ankle pain for 2 days. The
patient reports she was running down a hill at a park when she slipped on wet grass and forcefully plantar flexed and inverted her right
ankle. She immediately had pain with weight bearing and developed ankle swelling and lateral bruising within 2 hours. The pain and
swelling worsened the next day, and the patient was taken to her primary care provider. She was diagnosed with a lateral ankle sprain,
instructed to buy an over-the-counter brace, and xrays were ordered of the right ankle. She was referred to sports medicine for
interpretation of the xrays, definitive diagnosis, and treatment. The patient currently rates her pain 7/10 over the lateral and posterior
right ankle and describes it as achy, sharp, and throbbing. The pain is worsened with weight bearing and palpation and improved with
elevation and rest. She has been taking ibuprofen 600mg every 8 hours, which improves her pain. The patient has no prior history of
ankle sprain and denies knee or proximal lower leg pain, swelling, or bruising.
Physical Exam: No acute distress. Abnormal antalgic gait with right sided limp. Right foot and ankle exam: Soft tissue swelling over
medial and lateral ankle joint, dorsal foot, and toes. Ecchymosis over lateral ankle joint and foot. Tenderness to palpation over posterior
distal tibia, Achilles, ATFL, CFL, anterior joint line and medial malleolus. Small palpable ankle joint effusion. Full but painful ankle range
of motion, most painful with plantar flexion or external rotation. Strength is 4/5. Neurovascularly intact. Talar tilt and anterior drawer
tests negative. No crepitus or pain with fibular squeeze test.
Differential Diagnosis:
1.
2.
3.
4.
5.
6.
7.
8.
Ankle sprain
Trimalleolar fracture
Isolated malleolar avulsion fracture
Achilles tendon injury
Peroneal tendon strain, subluxation, or dislocation
Osteochondral defect of tibia or talus
Talus fracture
Maisonneuve fracture
Test Results: Right ankle radiograph 3 views: Nondisplaced isolated posterior malleolar fracture with extension to distal articular
surface with less than 10% joint surface involvement. Ankle mortise preserved without widening. Small joint effusion and moderate
lateral soft tissue swelling noted. CT right tibia/fibula without contrast: Minimally displaced ( < 2mm) isolated posterior malleolar
fracture without step-off at articular surface. No fibular fracture. Tibiotalar and subtalar joint effusions seen. Anatomic alignment of
ankle mortise.
Final Diagnosis:
1.
2.
Isolated posterior malleolar fracture without significant displacement.
Lateral ankle sprain.
Treatment: Patient placed in non-weightbearing short leg cast in 10 degrees equinus for 6 weeks with crutches for ambulation.
Outcome: Follow up xrays in cast at day #8 and day #15 showed no change in alignment of the fracture fragment. At 6 weeks from the
date of injury, the cast was removed and xrays showed satisfactory healing of the stable posterior malleolar fracture. The patient was
transitioned to a lace up ankle brace. She weaned off crutches in the following 7 days.
Follow-Up: After cast removal, physical therapy was prescribed for ankle stabilization, strengthening, and proprioceptive training. At 12
weeks from the date of injury, the patient was pain-free and exhibited full symmetric range of motion with mild ankle laxity and mild
deficit in strength and stability on exam. She continued physical therapy for another 2 weeks. At 14 weeks from the date of injury, she
began progressive hula dance training with complaint of mild lower leg fatigue with exertion. The patient was instructed to continue her
home exercise program for another 4-6 weeks.
Novel Treatment for Recurring Episodes of Left Shoulder Pain
Authors: Jason Matuszak, MD, FAAFP
Affiliation: Excelsior Orthopaedics, Amherst, NY
History: 87 year old female nursing home resident with left shoulder pain, erythema, and swelling for several weeks. She
has history of multiple episodes of septic bursitis treated with IV vancomycin. She is currently under treatment for the
same by infectious disease. Referred to orthopedic partner who feels she is a very poor surgical candidate.
Physical Exam: Fluctuant mass, with overlying erythema and warmth. Diminished range of motion and strength.
Differential Diagnosis:
Septic buritis and/or septic arthritis with extension to the bursa
Sterile pyogenic effusion secondary to collapse
Test Results: Ultrasound - cystic mass measuring 13 cm in diameter, with full thickness RC tear and communication with
glenohumeral joint
x-ray - severe rotator cuff arthropathy Arthrocentesis - 420 ml of purulent fluid, positive for MRSA
Final Diagnosis: MRSA Septic arthritis/bursitis
Treatment: Patient returned for percutaneous aspiration, with sterile saline lavage, followed by intralesion injection of
vancomycin. This was performed in serial fashion q 10 days 3 more times, then q2 weeks for 2 times. This was serially
followed with culture and colony count. Treatment course: 420ml of purulent fluid aspiration only 180ml of purulent
fluid, aspiration, lavage with sterile saline, and irrigation with 1g Vancomycin 120ml of purulent fluid, aspiration, lavage
with sterile saline, and irrigation with 0.5g Vancomycin 90ml of turbid fluid, aspiration, lavage with sterile saline, and
irrigation with 0.5g Vancomycin 50ml clear red-yellow fluid, aspiration, lavage with sterile saline and irrigation with 0.5g
Vancomycin 2 weeks 30 clear red-yellow fluid, aspiration, lavage with sterile saline, and irrigation with 0.5g Vancomycin
2 weeks 30 cc clear orange fluid aspirated only, no lavage, no vanco - TEST OF CURE
Follow up 18 months later - no
recurrences.
Outcome: Intravenous and intralesion vancomycin stopped and two weeks later repeat aspiration showed negative gram
stain, culture and colony count.
Follow-Up: Return to the office 18 months later - no recurrences Repeat ultrasound - no effusion, no bursitis
Persistent Knee Pain in a 12 Year Old Dancer
Authors: Tara Futrell, MD
Affiliation: University of Massachusetts Medical School, Worcester MA
History: 12 year old competative dancer developed gradual onset of left knee pain 2 months ago. It began after taking a
2 month break from dancing when she returned to classes. No injury or trauma. Noticed while walking for prolonged
periods. No swelling, radiation, numbness or tingling, locking or buckling. It does not wake her at night. She locates the
pain over anterior knee. It is worse with walking, exercising or prolonged sitting. It feels better with ice and ibuprofen.
Rested from dance then went to 1 month of PT with no improvement. Now pain is so severe she cannot walk with normal
gait or participate in dance. She denies any previous knee problems or injury to her knees.
Physical Exam: Ht: 62in Wt:115 lbs GEN: A&amp;O, NAD HEENT: NC/AT, conjunctiva clear Resp: normal respiratory
effort CV: No LE edema MSK: Antalgic gait favoring right leg. No effusion, redness or warmth. TTP over entire anterior
knee worse over patella tendon, medial and lateral joint lines. Unable to fully extend or flex leg. ROM from 20° to 90°
Guarding beyond this range. 5/5 strength of hip flexors, knee flexion and knee extension within her ROM. Negative
anterior and posterior drawer, Lachman, varus and valgus stress. Negative McMurry but limited due to her ROM. CV: 2+
distal pulses and no edema Neuro: sensation grossly normal
Differential Diagnosis:
Patellofemoral pain
Discoid meniscus tear
patellar osteochondritis dissecans
patellar tendonitis
Jumper's knee (Sinding-Larsen-Johannsen disease)
Osgood-Schlatter's disease
Hoffa's fat pad impingement
Synovitis
Pigmented villonodular synovitis
Symptomatic infrapatellar plica
Test Results: X-rays bilateral knees: No fracture or bony abnormality MRI: Extensive edema in the infrapatellar fat pad
adjacent to lateral femoral condyle. Focal fat impingement inferior to lateral facet of patella.
Final Diagnosis: Hoffa's fat pad impingement
Treatment: Placed in brace and non-weight bearing on crutches for 2 weeks then reevaluate.
Outcome: No better with rest. Given an ultrasound guided corticosteroid injection into knee.
Follow-Up: She has follow up in 2 weeks and this will guide our return to play decision.
An Unlikely Return to Division I Football
Authors: Mark J. Conroy, MD; Aaron V. Mares, MD
Affiliation: UPMC St. Margaret Primary Care Sports Medicine, Pittsburgh, PA
History: A 17 year-old male high school football and basketball player without a significant history comes in to an emergency
department complaining of a headache and lethargy for 2 days. He is a junior in school and being recruited for Division I football. His
family grew increasingly concerned when they noticed he developed trouble communicating and had repeated episodes of emesis.
Neither trauma nor pertinent family history is reported. There is no history of mental illness or substance abuse.
Physical Exam: Vitals: Afebrile, HR: 48, BP: 137/67, SpO2 100% RA, RR: 16 Gen: sleepy but arousable, follows some commands, opens
eyes to command, occasionally answers questions Skin: warm, dry HEENT: atraumatic with moist membranes, pupils equal round and
reactive to light Neck: nontender, no pain with flexion CV: regular rate and rhythm, no murmurs or rubs Resp: clear to auscultation,
no wheezes, rales, rhonchi Abd: soft, no guarding Neuro: face symmetric, trouble with word finding and comprehension, moves all
extremities
Differential Diagnosis:
Migraine
Ingestion (accidental or intentional)
Diabetic ketoacidosis
Seizure
Meningitis (bacterial or viral)
Hemorrhagic stroke
Sepsis
Dural venous thrombosis
Epidural abscess
Ischemic stroke
Test Results: CBC - WBC 7.7, Hgb 11.1, Hct 32.8, Plt 285 Metabolic Panel - Na 134, K 5.7, Cl 100, CO2 27, BUN 12, Cr 0.9 PT/PTT/INR 14.8/26/1.2 CT head - Acute intraparenchymal hemorrhage in the left temporal lobe with surrounding edema, midline shift of 7mm
Intracranial CT angiography - Tangled vessels adjacent to the posteroinferior aspect of the intraparenchymal hematoma consistent with
arteriovenous malformation (AVM) measuring 3.2x1.5x1cm. Cerebral angiogram - small left posterior temporal arteriovenous
malformation fed by 1 or 2 branches of the middle cerebral artery with possible tiny contribution from the posterior cerebral artery.
Final Diagnosis: Left temporal lobe intraparenchymal hemorrhage secondary to ruptured left middle cerebral arteriovenous
malformation
Treatment: The patient underwent a diagnostic and therapeutic cerebral angiogram with embolization of 2 branches of the M4 division
of his left middle cerebral artery. He was admitted to the ICU with a systolic blood pressure cap of 80 mmHg and had no further
episodes of intracranial hemorrhage. On hospital day 15 he was transferred to inpatient rehab for intensive physical and speech therapy.
One month after discharge he underwent gamma knife radiosurgery to obliterate the malformation.
Outcome: When he was discharged from the hospital, the patient still had a small visual field defect as well as slight expressive and
receptive aphasia. He was unable to return to school for the end of his junior year. The two Division I football programs who had
previously shown interest withdrew their offers. He continued therapy and his aphasia and visual field deficits resolved. An MRI at 6
months post-radiosurgery showed improvement in his AVM with near complete obliteration. At one year, there was MRI evidence
indicating that complete obliteration was likely.
Follow-Up: With neurosurgical clearance at 6 months the patient was able to begin non-contact aerobic and anaerobic activity with
specific restrictions against heavy weight lifting, dehydration, and upside down positioning. At one year, with apparent complete
obliteration of his AVM, he was cleared for full activity, including contact sports. There have been no further complications from his
AVM. He returned and played his senior season of football and was again recruited to play at a high level Division I program. He has
been a significant contributor on defense as a true freshman and has the potential for a professional career.
Doctor: That is Not My Hip!
Authors: Luis Alfredo Rodriguez, MD
Affiliation: UHS Sports Medicine. Binghamton, NY
History: 17 year old female track, cross country and rodeo runner who presents to the office with 7 weeks of right hip
pain. She recalls no specific injury but states her pain started after an 8 mile run. She averages 40-50 miles per week. The
pain is described as achy, localized to the right groin with radiation to her right lateral hip. Intensity is mild: 2/10. The pain
is worsened by: sitting or standing for a long time, climbing or descending stairs, walking and running. The pain is relieved
by: rest. Associated symptoms: decreased mobility, difficulty initiating sleep, nocturnal awakening, swelling. She was seen
at a walk-in center 5 weeks before presentation and was prescribed a course of physical therapy which did not improve
symptoms. LMP: 3-4 months ago, she stated she only gets 2-3 menstrual periods per year.
Physical Exam: Vital signs: BP: 116/65 mmHg. Pulse: 68 bpm. Respiratory rate: 15 bpm. Temp: 97.7°F. Height: 5'2''.
Weight: 102.6 lbs. BMI: 18.52 (17.4 percentile). Pulse ox: 98% RA. General: no acute distress. HEENT: unremarkable.
Lungs: clear to auscultation. CV: regular rythm, normal S1/S2 without extra sounds, murmurs, rubs or gallops. Abdomen:
soft, non-tender and non-distended with normal bowel sounds and no organomegaly. Extremities: normal pulses, no
edema or cyanosis. Musculo-skeletal: right hip: no identifiable effusion, no local erythema or ecchymosis. Active and
passive ROM is normal but pain is reproduced by FADIR. Tender to palpation mainly in the groin but also on the ASIS
and the greater trochanter. Strength is 5/5. No leg length discrepancy. Neurological exam: intact.
Differential Diagnosis:
Muscle strain (e.g. Iliopsoas, Quadriceps, gluteus)
Stress fracture of hip/pelvis
Avascular necrosis
Labral tear
Referred pain from intrapelvic structures
Test Results: X-rays: normal. MRI: no discernible hip pathology. Large hemorrhagic cyst of right ovary. Bicornuate,
bicollis uterus (congenital uterine didelphys Follow up pelvic US (15 days after MRI): unremarkable, no ovarian cyst(s)
identified.
Final Diagnosis: Functional hemorrhagic ovarian cyst causing referred pain to right hip.
Treatment: Non-weight bearing until MRI results became available (2 weeks). Weight bearing as tolerated after MRI was
reviewed. Started on OCP 1 month after pelvic ultrasound for cyst prevention. Allowed progressive return to athletic
activity. Educated her regarding menstrual irregularities and the female athletic triad.
Outcome: Symptoms resolved completely over a 3 week period. She presented to the clinic 6 months later with right
knee pain after a "twisting" mechanism, which was eventually diagnosed with the aid of an MRI as a right distal femur
stress reaction. This was treated with a period of rest leading to complete resolution of symptoms.
Follow-Up: The athlete has returned to pre-injury level of activity and has remained asymptomatic.
Persistent Visual Changes in an FCS Football Player
Authors: John Shelton, MD
Affiliation: Halifax Health Sports Medicine Fellowship Daytona Beach, FL; Florida State University School of
Medicine
History: A 23 year old college football player presented to training room rounds in mid August 2013, reporting sporadic
blurry vision since the first contact practice 2 weeks before He cannot recall a specific hit but noted his vision worsened
with the most intense exertion, improving when he rested He denied other symptoms. PMH: 2009 - blow to the L eye 4
years before with no effect on vision. Pain from the blow resolved within a week. 8 2009- L achilles rupture and
reconstruction 2011 - only concussion in a non-football confrontation resolving spontaneously. Further questioning
revealed sporadic bilateral paresthesias, sometimes from head to soles of feet and bilateral hand numbness on rising.
Review of his paper chart: PPEs in preseason 2011 through 2013 consistently showed 20/20 vision and BP - 120/60.
2011 - R stinger, resolved. Early 2013 he reported rare bilateral sciatica with toe touches and occasional blurry vision
while exercising. He was asked to get Visual acuity immediately when blurry When asked specifically about Multiple
Sclerosis, he recalled a workup at his prior college in 2012, suggesting MS. Records were obtained from his original school
which spelled out his case. 7/08 workup for headache included MR brain showoing 5 small scattered brainstem foci of
increased FLAIR reaction. Repeat MR in 2009 ordered by neurology showed no change. Ophthalmology exams x 2 during
this time confirmed normal eyes and acuity. He received no treatment then.
Physical Exam: Examination was normal including visual fields, acuity and neurologic exam.
Differential Diagnosis:
Concussion
Post concussion syndrome.
Traumatic eye injury. Doubtful, remote history, no loss of visual acuity, not bilateral.
Multiple Sclerosis
Test Results: MR brain to compare for progression of FLAIR foci. This showed new foci in both cerebral hemispheres and
the anterior pons along with enhancing lesions of both temporal lobes.
Final Diagnosis: Multiple Sclerosis Optic Neuritis by history
Treatment: Treatment was offered by Neurologist, he has declined all treatment to date. He has not followed up but
welcomes contact by phone.
Outcome: He has a great job on a management track. In spring 2014 he experienced double vision with "crossed eyes".
He consulted Ophthalmology and was sent him to ED for emergency IV steroid treatment but left AMA, continuing to
decline conventional treatment. currently he is asymptomatic and using 2-3 grams of coconut oil daily as described in his
internet reference.
Follow-Up: At the time of Diagnosis locally, he was not cleared to play, anticipating treatment. He graduated with no
plans of returning to contact sports. He will be periodically contacted for support and to offer treatment.
A Surprise Outcome In A Patient With Chronic Bilateral Lower Extremity Pain
Authors: Timothy Yu, MD; Jeffrey Roberts, MD; Kamalpreet Buttar, MD
Affiliation: Virginia Commonwealth University-Bon Secours St Francis Family Medicine Resideny Program,
Midlothian Virginia
History: A 37 year old female teacher, triathlete, and active Air Force Reserves, presents with bilateral lower leg pain.
;Medical history includes headache, depression, anxiety, environmental allergies, hypothyroidism and chronic pain
Presented five months earlier with similar complaints and was diagnosed with bilateral posterior tibial tendonitis. Tibia
and fibula x-rays were negative bilaterally that visit and was referred to physical therapy Her pain improved, but about
ten days ago she “took a step backwards&rdquo; after taking an Air Force physical fitness test. ;Now pain is more
localized to the medial shin bilaterally Pain is a 4 out of 10, worse after workouts and has no pain with rest or walking She
has tried Ibuprofen and various orthotics that has brought some relief. ;She denies trauma or injury to the region
Physical Exam: No ankle effusion noted Normal gait No tenderness on palpation in the ankle, foot or calf Tenderness
along the medial aspect of the distal tibia bilaterally No instability findings, strength 5/5, pulses intact, neurologically
intact and no rash noted. ;
Differential Diagnosis: Medial tibial stress syndrome, tibial stress fracture, exercise induced compartment syndrome,
muscle fascial hernia, tendinitis, periostitis, syndesmotic sprain and neoplasm
Test Results: Bilateral Tibial and Fibula X-rays: Negative for fracture or dislocation Bone Scan: Right anterior midshaft tibial stress fracture
Final Diagnosis: Mid-shaft Anterior Tibia Stress Fracture
Treatment: Initially refused a walking boot and crutches, but returned to clinic a week later as her pain did not improve
Instructed to be non-weight bearing and was given a walking boot and crutches Improved symptoms at three weeks and
was given home exercises, referred to physical therapy and instructed to start crutch assisted walking with the plan to
advance as tolerated Four weeks later her pain resolved and repeat x-ray revealed a non-healing stress fracture She was
placed back on crutches non-weight bearing, continued her walking book, and referred to orthopedic surgery.
Outcome: Intramedullary nailing and drilling of the right tibia
Follow-Up: Patient continued post-operative care with orthopedic surgery She did follow up fifteen months later for
right knee pain There were no significant findings on physical exam and X-ray revealed intact hardware and no active
fracture Etiology was thought to be muscular in nature and referred to physical therapy To date she has no active lower
extremity complaints
Shoulder Pain in a Young Football Player
Authors: Julia Iafrate, DO; David Soma, MD
Affiliation: Mayo Clinic, Rochester, MN
History: A 15 year old male presented to his primary care physician after sustaining a shoulder injury the previous
evening. He did not recall a specific hit that caused him pain during the game, but upon removing his jersey and pads
noted difficulty with left shoulder abduction and significant anteriolateral shoulder pain. Patient went home but awoke the
following morning with sharp pain and inability to abduct or forward flex his arm above shoulder height. Upon evaluation
in family medicine, the patient was sent to pediatric sports medicine for further work-up. Patient reported in the preceding
football games he had been primarily using his left shoulder to block due to a broken finger on his right hand.
Physical Exam: General: Awake and alert. No acute distress. Musculoskeletal: No obvious asymmetry of the shoulders.
Mild tenderness to palpation of the left AC joint and bicipital groove, significant tenderness over distal acromion.
Significant pain with abduction of the shoulder. Positive O'Brien's and Speed's tests on the left side. Negative Hawkins and
crossover scarf test. Sensation and pulses intact.
Differential Diagnosis:
Left clavicle fracture
AC joint sprain
Avulsion of the left acromion
Traction apophysitis
Deltoid tear
Biceps strain or tendonitis
Test Results: Xray shoulder - The apophysis of the left acromion is displaced laterally in comparison with the right
suggesting an avulsive-type injury. The shoulders are otherwise normal in appearance. AC joints do not appear separated.
Final Diagnosis:
Left acromion traction apophysitis or type IA avulsion
Left long-head of the biceps strain
Treatment: Patient was placed in a sling for comfort. Physical therapy was initiated with focus on range of motion,
biceps eccentric strengthening, and scapular stabilization exercises.
Outcome: Avulsion fractures are quite common in young adults, though scapular avulsion injuries are a fairly rare entity.
The mechanism is uncertain, but may have been due to opposed deltoid flexion in this patient. Intervention is based on
the severity of the avulsion. Type IA fractures are minimally displaced avulsion fractures and heal rapidly. They usually do
not require surgical intervention. Literature has shown patients with a type IA avulsion or traction apophysitis often do well
non-operatively if treatment begins acutely with restriction of activities that would otherwise cause prolonged recovery
and potential injury to the acromial cartilaginous growth plate.
Follow-Up: The patient did not return to football as the season had ended. Physical therapy consisted of increased
activity as tolerated over a period of 2-4 weeks. With recurrence of pain one might consider repeat radiographs to assess
healing and better discern presence of a fracture. An orthopedic evaluation may be warranted at that time.
Gymnast with Knee Pain
Authors: Forrest Allen, DO; Prasad Gourineni, MD; Chris Chapman, DO
Affiliation: Advocate Christ Children's Hospital, Oak Lawn, IL
History: A 14-year old female gymnast presented to her primary care doctor with complaints of right knee pain for one
month duration. She described the pain as dull and achy, and seemed to be worse with ambulation and when crossing her
legs. She was unable to specifically localize the pain but described it as a generalized discomfort around the inferior
portion of her knee. The pain was not present at night, but worse after physical activity. She denied prior injury to the
affected leg or specific injury during gym practice. Patient was taking an OTC pain reliever without much relief. She was
sent for initial x-ray which revealed an unidentified benign lesion at the proximal tibia. Patient was educated on
strengthening and stretching exercises. She was instructed to do squats in the morning and evening, and advised to follow
up in 3-4 weeks. Patient did return for follow up until approximately 4 months later, at which point she was complaining of
worsening pain in the same area. She was again sent for x-rays which showed the lesion intact but slightly larger in size.
Physical Exam: Patient was alert, well nourished and in no acute distress. Mild tenderness was noted at the medial
aspect of the right knee with palpation. There was no pain elicited with palpation of MCL or LCL. The ACL and PCL
ligaments appeared to be intact with negative anterior and posterior drawer tests. There was no warmth, swelling, or
errythema appreciated on exam. Patient retained full ROM with knee extension and flexion, along with normal internal and
external rotation of the hips bilaterally. Distal neurovascular exam was normal.
Differential Diagnosis:
Chondromyxoid Fibroma
Nonossifying Fibroma
Giant Cell Tumor
Simple Bone Cyst
Test Results: MRI of the right leg revealed an indeterminate lesion at the posteromedial aspect of the proximal diastasis
on the right tibia. The lesion measured 2X3 cm and had sclerotic margins without periosteal reaction or calcification.
Final Diagnosis: Biopsy of the lesion favored Giant Cell Tumor vs Nonossifying fibroma
Treatment: Patient underwent right proximal tibial lesion curettage with bone grafting
Outcome: At follow up from surgery, patient was weight bearing well and reported significant improvement in her pain.
Follow-Up: Patient was able to return to gymnastics 4 weeks after the procedure
Sudden Abdominal Pain While Playing Tennis
Authors: Kylee Phillips, MD, MBA; C. Daniel Hendrickson, MD
Affiliation: University of Michigan, Ann Arbor, MI
History: A 22 year-old Asian male Division I collegiate tennis player suffered sudden onset of upper abdominal pain and
vomiting while competing in a tennis tournament. He was initially evaluated by the certified athletic trainer with no
improvement of his pain from costal manipulation. He subsequently had two episodes of emesis and was transported to
the Athletic Medicine Clinic for further evaluation He noted sudden onset of sharp upper abdominal pain after serving
that was maximal at onset and constant. He had two episodes of non-bilious, non-bloody emesis at the tennis courts and
one episode en-route to the Athletic Medicine Clinic. The pain was worsened with movement and improved when lying
still. He felt well prior to the tennis tournament. He had significant past medical or surgical history He denied any prior
injury to his abdomen. He denied fevers, diarrhea or sick contacts. He denied testicular or scrotal pain, dysuria and
hematuria. He had no history of GERD, gastritis, PUD, pain after eating, or regular NSAID use. He denied alcohol or drug
use. His family medical history was unknown.
Physical Exam: Vital Signs: BP 120/76, HR: 80, RR: 18, T: 98.0°F General: Ill appearing male, uncomfortable HEENT:
Moist Mucous Membranes, no scleral icterus Cardiovascular: Regular rate and rhythm, 2+ femoral pulses, brisk capillary
refill Pulmonary: Clear to auscultation bilaterally Skin: Diaphoretic, no ecchymosis or jaundice Abdomen: Bowel sounds
present, diffuse guarding, rebound tenderness, severe tenderness to palpation in RUQ and epigastrium, no RLQ
tenderness to palpation, positive Murphy's sign Genitourinary: No scrotal swelling, no testicular tenderness to palpation,
testes in sagittal plane, no CVA tenderness Neurological: Oriented x 3, no neurologic deficits Musculoskeletal: No chest
wall tenderness to palpation, crepitus or deformity In Office Right Upper Quadrant Ultrasound: normal anterior
gallbladder wall thickness, no gallstones present, no shadowing, no pericholecystic fluid and no free fluid
Differential Diagnosis:
Pancreatitis
Perforated gastric ulcer
Nephrolithiasis Gastritis 1. Gastric or duodenal ulcer 2. Mesenteric Ischemia 3. Pneumothorax 4. Appendicitis 5.
Perforated Appendix 6. Cholecystitis 7. Cholelithiasis 8. Testicular Trauma
Test Results: Obtained After Transport to the Emergency Department CBC: WBC: 20.5 with 83% neutrophils Hgb/Hct:
16.4/48.9 Platelets: Normal BMP: Normal Lipase: Normal LFTs: Normal PT/PTT: Normal VBG: Normal Lactate: Normal
Urinalysis: Normal H. Pylori IgM Antibody: Negative Right Upper Quadrant Ultrasound: Normal Upright Chest X-Ray: No
free air under diaphragm CT Abdomen: Findings suspicious for perforated gastric ulcer with a small amount of free air.
Wall thickening involving the adjacent transverse colon is likely reactive. No abscess formation.
Final Diagnosis: Perforated Gastic Ulcer
Treatment: The patient was emergently evaluated by General Surgery and taken to the operating room for a diagnostic
laparoscopy. Laparoscopy revealed a 3mm prepyloric ulcer with perforation. Repair of the perforation was completed
using a vascularized Omental Flap (Graham patch) sewn over the area of perforation. After surgery, he was admitted to the
surgical floors with a nasogastric tube in place set to continuous low wall suction. Intravenous proton-pump inhibitor,
Piperacillin -Tazobactam and Flucanazole were started post operatively and continued for three days. Once the
nasogastric tube was removed and he was able to tolerate POs, he was started on empiric H. Pylori treatment with
Augmentin, Clarithromycin and Omeprazole. He was discharged on post-operative day five He completed his home
course of Augmentin and Claritromycin. He will remain on Omeprazole indefinitely.
Outcome: Patient was discharged home from the hospital without complications. He is tolerating food and has had no
return of pain or complications to date.
Follow-Up: At discharge his surgeon advised rest and no activity for four weeks. After four weeks he will be revaluated
by the surgical team and, if improved, will be allowed to slowly return to activity with progression to play.
Recurrent Stingers in a Collegiate Freshman Football Player
Authors: Kimbre Zahn, MD; Gregory Rowdon, MD; John Baldea, MD
Affiliation: Indiana University School of Medicine, Indianapolis, Indiana
History: 19 year old male football player presented for a routine pre-participation physical evaluation at an NCAA
Division I university He had previously played defensive back at a large Indiana high school He reported a history of 2-3
stingers per season involving both his right and left upper extremities, but never simultaneously Symptoms were transient
and did not result in any lost playing time No prior workup had been completed Athlete's medical history was significant
for a single concussion sustained two years previously, otherwise unremarkable.
Physical Exam: Cervical spine showed normal lordosis Non-tender to palpation over the spinous processes and
paraspinous muscles Full range of motion of the neck with flexion, extension, rotation, and lateral bending Upper
extremity strength testing was 5/5 and symmetric bilaterally Sensation was intact throughout Biceps and brachioradialis
reflexes were 2+ Spurling's test was negative.
Differential Diagnosis: cervical stenosis, disk herniation, cervical fracture, cervical dislocation, os odontoideum,
clavicle fracture, shoulder dislocation, acromioclavicular separation, thoracic outlet syndrome
Test Results: Imaging was indicated to rule out bony involvement secondary to his history of recurrence Radiographs
including AP, lateral, open-mouth odontoid, flexion and extension views were obtained X-rays revealed an abnormal
dens, with apparent separation of the odontoid process from the body of the axis Lateral view in neutral position showed
8mm of posterior subluxation Extension showed 11mm of posterior subluxation MRI was obtained and confirmed
chronic odontoid synchondrosis fracture and osseous nonunion resulting in os odontoideum, with 4mm of separation of
the dens in relation to the body of C2 There was 2mm of posterior subluxation of the dens in relation to the posterior
margin of C2, without canal compromise No cervical herniation, cord compression, cord changes, facet degeneration or
foraminal compromise.
Final Diagnosis: Os odontoideum
Treatment: Athlete was referred to neurosurgery for further recommendations on management of os odointoideum
Initial neurosurgical evaluation determined medical disqualification and recommended surgical intervention because of
the potential for cervical instability Athlete subsequently pursued a second opinion and medical disqualifcation was
confirmed After an in depth discussion about risks and benefits, surgery was not recommended at that time since patient
was asymptomatic T was felt that the recurrent stingers were unrelated to the os odontoideum Surgery would be
reconsidered in the future if the athlete were to become symptomatic.
Outcome: The student athlete remained on scholarship and acted as a manager
Follow-Up: Medical disqualification.
Don't Jump to Conclusions: Lower Extremity Pain in an Adolescent
Authors: Harrison Goodno, MD; Mark Locke, MD; Owen Speer, DO
Affiliation: University of South Carolina Department of Family and Preventive Medicine; Palmetto Health
History: 15 year old healthy Caucasian male presented with left leg pain after jumping across a ditch the previous day
Denied head trauma, tingling or numbness, pain in any other extremity, or prior history of left leg injury. He further denied
low back pain, bowel or bladder symptoms, weight loss, fever, or night sweats. After plain films of the ankle were
obtained at urgent care, he was provided crutches and referred to pediatric orthopaedics for further evaluation of a
possible left gastrocnemius tear.
Physical Exam: General: alert and oriented, no lymphadenopathy
Left lower extremity: pain with passive range of
motion of ankle, tenderness to palpation in proximal fibula, skin intact, palpable dorsalis pedis and posterior tibial pulses,
capillary refill less than 3 seconds, sensation intact throughout. Able to wiggles toes. Non-tender to palpation in remainder
of extremity.
Differential Diagnosis: -Maisonneuve fracture -gastrocnemius tear -patellar tendon tear -Bakers cyst -knee
ligamentous or meniscal injury -high ankle sprain -benign versus malignant bone tumor
Test Results: Plain radiographs reveal acute pathologic fracture through 3 cm x 2 cm lytic lesion in proximal fibula.
MRI with and without contrast reveal transverse nondisplaced pathologic fracture through lytic lesion in proximal fibular
metadiaphysis without soft tissue, air fluid levels or contact with common peroneal nerve.
Final Diagnosis: Unicameral bone cyst
Treatment: Initially placed in patella tendon bearing cast and sent for MRI One week after MRI obtained, was placed in
CAM boot and made weight bearing as tolerated.
Outcome: Followed with serial radiographs and fracture noted to be healing on subsequent films Lesion and fracture
did not undergo biopsy, require intralesional injection or surgery.
Follow-Up: Gradual increase in activity out of CAM boot and will follow with serial radiographs Ongoing as of
submission date.
Stem Cell Therapy for Partial ACL Tear
Authors: Adam Androlia, DO; Joshua Hackle, MD; Jeffery Guy, MD
Affiliation: University of South Carolina, Columbia, SC
History: A 21 year old Caucasian male Division I college football player sustained a valgus blow to his knee during
practice. This injury was associated with an audible "pop" and immediate pain and swelling of his knee. He was unable to
weight bear at that time and had no previous injury to this knee.
Physical Exam: Mild distress secondary to pain. Right knee had limited flexion secondary to pain. 2+ effusion. No pain
over tibial tubercle, non-tender over patellar tendon, medial and lateral facet of patella non-tender, non-tender over
quadriceps tendon. MPFL non-tender. Medial joint line non-tender, mild tenderness to palpation of lateral joint line,
positive dial test. MCL, LCL, PCL intact. Soft end point on Lachman and negative pivot shift.
Differential Diagnosis: ACL partial tear, ACL complete tear, posterior lateral meniscus tear
Test Results: MRI of right knee with out contrast: Initial read by non musculoskeletal radiologist read as complete tear of
ACL with posterior horn radial medial meniscus tear. Unstable free edge tear of the posterior lateral minescus. Bone
contusions of posterior femoral condyle and anterior tibia. Grade 1 MCL sprain. Moderate effusion. Second Read
between orthopedic surgeon and musculoskeletal trained radiologist showing only partial tear of ACL. Posterior horn
radial medial meniscus tear. Unstable free edge tear of the posterior lateral minescus. Bone contusions of posterior
femoral condyle and anterior tibia. Grade 1 MCL sprain. Moderate effusion.
Final Diagnosis: Partial ACL Tear
Treatment: Stem cell ACL injection. Five days after the injury the patient had stem cells harvested from iliac crest and
then 3cc of the bone marrow aspirate was injected back into his ACL via ultrasound guidance.
Outcome: The patient is continuing to improve after the procedure and performing ACL rehabilitation program. He
continues to have mild ACL laxity on exam but with a firmer end point 5 weeks after procedure he is running in straight
lines, with mild cutting and is wearing his ACL brace on affected knee. The rest of his knee exam is negative at 6 weeks
post injury.
Follow-Up: He progressed on schedule and returned to play in NCAA Division 1 football after 6 weeks of rehabilitation.
Repeat MRI pending after the season is complete to observe if structural healing has occurred.
Thigh Pain in a Long Distance Runner
Authors: Charles Ware III, MD; Darrin Bright, MD
Affiliation: Riverside Methodist Hospital, Primary Care Sports Medicine Fellowship Training Program,
Columbus, OH
History: A 42YO female fitness instructor and long distance runner presents to the sports medicine office with a four
week history of increasing left medial thigh pain. She describes the pain as a dull aching pain that has progressively
worsened. She denies any acute injury and recalls that initially the pain was only noticeable after running. Aggravating
factors include intermittent discomfort while sitting if her legs are unsupported and hanging off a table. The severity of the
leg pain has progressed and is now occurring during any weight-bearing activities. She is a veteran marathon runner and
had previously been properly fitted for running shoes. Her exercise regimen included strict adherence to a formal outlined
marathon training program, with the end goal of competing in the 2014 Columbus Marathon. She reports the majority of
training is on flat terrain and denies any extreme hills in her workout regimens. Other workout activities include instructing
Pilates and yoga classes throughout the week. Following our initial evaluation her insurance dictated that her future
workup would need to be completed with providers outside of our network. Prompt evaluation from an orthopedic
surgeon within her coverage plan led to future diagnostic testing. She was diagnosed with a left femoral stress fracture
which was confirmed with a second orthopedic surgeon within her network. The treatment plan was continued rest and
shutting down all forms of physical activity for 3-6 weeks. With the Columbus Marathon now two weeks away and
having been provided this unfortunate news she returned to our office for an additional opinion. Her past medical history
is negative for any previous stress injuries. She denies practicing any special diets. All other review of systems was
negative.
Physical Exam: She appears healthy, no apparent distress, and walks with a normal gait. All vital signs are normal.
Inspections of bilateral hips demonstrate no obvious deformities, effusion and appear stable without evidence of
dislocation. There is normal flexion, extension, internal and external rotation of the hip bilaterally. Pain is present with
resisted hip flexion, resisted adduction and terminal abduction.There is tenderness to palpation along middle third of the
femur. There is no tenderness to palpation along the greater trochanteric bursa, ASIS or AIIS. The piriformis and hamstring
muscles demonstrate normal flexibility and are nontender to palpation. Strength is 5/5 throughout with intact distal
neurovascular examination. Remainder of the multi-system physical exam is normal.
Differential Diagnosis: 1.) Stress Fracture: Femur (Shaft/Neck)/Pubic Ramus/Acetabulum 2.) Muscle Strain:
Adductor/Quadriceps/Sartorius/Gracilis 3.) Muscle Contusion 4.) Adductor Insertion Avulsion Syndrome (Thigh Splints)
5.) Neoplastic Process/Primary Bone Tumor 6.) Osteomyelitis
Test Results: X-Ray - B/L Hip & Femur: No acute findings identified. MRI - Pelvis: Stress related bone marrow edema in
the left ischial tuberosity at the attachment of the common hamstring. No avulsion fracture or tear of the left common
hamstring tendon. MRI - Left Thigh: Stress related cortical edema and periosteal edema along the medial cortex of the
mid left femoral diaphysis without a stress fracture. The muscles in the left thigh are normal.
Final Diagnosis: Left Adductor Insertion Avulsion Syndrome
Treatment: She was cleared to compete in the Columbus Marathon with the final days of her training regimen allowing
for cross training with biking and swimming.
Outcome: She completed the Columbus Marathon without any complications and set a new personal record of 3:28:00.
Follow-Up: Following the marathon she was instructed to abstain from running for 3-4 weeks. She was enrolled into
physical therapy with emphasis on adductor strengthening. She continued cross training guided by the presence of pain.
She remains pain free today.
Knee Pain in a Cross Country Runner
Authors: Joseph Mitchell, MD; Jason Hageman, MD; Robert Hosey, MD
Affiliation: University of Kentucky, Lexington, KY
History: A 12 year-old female cross country runner presented in consultation from her pediatrician with right knee pain
and swelling of 3 weeks duration. She related the start of her pain to a twisting injury during a fall while carrying items into
her house. Her pediatrician evaluated her at the time and noted right knee and calf were swollen. Radiographs were
negative, and an ultrasound was negative for DVT. Also, blood work including a complete blood count, comprehensive
metabolic panel, creatine kinase, sedimentation rate and C-reactive protein were normal. Her pain was described as
intermittent, diffuse, 7/10 on pain scale, and sharp in nature. Endorsed popping and clicking, but denied locking or
catching symptoms. The pain worsened with any weight-bearing activity. Ibuprofen provided minimal relief. Her family
history is significant for a brother with knee "arthritis".
Physical Exam: Healthy-appearing female with normal BMI. Afebrile. Examination of the right lower extremity revealed
normal alignment of the leg without erythema. Full range of motion of the right hip and ankle without pain. A small
effusion was present in the right knee. Palpation revealed medial joint line tenderness. She maintained full active range of
motion of the knee. Negative Lachman's, negative posterior drawer and stable to varus and valgus stress at 0° and 30°.
McMurray's, patellar apprehension and the quadriceps inhibition tests were negative. Strength testing of right lower
extremity was normal though discomfort with this. Neurologically, she had intact sensation to light touch in the deep
peroneal, superficial peroneal, saphenous, sural, and tibial nerve distributions. She had 2+ dorsalis pedis and posterior
tibial pulses.
Differential Diagnosis:
Meniscal or articular cartilage injury
Pigmented villonodular synovitis
Juvenile rheumatoid arthritis
Spondyloarthropathy
Infectious synovitis
Test Results: X-ray of right knee and tibia/fibula: --normal radiographs MRI right knee without contrast: --Large
effusion of right knee joint which communicated with a complex popliteal cyst that dissected inferiorly superficial to the
medial head of the gastrocnemius. --Otherwise, no internal derangement of knee including normal ligaments, menisci,
and cartilage Laboratory testing: --normal CBC, CMP, ESR, CRP --normal TSH --Antinuclear antibody was positive in a
speckled pattern at a titer of 1:320. Anti-Smith, anti-RNP, anti SSA, and anti-SSB antibodies were negative --negative
Rheumatoid factor --positive HLA-B27
Final Diagnosis: HLA-B27-associated Juvenile Arthritis
Treatment: She was referred to Pediatric rheumatology for management. On initial consultation, she was started on
Mobic 7.5 mg daily.
Outcome: At one month follow-up, she was still symptomatic. Decision was made to inject her right knee with
Aristospan, and she was advised to continue taking Mobic daily. At three month follow-up, she was completely
asymptomatic and her knee effusion had resolved. She was transitioned to taking Mobic on an as needed basis.
Follow-Up: She and her family made the decision not to compete in cross country this fall. Her six month follow-up with
Pediatric rheumatology is in one month.
A Jaw Dropping Play
Authors: Anthony Shadiack, DO; Jason Womack, MD
Affiliation: Primary Care Sports Medicine, Rutgers-Robert Wood Johnson, New Brunswick, NJ
History: A fifteen year old male high school wide receiver was seen on the sidelines at a football game after being hit
with an illegal, head-first, tackle to the left side of his helmet. The athlete was experiencing headache and facial pain. Pain
was achy, 8/10, no radiation, and worsened with attempting to open his mouth.
Physical Exam: On the sidelines the patient was found to be in moderate painful distress. His face was without bruising
or swelling. He had normal teeth occlusion and had no teeth avulsions. His mandible was minimally deviated to the left.
He could not open his mouth widely as it was causing him significant pain. He had tenderness to palpation of the area
overlying the temporomandibular joint (TMJ). The TMJ could not be palpated on the left side. He had normal ROM of his
neck and had no tenderness. His vision was normal. Muscle strength, sensation, and cranial nerves were intact.
Differential Diagnosis:
Mandibular fracture
Tripod fracture
Zygomatic arch fracture
Facial fractures (including Le Forte)
Orbital fractures
Nasoethmoidal fracture
Traumatic hemarthrosis of TMJ
Temporomandibular joint dysfunction
Mandibular dislocation
Fractured tooth
Temporomandibular joint sprain or strain
Test Results: CT head: Mandibular fracture and dislocation
Final Diagnosis: Mandibular fracture with dislocation
Treatment: Maxillo-mandibular fixation
Outcome: The patient tolerated the surgical procedure well and is still undergoing fixation
Follow-Up: This will largely depend on the extent of injuries and how well the patient progresses. Light activities can
begin during the fixation process. Return to full contact can occur roughly 8 weeks post injury.
Curious Case of a Foot Drop
Authors: Jay Umarvadia, MD, MBA; Steven Ogden, MD; Emilio Russo, MD
Affiliation: Louisiana State University Health Sciences Center (LSUHSC) - New Orleans; Rural Family Medicine
Residency, Bogalusa, LA
History: We present a healthy 18 year old Caucasian male who enjoys playing instruments, namely the piano and
keyboard. The patient reported he was in his usual state of health until two weeks prior to presentation when he began to
experience pain in his back. The pain was mild and the patient attributed to muscle secondary to moving heavy musical
equipment a few weeks prior. He presented to our rural family medicine residency clinic with increasing pain in his lower
back. Initially, his physical exam was essentially normal except for mild spinal point tenderness. The patient was prescribed
NSAIDS for lower back pain. After taking the medication, the patient returned to the family medicine clinic with similar
ongoing complaints. The patient was given a steroid injection with local anesthethic for presumed muscle spasm with
immediate relief. The patient was then sent home with two weeks of NSAIDS and muscle relaxants with a two week followup The patient still had lumbar back pain with minimal relief from NSAIDS and muscle relaxants. At this point, the patient
had x-rays and referred to physical therapy. Prior to seeing physical therapy, the patient's pain had become progressively
worse and he noticed a "knot in his pelvis", specifically a painful lump in his left buttock that felt firm and pain his lower
back, so severe that it awoke him from sleep and he experienced shooting pains down his left leg Furthermore, the
patient began to have a foot drop on the left side but did not return to clinic for further evaluation. The patient began to
use a cane to ambulate He was then evaluated by physical therapy, who further noted his neurologic deficits and mass in
his left buttock. The patient was then seen by family medicine clinic and immediately sent for an MRI of the pelvis that
same day. The MRI revealed a large lobulated densely enhancing mass appreciated left gluteal, posterior inferior
hemipelvis extending between the iliac and sacral bones with some abnormal enhancement therefore concerning for early
osseous involvement. Immediately, the patient was then sent to Radiation Oncology at which point he had developed
constipation and some difficulty with urination in addition to worsening leg pain, weakness and loss of sensation. The
patient was put on oral dexamethasone for symptomatic relief. The patient underwent CT guided biopsy. The pathology
was consistent for Ewing Sarcoma. The patient is currently undergoing aggressive chemotherapy at St. Jude Children's
Research Hospital.
Physical Exam: General: AAOx3, No acute distress HEENT: NC/AT, EOMI, PERRLA, no lymphadenopathy CV: S1/S2
positive, Regular rate and rhythm, no murmurs, rubs or gallops Lungs: CTA B/L, no wheezing, rales or rhonchi Abdomen:
Soft, nontender, nondistended, BS+ Ext: Large palpable mass noted in left gluteal region Neuro: Hyperreflexia on left
lower extremity (achilles, patellar tendons), Normal DTR on Right, foot drop on left with gait disturbance
Differential Diagnosis:
Neuroblastoma
Non-Hodgkin Lymphoma
Nonrhabdomyosarcoma Soft Tissue Sarcomas
Osteomyelitis
Osteosarcoma i.e. Ewing Sarcoma
Rhabdomyosarcoma
Test Results: Tests performed - pertinent results in case history, CBC, Blood cultures, CRP, ESR, LDH, Cytogenetic and
molecular studies, Histology, MRI Pelvis, Xray Pelvis, CT Chest, PET scan
Final Diagnosis: Ewing Sarcoma
Treatment: Chemotherapy with alternating courses of vincristine, doxorubicin and cyclophosphamide; and ifosfamide
and etoposide
Outcome: The patient is undergoing chemotherapy and extensive physical therapy in hopes to make a full recovery from
becoming wheelchair bound due to his deficits.
Follow-Up: The patient continues to play the keyboard, however not the piano due to his inability to use the foot pedal.
Platelet-Rich-Plasma Injection Therapy for Morton&#039;s Neuroma
Authors: Benjamin Oshlag, MD
Affiliation: Resident, Department of Emergency Medicine, Drexel University, Philadelphia, PA
History: A 42-year-old male presented to Sports Medicine clinic for ongoing multifactorial right foot and ankle pain over
the prior 2 years. The patient had previously seen both Podiatry and Orthopedics, with a work-up showing degenerative
joint disease of the 1st MTP, sesamoiditis, an intermetatarsal neuroma, and tarsal tunnel syndrome. He had had no success
with attempted treatments including taping, orthotics, steroid injections, and immobilization in a walking boot.
Physical Exam: On initial evaluation, he was found to have pain with forefoot squeeze and tenderness to palpation
between the 1st and 2nd MTP joints, as well as pain with passive range of motion of the 1st MTP. The patient also had a
positive tinel's in the tarsal tunnel. The rest of his exam was otherwise unremarkable.
Differential Diagnosis: Morton's Neuroma, Tarsal Tunnel Syndrome, Sesamoiditis, Degenerative Joint Disease
Test Results: The patient was noted on prior MRI to have a septated cystic lesion adjacent to the flexor tendons in the
plantar soft tissues, which was visualized again on office ultrasound. He also underwent an EMG, with normal results.
Final Diagnosis: Morton's Neuroma
Treatment: The patient initially received an ultrasound-guided injection of corticosteroids at the 1st MTP joint, with
minimal improvement in symptoms. He then received an additional ultrasound guided steroid injection into his tarsal
tunnel. On follow-up, he continued to report pain between his 1st and 2nd MT heads, with no improvement from the
corticosteroids. He requested platelet-rich-plasma (PRP) injection, which he had previously had for unrelated symptoms,
for treatment of the neuroma, and was injected under ultrasound guidance.
Outcome: This case demonstrates the potential benefit of PRP use in the treatment of neuromas. The indications for use
of PRP in the treatment of musculoskeletal complaints have expanded in recent years, and standard treatments had been
unsuccessful in relieving the patient's symptoms. PRP has previously been shown to be effective in nerve-related pain, and
further evaluation of its effectiveness in the treatment of Morton's neuromas is warranted.
Follow-Up: Over the next 1-2 months, the patient gradually increased his activities, with 90-95% improvement in
symptoms and return to full activity. The following year, the patient reported exacerbation of pain after going bowling
without orthotic inserts, and again received an ultrasound-guided corticosteroid injection at the 1st MTP. The patient
returned for follow-up 6 weeks later reporting minimal improvement in symptoms, and again received an ultrasoundguided PRP injection between the 1st and 2nd MTP heads On subsequent follow-up, the patient reported full resolution of
symptoms, and was back to his usual level of activity, playing basketball 3 times a week
Hockey Player Status Post ACL Reconstruction Presents with Arm Pain
Authors: John M. Childress, MD
Affiliation: University of Cincinnati, Cincinnati, OH
History: 20 year old hockey player presents to the sports medicine clinic status post ACL reconstruction one week prior
with a complaint of left shoulder pain Patient has no other pertinent past medical history He complained that 2-3 days
after his ACL repaired that he started to have nausea and vomiting, and upper body pain Patient stopped taking the
Celebrex and Vicodin and the vomiting stopped The pain then subsequently started to localize around the left shoulder
region Patient denies any trauma to this area Patient denies any fevers or chills or prior shoulder or neck problems
Patient denies having numbness or weakness of the extremity.
Physical Exam: General: Patient is alert and oriented x3 in no apparent distress. HEENT: Normal Cardiovascular:
Regular rate and rhythm no murmurs. Equal pulses in all extremities Lungs: clear to auscultation bilateral. Abdomen:
Soft, nontender, no masses Neuro: Cranial nerves II through XII intact. No focal deficits. Muscle strength 5 out of 5 in all
extremities. Sensation grossly intact Musculoskeletal: There is no pain to palpation over the cervical spine. Patient has full
range of motion of his neck without pain. There is no paraspinal tenderness. The left shoulder is without any deformity or
swelling. Patient has mild tenderness to palpation along the anterior deltoid and scapular region on the left. Patient shows
full range of motion in the shoulder but with pain at end of range of motion in abduction, forward flexion and internal
rotation. There is no significant weakness of the rotator cuff tested, although irritation is noted. Right knee shows mild to
moderate effusion with a clean, dry incision over the anterior knee.
Differential Diagnosis:
1)
2)
3)
4)
5)
6)
7)
8)
Myofascial irritation
cervical radiculopathy
rotator cuff tendinopathy
iatrogenic intraoperative injury
neuropathy
brachial plexus injury
Parsonage-Turner syndrome
Brachial plexus mass
Test Results: Chest x-ray: Normal left shoulder x-ray: Normal CBC and complete metabolic panel: Normal. Lyme
titer negative VDRL: Negative Cytomegalovirus PCR: Negative MRI cervical spine:, C6-7, a small right paracentral disc
protrusion is observed but there is no evidence of cord compression or nerve root displacement MRI left brachial plexus:
Intramuscular edema involving the supraspinatus and infraspinatus muscles on the left. Mild left shoulder girdle muscle
atrophy is identified involving the supraspinatus and infraspinatus muscles. Suggestion of early denervation change. No
evidence of brachial plexus region mass Cerebral spinal fluid analysis: Within normal limits. EMG left upper extremity:
Irritability and no motor unit function seen at the infraspinatus and supraspinatus muscles only, and marked decreased
number of motor units seen in the deltoid muscle with attempted full muscle contraction today. Normal nerve studies of
the left median and ulnar nerves are seen
Final Diagnosis: Parsonage-Turner syndrome
Treatment: Initial treatment regimen was started with a 10 day prednisone taper as well as gabapentin 300 mg 3 times
a day for 10 days. After consultation with a neurologist from the Mayo Clinic it was recommended that the patient
receive 1000 mg of IV Solu-Medrol 5 times a week for 4 weeks.
Outcome: Patient still remains symptomatic with marked atrophy and weakness of the deltoid and supraspinatus region.
Follow-Up: The patient has not yet returned to play and is undergoing physical therapy, working on strengthening and
range of motion exercises for the left shoulder
Snap, Crackle and Pop: An Unusual Case of Shoulder Pain in an Active 64 Year Old Male After a
Bicycle Accident
Authors: Michael Murphy, MD; Owen Speer, DO; Charles Lamar, MD
Affiliation: Palmetto Health/University of South Carolina Department of Family and Preventative Medicine,
Division of Sports Medicine, Columbia, SC
History: Patient is an active 64YO male with past medical history of cervical radiculopathy who presented with a chief complaint of left
sided shoulder pain for four days following a bicycle accident. Patient is a causal cyclist who decided to ride a recently acquired older
model bicycle. While he was cycling, his brakes failed and he fell over the curb into the grass onto the left side of his chest. He was not
wearing a helmet, but only had a small abrasion on his forehead and had no loss of consciousness, vomiting, somnolence, headaches,
focal neurologic symptoms or any other signs of traumatic brain injury. He had an aching shoulder pain which was worse with
movement and an aching pain in his left, anterior-superior chest wall that was worse with coughing and deep inspiration. He described
mild shortness of breath with exertion, but breathed comfortably at rest. He was treating his pain at home with ibuprofen 800mg TID
with good relief. All of his symptoms had been improving over the four days prior to being seen in the office.
Physical Exam: Vital Signs: HR88, BP152/72, RR16, Pulse ox 98% on room air, temperature 97.6°F. General: Well appearing and in no
acute distress. Neck: Supple with crepitus over anterior neck and moderately tender to palpation over the anterior neck without
tenderness of the spine. Respiratory: Non-labored with diminished breath sounds over the left lower lung field. Symmetrical chest wall
expansion without paradoxical breathing. Chest: Subcutaneous crepitus palpated from anterior neck inferiorly to shoulders, anterior
chest, and superior abdomen. Patient had mild tenderness to palpation with mild ecchymosis over the left superior-lateral chest wall.
Cardiovascular: Normal rate, regular rhythm with 2+ pulses equal in all extremities. Musculoskeletal: Mild tenderness to palpation at
the right radial styloid with full range of motion of right wrist with 4/5 strength. Neurologic: No focal deficits present. Cranial Nerves IIXII intact with normal deep tendon reflexes.
Differential Diagnosis:
Pneumothorax
Esophageal perforation
Displaced rib fracture
Flail chest
Tracheal rupture
Pulmonary laceration
Bronchial rupture
Penetrating wound
Gas gangrene
Test Results: Chest x-ray showed numerous left sided contiguous over-riding rib fractures with extensive subcutaneous emphysema
and pneumomediastinum that was concerning for possible flail chest. X-ray of the soft tissue of the neck showed extensive
subcutaneous emphysema. CT scan of the chest showed acute displaced fractures of left sided ribs 2-6 with extensive
pneumomediastinum, hemorrhage into the anterior mediastinum and hemothorax with complete collapse of the left lower lung and
extensive subcutaneous gas, but negative for pneumothorax.
Final Diagnosis: Acute displaced fractures of the posterolateral segments of the second through sixth ribs, left second rib costal
cartilage fracture with marked displacement, extensive pneumomediastinum, moderate anterior hemomediastinum, and left lower lobe
hemothorax w
Treatment: Patient was admitted to the surgical trauma intensive care unit for workup and observation. No surgical intervention was
performed. Patient elected for conservative management of hemothorax instead of chest tube placement in light of mild
symptomatology. Pain was controlled with hydrocodone.
Outcome: Esophagram was negative for perforation. Repeat chest x-ray three days later showed decreasing subcutaneous emphysema.
Patient tolerated full diet and was discharged home on hospital day three.
Follow-Up: Patient followed up in trauma clinic 16 days after the initial injury and was cleared for light activity with gradual return to
normal activity as pain allowed. Patient followed up with his primary care physician 28 days after initial injury with return to normal
activity without any residual symptoms.
A Rare Cause of Claudication in a 45 Year-Old Runner and Cyclist
Authors: Clark Madsen, MD; Stephen Scharmann, MD
Affiliation: McKay-Dee Family Medicine Residency, Ogden, Utah
History: A 45 year-old professor who is an avid cyclist and runner began experiencing right calf pain resembling
claudication during long rides and runs. The pain resolved with rest. He also developed parasthesias and foot drop which
improved after rest. His symptoms continued to worsen to the point that he sought medical attention at the university
training room.
Physical Exam: There were no skin changes in the right leg, no palpable cord or edema noted. No bony or soft tissue
tenderness. He had normal strength other than slight weakness with plantar flexion. He had a negative straight leg raise
and otherwise normal neurological exam. Popliteal pulses were brisk and symmetric as were the posterior tibial and
dorsalis pedis pulses.
Differential Diagnosis: Exertional compartment syndrome, tibial stress fracture, peripheral nerve entrapment, lumbar
nerve impingement with radiculopathy, atherosclerotic peripheral vascular disease, popliteal artery entrapment syndrome,
and cystic adventitial disease.
Test Results: EMG testing was normal. Bilateral compartment testing showed elevated anterior compartment pressures
and a bilateral fasciotomy was performed. This did not resolve the symptoms and the patient next underwent an exercise
arterial ultrasound that showed increased resistance with ankle dorsiflexion consistent with popliteal artery entrapment.
However, an MRI of the leg showed numerous multi-lobulated cystic lesions involving the popliteal artery implying a
diagnosis of cystic adventitial disease.
Final Diagnosis: Cystic adventitial disease of the popliteal artery was confirmed by surgical pathology. It is a rare
condition (~400 reported cases) presenting predominately in men 40-50 years of age. It involves cysts forming in the
adventitial layer of the popliteal are felt
Treatment: The patient was scheduled for popliteal artery resection. During the surgery the cystic structures were found
to be far more extensive than previously thought and were adherent to the surrounding tissue. Therefore a saphenous
vein graft was harvested and a femoral-tibial artery bypass was performed. The graft had to be passed anterior to the tibia
to reach the tibial artery.
Outcome: Seventy percent of patients do well after resection of the affected artery segment, usually the popliteal artery.
Our patient required a complete bypass of the artery and developed a post-operative thrombosis requiring a
thrombectomy but recovered well afterwards.
Follow-Up: The patient recovered quickly and within weeks was running and biking again symptom free. He enjoyed
several months of improved symptoms before he developed venous congestion in his leg possibly related to his extensive
surgery. He is currently biking, but at reduced activity and requiring compression and lymphatic massage of his right leg
to reduce his symptom load.
An Interesting Confounding Factor to Knee Pain
Authors: Sufian Zayed, MD; Joseph Garry, MD
Affiliation: University of Minnesota Medical Center, Minneapolis, MN; University of Minnesota Department of
Family Medicine and Community Health, Division of Sports Medicine
History: MH is a 67 year old Somali speaking male who presents to the clinic with complaint of worsening right knee
pain for approximately 3 months. This is different from his chronic joints pain. He reports that he is no longer able to enjoy
his daily walks. The pain is located around his patella, aching in nature, except when severe, he reports it as burning. At
rest, the pain is 2-3/10, with worsening to 6-7/10 with activity. Pain is exacerbated by prolonged walking or sitting,
ascending stairs, weight bearing and change in temperature. It improves with changing position, ice and honey. The
patient was unsure about his medical history, and after extensive medical records review, he's found to have a significant
history of Paget's disease of bone, HTN, and PTSD. He is married, retired, denies EtOH, tobacco abuse or illicit drug use.
Physical Exam: CONSTITUTIONIAL: healthy, alert and no distress
Vital signs stable HEAD: NC, AT, no masses,
tenderness or abnormalities Heart, lung, abdomen, skin normal GAIT: normal Stance: normal NEUROLOGIC:
Normal muscle tone and strength, reflexes normal, sensation grossly normal. MSK: Right knee: FROM, strength normal,
sensation normal, no effusion, positive medial/lateral patellar facet tenderness, positive patellar inhibition, negative
apprehension; no pain or laxity with valgus stress testing in 0° or 30° of knee flexion, no pain or laxity with varus stress
testing in 0° or 30° of knee flexion, negative lachman, negative posterior drawer, diffuse medial joint line tenderness,
Negative lateral joint line tenderness, negative bounce test Left knee: normal exam
Differential Diagnosis:
Paget's disease progression
Osteoarthritis
Fibromyalgia
PTSD
Test Results: Previous Imaging studies:
NM Whole body bone scan 8-2-07 Abnormal bone finding. Cannot rule out
atypical fibrous dysplasia, atypical Paget's disease, or metastatic disease including prostate cancer. Likely degenerative
changes in the left ankle and right first metatarsal phalangeal joint. XR Pelvis with left hip 6-9-09 Cortical and
trabecular thickening within the proximal left femur. Similar in appearance to the study performed on 8/13/2007.
Consistent with known history of Paget's disease. 2. 2.4 cm right sided soft tissue nodular density as described above,
recommend clinical correlation. Most recent imaging study: XR 7/17/2014 - 3 views Osteophytic spurring along the
medial aspect of the left knee medial tibial plateau, small right knee joint effusion, and spurring in the patellofemoral joint
compartment
Final Diagnosis: Osteoarthritis in context of Paget's disease of the bone
Treatment: Patient has been seen in clinic on multiple occasions for joint and bone pain, without significant relief, but
with multiple XRs done and with symptomatic treatment. Patient's symptoms appears to be more consistent with
Osteoarthritis of the knee than progression of Paget's disease. Discussed options for treatment for osteoarthritis and he
elected for use of intermittent analgesics and PT for instruction in patellar taping. He was given a prescription of Ibuprofen
600 mg Q8H PRN for pain. No concern for PTSD associated somatoform of fibromyalgia given that the patient has a
documented organic cause of his previous visits.
Outcome: Improved pain and ability to return to baseline physical activity with tolerable pain levels
Follow-Up: Symptoms under control on follow up visit, knee taping appears to have helped significantly in allowing him
to return to his daily walking routine and activities. Patient will follow up if pain worsens, at which point, a corticosteroid
injection would be recommended
Midline Neck Pain - Never Normal
Authors: John Breck, DO; Seth Smith, MD
Affiliation: University of Florida, Gainesville, FL
History: 14 year old male junior varsity football player presented to the training room after practice complaining of
headache, nausea, and dizziness after sustaining a head to head collision during hitting drills. He remembered the
previous hits in practice but did not remember the third time through the drill. He remembers being on the ground. He
has two prior concussions, the most recent being approximately one year ago. He believes this feels very similar. He
denies any weakness, numbness, or tingling. No vomiting or vision changes. No photophobia. Feels his gait and balance
are normal.
Physical Exam: Gen: Well-nourished young man, no acute distress, but does seem somewhat foggy and has difficulty
following directions during the exam. HEENT: normocephalic, no obvious trauma, swelling or bruising of the scalp or
head. Neck: Mild cervicaal spine midline tenderness from C2-C4. Eyes: Clear, White Sclera, no injection. Pupils are equal,
round, and reactive to light. Extraocular muscles are intact, but there is some delay in tracking. Posterior oropharynx is
clear, moist mucous membranes. Neuro: CN2-12 intact. 5/5 muscle strength in upper and lower extremities. Tandem gait
is slow and mildly off balance. No past pointing on finger to nose. Vestibular exam reveals increased HA with vertical gaze
tracking. Scat3 Scores (post injury): 31, 5/5, 13/15, 3/5, 0/1/1, 1/1, 4/5
Differential Diagnosis:
Sports related concussion with cervical muscle strain
cervical ligamentous injury
posterior cervical elements bone contusion
fracture
Test Results: Cervical spine XR: On the odontoid view there is offset of the lateral masses of C1 on C2. In this patient
with point tenderness at C1-C2 recommend CT cervical spine for further evaluation. Cervical Spine CT: Redemonstrated
asymmetry of the lateral masses of C1 on C2 and straightening of the cervical alignment concerning for ligamentous injury
around the dens. Recommend MRI to further characterize. No acute fracture within the cervical spine. - Congenital fusion
of the C2-C3 vertebral body and facet joints bilaterally. MRI: Findings are consistent with transverse and alar ligament
injuries on the left between the dens and the lateral mass of C1. - Developmental fusion anomaly of the posterior
elements of C2 and C3. Incidentally this may have contributed to altered biomechanics predisposing the patient to the
above ligamentous injury. - Low volume central disc extrusion at C5-C6, which does not cause significant canal stenosis.
Final Diagnosis: 1. Sports Related Concussion
2. Alar and transverse ligament strain in the setting of congential fusion
of C2-C3
Treatment: Neurosurgery referral. Recommended cervical collar continuously for two weeks then repeat evaluation.
Outcome: - Neck Pain resolved, Cervical ROM and strength back to normal. - Concussion symptoms continue
Follow-Up: Repeat neurosurgery evaluation at two weeks post injury revealed normal appearing flexion extension
cervical spine xrays and the patient was cleared to return to sports from a cervical spine perspective. This recommendation
brings up an interesting discussion reguarding return to collision sports in patients with congenital spine abnormalities.
Torg and Ramsey-Emrhein addressed this in the late ninties and noted that upper cervical spine congenital fusions were
an absolute contraindication to participating in collision sports. However, the cervical spine and return to play is a moot
point at this time as the patient is still experiencing concussion symptoms and has not returned to any sports related
activity.
Collegiate Swimmer with Shoulder Pain
Authors: David K. Lisle, MD
Affiliation: Division of Sports Medicine, Department of Orthopaedics and Rehabilitation University of Vermont
Medical Center, Burlington, VT
History: A 19 year old right-hand-dominant female collegiate swimmer presented to the training room with a 2 week
history of gradual onset worsening right shoulder pain. Her pain localized to the shoulder, clavicle and axilla. She was
unable to swim and she reported having symptoms of shoulder pain with day-to-day activities such as washing her hair.
Her pain was aching and radiating to the arm and worsened with overhead or repetitive use. She denied trauma or prior
shoulder or neck problems. She denied left sided symptoms. Quick DASH: 75; PMH: negative; PSH: negative; Medications:
None; Allergies: NKDA
Physical Exam: 5 feet 5 inches, 121 lbs Neck: no carotid bruits, full range of motion, negative Spurlings Right shoulder
exam: no atrophy, discoloration or asymmetry. On palpation, mild tenderness to supraspinatus insertion. No tenderness
to bicipital groove, AC joint or coracoid process. Range of motion: 180° forward elevation, 180° abduction, 80° external
rotation with arm at side and 110° external rotation with arm at 90° abduction. Internal rotation to T2 bilaterally. Strength
testing: 5/5 empty can, external rotation and subscapular lift off. 1+ anterior and 1+ posterior translation on load shift
testing and 1+ sulcus sign. Hawkins test positive. Neurologically intact to light touch in the radial, ulnar and median nerve
distributions. Radial and ulnar pulses 2+.
Differential Diagnosis:
Multidirectional glenohumeral instability
Impingement syndrome
Cervical radiculopathy
Test Results: Plain films of right shoulder: normal.
Final Diagnosis: Right multidirectional glenohumeral instability
Treatment: Rotator cuff, scapular stabilizer and postural rehabiliation was initiated. After 3 weeks of treatments,
symptoms improved and she returned to swimming. Symptoms returned and worsened. She complained of pain in the
right shoulder with aching down the medial upper arm and forearm with numbness to the 4th and 5th digits. She
described an inability to keep her fingers together while swimming. Right shoulder examination revealed tenderness to
the supraclavicular region. Full range of motion and 5/5 motor strength throughout except weakness to the interossei
musculature and diminished sensation to light touch in the ulnar nerve distribution. Adson's test positive. Straight arm
raise test positive with attenuation of radial pulse and positive ulnar nerve paresthesia. Roos test positive. Cervical spine
films revealed a cervical rib on the right. EMG testing of right upper extremity: negative. Ultrasound duplex revealed no
vascular compromise. A guided lidocaine injection to the scalenes was performed with resolution of symptoms and
ability to swim without pain for 20 minutes. Revised diagnosis: neurogenic thoracic outlet syndrome. Rehabilitation
was focused on Edgelo protocol with breathing and postural work. She continued shoulder rehabilitation. After 3 months,
she noted minimal improvement in symptoms with ADLs and was unable to swim. Quick DASH remained 75. She
underwent surgery by supraclavicular approach involving first rib resection, anterior and middle scalenectomy and brachial
plexus neurolysis.
Outcome: Her post-operative course was uneventful. She noted immediate decrease in her shoulder and arm pain.
At
her 2 month post-operative check, she had no shoulder pain. Bilateral upper extremity EMG were normal with provocative
maneuvers. On examination, Adson's test and straight arm raise test were negative. Quick DASH was 11. She was released
to full activities.
Follow-Up: At her 6 month post-operative evaluation in the training room, she brings her cervical rib in a bell jar. She
has no shoulder pain. She has returned to competitive swimming without limitations. Her only examination finding is mild
weakness to the interossei musculature with trouble keeping fingers together when swimming. She has been allowed to
buddy tape her 3rd, 4th and 5th fingers for training and competition.
In a Football Player with a Seronegative Spondyloarthropathy, Does Exercise Accelerate Erosive
Joint Disease?
Authors: Lauren Porras, MD; Dave Berkoff, MD; Mario Ciocca, MD; Harry Stafford, MD
Affiliation: University of North Carolina Chapel Hill, Chapel Hill, NC
History: The patient is an 18yo football player with history of hospitalization for complex regional pain syndrome who presents with a
one-year history of back pain and foot pain. He suffered a trauma to the right great toe when he planted and cut on his right foot and
felt a pop and had acute pain in the right great toe. He was diagnosed with turf toe and treated with a boot and crutches. An MRI of the
right foot showed an incidental finding of synovitis at the 5th MTP with erosive changes. Three days after his right toe trauma, he
developed pain in the left foot along the plantar surface of the MTPs, worse with activity He endorses morning stiffness in the mid
lower back lasting 5-10 minutes, and bilateral feet swelling roughly every other day, which he manages with elevation and ice. He
reports chronic swelling around the right SC joint, but denies pain.
Physical Exam: General: Athletic young AA male in no distress CV: RRR, no m/r/g Pulm: CTAB Skin: no rash MSK: TTP at
sternoclavicular joint. R SC joint is swollen, but not warm or tender. There is normal range of motion of the neck, shoulders, wrists, and
hands. There are no nodules or synovitis of the upper extremities. There is good range of motion of the hips and knees without any
synovitis or tenderness. There is exquisite tenderness at right 1st MTP, also very tender at 2nd, 3rd and 5th MTP on right. Tender at left
3rd and 4th MTP. There is definite synovitis of the affected MTPs on both feet. No ankle or midfoot TTP.
Differential Diagnosis:
Rheumatoid Arthritis
Systemic Lupus Erythematosus
Still's Disease
Psoriatic Arthritis
Seronegative Spondyloarthropathy
Reactive Arthritis
Crystal-induced Arthritides
Primary Osteoarthritis
Test Results: CBC: Normal HLA-B27: Positive CCP: Negative ESR: 2 Rheumatoid Factor: Negative CRP: 0.7 Urinalysis: 1.019/2+
Leukocyte Esterase/29 WBC Urine PCR: + Chlamydia Xray of Foot: Erosion of distal-lateral fifth metatarsal MRI of Foot: Effusion at
the first MTP joint and tenosynovitis of the flexor hallucis longus. Effusion at the fifth MTP joint, along with erosive changes and
abnormal signal and enhancement throughout the fifth metatarsal shaft. CT of Chest: Asymmetric erosive change with associated
osseous overgrowth at the sternoclavicular joints, right worse than left.
Final Diagnosis: HLA-B27 Positive Seronegative Spondyloarthropathy/Chronic Reactive Arthritis
Treatment: The patient's symptoms improved on Prednisone but there was concern for the large load on his joints that he would have
as an offensive lineman. He eventually was transitioned to Etanercept.
Outcome: There was concern for progression of his erosive arthropathy with high-impact exercise and his risk of infection given
frequent contact with others. In a study by De Jong et. Al published in 2004, it was noted that the progression of radiologic joint
damage of the hands and feed were not increased by long term high intensity weight bearing exercises. In fact, these exercises may
have a protective effect of the joints of the feet. In a study published in 1994 by Hakkinen et al, the authors found that dynamic strength
training in early arthritis increases the neuromuscular performance without detrimental effects on disease activity or joint damage.
Follow-Up: It was felt given previous studies, the patient could return to football. With Etanercept, clinical swelling of his SC joint, foot
pain, and back stiffness resolved, and he was able to maintain weight/lift/practice and compete a whole season without difficulty.
Ice Hockey Player and Winter 2014: Shoveling More Than Snow?
Authors: Christina Master, MD; Matthew Grady, MD
Affiliation: Division Pediatric Orthopedics and Sports Medicine, The Children's Hospital of Philadelphia,
Philadelphia, PA
History: 14 yo ice hockey player complains that his right arm hurts after shoveling driveway. Mom thinks that he just
does not want to shovel snow. He further complains that his arm is numb and tingling after shoveling snow. However,
upon further history, he has pain in right arm when blood pressure taken in that arm and also has weakness with pullups
in that arm - he can only do pull-ups with band-assist. Patient generally avoids right arm overhead activities due to
discomfort. Is right-hand dominant for writing but plays ice hockey left-hand dominant. He does not notice any problems
when playing ice hockey. Past history of snoring (but not OSA) and Supraventricular tachycardia
Physical Exam: BP 155/70 on right - painful - and 126/58 on the left No cardiac murmur Positive Adson's test, Halstead
maneuver, Wright hyperabduction test for pain and diminished radial pulse Use of scalenes with deep breaths (vs.
diaphragm)
Differential Diagnosis: BP 155/70 on right - painful - and 126/58 on the left No cardiac murmur Positive Adson's
test, Halstead maneuver, Wright hyperabduction test for pain and diminished radial pulse Use of scalenes with deep
breaths (vs. diaphragm)
Test Results: Chest Xray - no cervical rib Cardiac echo for SVT revealed no coarctation of aorta Dynamic ultrasound
reveals compression of subclavian artery with Adson's test, Halstead maneuver and Wright hyperabduction test and
reproduction of notable right arm pain, weakness and numbness, tingling, no compression of brachial plexus with
maneuvers
Final Diagnosis: Thoracic Outlet Syndrome with hypertrophy of scalenes and compression of subclavian artery
Elevated BP on the right likely secondary to pain with claudication
Treatment: Physical therapy for stretching scalenes and pectorals, and trapezius, scapulothoracic strengthening exercises
to improve posture Breathing retraining to use abdominal muscles and not scalenes with deep breaths
Outcome: Improvement of symptoms with overhead activities after physical therapy
Follow-Up: Full return to ice hockey and shoveling snow
Non-Traumatic Paraplegia in a First Time Surfer
Authors: Vince Si, MD; Nayeema Chowdhury, DO; Joan Gold, MD
Affiliation: NYU Medical Center, New York, NY
History: 14 year old male with no significant medical history who presents with acute low back discomfort and
progressive bilateral lower extremity weakness during a surfing lesson. The patient and his family were on vacation and
were receiving surfing lessons for the first time. During his lesson, he noted an initial “twinge&rdquo; in his lower back
while working with smaller waves but continued on with the lesson. Shortly after, he had attempted to stand and catch a
larger wave but felt his lower back and legs give out. He landed onto the surfboard on his buttocks and continued to
experience weakness in his legs. He lowered himself back onto the board in a prone position and informed his surfing
instructor of his symptoms. The two remained in the ocean, and the instructor advised him to remain prone until the
discomfort subsided. After 20 minutes, the pain persisted, and the decision was made to stop the lesson and paddle back
to shore. The patient was able to stand up but was unable to walk secondary to weakness and pain, requiring the
assistance of others. He was taken to his car by his father and remained supine while the rest of his family members
finished their lessons for another 40 minutes. Upon return to their hotel, the patient noticed progressive weakness and
was now unable to stand on his own. His parents called EMS at this point, and he was brought to the ED.
Physical Exam: Vitals unremarkable, BMI 20.3. Gen: lying comfortably on exam bed, in NAD. Normal medical exam.
Neurologic exam: Motor: 5/5 strength in bilateral extremities throughout, 3/5 in hip flexion bilaterally, 2/5 hip abduction
and adduction, 2/5 knee extension and flexion, 0/5 ankle dorsiflexion and plantarflexion bilaterally. Sensory: diminished to
light touch from L1 dermatome downwards bilaterally, diminished to pinprick from L2 dermatome downwards bilaterally,
vibration and proprioception intact in all 4 extremities. Reflexes: 2+ and symmetric throughout.
Differential Diagnosis: Vascular: primary and secondary ischemia of the spinal cord (surfers myelopathy, anterior cord
syndrome, posterior cord syndrome), spinal hemorrhage (epidural, subdural, subarachnoid, intraparenchymal), vascular
malformations; inflammatory: transverse myelitis, multiple sclerosis, abscesses (epidural, subdural); disc prolapse;
neoplasm; Guillain-Barre Syndrome.
Test Results: Laboratory: complete blood count and chemistry unremarkable. CT Lumbar spine without contrast:
unremarkable. MRI Thoracic Spine: abnormally increased T2 signal within the central cord from T8-T11. Suggestion of a
small central disc protrusion at C5-C6 level that does not cause mass effect upon the cord.
Final Diagnosis: Surfer's Myelopathy.
Treatment: The patient was admitted to an inpatient pediatric floor and was managed conservatively without steroids.
He was placed on lovenox for DVT prohylaxis and an indwelling urinary catheter was placed for his neurogenic bladder.
Medical course was uncomplicated, and he then underwent acute inpatient pediatric rehabilitation at Rusk Rehabilitation.
Outcome: On admission to inpatient rehabilitation, he was classified as a T12 ASIA impairment scale (AIS) C. Upon
discharge home, he improved to a T12 AIS D and was at an independent or supervision level for his ADLs and ambulation
with lofstrand crutches and bilateral AFOs. He has also regained full recovery of his bowel and bladder function.
Follow-Up: He continued with outpatient rehabilitation services and also started hyperbaric oxygen therapy (HBOT) in
order to expedite recovery as it can promote angiogenesis and increase neuroprotective factors in the spinal cord. He
completed his outpatient therapies and received 20 sessions of HBOT and is currently independent in ADLs, and although
he remains with residual foot drop, he is currently ambulating without an assistive device. He has been cleared to return to
school but is currently restricted from participation in gym class and sports
Division 1 Prospect with a Shattered Dream and a Fourth Chance to Walk Again
Authors: Jason Perry, MD; Ricardo Colberg, MD
Affiliation: Andrews Sports Medicine and Orthopedic Center, American Sports Medicine Institute, Birmingham,
AL
History: An 18-year-old high-school senior linebacker presented for evaluation of left neck and shoulder pain. He
described a mild, constant throbbing in the left neck with radiation to the left shoulder, and weakness with decreased
sensation over the left shoulder. Symptoms were better with rest and worse with overhead activity. Four days prior
during a football game he hyperextended his neck during a tackle. He felt shooting, electric pain into all extremities. He
was unable or move both arms and legs. Symptoms lasted three minutes with gradual return to baseline. There was no
loss of consciousness. He was not spine-boarded or taken to the ER. A similar episode occurred two years prior, in which
he made a tackle and developed inability to move. He was spine-boarded, taken to the ER, and had a cervical spine CT
scan that was normal. He returned to play once his strength recovered. A third episode occurred two weeks prior to
presentation, in which he felt sharp neck pain with radiation and tingling to his arms. Prior episodes were diagnosed as
"stingers", and he was counseled that symptoms would resolve. Actively in pursuit of Division 1 scholarship, he returned to
play.
Physical Exam: On exam, there was tenderness to palpation over the left cervical paraspinal muscles with guarding.
Simultaneous neck flexion and side bending to the right reproduced symptoms. Cervical spine range of motion (ROM) was
normal. Spurling's test was negative. There was decreased sensation to light touch over the left C5 dermatome. Strength
was 5/5 and deep tendon reflexes were 2/4 throughout. Exam of the left upper extremity was otherwise unremarkable.
Differential Diagnosis:
cervical cord neuropraxia
cervical spinal stenosis
intervertebral disc herniation with cervical radiculopathy
cervical cord syrinx
brachial plexitis
Test Results: Radiographs of the cervical spine showed normal bony alignment with preservation of cervical lordosis, no
fractures, and no evidence of instability. Torg ratio was 0.86. The patient was held out from sports and cervical spine MRI
was obtained, which was read by an attending radiologist as normal. Review of images with an orthopedic spine surgeon
noted absence of cerebrospinal fluid from C2-C5 with mild enlargement of the spinal cord at C3-C4 consistent with
functional spinal stenosis. There was no cord edema. AP diameter measured 8.3mm at C3 and 8.5mm at C4, suggesting
congenital spinal stenosis. No syrinx, disc herniation, or evidence of nerve root injury was seen.
Final Diagnosis: Cervical cord neuropraxia secondary to cervical spinal stenosis.
Treatment: He was educated on absolute contraindications of contact sports and relative contraindications of limitedcontact sports given risk for quadriplegia.
Outcome: Symptoms were completely resolved at his 3-week follow-up visit
Follow-Up: In this patient with multiple episodes of transient quadriparesis and functional spinal stenosis on imaging,
return to contact sports is absolutely contraindicated. The patient was encouraged to participate in noncontact sports.
Preseason Palpitations in a Collegiate Football Player
Authors: Kimberly Lock, MD; Robert Hosey, MD
Affiliation: University of Kentucky, Lexington, KY
History: A nineteen year old collegiate offensive lineman presented to the training room with palpitations after a
preseason football scrimmage. He described the feeling as his "heart was beating harder than normal". He reported it
started about ten minutes after the scrimmage had ended and was continuous. He also endorsed shortness of breath and
difficulty taking a deep breath. He denied having any palpitations or chest pain during practice. He denied any prior
occurrence of similar symptoms. His past medical history was significant for a heart murmur when he was younger for
which he was cleared by a cardiologist to play high school football. He denied any family history of sudden cardiac death.
He denied any caffeine intake or use of supplements. He denied any recent alcohol intake or recreational drug use.
Physical Exam: Blood pressure: 130/82 mmHg. General: alert, oriented, mild distress, anxious. Cardiovascular:
irregularly, irregular rhythm. Rate 100s. No murmurs. No carotid bruits. Respiratory: lungs clear to auscultation bilaterally.
Respirations nonlabored. He was sent to the emergency department for further evaluation and management.
Differential Diagnosis:
1)
2)
3)
4)
5)
Atrial fibrillation
Frequent premature atrial contractions
Frequent premature ventricular contractions
Supraventricular tachycardia
Atrial flutter
Test Results: EKG: irregularly, irregular rhythm with rate of 94 bpm. QTc 425 ms. CBC normal. CMP significant for
sodium 149, potassium 3.9, creatinine 1.4, glucose 109. INR 1.2. TSH and free T4 normal. Echocardiogram: borderline
concentric left ventricular hypertrophy, ejection fraction 60% with normal wall motion and diastolic function. No thrombus.
No atrial septal defect. Left atrium normal in size. Valvular abnormalities included trace mitral regurgitation and trace
tricuspid regurgitation. 24 hour holter monitor: average heart rate of 71 bpm with a range of 36-176 bpm, few PACs, and
rare PVCs. An isolated rhythm of Wenckebach with 2:1 AV node conduction delay was also noted. No additional runs of
atrial fibrillation.
Final Diagnosis: Atrial fibrillation
Treatment: In the emergency department, he was given one liter normal saline bolus. Cardiology was consulted.
Chemical cardioversion was attempted with 1 mg ibutilide and 2 g magnesium sulfate IV push both of which were
unsuccessful. He was then given 10 mg metoprolol IV push and an additional dose of 1 mg ibutilide without successful
conversion to sinus rhythm. He rested in the ED and spontaneously converted to normal sinus rhythm several hours later.
Outcome: He was seen by cardiology four days after initial presentation and diagnosed with lone atrial fibrillation and
hypertension.He was counseled to stop all stimulants and started on amlodipine 5 mg daily for his hypertension.
Follow-Up: He was seen by cardiology one month later for follow up. He had no recurrence of symptoms of palpitations,
exercise intolerance, or lightheadedness. Amlodipine was increased to 10 mg daily for improved blood pressure control.
He was counseled that if he had any recurrence of symptoms in the future, he would need to undergo an
electrophysiology study with possible ablation. He was cleared for all activities one month after initial presentation and
played football for the remainder of the season without any recurrence of symptoms.
Chest Pain and Shortness of Breath in Collegiate Baseball Pitcher
Authors: Skye Heston, MD; Bret Jacobs, DO; Pamela MacNett, MS, ATC, LAT
Affiliation: Pennsylvania State Medical Center Family Medicine Program, Hershey, PA
History: 22-year-old right-hand dominant collegiate baseball player presented to athletic trainer with abrupt onset of
left-sided chest pain and shortness of breath during practice. The patient was throwing lightly and denied any injury or
trauma. Pain was worse with movement and deep breaths. In the training room patient appeared uncomfortable and had
mild tachypnea and tachycardia, other vital signs were normal. No significant past medical history except ADHD for which
he took Adderall. No significant family history. The patient was promptly taken to the ED at the recommendation of the
team doctor. In the ER, the patient had a chest x-ray and was given oxygen and IV pain medications.
Physical Exam: In general, he was alert and in moderate distress. Cardiac exam with tachycardia, normal S1 and S2 and
no murmurs, rubs, or gallops. No chest wall tenderness, ecchymosis, crepitus. No subcutaneous emphysema. Lung exam
with tachypnea and shallow respirations, otherwise normal breath sounds. Abdominal exam without tenderness, bowels
sounds present. Musculoskeletal exam with good strength in all extremities. Neurovascularly intact upper extremity and
lower extremity bilaterally.
Differential Diagnosis:
Spontaneous Pneumothorax
Pulmonary embolism
rib fracture
A-C separation
Muscle strain
Test Results: Initial chest x-ray without any acute abnormalities. CT chest with contrast revealed no signs of pulmonary
embolism, small left apical pneumothorax, and two nodules within the right lung which were most likely postinflammatory. Repeat chest x-ray in the ER four hours later showed unchanged small left apical pneumothorax.
Final Diagnosis: Spontaneous Pneumothorax
Treatment: Evaluated by CT Surgery Service in the ER. He was discharged from ER with oral pain medication and
recommendation to follow-up with team physician. After 3 days of rest, he presented to sports medicine clinic for
evaluation. He continued to complain of occasional discomfort and burning sensation in his left chest. He denied further
shortness of breath and did not required use of his pain medication. Chest x-ray obtained in the office again showed small
apical pneumothorax, as well as biapical subpleural blebs. Pneumothorax did appear slightly smaller than previous
evaluation. He was kept out of all athletic activities and scheduled for follow-up appointment with repeat chest x-ray in
two weeks.
Outcome: Patient traveled with baseball team for Spring Break trip, but did not participate in any activities. At follow-up
appointment chest radiograph showed complete resolution of pneumothorax. He was permitted to start light
cardiovascular activity and light throwing at that time. Four weeks from injury he was doing more intensive activities, but
still not back to full participation.
Follow-Up: Four and a half weeks after injury he threw his first bullpen session. He returned to full baseball participation
five weeks after injury. He had regular follow-up with trainer and team doctor in the training room, but had no further
issues or symptoms after resolution of the pneumothorax on chest x-ray.
High School Football Player with an Older Man's Injury: A Unique Case for Surgical Correction
Authors: Todd McGrath, MD; James Kramer, MD; Karl Fields, MD
Affiliation: Cone Health Sports Medicine, Greensboro, NC
History: A 15-year-old JV football player presented one day after sustaining an injury to his left shoulder. He was
uncertain as to the mechanism of injury, only that he was tackled and injured his shoulder. He was seen in an urgent care
setting, had unremarkable x-rays and was placed in a sling. Review of the urgent care note reveals his physical
examination was markedly limited secondary to pain. With continued pain overnight he presented to the orthopedic clinic
for further evaluation. He complains of pain in the anterior aspect of his shoulder. No feeling of instability, paresthesias or
neck pain. The patient denies prior shoulder/arm pain or injury.
Physical Exam: Left shoulder examination reveals slight asymmetry in the proximal bicep region. No tenderness to
palpation over the scapula, acromioclavicular joint or deltoid muscle. His range of motion reveals he is able to abduct and
flex the shoulder to 90 degrees, but is limited beyond that secondary to pain. Hawkins, Yergasons and Speeds tests are
positive. Obrien's test is equivocal. He has limited supination of the forearm and holds the elbow in flexion with limited
movement of the joint secondary to pain. A palpable defect and tenderness are noted in the proximal biceps region with
evidence of a Popeye deformity. Neurovascular exam is intact and equal to the unaffected arm.
Differential Diagnosis:
Biceps tendon rupture
Rotator cuff tear/tendonopathy
Labral pathology/SLAP tear
Shoulder subluxation
AC separation
Muscle strain
Hematoma/muscle contusion
Test Results:
1.
2.
Musculoskeletal ultrasound was performed in the office, which revealed evidence of proximal bicepital tendon
rupture.
MRI was done preoperatively to confirm degree of rupture. MRI revealed complete tear of the long head of the
biceps tendon from the superior labrum with 5cm of retraction.
Final Diagnosis: Rupture of the long head of the biceps tendon
Treatment: Given the patient's age and activity level this is a unique presentation of an injury commonly seen in much
older patients with more degenerative tendonopathy. While older patients rarely warrant surgery, in a young active athlete
surgical correction is indicated. Shoulder arthroscopy was performed to debride the nub of tendon from the unaffected
labrum. The rotator cuff and joint looked normal. An open biceps tenodesis was then performed to reestablish integrity of
the biceps tendon mechanism.
Outcome: The patient did very well with the procedure and there are no complications to date. Biceps tendon rupture
rarely occurs in young athletes although case reports document this in body builders, arm wrestling and rock climbers.
Overhead throwing athletes have biceps tendon injuries but usually only partial tears This is a unique presentation of an
injury commonly seen in older patients or very active individuals in middle age Surgical correction is reserved for young,
active athletes and thus was warranted in this case.
Follow-Up: This was a season ending injury, however, he should be able to return to full activity next season.
Myo My - a Rare Case of Myositis Ossificans
Authors: Leena Patel, DO
Affiliation: Kaiser Permanente Los Angeles, Department of Sports Medicine, Los Angeles, CA
History: A 53 year old female presents with 4 months of left groin pain. Started suddenly while doing lunges at the gym.
Since initial injury, she has had constant pain with varying levels of intensity. Aggravated by butterfly stretch, resting leg on
edge of bath tub. Temporary relief from rest, ice, anti-inflammatories.
Physical Exam: Tenderness at adductor longus insertion. Tenderness along iliopsoas +thomas test, stiff single joint hip
flexion, lacking 10 degrees Normal gait. Weak gluteal strength.
Differential Diagnosis:
1. adductor strain
2. adductor tear
3. iliopsoas strain
4. adductor tendinosis
5. iliopsoas tendinosis
Test Results: initial visit xray hip - normal MRI pelvis - partial tearing, edema, and fluid signal at the left quadratus
femoris muscle centered at the ischiofemoral space and quadratus femoris space. Also a bulbous masslike effect at the site
of edema raising the possibility of myositis ossificans. The pubic symphysis shows mild sclerosis consistent with mild
enthesophytes. The origin of the abductors and the abdominal muscles are without tear 6 weeks after initial visit xray hip
- normal
Final Diagnosis: myositis ossificans and partial tear of quadratus femoris
Treatment: Physical therapy for gluteal activation and hip flexor stretching 1 month after initial visit, seen in office for
ultrasound to localize myositis ossificans and consider corticosteroid injection for symptom relief. Not well visualized due
to depth of lesion so injection aborted. Discussed with radiology, recommended CT to further evaluate myositis ossificans
and possible CT-guided injection for pain relief.
Outcome: 6 weeks after initial visit (6 months after initial injury), symptoms were mostly resolved
Follow-Up: Able to return to gym workouts and remained asymptomatic on left side. Developed right sided trochanteric
bursitis about 2 months later.
Is an ACL Tear a Career Ending Injury?
Authors: Kristin Ernest, MD
Affiliation: Houston Methodist, Houston, TX
History: Patient is a 36YO female with chronic L-knee pain. Her initial injury occurred 18 years ago and consisted of an
ACL tear, medial meniscus tear and MCL sprain. She underwent ACL reconstruction w/patellar tendon autograft and
medial meniscus repair. She progressed through rehabilitation despite persistent knee pain that began worsening 3
months postoperatively. On follow up visits her ACL graft was reportedly "loose". Due to persistent knee pain, xrays were
ultimately obtained for evaluation and a fracture of L-patella was identified. Rehabilitation was discontinued and she was
placed in a knee immobilizer for 4wks. She returned to physical therapy but was unable to resume activity to the level
prior to initial injury and thus retired from competitive athletics and lost her basketball scholarship. Her L-knee pain
persisted for 14 years w/intermittent edema. She denies injury. She is unable to identify specific aggravating factors and
reports constant knee pain. She denies alleviating factors. In 2012 she woke one morning unable to bear weight on her left
leg due to severe pain. She was diagnosed w/severe osteoarthritis of her L-knee and after failing nonsurgical therapy, a
total knee replacement was recommended. A total knee replacement was performed in March 2012 when she was 34YO.
Her surgery was complicated by wound dehiscence and poor bone healing. She went for second opinion and a revision of
her total knee replacement was recommended. In 2013 at 35YO a second total knee replacement was performed with a
rod inserted in her tibia. She was placed in a knee immobilizer postoperatively due to the history of poor wound healing.
After her incision was healed she was taken to the operating room for manipulation of her left knee under anesthesia to
regain complete range of motion. Her knee pain persisted despite the revision and on repeat xrays no bone healing was
visualized, leaving the surgical hardware loose.
Physical Exam: Her presenting physical exam in 1998 of her left knee was significant for positive Lachman, positive
McMurray's and positive laxity and pain with valgus stress at 0° and 30°. Currently in 2014 her exam is significant for
persistent left knee effusion and pain with range of motion.
Differential Diagnosis: -Systemic Lupus Erythematosus -Osteoarthritis -Hyperparathyroidism -Benign Joint
Hypermobility Syndrome -Marfan's Syndrome -Ehlers-Danlos Syndrome
Test Results: 5/2014: Positive genetic testing for Ehlers-Danlos Syndrome - Hypermobility
Final Diagnosis: The final diagnosis is Ehlers-Danlos syndrome - Hypermobility.
Treatment: There is no curative treatment available for patients with Ehlers-Danlos Syndrome. The primary approach in
hypermobility patients is therapy and nonoperative management. In this particular case, further treatment and workup is
currently on hold pending discussion regarding possible parathydroidectomy or left above knee amputation.
Outcome: No genetic testing was completed prior to 2014 and the EDS testing was undertaken, per patient request. If
testing occurred earlier, a more conservative, nonsurgical approach would have been pursued given the poor healing in
this patient population. The musculoskeletal conditions diagnosed were treated appropriately for a patient without EDS,
however this patient did not respond as predicted following any of the surgical procedures performed. A total knee
replacement would have been avoided in this patient if the diagnosis of Ehlers-Danlos Syndrome had been made earlier.
Follow-Up: It is recommended that patients with Ehlers-Danlos Syndrome not participate in contact sports. Low-impact
activities are preferred to minimize joint stress and specialized therapy for joint stabilization prior to sport participation to
prevent injury could be considered.
Percystant Wrist Pain in a Weightlifter
Authors: Matthew Negaard, BA; Christopher Hogrefe, MD
Affiliation: Department of Emergency Medicine, University of Iowa Carver College of Medicine, Iowa City, IA;
Dept. of Emergency Medicine, Dept. of Medicine - Sports Medicine, and Dept. of Orthopaedic Surgery - Sports
Medicine, Northwestern Medicine, Chicago, IL
History: A 25YO, R-handed male with no past medical history presented to Sports Medicine Clinic with 1-mth of dorsolateral L-wrist
pain. Patient was competitive weight lifter. Denied precipitating events or previous wrist pain. The patient reported sharp pain that
progressed over the past month, now rated at an 8 out of 10. Wrist extension during weight lifting significantly increased pain. He
noted decreased grip strength and marked pain w/continued weight lifting. He was not taking any pain medication. He denied any
erythema, ecchymosis, swelling, numbness, tingling, weakness, clicking, locking, popping or instability of the L-wrist. The patient denied
fever, chills or other joint pains.
Physical Exam: Visualization of L-wrist was without erythema, ecchymosis, swelling and/or atrophy. There was localized tenderness over
the left anatomic snuff-box and scaphoid tubercle. He had full range of motion of the L-wrist, though deep flexion elicited pain. He
endorsed mild pain w/radial deviation and no pain with ulnar deviation; no laxity was appreciated with these movements. Radial pulses
were 2+. Sensation was intact. Strength was 5/5 and symmetric throughout the wrists and hands. Shuck testing was negative.
Differential Diagnosis:
Scaphoid stress fracture/fracture
Intersection syndrome
Scaphoid avascular necrosis
Carpal metacarpal osteoarthritis
Scaphoid cysts
Scapholunate instability
Scapholunate dissociation
De Quervian's tenosynovitis
Carpal boss
TFCC tear
Test Results: X-Ray: Multiple cysts within the left scaphoid without evidence of a fracture MRI: Multiple cysts within the left scaphoid
that prevented visualization of potential fractures. Irregularity of the volar surface of the scaphoid. Bone marrow edema was seen in the
scaphoid, lunate, capitate, and base of third metacarpal. Central fenestration versus perforation of the TFCC near the radial attachment
was also seen. A CT was suggested to better assess for a fracture CT: Cystic changes of the scaphoid with no acute fractures. A 1 mm
ossicle in the scapholunate interface was identified as well as a 4 mm ossicle in the volar aspect of the lunocapitate interface.
Final Diagnosis: Repetitive trauma from weight lifting causing micro-fractures in the scaphoid leading to multiple bone cysts.
Treatment: Initially, patient was treated with an EXOS splint to protect the scaphoid pending the MRI, which could not rule out
fractures due to the multiple cysts within scaphoid. Therefore, a CT scan was secured, which also failed to rule out fractures. Patient was
advised to avoid weight lifting and wear EXOS splint until follow-up in 4-wks. At follow-up, he reported decreased pain and was allowed
to lift weights with the splint on but without directly loading the wrist joint. Upon re-evaluation 2-wks later, the patient's symptoms
persisted and his desired activity level was limited. After 6-wks of conservative treatment featuring rest/modified activity and the EXOS
splint, he was referred to an Orthopaedic Hand Surgeon, who elected to take the patient to the operating room. His operative findings
and treatment included: 1. L-scaphoid intraosseous cysts; curettage and bone grafting of left scaphoid w/distal radial autograft. 2. LPalmer type 1B triangular fibrocartilage complex tear; debridement and repair. 3. Partial L-scapholunate ligament tear; repaired
arthroscopically. 4. Two L-wrist intraarticular loose bodies at scapholunate interval; removed.
Outcome: Patient had limited flexion and extension of the wrist at his 4-wk post-operative visit. Therefore, he initiated occupational
therapy 3x/wk for 6-wks. At his 12-wk follow-up visit, he had full, painless range of motion of the wrist.
Follow-Up: Twelve weeks post-operatively he returned to competitive weightlifting without discomfort and/or limitation. He was
discharged and has remained pain-free, fully engaged in competitive weightlifting.
Hip Pain in a 16 Year Old Field Hockey Goalie
Authors: Catherine Reese, MD; David Harsha, MD
Affiliation: St. Vincent Hospital - St. Vincent Sports Performance - Family Medicine Residency, Indianapolis, IN
History: A 16 year old field hockey goalie presents with five years of intermittent, worsening hip pain in the posterior and
anterior aspect of her left hip. She also reported left groin pain and catching and would worsen when she shuffled
sideways. She described "giving out" at times on inclines or going up stairs. Pain was usually relieved by rest from
competition. The pain worsened during "split saves". A rest period, physical therapy and chiropractic manipulation
provided little relief. She competed, but experienced increased pain after participation. She noted neurologic complaints,
and denied back pain. PMHx: fractured clavicle at birth. No hx of alcohol or tobacco use. FMHx: her mother reported
developmental dysplasia of the hip.
Physical Exam: Vital signs stable. No acute distress. Normal gait. Skin: intact with no erythema, ecchymosis or increased
warmth. Symmetric limb lengths. Stands with increase in left hip internal rotation. ROM limited in passive flexion and
external rotation, and internal rotation of left hip compared to right. Pain is caused by flexion, adduction, and internal
rotation. Positive McCarthy, Positive scour test. Negative Stinchfield. NV: normal sensation to light touch and strength in
hip abductors, gluteus maximus, hip external rotators 3/5 strength on left vs. 4+/5 on right. Normal distal pulses.
Differential Diagnosis:
1.
2.
3.
4.
5.
6.
7.
8.
Labral tear
Femoral Acetabular Impingement
Stress fracture
Iliopsoas bursitis/tendinopathy
Neoplasm
SCFE
Legg-Calvé-Perthes
Developmental dysplasia of the hip with late diagnosis
Test Results: AP pelvis and frog lateral view left hip 7/14/2011: L hip dysplasia with lateral uncovering of femoral head.
Center edge angle measures 20°. on left and the right it measures 35°. Acetabular index is about 10°. No other acute
osseous abnormalities. MRI left hip 10/2010: unremarkable MR arthrogram left hip 6/6/2011: anterosuperior labral tear
without any hyaline cartilage disease.
Final Diagnosis: left hip labral tear with underlying femoral acetabular dysplasia.
Treatment: She was referred to an orthopedic physician and underwent a periacetabular osteotomy 11/29/2011 to
correct the underlying dysplasia. Her labral tear was not repaired or trimmed.
Outcome: Post-op recovery was uneventful. She completed 25 weeks of physical therapy to with aquatics, light weights,
progressing to jumping, jogging, cutting and landing.
Follow-Up: 3 years later the patient is playing goalie in field hockey in division 3 collegiate athletics. She had some
residual hip pain after her surgery but as time passes it continues to improve. She still has some catching in her hip and
soreness, which is thought to be due to her labral tear. She is no longer in physical therapy but does the training and
conditioning with her team.
College Soccer Player With Sun Anaphylaxis
Authors: Amir Hendiani, MD; A. MacQueen, MD
Affiliation: UHS Sports Medicine, Binghamton, NY
History: 17 years old female soccer player presented at pre-participation physical examination (PPPE) to receive
clearance to play. She admitted to having a "sun anaphylaxis" that began as erythematous papules on her sun-exposed
areas. She had been seen by a dermatologist who started her on tapering doses of prednisone then started her on betacarotene and hydroxychloroquine. However, the athlete could not elaborate on her diagnosis or management plans. She
was not cleared to play and was asked to share clinicians notes and laboratory findings.
Physical Exam: At the time of PPPE, she had no abnormal physical findings.
Differential Diagnosis:
Lupus Erythematosus
Solar Urticaria
Drug-induced Photosensitivity
Phytophotodermatitis
Erythropoietic Porphyria
HIV Photodermatitis
Polymorphous Light Eruptions (PMLE)
Test Results: Negative ANA, dsDNA, SSA, SSB, HIV and urine porphyrins.
Final Diagnosis: Upon further questioning, she denied any history of mucosal edema or signs of upper airway
compromise. Athlete had severe photosensitivity but no true anaphylaxis. Based on history, physical examination and
laboratory findings, she had been diagnosed with
Treatment: She was continued on hydroxychloroquine and beta-carotene. We advised her to use protective clothing,
apply high-SPF sunscreens and to undergo annual opthalmic evaluations to rule out hydroxychloroquine-induced
neurologic and/or opthalmic toxicities.
Outcome: She had responded well to medications without any known complication.
Follow-Up: She was cleared to participate in collegiate sport. Athlete played soccer without recurrence of skin eruptions.
Medial Forefoot Pain in a Soccer Player
Authors: Luci Olewinski, MD; Morteza Khodaee, MD
Affiliation: University of Colorado Department of Family Medicine, Denver, CO
History: 29 year old man presents with left medial forefoot pain He is an avid soccer player, and approximately one
week prior to presentation he struck his foot against the ground while running during a soccer game and immediately felt
pain to the left plantar medial forefoot He presents in a walking boot requesting further evaluation. PMH of left ACL
repair but no previous injury to the left foot. Other medical problems include anxiety and depression managed well with
fluoxetine and rare use of alprazolam.
Physical Exam: Height: 6' 0" Weight 204 lbs. Healthy appearing man in no acute distress Left foot: Neutral alignment of
foot with normal longitudinal arch. Intact sensation and less than 2 second capillary refill to all toes. Mild swelling across
forefoot with notable tenderness to palpation along the medial and plantar aspect of the first metatarsophalangeal joint.
Pain is worsened by dorsiflexion of first toe more so than plantar flexion.
Differential Diagnosis:
Metatarsophalangeal joint sprain
Acute sesamoid fracture
Sesamoid stress fracture
Bipartite sesamoid
Sesamoiditis
MTP osteoarthritis/hallux rigidus
Test Results: Left Foot Xray: Bipartite medial sesamoid vs. transverse fracture.
Left Foot Noncontrast CT Scan:
Bipartite medial sesamoid. Slightly sharp margins of the medial sesamoid, as well as the lack of cortication in the adjacent
fragments, most likely represent fracture. No evidence of AVN. The lateral sesamoid remains intact.
Final Diagnosis: Displaced Tibial Sesamoid Fracture
Treatment: Open reduction and internal fixation of the sesamoid with one 2mm screw at approximately 2 weeks after
injury.
Outcome: At two weeks post-op he is doing well with minimal complaints of pain.
Follow-Up: Gradually advancing his weight bearing status.
Elbow Pain Without an Injury in a 13 Year Old Female Gymnast
Authors: Brian Hinkley, DO; Brooke Lemmen, DO; Andrew Schorfhaar, DO
Affiliation: Michigan State University Sports Medicine, East Lansing, MI
History: 13 year old female presented for a chief complaint of right elbow pain She states it has been ongoing for 8
weeks with no acute injury Her pain is worse with full extension, especially with weight-bearing during gymnastics She
admits to occasional popping, but denies locking, instability, swelling or bruising She has tried ice, NSAIDs and OMT with
no significant relief She denies any previous injuries or surgeries.
Physical Exam: Gen - well developed, well nourished, no acute distress MSK - No thenar/hypothenar atrophy
bilaterally Grip strength, thumb, wrist pronation/supination/flexion/extension resisted range of motion - 5/5 bilaterally
Right Upper Extremity elbow flexion/extension resisted range of motion - 4/5 Right elbow - No tenderness to palpation
olecranon, medial/lateral epicondyle, capitellum Palpable snap superior to olecranon with open/closed chain extension
Negative Tinel's ulnar nerve Negative varus/valgus/milking sign Internal/external rotation right upper extremity resisted
range of motion - 4/5 pain free Right Empty Can pain free - 4/5 Right Speeds pain free - 4/5 Left full active range of
motion elbow flexion/extension Left upper extremity strength 5/5 Negative Spurlings Intact cervical active range of
motion Neuro - Reflexes 2+/4 Upper extremity/Lower Extremity, Distal Neurovascular status intact bilaterally - no ulnar
nerve focal deficits Skin - intact w/o lesions/rashes Psych - Alert and cooperative, normal mood and affect, normal
attention span and concentration
Differential Diagnosis: Elbow weakness secondary to rotator cuff/shoulder weakness, Triceps strain, Somatic
dysfunction of Shoulder and Upper extremity, Medial/Lateral epicondylitis, Osteochondritis desicans lesion, Loose body,
Juvenile Rheumatoid Arthritis, Ulnar nerve dislocation/subluxation, Cervical neuropathy, Anconeus Muscle strain, Pronator
teres strain/syndrome
Test Results: Patient was started in physical therapy for scapular stabilization She had improvement in shoulder
strength, but still had noticeable snapping and pain with elbow extension after 4 weeks. XR Right elbow - Negative,
without evidence of osteochondral abnormality Dynamic US - Popping corresponds to an abrupt protrusion of what is
likely thickened synovium at the posteromedial aspect of the elbow on extension The tissue contracts the ulnar nerve
when this occurs. MRI was then ordered to further assess the soft tissues to confirm thickened synovium, with referral to
orthopedic surgeon for possible intervention MRI Right elbow without contrast - No chondral/osteochondral lesion or
loose body Small region of presumed synovial thickening at posteromedial elbow, presumably accounting for the findings
described on recent US Small accessory muscle overlying the cubital tunnel consistent with anconeus epitrochlearis.
Orthopedic evaluation - found to have ulnar nerve subluxation at terminal extension
Final Diagnosis: Right ulnar neuritis with subluxation of ulnar nerve
Treatment: Due to her symptoms preventing her from doing the things she enjoys (gymnastics), the patient wanted
surgery.
Surgical transposition of the ulnar nerve was completed without complications.
Outcome: Surgical transposition of the ulnar nerve was completed without complications. She is awaiting her follow up
appointments with the Orthopedic Surgeon.
Follow-Up: At this time she is awaiting her postsurgical follow up appointments. She will likely be out of activities for at
least 6-8 weeks, potentially up to 3 months pending her recovery and her level of activity in gymnastics.
Typical Day in the Kneeborhood
Authors: Benedict Ifedi, MD; Nader Ayub, DO; Randolph Taylor II, MD
Affiliation: Memorial Family Medicine Residency Program, Sugar Land, TX
History: 12 yo male with no PMH, active participant in karate who presented with a 3 week history of left knee pain,
acutely worsened 2 days prior to his presentation in the office 3 weeks prior to presentation, patient was kicked in left leg
during a karate match and fell onto a bent left knee. After injury he was unable to continue play. He endorsed pain
radiating down the length of his leg, but denied any popping sensation. The patient tried ibuprofen at home with minimal
relief of his pain. ROS per history, otherwise negative for any other constitutional symptoms.
Physical Exam: Left Knee difficulty bearing weight onto Left leg with antalgic gait, pain to palpation over Left tibial
tuberosity, no swelling or erythema noted, remainder of knee exam was negative. RLE normal ROM with no TTP
Differential Diagnosis:
Avulsion Fracture of Tibial Tubercle
sgood Schlatter Disease
Patellar Tendonitis
Osteochondritis Dissecans
Sinding Larsen Johansen Disease
Test Results: Xray L Knee -slight fragmentation of anterior tibial tubercle representing normal variation vs. undisplaced
small avulsion fracture B/L Knees - slight fragmentation of the ossification center of the greater tibial tubercle, this is
symmetrical finding, most likely normal variation. Patient given crutches and instructed to be non-weight bearing on left
leg with use ice and ibuprofen as needed for pain Pedi Ortho visit 5 days after clinic visit Confirmed x-ray showed
fragmentation of the tibial tubercle apophysis Impression: Left knee pain secondary to Osgood-Schlatter’s Disease
Suspected patient presentation due to growth-related pain
Final Diagnosis: Osgood-Schlatter's Disease
Treatment: Recommended hamstring stretches, NSAIDS, ice, patellar tendon band, no PE for 2 weeks, return to activities
as tolerated.
Outcome: Osgood-Schlatter Disease (OSD) is a traction apophysitis of the tibial tubercle caused by repetitive strain on
and chronic avulsion of the secondary ossification center of the tibial tuberosity. OSD is commonly seen in active
adolescents (boys 12-15 years; girls 8-12) who participate in sports involving jumping, running, and repeated knee
extension. Patients typically present with gradual onset of pain, swelling and tenderness over the anterior knee, with pain
worsened by activities that involve extending the knee against resistance. Our patient's diagnosis was more difficult to
determine because the patient was not running and had an acute traumatic event prior to having pain, instead of the
usual overuse that leads to OSD. Diagnosis is predominately clinical, as on physical examination findings include
tenderness and prominence overlying the tibial tubercle, reproduction of pain with knee extension or squatting, and
possible tightness in hamstrings and quadriceps muscles. Obtaining plain radiographs of the knee is recommended in all
unilateral cases to rule out other possible causes of knee pain such as tibial apophyseal fracture, infection, or tumor (4,6).
Again this patient had an inciting trauma so bilateral x-rays of the knee were key in differentiating between an avulsion
fracture vs OSD. Overall prognosis for OSD is good as the disease process is self limited in greater than 90% of patients.
Conservative treatment of OSD is the gold standard and consists of application of ice, limitation of activities, oral antiinflammatory medications, protective knee padding and physical therapy. Potential complications of OSD include patellar
tendon avulsion, and possible genu recurvatum (hyper-extension) of the knee, early onset of arthritis of the knee, and
possible persistence of pain into late adolescence or adulthood and limitation of activity due to a small ossicles formed
that may impinge on the patellar tendon.
Follow-Up: Return to activity is dictated by gradually increasing patients' exercise levels as tolerated and making
periodic adjustments as needed based on symptoms experienced. Our patient was cleared for activity after 3 weeks of
conservative management.
Pain in the Butt
Authors: Payton Fennell, DO; Brent Messick, MD, MS
Affiliation: Cabarrus Sports Medicine Fellowship, Concord, NC
History: A 17 year old high school football player with no past medical history presented with a one month history of left
hip pain. His pain began following cutting drills when he felt a sharp pop followed by lateral left hip pain. Patient stated
that the pain was exacerbated by sitting, walking, and running. In addition, patient stated he felt left lateral foot
numbness with prolonged standing. He had been treated for iliotibial band syndrome for the past month by his athletic
trainer with no relief or improvement. He took ibuprofen without relief.
Physical Exam: Patient presented ambulating without assistance. Patient had full active range of motion of the hip,
strength 5/5 and no instability. He had tenderness over his greater trochanteric bursal area and hip flexors, positive Ober
and FADIR test. He had no pain to the piriformis area or SI joints, negative Thomas and FABER test. Lower back showed
normal anatomical alignment with non-painful full range of motion. Knee exam was within normal limits. No
neurovascular deficits and reflexes were 1+ and symmetric.
Differential Diagnosis: Labral tear, hip flexor strain, radiculopathy, snapping hip syndrome, greater trochanter bursitis,
femoral neck stress fracture
Test Results: Bilateral AP pelvis x-ray showed normal anatomical alignment and no acute fractures or cortical
irregularities. MR arthrogram showed a linear decreased T1 signal suggestive of a non-displaced fracture involving the left
S1 sacral segment with adjacent marrow edema. Fracture line extended into the left S1 sacral foramen. Lab work showed
a normal BMP, calcium level and Vitamin D25 Hydroxy.
Final Diagnosis: Sacral Stress Fracture
Treatment: The patient was instructed to avoid any weight-bearing activities except for normal daily activity. Patient was
placed in a LSO brace and advised to return in four weeks following a limited lumbosacral CT to evaluate healing. At six
weeks, he was started on a hip rehabilitation program with his athletic trainer and slowly advanced back to football.
Outcome: When asked specifically, he did endorse buttock pain with prolonged standing. Upon reexamination, he had
tenderness to his left sacrum. At his four week follow-up, the lateral hip pain had decreased significantly. However, he
continued to have minimal tenderness to the left sacrum and occasional foot numbness with prolonged standing. The CT
scan showed linear sclerosis to the fracture site with minimal lucency along the fracture line. Patient was given a diagnosis
of a zone two sacral stress fracture.
Follow-Up: Patient finished off his senior year playing football approximately 8 weeks following the diagnosis. No
further complaints.
Snap Judgment: When Time is Muscle
Authors: Heather Hammonds, MD
Affiliation: Baylor Scott and White-Hillcrest; Southwest Sports Medicine and Orthopedics & Waco Family
Medicine Residency Program, Waco, TX
History: 16YO High School male running back reports to SM clinic with 1-day history of severe lateral lower leg pain after sustaining
inversion injury in football scrimmage 1-night prior. After injury, was evaluated by school trainer and received cryotherapy.
Recommended to return to trainer the next day to receive treatments. Pain progressed through the night prompting him to be seen in
local ER. He was noted to have negative Xrays, tenderness at lateral ankle but intact neurovascularity so recommended to follow up in
sports clinic the following day. Due to pain out of proportion, he was worked into clinic. Unable to weightbear. Has been utilizing
crutches since last night. Reports difficulty in finding comfortable position for his leg. The lateral leg is source of greatest pain. Reports
numbness over lateral aspect of ankle. He also reports having mild ankle sprain 1-wk earlier but able to progress back to play under
guidance of school's athletic trainer without any significant concerns.
Physical Exam: General: Alert and oriented, no acute distress, but with visible discomfort R-LOWER LEG EXAMINATION:
Observation: Swelling noted to the lateral aspect of the leg Palpation: Noted firmness to soft tissue of the lateral aspect of the leg.
Tenderness to the fibular head as well as distal fibular shaft. He is most tender in the mid-portion of the lateral leg/fibular region. No
tenderness to the lateral malleolar region, however patient indicates the region is numb. There is swelling to this area. Strength testing:
2/5 with ankle dorsiflexion - patient indicates is not related to pain just a frank inability to dorsiflex. Neurological: decreased sensation
over R-lateral aspect of the lower leg and ankle region as well as the first dorsal web space. Vascular: 2+ Posterior Tibialis pulse; 2+
Dorsalis Pedis Skin Examination: No erythema. Noted tightness to skin of lateral foreleg. Surgery Findings: Upon incision of lateral
compartment and fascia there was immediate extrusion of the hematoma from that compartment under pressure. After further
lengthening of the incision and lavage to the area, the peroneus longus muscle was noted to be completely avulsed from it's origin. The
muscle belly was void of viability (color, non-contractibility, and inability to bleed) and was resected to its myotendinous junction.
Differential Diagnosis:
1.) Fibula Fracture
2.) Ankle Sprain
3.) Peroneus Tendon Syndromes
4.) Common Peroneal Nerve Entrapment
5.) Acute Compartment Syndrome
Test Results:
1.) R-ankle xray: negative for fracture, intact ankle mortise R-knee xray: slightly increased prominence of fibular physis compared to
tibial physis
2.) R-lower leg compartment testing with U/S guidance: Lateral: 134 mmHg Anterior: 65 mmHg Superficial Posterior: 43 mmHg
Final Diagnosis:
1.) Acute Compartment Syndrome
2.) Peroneus Longus Proximal Muscle Rupture
3.) Acute Ankle Sprain
Treatment:
1.) Emergent decompressive fasciotomy of anterior and lateral compartments of R-lower leg
2.) Surgical removal of non-viable peroneus longus muscle origin (to myotendinous junction) with subsequent tenodesis to peroneus
brevis
3.) Non-weight bearing for 6-wks
4.) Rehabilitation for full ROM/strength/proprioception of ankle and subsequent progression back to sports
Outcome: Patient recovered from surgery w/good healing and return of normal neurovascularity to anterior and lateral compartments
of R-leg. Able to progress through his progressive rehabilitation program to near full strength of his secondary evertors.
Follow-Up: Patient's formal rehabilitation program began at 3-wks w/ROM and gentle strenghtening. Over course of subsequent 8-wks,
able to go from non-weightbearing to full contact play and returned to last high school football game on 11/7/14. Noted to have some
residual deficiencies in eversion when compared to the contralateral. He is to continue to work w/high school trainer to build up
strength. He also plays basketball and is sprinter/high jumper so we'll continue to follow his return to full spectrum activities as year
evolves.
Red Urine. Red Shirt. Who Decides?
Authors: Jennifer Hopp, MD; Vijay Jotwani, MD
Affiliation: Houston Methodist, Houston, TX
History: A.C. is a 22 year old collegiate football player with past medical history significant for IgA nephropathy who
developed painful swelling of his anterior left thigh and gross hematuria. Patient notes the swelling and redness
developed one day ago, and his urine had been progressively darker over the past five days. His only medications include
Zyrtec, fish oil 1gm, multi-vitamin, and losartan 25mg. Denies fever, chills, abdominal pain, no recent illnesses or foreign
travel, no headache, blurred vision, SOB, DOE, sore throat. Positive for rhinorrhea and dry cough. Denies the use of
supplements, tobacco, alcohol or illicit drugs. Admits to being sexually active with one partner and uses condoms. Allergic
to PCN and avoids NSAIDs
Physical Exam: VS: 98.1°, 171/84, 84, 16, 99% on RA Ht 6'5" Wt 270lbs Gen: White male in no acute distress. CV:
RRR, no murmurs, gallops or rubs. Good distal pulses. Pulm: CTAB, no rhonchi or rales. Back: No CVA tenderness. Abd:
Normal BS. No tenderness no guarding. Extr: L leg held in extension; 1+ pitting edema from hip to mid-calf; painful to
touch. Skin: Erythematous macule over anterior L thigh, extending from medial thigh to knee. No pustules or vesicles.
Warm to touch. No visible breaks in the skin.
Differential Diagnosis:
Cellulitis causing IgA Nephropathy Hematuria
Viral Myositis with Rhabdomyolysis
Herpes Zoster
Acute Tubular Necrosis
Exercise Induced Hematuria
Test Results: UA: Color Red, Cloudy Blood 3+ Protein 1+ WBC 7 ; RBC &gt;200 ; LE Small BMP: Na 125 K 4.3 Cl 97
CO2 20 BUN 52 Cr 3.4 Glu 110 GFR 24 CBC: WBC 22.14 Hgb 11.0 Hct 33.6 Plt 250 US LLE: Edema and thickening
of the subcutaneous tissues. No defined abscess or hematoma identified. Vessels are patent. Blood Cx: Negx2, after 5
days of growth CK: 71
Final Diagnosis: Cellulitis with Acute Tubular Necrosis secondary to IgA Nephropathy
Treatment: Patient was started on broad spectrum antibiotics, which was later switched to vancomycin. Given his renal
function in the hospital (peak Cr 3.7, GFR 21), the vancomycin dose was adjusted daily. He remained hospitalized for six
days, and discharged with an outpatient regimen of levofloxacin and minocycline. He gained twenty pounds of fluid while
admitted. He continued to improve and his creatinine returned to near baseline (1.3) three weeks after discharge.
Outcome: After discharge, the patient began to aggressively train for the upcoming football season, working out fifteen
hours each week. One week after discharge, he developed atypical sub-sternal chest pain. The pain was initially
intermittent, and he did not report it as he felt this was related to training. His chest pain became more constant and six
weeks after discharge, he was seen by a cardiologist. He was diagnosed with pericarditis just two weeks before the start of
summer training.
Follow-Up: Still with Stage III, CKD (Cr 1.59, GFR 59), and now with pericarditis, his nephrologist recommended against
playing football given his current state of health. For athletes with chronic kidney disease, return-to-play decisions are
made based on each individual case. The previous football season had moved him from Stage II to Stage III, CKD. After
developing cellulitis, fluid overload and pericarditis, a long discussion was had between the patient, his coaches and his
health care providers about the upcoming football season. The patient decided to retire from football for his health. He
remains in Stage III, CKD (Cr 1.6, GFR 54) and is interested in a career in Sports PT. He is currently acting as an assistant
coach in his last semester at Rice University.
The Stiff-Necked Basketball Player Who Sees Double
Authors: Kyle Ott, MD
Affiliation: Wright State University Family Medicine, Dayton, OH
History: Patient is a 15 year old male High School basketball player, referred by Athletic Trainer, presenting to be
evaluated for post-concussive like symptoms. During the past month, headaches and transient double vision have been
his predominant complaints. Patient recalls his occipital skull impacting the hardwood gym floor after getting his feet
swept out from under him during practice about 3 weeks ago. He denies loss of consciousness, yet is unsure if he
experienced nausea, confusion, poor concentration, dizziness, or further symptoms at that time. He has just recently
notified the Athletic Trainer. Presently, he notes occasional headaches. Brief occurrences of double vision, typically one
minute in duration, continue. He reports intermittently experiencing this upon awakening in the morning. Double vision
has additionally been preceded on occasion by increased physical exertion and when landing on his butt in the setting of
basketball play. Past medical, family, and social histories are non-contributory.
Physical Exam: Vital signs are within normal limits for age. A focused HEENT and neurologic exam was conducted and
remarkable for patient developing diplopia upon visual field testing when asked to look to his extreme left. No
disconjugate gaze was appreciated. Incidentally, his neck seemed extremely stiff with inability to fully extend, flex, or
laterally bend. Upon illustrating difficulty with neck rotation, he notes that teammates and coaches have commented on
this for years. In fact, when spoken to, he has to actually turn his whole body to look at them. This neck issue, however,
does not appear to be an acute injury which is related to this current vision problem.
Differential Diagnosis:
1.
2.
Diplopia: Primary ophthalmologic condition, Concussion with persistent symptomatology, Hematoma, Scar tissue,
Mass (benign vs malignant), or Vascular anomaly (intracavernous carotid aneurysm, AVM).
Restricted Cervical ROM: Congenital abnormality/stiffness.
Test Results: Aside from holding patient out for one week (until follow up re-evaluation) and advising brain rest, XR (cspine series) was ordered for the incidental finding of chronic neck stiffness. This showed fusion of C3-4-5 (likely
congenital) with moderate foraminal narrowing which Radiology recommended further evaluation via MRI spine. MRI
spine revealed similar c-spine pathology, however made note of what appeared to be an incompletely visualized brain
mass in the peripheral field of this study limited to targeting the c-spine. Radiology subsequently recommended "MRI
brain with gadolinium ASAP". Brain MRI was obtained, illustrating a large bilobed mass on the left side of the skull base
with enhancing cystic and solid components, producing significant mass effect.
Final Diagnosis: Intracranial mass of uncertain clinical significance
Treatment: Patient was referred to a tertiary care facility with neurosurgery. He underwent incomplete tumor resection
with decompression of the brain.
Outcome: At three months post-op, repeat MRI Brain showed increased size of the residual tumor. Patient underwent
further debulking plus radiotherapy. One year out, only a scant area of enhancement remained with eventual full
resolution upon subsequent follow-up imaging. Pathology report was conclusive for WHO grade I Schwannoma
(neoplastic proliferation of schwann cells that classically manifests as a solitary intracranial, intraspinal, or deep soft-tissue
lesion with peak incidence in the 3rd decade which may occur sporadically or associated with neurofibromatosis type 2;
rarely malignant; symptomatic lesions should be surgically resected)
Follow-Up: Neurosurgery later cleared patient for full return to sport, requiring use of a protective headgear. Issues in
this case that may contribute further discussion include the following challenges with concern for the integrity of his skull
and intracranial jostling of the brain: When should this patient return to athletics (if at all)? In what sports should he return
(degree of contact)? What protections are necessary?
Knee Pain in a 10 Year Old Female
Authors: Danielle Magrini, DO; Kyle Nagle, MD, MPH; Jay Albright MD
Affiliation: University of Colorado and Children's Hospital - Colorado, Aurora, CO
History: 10 year old female accompanied by her mother arrives to the sports medicine office with a chief complaint of
left knee pain that began nearly three and a half weeks prior to this initial visit. There was an acute injury to initiate the
onset of pain. The pain occurred when she was playing basketball outside at the end of school when she jumped, landed,
then fell onto a bent knee. She does not remember feeling a "pop" or any other sensation in her knee, but had immediate
pain and by the time she got home she had a large effusion She had never had any prior knee pain, swelling or prior
injury to this knee Patient was seen by sports medicine then orthopedics, imaging was completed and diagnosis revealed
No significant past medical or surgical history Family history was significant for a father, brother and sister all with
osteogenesis imperfecta.
Physical Exam: MSK: Left Knee Examination of the left knee reveals no ecchymosis, no swelling, and no gross
deformity. There is a large knee effusion. Range of motion of the knee is from -5° to 125° compared to -5° to 130°. There
is no tenderness to palpation of the patella, patellar tendon, tibial tuberosity. There is no Hoffa's fat pad tenderness. There
is no tenderness of the medial or lateral patellar facets. There is mild tenderness with patellar grind. There is mild
apprehension with patellar distraction There is no focal medial or lateral joint line tenderness There is no ligamentous
laxity with a negative posterior drawer, varus and valgus stress testing. Lachman's and anterior drawer are equivocal due
to guarding. There is no pain with full flexion. McMurray’s test is negative. Distally neurovascularly intact with brisk pulses,
2 second capillary refill, and a normal motor and sensory nerve examination in the affected limb. Opposite limb: no
swelling, deformity, joint effusion palpable, tenderness to palpation, or decreased range of motion. Full strength. Skin
intact.
Differential Diagnosis:
Internal derangement such as a partial ACL
impaction fracture
tibial eminence fracture
patellar dislocation
JIA
PVNS
Test Results: X-rays - IMPRESSION: Intra-articular calcifications with joint effusion and probable synovial thickening.
MRI recommended for further evaluation. MRI: IMPRESSION: Multiple intra-articular fragments consisting of bone signal
as well as a chondroid signal. The fragments are various sizes. Synovial osteochondromatosis is the primary consideration
but usually the intra-articular lesions are similar size. No mass identified to suggest synovial sarcoma but the distinction
between these 2 can be difficult. There is some irregularity and focal defects of the distal medial femoral articular
cartilage. There is synovial hypertrophy as well. No hemorrhage seen to suggest PVNS. Does the patient have other
changes that might be suggestive of juvenile idiopathic arthritis? However this would be atypical appearance for such.
Surgical Findings: Multiple loose bodies (100+) of varying sizes, with appearance of arising from the synovial lining.
Gram stain and culture-operative biopsies: SMEAR: FEW POLYMORPHONUCLEAR CELLS DETECTED. NO ORGANISMS
DETECTED
Final Diagnosis: Status post left knee arthroscopy with loose body removal, biopsy with permanent sections, and
synovectomy consistent with synovial osteochondromatosis.
Treatment: Arthroscopic knee removal or all loose bodies and extensive synovectomy.
Outcome: Hinged knee brace locked in extension with weight bearing as tolerated for 6 weeks, PT for Range of Motion,
Hip, core and knee strengthening, progressing to return to activity training. Post-operatively patient did very well. No
recurrent swelling, no mechanical symptoms, no pain.
Follow-Up: Return to activity: gradually progressed in PT for improved ROM, strength training and has been cleared for
full return to activity.
Leg Weakness in a Professional Diver
Authors: David Ross, MD; Rahul Kapur, MD; John Vasudevan, MD
Affiliation: Hospital of the University of Pennsylvania, Philadelphia, PA
History: The patient is a 29 year old female professional diver who presented to sports medicine with left leg weakness
for two months duration. Her weakness started after a vaginal delivery of her first child at home with the assistance of a
midwife. During the delivery, she was lying on her left side with her hips and knees flexed continuously for four hours.
Immediately after delivery, she noticed difficulty with ambulation. She initially saw a chiropractor for therapy, without any
relief. The patient described the weakness as a "funny numbness" down the anterior thigh. She had difficulty and fatigue
with stair climbing and squatting. She also described decreased sensation in her anteromedial thigh. She denied any hip,
leg, knee, or back pain. She was particularly worried about her unsteadiness because of her occupation as a professional
diver, which required walking up stairs with a 60lb oxygen tank and climbing in and out of an aquarium with her
equipment.
Physical Exam: The patient's initial exam was notable for full range of motion of her lumbar spine, hip and knee joints.
She had no bony tenderness or paraspinal tenderness in her lumbar spine. She had decreased sensation to light touch
down her anterior thigh to the dorsal foot on her left side. Her strength was 3+/5 with knee extension on left side. There
was no effusion. There was no defect noted of her distal quadriceps or tendon. She had a negative straight leg raise
bilaterally. Her deep tendon reflexes were 2+ bilaterally.
Differential Diagnosis: Predominantly neuronal and muscular etiologies such as a femoral neuropathy, lumbosacral
radiculopathy or plexopathy, or a quadriceps muscle/tendon injury that would have developed secondary to her
prolonged hip flexion during her vaginal delivery. A less likely cause could have been an inflammatory myopathy such as a
polymyositis or inclusion body myositis given the proximal muscle involvement.
Test Results: MRI imaging of the hip and lumbar spine were notable for diffuse mild edema within the left pectineus,
rectus femoris, sartorius and vastus intermedius muscles in the distribution of the anterior division of the femoral nerve,
suggesting an entrapment neuropathy. The patient's imaging was also notable for mild degenerative disk disease at L4-5
and L5-S1. An EMG study was obtained given her weakness and MRI findings, which suggested subacute left femoral
neuropathy with denervation on needle exam and significant decrease of motor conduction amplitude. The location of
injury based on these studies was thought to be distal to the innervation of the iliopsoas muscle and proximal to the level
of inguinal ligament. The presence of observable motor conduction response and active recruitment of motor units on
needle exam indicated a favorable prognosis for recovery with appropriate therapy.
Final Diagnosis: The patient was diagnosed with a left Femoral Nerve Entrapment secondary to prolonged left sided hip
flexion.
Treatment: The patient was enrolled in aggressive physical therapy with twice weekly sessions focused on increasing her
leg strength by climbing and lifting. She was enrolled for ten months.
Outcome: The patient regained her strength with physical therapy focused on weight lifting and ladder climbing. She
was re-evaluated with an EMG eight months after her initial presentation, which showed evidence of substantial reinnervation of the left femoral nerve innervated muscles. The fibrillations on initial EMG due to the femoral neuropathy
had resolved. These results were consistent with her improved strength.
Follow-Up: After ten months of therapy, the patient was able to return to work at the Aquarium and climb up stairs with
her 60lb oxygen tank and dive without any difficulty.
Left Knee Pain After ACL Reconstruction
Authors: Shane Drakes, MD; Se Won Lee, MD; Tony Wanich, MD
Affiliation: Montefiore Medical Center, Bronx, NY
History: 53 year old male who presented with chronic left knee pain after undergoing anterior cruciate ligament (ACL)
reconstruction three years prior to presentation. His original ACL injury had been caused by a fall with direct trauma to the
knee. He reported that his pain was in the posterior aspect of the knee/thigh and there was intermittent radiation to the
calf. The pain was described as aching and deep and accompanied by tingling and cramps in the calf. The pain was
aggravated by prolonged walking and standing and alleviated by use of nonsteroidal anti-inflammatory medications.
Physical Exam: The patient was noted to have minimal effusion and a surgical scar on the medial aspect of the left knee.
Flexion of the left knee was limited to 110 ° and extension was normal. Strength in the left lower limb was normal except
for knee flexion which was 4/5. There was a visible defect in the musculature of the posteromedial aspect of the left thigh
when knee flexion was performed against resistance. There was tenderness in the posteromedial aspect of the left thigh
and the patient was found to have tight hamstrings bilaterally. There was hyperesthesia in the lateral aspect of the left leg.
The straight leg raising test was negative and there was no Tinel’s sign elicited in the posteromedial aspect of the left
thigh.
Differential Diagnosis:
Sciatic nerve injury
Hamstring muscle injury
ACL graft failure
Test Results: MRI was remarkable for the presence of a medial meniscus tear and a Baker's cyst and absence of the
semitendinosus tendon. The ACL graft was intact. Musculoskeletal ultrasound demonstrated hyperechoic tissue in close
proximity to the left sciatic nerve in the posterior thigh and on sonopalpation this corresponded to the area of the
patient's maximal pain.
Final Diagnosis: Left thigh pain due to semitendinosus muscle retraction with sensory symptoms due to sciatic nerve
irritation
Treatment: Physical therapy was prescribed for eight sessions.
Outcome: After physical therapy, the patient reported some improvement in his pain and decreased muscle tightness
was noted by the therapist after his course was completed. He was discharged to a home exercise program.
Follow-Up: The patient continued his usual activities with some improvement in his pain but he still reported sensory
symptoms. In view of his continued sensory symptoms, electrodiagnostic testing was recommended but he defaulted from
follow-up before testing could be performed or any other treatment could be started.
Just Another Baker's Cyst, Right?
Authors: Brandon Kakos, MD
Affiliation: Mercy Health Orthopedics and Sports Medicine, Cincinnati, OH
History: 47 year old caucasian male who presents with a two year history of intermittent left posterior knee pain/mass
Initial symptom onset occurred without traumatic or inciting event. He complained of left posterior knee pain and swelling
with tenderness to direct palpation. He was able to complete his activities of daily living and fully participate in his
recreational soccer league (midfielder) with minimal deficit. The pain was sharp in nature with no radiations, however he
did have occasional popliteal dysaesthesias. Direct palpation of the popliteal fossa mass and full knee flexion exacerbated
his symptoms, while rest, ice and sporadic anti-inflammatories were relieving factors. His symptoms remained tolerable
and while undergoing evaluation for chronic right knee pain, he incidentally mentioned his left popliteal mass/discomfort
An ultrasound guided aspiration was attempted and three distinct hypoechoic loculated cysts within a heterogeneous
appearing mass were discovered. Upon attempts at aspiration of the cysts he complained of severe popliteal "shock like"
pain/dysaesthesias. 5 cc of serous fluid was obtained and sent for analysis An MRI was then ordered for further
evaluation. PMHx: ADHD PSHx: None Medications: Ibuprofen PRN; Atomoxetine 25mg QD Allergies: No known drug
allergies Family History: CVA=Father; Skin Cancer=Brother & Sister Occupational History: Finance Social History:
Denies any Tobacco/Illicit Drug Use; Social EtOH use; Plays recreational soccer
Physical Exam: Inspection: 2cm x 2cm medial popliteal fossa mass, no erythema; ROM: Active/Passive arc of motion: 0130°; Palpation: mild tenderness to palpation over the popliteal fossa mass, the mass is firm and freely mobile; Strength:
5/5 Quad, 5/5 Hamstring, 5/5 Gastroc/Soleus; Provocative Testing: negative lachmans, negative anterior drawer, negative
McMurrays, negative pain/laxity with varus/valgus stress at 0° and 30°; Neurovascular: +Tinels over the popliteal fossa,
DP/PT pulses intact
Differential Diagnosis: 1. Bakers Cyst 2. Popliteal/Gastroc Strain 3. Malignancy - Myxoma, Soft Tissue Sarcoma,
Neurogenic Tumor 4. Vascular - DVT; Popliteal Artery Aneurysm 5. Peripheral Nerve Sheath Tumor (Neurolemmoma)
Test Results: Synovial Fluid Analysis: Appearance=Clear/Yellow
WBC=248; RBC=112; Neutophils=7;Lymphs=89;
Monocytes=4 Total Cell=27 Gram Stain= No Organisms; Culture= No growth U/S: Heterogeneous well circumscribed
popliteal fossa mass containing three hypoechoic loculated cystic structures MRI: 1. Well encapsulated complex soft
tissue mass, posterior aspect of the knee, interposed between the medial and lateral heads of the gastrocnemius muscle,
without evidence of muscle invasion 2. No evidence of internal derangement. 3. Grade 4 chondromalacia patella. 4.
Small intra-articular joint effusion. Pathology of Mass: Spindle cell neoplasm of mild-moderate cellularity with a whorling
pattern. No definite capsule. Immunostaining: Strongly positive for vimentin and moderately positive for S-100. Negative
for muscle specific actin. Features are consistent with a schwannoma.
Final Diagnosis: Left Popliteal Schwannoma
Treatment: Surgical Excisison and Biopsy
Outcome: One week after surgery the patient was completely pain free and had full knee range of motion, strength and
function without tenderness to palpation.
Follow-Up: Six months after surgery he continued to be pain free without further ailments/complaints. He was able to
resume recreational physical activity without deficit.
Unexpected Cause: Exertional Foot Pain in a Young Athlete
Authors: Marcel Bahr,MD
Affiliation: Summa Center for Sports Health, Akron, OH
History: In May 2014 a 17 y/o high school football player presented to our office with the chief complaint of a two year
history of intermittent right plantar foot pain. The athlete had undergone right patellar realignment surgery in September
2013 but his past medical history is otherwise negative. The pain is described as exercise-related burning and tightness,
coming on minutes into running or jumping activity and quickly leading to cessation of activity. The pain will then resolve
almost immediately only to come back with return to activity. There is no pain with walking or standing. After having seen
a podiatrist previously, physical therapy for suspected plantar fasciitis did not yield improvement. X-rays of his right foot
dated 06/2013 demonstrated no gross abnormality; An MRI of his foot dated 05/2014 also did not show significant
abnormality.
Physical Exam: Initial physical examination of his right foot and ankle is normal regarding inspection, range of motion,
strength, and neurovascular status. There is no tenderness to palpation over the plantar fascia or to testing of FHL and
FDL. After exercise some discomfort is noted with palpation of the lateral plantar midfoot.
Differential Diagnosis:
Plantar fasciitis
Stress fracture
Exertional compartment syndrome of the foot
Test Results: Weight bearing X-rays of his right foot are negative for acute osseous abnormality. A bone scan is negative
for an acute bony process. Foot lateral and central compartment pressures post exercise are elevated but in consultation
with a foot and ankle orthopedic surgeon found to be not high enough to warrant a release. The medial compartment
pressure is subsequently tested - also upon parent's request - and found to be normal. Nerve conduction studies are
negative. Over the next 4 months symptoms worsen with new pain involving the achilles tendon area and subsequently his
entire lower leg ultimately also at rest. Following lower leg symptomatology in September pressure testing of all four
compartments yields non-significant results. At this time an ankle - brachial index is ordered. It is 0. 38 on the right and 1.1
on the left. At rest, segmental wave forms in the right ankle are category 3-4, and category 5 in both the transmetatarsal
and first digit. There is severe femoro popliteal occlusive disease.
Final Diagnosis: Chronic limb ischemia secondary to peripheral arterial disease, popliteal artery entrapment syndrome
Treatment: The patient under goes angioplasty (attempted) andangiography with catheter - directed intra-arterial
thrombolysis with tPA on 10/02 which remains unsuccessful after about 60 hours and is stopped after development of a
left groin hematoma. On 10/09 he under goes right AK popliteal to peroneal and posterior tibial bypass surgery with great
saphenousveindrom his left leg.
Outcome: The patient is discharged on Coumadin for by pass patency and does home physical therapy soon thereafter.
He has been recovering well from surgery.
Follow-Up: At the present time he does fine with walking and jogging.
Young Volleyball Player with Persistent Knee Pain and Swelling After ACL Reconstruction
Authors: Daniel Tsukanov, DO; Gerard Malanga, MD
Affiliation: Montefiore Medical Center, Bronx, NY
History: BW is a 20 year old female who in 2011 injured her left knee while playing college volleyball. She was diagnosed
with an anterior curciate ligament (ACL) tear and underwent ACL reconstruction with a patellar tendon autograft. Post
surgery she developed knee swelling, pain and fever and was diagnosed with an infected joint. She had two arthroscopies
with a course of antibiotics and after failure to improve in therapy underwent lysis of adhesions under anesthesia in 2012.
She attempted to return to volleyball but had exacerbation of pain and swelling Repeat fluid analysis indicated an
ongoing infection and in late 2012 she underwent removal of hardware and a second ACL reconstruction. After surgery
she continued to have pain, limited mobility and was unable to return to sports. She had difficulty with daily ambulation
and was not able to exercise.
Physical Exam: Left Knee: Inspection: Healed incision with erythema. Mild effusion and valgus alignment. Range of
Motion: 0-130° pain free Palpation: Tenderness posterior-medial joint line Muscle Strength 5/5 throughout, DTR's 2/4,
Special Testing negative for anterior drawer, posterior drawer, Lachman, valgus and varus stress testing. Gait: non
antalgic without assistive devices
Differential Diagnosis: Osteoarthritis, Synovitis, Meniscal Tear, Infection
Test Results: MRI 03/04/2014 - High grade full thickness cartilage loss with underlying cystic changes and bone
marrow edema worst at the medial compartment. Status post ACL graft reconstruction which heterogeneous in signal.
Status post prior menisectomy. US 6/25/2014 - Medial meniscus partial tear. Reduced cartilage and cortical irregularities
on the medial femoral condyle.
Final Diagnosis: Severe Osteoarthritis
Treatment: Patient had initially trialed NSAIDS and two Synvisc One injections without any improvement in symptoms. In
our clinic she elected to have a bone marrow concentrate and platelet rich plasma injected into her left knee on
7/23/2014.
Outcome: Initially patient had significant improvement in symptoms with daily walking and awas able to strength train
pain free. She attempted to use a treadmill for 25 minutes on two occasions and had exacerbation of symptoms. A repeat
PRP injection was performed 10/17/14.
Follow-Up: Currently patient has returned to her university and is able to engage in daily activities without significant
pain. She is able to participate in strength training and using a stationary bike.
Persistent Weakness Post-Concussion
Authors: Randon Hall, MD, MBA
Affiliation: Phoenix Children's Hospital, Department of Orthopaedics and Sports Medicine Phoenix, AZ
History: A 13 year old female presented to sports medicine clinic with persistent weakness after a concussion. Eight
weeks prior to presentation she fell and struck her head while trying to get into a car when she lost her balance. She
developed persistent, dizziness, fatigue, nausea, difficulty with short-term memory, and difficulty with focus and
concentration at school. She was initially seen by neurosurgery, and an outpatient concussion treatment protocol was
started. She was then referred by her PCP to sports medicine clinic for persistent weakness which was thought to have
been present prior to the injury but worsened since the concussion. Upon further questioning, she reports having the
gradual onset of unsteadiness while standing as well as extreme difficulty lifting up objects over the past two years.
Specifically, she reports having difficulty lifting herself up out of the bathtub. Prior to the injury, she has noted some subtle
changes in her speech, as well as difficulty with pronunciation of certain letters. She reports intermittent blurry vision and
occasional double vision. She can no longer be active in sports and currently reports difficulty with walking. Her symptoms
of headache, nausea and memory problems have resolved. She is taking no daily or prn medications. She has no pertinent
family history. She is an eighth grader who has had a recent decrease in her school performance. Review of systems was
positive for weight loss, blurred vision, shortness of breath and depression.
Physical Exam: Vital signs: HR 90/min, BP 120/69 RR: 26/min pulse GEN: Patient was well appearing, well developed
and in no acute distress. HEENT: Oral examination showed 2+ tonsils. Normal tongue movements. Midline uvula. NECK:
Neck was non-tender with normal range of motion. NEURO: Alert and oriented to person, place and time. Cranial nerve
exam showed normal eye tracking and symmetrical facial expression. She is unable to puff out her cheeks with or without
resistance. She is unable to whistle. Voice is noted to be nasal in nature. Muscle tone and bulk is normal. 3+/5 strength
with forward flexion and abduction of shoulders bilaterally. 4/5 strength bilateral shoulder shrug. 4/5 hip flexor strength
bilaterally with a positive Trendelenburg test. Upper and lower extremity reflexes normoactive bilaterally. Normal gait.
SKIN: no lesions or rashes
Differential Diagnosis:
Botulism
Myasthenia Gravis
Lambert-Eaton
Mitochondrial Myopathy
Multiple Sclerosis
Depression
Hypothyroidism
Dermatomyositis
Intracranial Mass
Test Results: Labs: CBC: WBC 8.3 Lymphocytes 33%, otherwise normal. CMP normal. ESR 41. RF negative. ANA
Negative. TSH 0.39, T4 1.23 Acetylcholine receptor binding antibody 125.13 (critical high). Imaging: MRI Brain C,T and L
spine - No acute abnormalities or demyelinated lesions. EMG: No sensory deficits, greater than 20% decrement following
2 Hz repetitive stimulation for 3 baselines obtained for the left median nerve, left musculocutaneous nerve and peroneal
nerve.
Final Diagnosis: Acetylcholine Receptor Antibody Positive Generalized Myasthenia (Myasthenia Gravis)
Treatment: She was treated inpatient with IVIG and transitioned to low dose prednisone and a cholinesterase inhibitor.
Outcome: She showed dramatic improvement, however every 3-4 weeks after multiple treatments with IVIG her
symptoms would exacerbate. She has since undergone an uncomplicated thymectomy and remains on low dose
prednisone and a cholinesterase inhibitor.
Follow-Up: Patient is under the care of Neurology and being medically managed with low dose prednisone and a
cholinesterase inhibitor. She is not active in sports at this time.
One Year of Abdominal Pain Gone After One Ultrasound-Guided Cortisone Injection
Authors: Kristin Duncan, MD; Jonathan Schultz, MD
Affiliation: University of Missouri Kansas City Department of Family Medicine and Community Medicine,
Kansas City, MO
History: A 57 year old female with PMH of hypothyroidism and GERD with a hiatal hernia was referred to the Sports
Medicine Center by her PCP after having upper abdominal pain for greater than one year Prior to seeing her PCP, she was
evaluated multiple times by another physician who obtained an abdominal x-ray, diagnosed her with constipation, and
gave her a laxative with no improvement in her symptoms. When first evaluated by her PCP, she described a pressure-like
sensation underneath her rib and sometimes below her sternum She reported her abdomen felt tight and that these
symptoms abruptly began about one year prior She complained of feeling bloated and was taking Prilosec without relief
of her symptoms Her PCP ordered a CT of the abdomen and pelvis, abdominal ultrasound, and EGD that were all normal
Upon evaluation in the Sports Medicine Center, she noted her pain started one year ago after running a tiller in her
garden She stated that light massaging helps the pain but harder pressure will reproduce her symptoms She noted it was
made worse by turning at the waist, raising her knees, doing tasks with her arms, and valsalva She stated that the pain
radiated to the middle of her back She had been taking Ibuprofen, then Meloxicam and Gabapentin that provided
minimal relief She denied any history of trauma Her main goal was to return to gardening.
Physical Exam: She was a healthy appearing, overweight woman in no acute distress Lungs clear to auscultation.
Cardiac exam revealed normal rate and rhythm Abdominal exam revealed normal bowel sounds, tenderness to palpation
at the tip of the xiphoid, and no detectable hepatosplenomegaly She did have tenderness to palpation in the upper
abdomen at the liver edge and some epigastric tenderness No rebound or guarding Musculoskeletal exam revealed a
normal appearing sternum without pectus excavatum or carinatum The xiphoid tip was tender to palpation No
deformities, swelling, erythema, or ecchymosis was noted Normal chest expansion Muscle strength testing of the
pectoralis was 5/5 bilaterally and did not reproduce symptoms Neurovascular testing revealed normal sensation and
perfusion in the sternal area.
Differential Diagnosis: GERD, Functional Dyspepsia, Angina, Xiphodynia
Test Results: A CBC, CMP, and Hepatitis panel were normal The CT of the abdomen and pelvis was essentially
unremarkable except for some small, scattered nodules on the liver The ultrasound of the abdomen demonstrated small
cysts on the liver too small to characterize.
Final Diagnosis: Xiphodynia
Treatment: The patient received an ultrasound guided 40 mg Kenalog/2 mL 1% Lidocaine injection of the xiphoid
process She was given a topical compound of Gabapentin/Diclofenac, Cyclobenzaprine, Baclofen, and Bupivacaine She
was instructed to stop Meloxicam and limit strenuous activity for 2 weeks.
Outcome: She reported 97% improvement of her symptoms after this injection and resumed gardening without pain.
Follow-Up: The patient returned to the Sports Medicine Center 1 month and 4 months following her injection She is
continuing to be observed and could consider repeat injection if symptoms recur.
Ultrasound Diagnosis of Atypical Tennis Leg
Authors: Erin Voss, MD
Affiliation: Cone Health Sports Medicine, Greensboro, NC
History: An otherwise healthy 32-year-old male recreational athlete presents with acute right calf pain that started while
playing tennis the previous day. He felt a pop and sudden pain that caused him to stop playing. The morning of his visit,
he noticed increased swelling, stiffness, and pain prompting him to come to the office. He is unable to bear weight on the
leg due to pain. He has been using a compression sleeve for the last hour. He was recently treated for UTI with
ciprofloxacin.
Physical Exam: The patient is a well-developed male in mild distress. He is non-weight bearing. On inspection there is
obvious enlargement of the right calf muscle as compared to the left. Right calf is tense on palpation. On active range of
motion, there is minimal to no dorsiflexion at the ankle. Approximately 15° of passive dorsiflexion can be achieved with
assistance from the examiner. Sensation intact distally. Dorsalis pedis and posterior tibial pulses 2+.
Differential Diagnosis: - Calf Strain (gastrocnemius, plantaris, soleus) - DVT - Compartment syndrome Thrombophlebitis - Achilles Rupture
Test Results: Musculoskeletal ultrasound was performed in longitudinal and transverse views. This demonstrated intact
Achilles tendon and gastrocnemius muscle with large hypoechoic hematoma deep to the gastrocnemius with no intact
soleus muscle fibers visualized. Hematoma measures 5 cm in width, 4 cm in depth, and 8 cm in length. Doppler images of
dorsalis pedis and posterior tibial arteries show normal flow.
Final Diagnosis: Soleus rupture and large hematoma with concern for impending compartment syndrome
Treatment: The patient's lower leg was wrapped with a compression wrap and he was placed in a full length walking
boot. He was advised on aggressive icing and elevation as well as avoidance of heat and NSAIDs. There was concern for
impending compartment syndrome based on his calf tightness and significantly limited dorsiflexion, so case was discussed
with orthopedic surgeon. Based on presence of good distal blood flow, decision was made for conservative care with daily
office visits for reexamination. He was counseled on symptoms of compartment syndrome.
Outcome: Patient had daily office visits for 3 days. He experienced improvements in his pain and ankle dorsiflexion as
well as decreased calf tightness. He improved without further intervention.
Follow-Up: Patient was allowed to wean out of the walking boot. He was instructed to avoid sporting activity until his
pain had fully resolved and he had regained full range of motion.
Sport-Related Chest Trauma in a Recreational Student Athlete
Authors: David Smith, DO
Affiliation: Notre Dame University Health Services, South Bend, IN
History: A 23 year old healthy man presented with acute chest pain. While playing defensive line in a flag football game,
he collided with a teammate as he was chasing the opposing quarterback. The teammate was at a full sprint when he
struck the patient's central chest with his forehead. He felt immediate pain in his chest and removed himself from play.
Over the next hour, his pain worsened and he began to experience dyspnea. He sought treatment at the emergency
department shortly after the game. In the emergency room the pain was described as sharp, constant, focal, nonradiating, and was rated 9/10. The pain was exacerbated by deep inspiration and movement of his arms. It was not
notably positional. He denied palpitations, nausea, sensory or motor issues, or known head trauma.
Physical Exam: When examined he was in notable pain but no distress. HEENT exam revealed normal tympanic
membranes and oronasopharynx. Pupils were equal, round, and reactive to light. Cardiac exam showed regular rate and
rhythm without murmur or rub. Pulmonary exam revealed equal breath sounds throughout, no wheezes, rales, or ronchi,
no paradoxical breathing. The sternum was tender with palpable crepitus present, no signs of flail chest. The overlying
skin was without ecchymosis or edema. Neurological exam was grossly normal. The remainder of the exam was within
normal limits.
Differential Diagnosis: Initially: pericardial effusion, pneumothorax, sternal/rib fracture, aortic injury, pulmonary
contusion and arrhythmia.
Test Results: Preliminary workup included normal serum chemistry and CBC. Troponin was 0.06. EKG showed normal
sinus rhythm with no ectopy or axis deviation. Chest radiograph was performed showing no pneumothorax, mediastinal
shift, cardiomegaly, or pleural effusion. The lateral view raised suspicion for a sternal fracture, prompting further
evaluation with a CT of the chest with contrast. CT showed a displaced fracture of the mid-sternum that was depressed 1
cm. No vertebral or rib fractures were present. A substernal hemorrhage was seen with no organized hematoma. There
was no evidence of aortic rupture, dissection, or aneurysm. No pulmonary infiltrate or infarct was seen.
Final Diagnosis: Closed, displaced, depressed fracture of the sternum.
Treatment: Immediate treatment consisted of pain control with hydrocodone/acetaminophen 325/5mg tablets as
needed every 4-6 hours for pain. He was given instructions to follow up the next day. At this point, care was assumed by
the student health staff. Due to the fracture location and level of displacement, conservative care was indicated.
Treatment was directed at pain control and restoration of pain free breathing. He was instructed to limit his motion,
within reason, and remained out of class for the remainder of the week. He was given a handicap parking pass for the
next month.
Outcome: One week after initial injury the patient no longer required oral narcotics for pain. Pain persisted with certain
arm movements, but was less frequent and tolerable. At 2 weeks post-injury he remained without complications and
continued to have decreased pain. Sternal crepitus was no longer present. He was able to attend class regularly and
breathe pain free.
Follow-Up: Weekly follow up was mandatory to assess for associated complications that may arise including myocardial
contusion, expansile hematoma, and pulmonary or pericardial effusion. Ultimately, the goal is to follow him clinically and
slowly return him to exercise when pain free. At the time of submission he is near pain free and without complications.
The Case of Bilateral Upper Extremity Edema
Authors: Andrey Samal, MD; Peter Wenger, MD
Affiliation: Capital Health Regional Medical Center, Trenton, NJ
History: 24-year-old female with chronic neck and upper thoracic back pain, no medications, except oral contraceptive
pills (OCP), presented with right greater than left upper extremity edema that started two days after strenuous TRX upper
extremity workout.
Physical Exam: Initial evaluation in the office revealed 4+ pitting edema from the upper arm to the wrist on the right
and 1+ edema down to the elbow on the left arm. Normal pulses were palpable over brachial, radial and ulnar arteries
bilaterally. No bruises or other skin abnormalities were found and there was no lymphadenopathy noted.
Differential Diagnosis: Axillary dissection, axillary vein acute thrombosis, superior vena cava syndrome, thoracic outlet
syndrome, Paget-Schroetter syndrome, neck/upper chest mass.
Test Results: A STAT MR angiography of the neck and the upper chest was performed the next day and showed focal
narrowing of the right subclavian artery in the region where it was crossing the first rib A suspected filling defect was
described in the adjacent subclavian vein that could represent a venous clot. The findings were worrisome for thoracic
outlet syndrome with Paget-Schroetter disease. The patient was immediately referred to a vascular surgeon, who after a
physical examination performed a limited in-office vascular ultrasound which was normal A formal bilateral upper
extremity arterial and venous duplex scan was performed and provocative maneuvers confirmed the diagnosis of venous
impedance and thoracic outlet syndrome (TOS) No clot or other abnormality was identified at the time of this study which
was approximately 4 weeks after the initial presentation. Concurrently, the work-up for thrombophilia was performed and
turned out to be negative.
Final Diagnosis: Paget-Schroetter syndrome
Treatment: The patient underwent the first rib resection for decompression of her thoracic outlet on the right
Outcome: After a course of traditional post-operative PT her symptoms resolved.
Follow-Up: She has resumed all her normal activities without issue.
A Case of Medial Knee Pain in an Adolescent Runner
Authors: Jacob Ringenberg, MD; Dave Sealy, MD; Dan Divilbiss, MD
Affiliation: Self Regional Hospital, Montgomery Center for Sports Medicine, Greenwood, SC
History: A 15-year old competitive cross-country runner presented to the sports medicine clinic with gradually increasing
medial left knee pain over the previous four weeks The onset of her pain was insidious as she slowly increased her
running distance to 40 miles per week. The pain was localized over the adductor tubercle of the femur, radiating toward
the insertion of the pes anserine tendons At the time of evaluation, the pain was mild, only present toward the end of
long runs, and not limiting her activity She denied history of prior knee pathology and was otherwise healthy.
Physical Exam: On examination, the left knee was normal in appearance. There was no swelling, erythema, or joint
effusion Pain was elicited with palpation of the distal pes tendons and medially at the myotendinous junction of the
adductor magnus Pain was elicited with resisted adduction of the leg Active range of motion was not limited Patellar
apprehension, Lachman, varus stress, valgus stress, medial McMurray, and lateral McMurray were negative. Treadmill was
used to exacerbate her pain and to evaluate her gait She was found to have significant pronation of her feet bilaterally at
the stance phase.
Differential Diagnosis: Highest probability: pes anserine bursitis and tendinosis of the adductors, gracilis, sartorius, or
semitendinosis. Other conditions considered, but less likely: tibial plateau stress fracture, medial femoral condyle
avulsion or stress fracture, patellofemoral pain syndrome, medial meniscal tear, MCL sprain, medial plica syndrome, bony
lesion, reverse Segond fracture, quadriceps/adductor hematoma, or referred pain from the hip.
Test Results: The patient underwent a video gait analysis that confirmed over-pronation bilaterally and was placed in
supination shoe wedge inserts Over the next four weeks, her pain continued to worsen and began to limit her running
She underwent re-evaluation that included sports ultrasound of the medial knee which revealed a heterogeneous
protrusion from the medial femoral condyle with overlying hypoechogenicity On plain film of the knee, a benign
appearing bone lesion consistent with a solitary osteochondroma was identified Patient underwent MRI which revealed
extension of the osteochondroma into the border of the vastus medialis and adductor magnus resulting in external
compression and muscle edema The cartilaginous cap measured less than 2 mm. There was a trace amount of fluid
within the pes anserine bursa with normal appearing pes anserine tendons at the insertion.
Final Diagnosis: Patient was diagnosed with a benign, solitary osteochondroma with associated extension into the
vastus medialis and adductor magnus resulting in pain and muscle edema with running Her over-pronating gait likely
contributed to her symptoms and cause
Treatment: The patient underwent excision of the bony lesion from the medial supracondylar metaphysis of the femur
The lesion was dissected from the surrounding vastus medialis and adductor magnus She continued to wear a supination
wedge insert to correct her over-pronating gait She participated in home physical therapy with a focus on quadriceps and
adductor strengthening.
Outcome: Pathology confirmed a benign lesion consistent with a solitary osteochondroma There was a thin
cartilaginous cap present measuring less than 2 mm. The post-operative course was unremarkable.
Follow-Up: After a period of eight weeks of relative rest, the patient was able to return to light running and heavy crosstraining She was able to return to her typical running pace twelve weeks after resection She is currently six months postop and remains pain free with running.
An Unusual Case of Shortness of Breath in an Adolescent Basketball Player
Authors: Luke Widstrom, DO; Dave Olson, MD
Affiliation: University of Minnesota - North Memorial Family Medicine Residency, Minneapolis, MN
History: A previously healthy 15 year old male presents to the emergency department with a chief complaint of coughing
and shortness of breath for two days Additionally he describes wheezing and a pain in his chest and anterior lower neck,
especially with swallowing He states that the evening before his symptoms started he was playing pickup basketball and
was forcefully elbowed in the chest, causing significant pain When he woke up the following morning he had developed a
frequent harsh cough He did have two episodes of post-tussive emesis on the day of presentation Other than a mild
headache, his review of systems is otherwise negative The patient recalls no recent sick contacts. The patient is
previously healthy, has no allergies and takes no medications. His sister has asthma, but family is otherwise healthy.
Physical Exam: VS: Temperature 98.5, Blood Pressure 144/78, Pulse 94, Respirations 20, O2 Sat 95% Constitutional:
Well-developed and well-nourished. Appears uncomfortable. HEENT: NCAT. PERRL, no icterus or conjunctivitis. Bilateral
TM's gray and translucent. MMM. No oropharyngeal erythema or exudate. No nasal congestion. Neck: Palpable crepitus
to neck and upper chest. Normal ROM. Neck supple. No tracheal deviation. No lymphadenopathy. Cardiovascular: RRR,
no murmurs, gallops, or rubs. Pulmonary: Expiratory and inspiratory wheezing throughout with diminished air
movement. Normal respiratory effort, no tachypnea. No chest wall tenderness. Abdominal: Soft, non-tender, nondistended, no hepatosplenomegaly. Bowel sounds are present. No rebound or guarding. Musculoskeletal: Normal range
of motion, 5/5 strength in all extremities. Neurological: A & O x 3. Moves all extremities. No focal deficits. Skin: Skin
is warm and dry. No rashes or erythema. Psychiatric: Normal mood and affect.
Differential Diagnosis: Pneumothorax, pneumomediastinum, esophageal perforation, viral respiratory infection,
pneumonia, new onset asthma.
Test Results: Chest X-Ray: Lungs are clear with no edema or infiltrate. No effusion or pneumothorax of the pleural
space. No bony abnormality. Abnormal subcutaneous emphysema and air lucencies are overlying the upper mediastinum.
Recommend CT of the chest and low neck for further assessment. CT chest with contrast: Impression: 1. Subcutaneous
emphysema throughout the anterior low neck and axillae bilaterally extending into the mediastinum. Pneumomediastinum
present. Epidural pneumatosis. Presumed barotrauma. No discrete lesion is identified. Specifically no discrete esophageal
abnormality 2. If there has been prolonged vomiting, barium swallow may be considered to confirm esophagus is intact.
3. No pneumothorax.
Final Diagnosis: Traumatic pneumomediastinum
Treatment: Initially, the patient was treated symptomatically with IV fluids and ondansetron for vomiting and
dehydration He was given albuterol nebulizer treatments and oral prednisone for his wheezing and bronchospasm
component, and he was admitted to the inpatient service When he did not improve with this treatment, the chest CT was
ordered to rule out esophageal rupture as well as pneumonia Once the definitive diagnosis of pneumomediastinum was
made, his activity was limited and he was given benzonatate for his cough to decrease lung pressures He was also placed
on oxygen by nasal cannula at 2L Hydrocodone prn was provided for pain. Albuterol nebulizer treatments were scheduled
while inpatient, and the prednisone burst was continued.
Outcome: Daily chest x-rays were performed during hospitalization, and by hospital day four the pneumomediastinum
had resolved and subcutaneous emphysema has significantly improved The patient was discharged to home with a seven
day steroid burst and benzonatate for his cough, and he was provided with instructions to limit activity and follow up in
one week.
Follow-Up: Limited activity during first week post-hospitalization Return to full activity with stable chest x-ray at one
week follow-up.
Acute Iliopsoas Hematoma in a High School Wrestler
Authors: Josh Sole, MD; Travis Rhodes, DO; Jason Mogonye, MD
Affiliation: John Peter Smith, Fort Worth, TX
History: 17 year-old male wrestler presented with anterior left hip and groin pain for one week. He initially experienced a
"clicking" sensation in his anterior hip and groin region "every time he took a step". The "clicking" had started two weeks
earlier, however, one week prior to seeking medical treatment, he began having pain in the same area of his left hip and
groin following wrestling practice. At presentation, the "clicking" had completely resolved but his pain symptoms
persisted. He endorsed a possible direct blow to his left thigh and groin region during wrestling practice, but he was
unable to recall any specific trauma. He denied any overlying ecchymosis. Pain was rate 6-7/10 and worsened with flexion
and abduction of the thigh, with walking, and with climbing stairs. Pain did not radiate anywhere else. He had tried ice and
OTC NSAIDs without relief. His history was negative for anticoagulant or antiplatelet therapy. He denied any personal or
family history of bleeding diathesis or coagulopathy.
Physical Exam: Gait was mildly antalgic on left. There was tenderness to palpation in the anterior thigh and groin in the
distal portion of the iliopsoas myotendinous junction (distal to the ilioinguinal ligament and proximal to the lesser
trochanter) Left hip ROM was within normal range, except hip flexion, which was limited to 70 degrees secondary to pain
inhibition. Left lower limb strength was 5/5 in all tested muscles, except left hip flexion and abduction at 4/5. FABER was
negative for sacroiliac joint pain or anterior hip impingement pain. Ober’s test was negative for iliotibial band tightness
Skin in the affected region was devoid of erythema, scars, or ecchymosis. Femoral pulse was present bilaterally and
femoral nerve innervation was preserved to sensory testing. Lumbosacral spine examination was within normal limits
including ROM, SLR, and gait mechanics, other left limb antalgia.
Differential Diagnosis: Femoracetabular impingement, rectus femoris strain, quadriceps tendinopathy, iliopsoas strain,
iliopsoas tendinopathy, iliopsoas hematoma
Test Results: We performed bedside diagnostic ultrasonography of the left groin and thigh. This revealed a
circumscribed hyperechoic mass within the iliopsoas muscle belly disrupting the normal muscle and iliopsoas tendon
echotexture in short-axis. The sonographic appearance was consistent with an intramuscular hematoma. Color Doppler
flow was present in both long and short axis within the hematoma. There may also have been hyperechoic fluid
circumferentially around the iliopsoas tendon secondary to the hematoma. The rectus femoris, quadriceps muscles, and
femoral neurovascular bundle appeared normal.
Final Diagnosis: Iliopsoas hematoma
Treatment: The patient was advised to utilize cryotherapy intermittently as needed to relieve pain. Rehabilitation
recommendations were discussed with high school ATC. We recommend no return to wrestling or recreational sport
activity until pain free.
Outcome: Patient continued home rest, rehab and stretching and pain completely resolved 10 days following initial
consultation, and 4 days prior to follow-up appointment. Repeat bedside diagnostic ultrasound was performed at followup and demonstrated nearly complete resolution of the iliopsoas hematoma with slightly enlarged diameter of distal left
iliopsoas muscle compared to the right.
Follow-Up: We initially recommend no return to wrestling or recreational sport activity until pain free. Follow-up was
scheduled two weeks out with repeat diagnostic ultrasound to assess maturation and progression of hematoma. Serial
bedside ultrasound exam revealed nearly complete resolution of the iliopsoas hematoma with slightly enlarged diameter
of distal left iliopsoas muscle compared to the right. Patient was instructed to follow-up as needed if symptoms recurred.
Medial Thigh Pain in a Teenage Multisport Athlete
Authors: Alysha Taxter, MD; Matthew Grady, MD
Affiliation: The Children's Hospital of Philadelphia, Philadelphia, PA
History: The subject was a previously healthy football player until age 12 when he developed left groin pain without
preceding trauma. He was diagnosed with tight hamstrings and ischial tuberosity apophysitis. He completed a formal
course of physical therapy with slight improvement of his pain. Over the next three years, he continued to have pain
about his medial thighs and progressed to having pain about his greater trochanters, anterior and medial knees, and low
back His pain worsened with activity and was not relieved with non-steroidal anti-inflammatory medications or
acetaminophen He discontinued football and basketball secondary to pain He never reported joint swelling,
conjunctivitis, fevers, rashes, or unexplained weight loss. He is stiff for 15 minutes in the morning and has difficulty walking
up the stairs because of pain.
Physical Exam: Exam findings notable for normal genital exam without evidence of an inguinal hernia. There was
bilateral hip pain with passive range of motion, decreased internal and external rotation, bilateral sacroiliac tenderness
with lumbar flattening upon forward flexion and modified Schober of 23 cm Greater trochanter, patellar, and plantar
enthesitis was present He also had early boutonniere deformities on index, long, and ring fingers bilaterally without
effusions.
Differential Diagnosis: Inguinal hernia, sports hernia, adductor or psoas strain, slipped capital femoral epiphysis,
avascular necrosis, stress fracture, impingement syndrome, lesser trochanter apophysitis, enthesitis, sacroiliitis.
Test Results: Radiographs at age 12 showed normal pelvis and hip joints without avascular necrosis or slipped capital
femoral epiphysis. Repeat radiographs at age 15 showed subtle changes within the femoral heads and necks bilaterally
that may be associated with hip impingement. MRI showed bilateral acute on chronic sacroiliitis and diffuse asymmetric
enthesopathy with edema at the muscle attachments involving the bilateral ischial tuberosities, left anterior superior iliac
spine, greater and lesser trochanters and no evidence of femoral-acetabular impingement. He was HLA B27 positive.
Final Diagnosis: Enthesitis-related arthritis, which is a type of juvenile idiopathic arthritis. Enthesitis-related arthritis is
characterized by arthritis and enthesitis (tenderness at sites of tendon insertion); or arthritis or enthesitis with at least 2 of
the followin
Treatment: Because of his severe pain, he completed a three week steroid taper to rapidly halt his inflammatory arthritis.
Due to his axial spine involvement and long-standing arthritis as evidenced by erosions, we initiated etanercept, an antitumor necrosis factor biologic agent, as non-biologic disease-modifying anti-rheumatic drugs (DMARDs) do not
adequately treat axial disease. He restarted physical therapy to increase lumbar and hamstring flexibility and increase core
strength.
Outcome: His pain resolved within one month of pharmacotherapy initiation He continues in a formal physical therapy
program.
Follow-Up: He returned to multiple sports and continues to participate in both a formal and home physiotherapy
program. Since no anklyosisis was present on MRI imaging, we did not limit his participation in athletics. However, there
are no formal return to play guidelines in patients with juvenile idiopathic arthritis, and future investigation should better
define criteria necessary to safely return to this population to play.
Recurrent Hip and Knee Pain in a Soccer Player
Authors: Keith Heck, DO; Andrew Reisman, MD; David Webner, MD; Kevin DuPrey, DO
Affiliation: Crozer-Keystone Health System, Springfield, PA
History: A 25 year-old collegiate soccer player presented to the sports medicine clinic with two years of left hip and knee
pain. He initially presented with left groin pain that worsened in flexion and internal rotation. A trial of therapy and intraarticular corticosteroid injection failed to relieve his symptoms MRI revealed anterior labral tear for which arthroscopic
acetabular micro-fracture with labral and CAM debridement was performed. Over the next year, he developed multiple
episodes of left knee pain refractory to physical therapy, aspiration and corticosteroid injection. An MRI revealed evidence
of synovial thickening. Subsequently, arthroscopic surgery was performed with partial synovectomy and popliteal cyst
drainage. One month post-knee surgery, he developed a recurrent effusion refractory to conservative treatment. Repeat
MRI showed a moderate intra-articular effusion and baker’s cyst, which led to an open excision of the cyst. Four months
following excision, he developed anterior knee pain. Repeat MRI showed proliferative nodular synovitis. During the
treatment of our patient’s knee effusion, his left hip pain returned and worsened. Prior to his hip and knee symptoms
developing, he was diagnosed with bilateral popliteal artery entrapment and underwent surgery to release the popliteal
arteries Treatment following his multiple knee procedures included oral doxycycyline, PRP, and consistent therapy. Given
his recurrent hip pain, his surgeon repeated a left hip arthroscopy, which demonstrated numerous villi of the synovium.
Physical Exam: Initial and subsequent left hip examinations were similar. He exhibited moderate pain with log rolling
and end flexion. FADIR and FABER elicited groin pain. Hip internal range of motion was normal and symmetric bilaterally.
There was no tenderness to palpation. Resisted strength testing exhibited painless 5/5 hip flexion, abduction, adduction
and external rotation. Initial and subsequent left knee examinations were similar with repeat visits. Range of motion was
full at 0/130°. There was a moderate intra-articular effusion with persistent fullness in the popliteal space. Skin was absent
of erythema and warmth. There was no tenderness to palpation. Ligaments were intact.
Differential Diagnosis:
1. Recurrent Acetabular Labral Tear
2. Osteochondritis Dissecans Lesion
3. Sympathetic Knee Effusion
4. Spontaneous Osteonecrosis of the Knee
5. Gout
6. Inflammatory/Lyme Arthritis
7. Septic Arthritis
8. Deep Vein Thrombosis
Test Results: X-Ray Left Knee: No fracture X-Ray Left Hip: No fracture or degeneration. Localized protuberance along
lateral margin of femoral head consistent with Femoral-Acetabular Impingement. -MRI Knee #1: Medial gastrocnemius
strain. Small Joint effusion. -MRI Knee #2: Large popliteal cyst. Large effusion with synovitis. -MRI Knee #3: Moderate
proliferative nodular synovitis. Nodular and septated popliteal cyst. -MRI Hip Arthrogram: Anterior-superior acetabular
labral tear. Anterior acetabular subchondral edema with chondromalacia. - Bone scan: Increased uptake at the left hip
and left knee. -Hip synovial pathology- Macroscopically, numerous villi observed. Microscopically, papillary synovial
hyperplasia and chronic inflammation with plasma cells with pattern seen in setting of auto-immune disease.
Negative/Normal Labs: Following were all negative or normal reference range: HBsAg, Hep A IgM,Hep B Core Ab IgM,
HCV Ab, rheumatoid factor, ACE level, Anti-CCP, Lyme titers, ANA direct, anti-DS Ab, RNP ab, Smith Ab, anti-scleroderma70 Ab, Anti-SS-A/B, Anti-chromatin Ab, Anti-Jo-1, Anti-centromere B Ab, CRP, ESR, CBC, CMP, Ferritin, Quantiferon Gold,
HLA B27, STD Panel Positive ANA titer-homongenous pattern, titer 1:160
Final Diagnosis: Seronegative Rheumatoid Arthritis
Treatment: Prednisone, Methotrexate and Infliximab
Outcome: Awaiting clinical response to immunosuppressants.
Follow-Up: Patient will follow up with Rheumatology in one month. He will continue with therapy. He will advance
activities as tolerated.
Central Nervous System Pathology Presenting as Left Ankle Sprain
Authors: Hersch Bhatia, MD; Jason Ramirez, MD
Affiliation: University of Maryland Family and Community Medicine Residency Program Baltimore, MD
History: 49 yo M, no PMH, who presented with left ankle sprain, right leg pain and numbness, and erectile dysfunction.
Left ankle was sprained 1 month prior when he was stepping off a curb. He had initial swelling and was unable to put
pressure on it. He used crutches for a few weeks, took ibuprofen, iced, and elevated his ankle. He had little improvement in
symptoms, but he needed work clearance, so he came in. At presentation he was able to walk on it, but he was still
experiencing pain with walking. On review of systems he reported right leg pain and numbness below the knee that had
been slowly worsening over the past couple years, and was much worse over the past 6 months. He described it as pins
and needles. He could not feel the brake/gas pedals while driving. He reported dragging his right foot while walking. He
reported bilateral leg weakness. He also reported erectile dysfunction that worsened over the same time period. At first he
could not maintain an erection, but then lost ability to get an erection. He complained of diminished sensation in his groin
area. He reported having trouble holding his urine. He did not have bowel incontinence.
Physical Exam: Neuro: Cranial Nerves II through XII grossly intact bilaterally Upper extremity strength 5/5, sensation
intact, reflexes normal Lower extremity: Right extremity weaker than left extremity, 4/5 with knee extension, flexion,
dorsal flexion, and plantar flexion. Decreased sensation to light touch and pinprick below the right knee. Absent sensation
in his right toes. Patellar and Achilles tendon reflexes 4+ right, 3+ left + clonus, + babinski's Tandem, gait, and heel-toe
walking are normal Genitourinary: Decreased sensation of his penis and scrotum and surrounding groin area.
Musculoskeletal: Cervical back: He exhibits no tenderness. Thoracic back: He exhibits no tenderness. Lumbar back: He
exhibits no tenderness. Left ankle: TTP on the lateral malleolus and dorsal part of the foot. No swelling, ecchymosis.
Unable to dorsal flex or plantar flex 2/2 to pain. Intact sensation. Warm, pulses intact bilaterally.
Differential Diagnosis:
1.
2.
3.
4.
Multiple Sclerosis
Spinal Tumor
Transverse Myelitis
Dorsal Arachnoid Web
Test Results: MRI results: Brain, cervical, and lumbar spine unremarkable without any evidence of spinal cord pathology.
In the thoracic spine, there is a intramedullary non-enhancing lesion at T5-T7 with surrounding edema.
Dorsal thoracic arachnoid web at T5-6 causing significant cord compression.
CT myelogram:
Final Diagnosis: Thoracic Dorsal Arachnoid Web
Treatment: Surgical intervention: T5, T6 laminectomy, intradural opening and intradural exposure for resection of the
arachnoid adhesions.
Outcome: Tolerated surgery well
Follow-Up: At 6 week follow up, right leg weakness completely resolved and he had improved strength. He had regained
some of the sexual function and genital sensation. He was ambulating well and using the treadmill. Sensation was grossly
intact to light touch throughout. At 3 month follow up, he had continued improvement of strength, sensation, and sexual
function. His weight training regimen was increased at this time. At 6 month follow up, he was at full strength. He was
still hyper reflexive in the lower extremities. He had neuropathic pain issues in the right leg, which was treated with
gabapentin and flexeril. He is on disabilitiy until the 1 year mark and will be re-assessed at that time.
Mirror, Mirror
Authors: Brent S.E. Rich, MD, ATC
Affiliation: Utah Valley Sports Medicine, Brigham Young University Team Physician Provo, UT
History: KJ is a 51 year old male with chronic pain in his left (non-dominant elbow) He had a cortisone injection into his
common extensor tendon about a year ago with good relief for about 9 months He would like to receive another
injection He is right hand dominant and has no pain in his right arm.
Physical Exam: Alert and oriented, well developed male. Vital signs stable Neurovascularly intact. Pain over the
lateral elbow to palpation. Pain with 3rd finder extension and wrist extension that reproduces his pain. Of note, when he
moves his right arm, his left arm involuntarily reproduces the same movements as his right arm.
Differential Diagnosis:
Common Extensor tendon tear
Lateral epicondylitis
Etiology unclear why both arms move with contraction
Test Results: 3 view elbow radiograph: Normal, no evidence of bony abnormality Musculoskeletal ultrasound:
Thickening of the common extensor tendon without tear
Final Diagnosis: L. (nondominant) lateral epicondylitis Mirror Movement Dystonia
Treatment: Cortisone injection provided with 1 cc of 40mg/cc kenalog under ultrasound guidance Discussion regarding
dry needling or PRP to be performed in the future Stretching, strengthening and compression strap to use prn.
Outcome: Mirror movement dystonia is a rare congenital condition where certain limbs perpetually act in tandom The
genetic mutation causes sufferers to create less of a protein that directs brain signals to either the left or right side of their
bodies. Because of that, both sides of the body sometimes get the same signal -- though not always with the same
intensity. KJ has had this all his life and had to sit on his hand to write or the left hand would mimic what his right hand
would do and he would get teases He cannot do certain activities, such as climb the monkey bars or play the piano His
dominant arm is strong enough that he does not get an overuse injury, but the weaker, non-dominant arm is always
contracting when he uses his right arm Will show video where he can write his name backwards with the nondominant
arm while writing it forward with the dominant one.
Follow-Up: Follow up as needed Consider PRP for more definitive treatment when cortisone injections fail Of note, he
has three generations of family member who have varying degrees of this disorder.
A Real Pain in the Neck
Authors: Sarah Merrill, MD
Affiliation: University of California San Diego
History: 46YO male with history of mild low back pain and 2 episodes of iritis was referred to SM clinic for 7-wk history of R-sided neck
muscle spasm with no inciting injury or trauma. Patient thought pain may have initially started due to holding phone with his shoulder.
Was seen by his PCP and initially given stretches to do and muscle relaxants. Reported he tended to have pain at the occiput that
radiates to the sternum. Pain worse w/movement of the shoulder, raising his arm. Pain radiated to the ear and jaw and recently to the
clavicle. Pain did not get better over the course of several weeks. Returned to PCP and was given baclofen and referred to physical
therapy, ibuprofen and norco for pain. Pain improved somewhat, but continued to have pain that waxes and wanes throughout the day.
Neck felt stiff with decreased movement. He had tried heat and self massage without improvement in symptoms. Not been able to
schedule physical therapy yet. Pain was waking patient up at night, and severe 10/10. Temporized by norco but not relieved. Had a
history of similar neck pain, which generally resolved over the course of a few days. No prior history of neck injuries. No weakness. No
numbness or tingling of arms or legs. Patient also started developing pain and swelling at the sternum. Pain did not radiate. No chest
pain, shortness of breath.
Physical Exam: General Appearance: healthy adult male, alert, no distress, pleasant affect, cooperative Cervical Spine: INSPECTION:
holds self with neck tilted to L-side, no bony step-offs noted, no loss of lordosis PALPATION: no midline tenderness to palpation.
+diffuse paraspinal muscle and SCM tenderness to palpation and spasm on the R-side ROM: decreased rotation to the left,
sidebending to the left. FROM extension, flexion, side bending and rotation to the right. STRENGTH TESTING: 5/5 muscle strength in
upper extremities. Including radial, ulnar and median nerve NEUROVASCULAR: sensation and distal pulses intact to the upper
extremities bilaterally. SPECIAL TESTS: Negative Spurlings test Negative Roos Negative Adson's Chest: +notable swelling of Rsternoclavicular joint with +tenderness to palpation, no overlying erythema or ecchymosis Pulm: clear to auscultation bilaterally, no
wheezes or crackles Cardiovascular: regular rate and rhythm, no murmurs, rubs or gallops, no carotid bruits, no carotid tenderness to
palpation
Differential Diagnosis:
Muscle spasm
cervical radiculopathy
carotidynia
disc herniation
osteomyelitis
autoimmune disorder
seronegative arthopathy
Test Results: Labs: CBC, CMP, ESR, CRP within normal limits. RF, CCP, smith Ab, ANA, anti-double stranded DNA negative. HLA-B27
positive. Cervical neck xray - IMPRESSION: Mild multilevel facet and uncovertebral hypertrophy and minimal C7-T1 degenerative disc
disease. MRI Cervical Spine: 1. Right sternoclavicular joint effusion w/irregularity of the distal clavicle and mild surrounding soft tissue
edema. Findings may represent early osteoarthrosis, but inflammatory arthritis (i.e. septic arthritis, rheumatoid arthritis or other
seronegative spondyloarthropathies), cannot be entirely excluded. Recommend clinical correlation and consider joint aspiration as
needed for definitive diagnosis. 2. Mild degenerative changes of the cervical spine in particular at C4-6, without significant central
canal or neural foraminal narrowing.
Final Diagnosis: Seronegative arthopathy of cerivcal spine and R-sternoclavicular joint with resultant neck muscle spasms.
Treatment: Corticosteroid injection was placed at sternoclavicular joint w/short improvement in pain and swelling at that joint. Referred
to Rheumatology, where further imaging was obtained to look for possible satellite lesions; none were found. Was felt by Rheumatology
symptoms represented seronegative arthropathy. Patient started on sulfasalazine 1000mg 2x/day.
Outcome: Patient's symptoms have improved on curent treatment regimen of sulfasalazine, ibuprofen and physical therapy. Continues
to have mild neck pain and stiffness; sternoclavicular pain and swelling have resolved.
Follow-Up: Awaiting return to cycling pending return of full range of motion of the neck. Scheduled to follow up in clinic in 1 month.
Nine Year-Old with Atraumatic Injury Presents with Foot and Ankle Pain
Authors: Kristin Hine, MD; Justin Lee, MD
Affiliation: East Carolina University, Greenville, NC
History: 9 year-old male presents to clinic with left foot and ankle pain for 1 year He denies a known injury The pain is
primarily located over the lateral ankle and described as a constant dull ache/throb with intermittent sharp pains He
endorses some intermittent lateral ankle/foot swelling and has pain with most activities He takes occasional Ibuprofen
and Tylenol that provides minimal pain relief He has known flat feet and his mother and sister who accompany him state
that "another family member required surgery for something similar." He denies fevers, chills, sweats, or weight loss He
does not play any competitive sports.
Physical Exam: BP 128/79, Pulse 83, Height 5'0", Weight 147lbs, BMI 28.71 General: no apparent distress Left ankle
exam: very guarded on exam, range of motion shows mild stiffness with plantar/dorsiflexion; however, he has severe
limitation with eversion/inversion. No swelling or ecchymosis, normal strength, tenderness to palpation over cubiod and
sinus tarsi region; remainder of exam normal. Bilateral foot exam: shows severe pes planus with hindfoot valgus left >
right. Left lower extremitiy with mild posterior tibial dysfunction. Gait analysis shows severe overpronation left > right. No
leg length difference.
Differential Diagnosis: 1. Tarsal coalition 2. Osteochondritis dissecans of talus 3. Peroneal tenosynovitis/tendinosis
4. Calcaneal stress fracture/reaction 5. Cuboid stress fracture/reaction 6. Fibular stress fracture/reaction 7. Cuboid
syndrome 8. Ewing sarcoma of calcaneus 9. Chronic osteomyelitis of calcaneus
Test Results: 1. Xray 3-view left ankle: no acute abnormality. Pes planus with soft tissue swelling of the dorsal aspect of
the hindfoot No fracture. Mortise intact. 2. X 3-view of left foot: no acute abnormality Pes planus with soft tissue
swelling of the dorsal aspect of the hindfoot. No fracture. 3. MRI left ankle: severe pes planus and hindfoot valgus No
evidence of tarsal coalition Moderate stress response anterior calcaneus and adjacent cuboid without stress fracture.
Final Diagnosis: Calcaneocuboid stress reaction; congenital pes planus
Treatment: Patient placed in walking boot X 3 weeks He was referred to physical therapy, but patient never attended
any sessions.
Outcome: Patient was compliant with boot and was essentially pain free at his followup visit 3 weeks later He was then
fitted for custom orthotics He was scheduled to followup 2 months later, but has unfortunately not returned to clinic.
Follow-Up: Gradually increase as tolerated in his orthotics He has not followed up in our sports medicine clinic.
Recurrent Knee Pain in an Adolescent
Authors: R. Ryan Carreon, MD; Kenneth Taylor, MD
Affiliation: UCSD Family & Sports Medicine Rady Children's Hospital 360 Sports Medicine, San Diego, CA
History: SL is a 14 y/o male with a self reported history of "loose joints". Two days prior to being seen in clinic he had
played tennis with a family member. He is usually inactive. That evening he started having bilateral knee pain, swelling,
and stiffness, worse in the right knee. He described his pain as a diffuse and sharp, aggravated by all activity, improved
with rest. He was taken to urgent care and referred to pediatric ortho for "chronic changes in his right knee", as reported
by his mother. He denied mechanical symptoms, but endorsed some swelling of his right knee.
Physical Exam: Trace effusion of right knee. Larger than expected joints of knees, elbows, and ankles. No TTP of
bilateral knees. FROM. Neuorvascullarly intact. Negaive patellar grind and glide. Normal laxity with bilateral lachman's,
posterior drawer, and varus and valgus stress. No pain with McMurray's. Mild left chest wall pectus carinatum. Exam of
bilateral hips and ankles otherise unremarkable.
Differential Diagnosis:
Patellar Subluxation
Partial ACL tear
Small meniscal tear
OCD
JRA
Reactive Arthritis
Genetic Disorder
Test Results: CBC WNL; CMP WNL; ESR 18; CRP 3.5 XR Right Knee: -Marked diffuse cortical irregularity of the
weightbearing surfaces of the medial and lateral femoral condyles and posterior patella as above. These appear chronic in
nature, though underlying osteochondral defects cannot be excluded. XR Left Knee: -Extensive abnormality of the femoral
condyles and to lesser extent the tibial plateaus. This is characterized by deficiency of bone and osteochondral
irregularities including several focal osteochondral defects. The findings are relatively similar to the contralateral knee. MRI
Right Knee: Diffuse heterogeneity of the articular cartilage with abnormal signal and morphology at the articular cartilage
epiphyseal cartilage interface. Additional areas of subchondral bone marrow edema, joint effusion and marked
hypertrophic synovitis likely reflect sequelae of chronic repetitive injury. XR bilateral Hips: - flattening of the superior
lateral right femoral head. Lack of sphericity of the left femoral head.
Final Diagnosis: Multiple Epiphyseal Dysplasia
Treatment: Appointment scheduled with Genetics January 2015. Educated to not participate in sports that involve
loading the joint. Final treatment to be determined after genetics evaluation.
Outcome: To be determined
Follow-Up: To be determined.
Forty Yard Disaster: A Groin Injury in a College Football Player
Authors: Neil Patil, MD; Jennifer Daily, MD; Raymond Shea, MD; Jeffrey Stimac, MD; Paul McKee, MD; Jessica
Stumbo, MD
Affiliation: Primary Care Sports Medicine Fellowship, University of Louisville and Kentucky - One Health
History: 21-year-old male collegiate football player was running full speed in a forty yard timing event and developed
acute sharp pain in his right groin He felt a pop that was associated with the pain The pain was described as sharp,
stabbing, and shooting at time of presentation There was no report of recent trauma to the affected side. The patient
reported the pain as constant and rated it an 8/10 alleviated only by pain medicine. Treatments included pain medicine,
ice, rest, elevation, and crutches.
Physical Exam: Gen: Male, fit, well built, well developed, well nourished in moderate distress due to pain
MSK: Right
hip/thigh in comparison to left hip/thigh. Tenderness to palpation over the anterior and lateral aspect of the hip.
Decreased active and passive range of motion. Pain with hip flexion and knee extension against resistance. Pain with
weight bearing on right leg. No gross deformity. Neurovascular: 2+ posterior tibial and dorsalis pedis pulses, capillary
refill &lt;2 seconds Neuro: reflexes intact, CN II-XII intact, No focal deficits, sensation intact Skin: No erythema, rash, or
lesions noted, normal hair growth
Differential Diagnosis: -Quadriceps Muscle Rupture -Quadriceps Muscle Strain -Sartorius Strain -Labral Tear Femoral Neck Stress Fracture -Femoroacetabular Impingement -Inguinal Hernia -Transient Synovitis -Osteonecrosis
Test Results: MRI of Right Thigh: 1. Proximal detachment/avulsion of both the direct and indirect heads of the right
femoris muscle/tendon origins from the anterior inferior iliac spine and supraacetabular ridge, respectively, with 5-6 cm
tendon retraction. 2. Mild grade 1 strain of the anterolateral aspect of the right Sartorius muscle.
Final Diagnosis: Right Rectus Femoris Detachment/Avulsion
Treatment: Medication for pain control. Surgery to repair the avulsed right rectus femoris tendon Crutches and a knee
brace, which was locked in extension, were prescribed to the patient for all weight bearing activity status-post surgery
Rehabilitation with physical therapy for strengthening, flexibility, and range of motion exercises.
Outcome: After several weeks of rehabilitation, he was able to get back to full activity and full game participation.
Follow-Up: On follow up visit, the patient demonstrated mild pain that was much improved with minimal tenderness to
his right groin A repeat MRI three months later showed satisfactory postoperative appearance of the right rectus femoris
avulsion repair to the anterior inferior iliac spine Patient was able to advance to full activity due to findings of repeat MRI
He was cleared to return to full activity as tolerated.
Knee Pain and Swelling in a 14 Year Od Soccer Player
Authors: Kristen Michael, DO; Margaret Gibson, MD
Affiliation: University of Missouri Kansas City Department of Family Medicine and Community Medicine,
Kansas City, MO
History: A 14-year-old male, soccer player, presented to our Sports Medicine Clinic with complaints of right knee pain
that had been occurring for the past two weeks The patient described the pain as a stabbing pain in the anterior portion
of his knee He could not recall any specific injury or movement that caused the pain but did remember that it began
while he was at soccer practice He said that he was able to continue with practice that day but as the week progressed he
had worsening pain and therefore approximately 5-6 days after the pain began he had to stop playing soccer He denied
any clicking, popping, or instability of his knee He reported that he noticed swelling on the anterior aspect of his knee He
said that the swelling appeared five days after onset of his pain He reported that activity, specifically sports, exacerbates
his pain He had been using ice on his knee and taking anti-inflammatories for the pain and had some relief of his
symptoms He denied any previous injuries to the knee and denied ever having similar pain in other joints Family history
is negative for rheumatologic conditions.
Physical Exam: Patient is a well-nourished, well-developed 14-year-old male At the time of exam he was afebrile and in
no acute distress Height 5' 5" and weight 125lbs He was able to ambulate without assistance Inspection of the knee
revealed a large knee effusion There was no erythema, or ecchymosis Patient was tender to palpation over the distal
quadriceps, vastus lateralis, vastus medialis, MCL and the medial joint line Range of motion of right knee was 0-100°,
compared to 0-110 on the left Muscle strength of right lower extremity was approximately 60% in the quadriceps and
hamstrings when compared to left He had intact anterior and posterior drawer test He had negative Lachman There was
no pain with varus or valgus strain at 0 degrees or at 30° There was mild discomfort with McMurray's testing, with pain
located over the medial joint line, but no clunk or pop Patient had pain with Thessaly testing Sensation was intact to light
touch.
Differential Diagnosis: Meniscus injury, Physeal injury, Bone contusion, Synovitis, Juvenile arthritis, Lyme disease
Test Results: An X-ray of the right knee showed closing growth plates but was otherwise unremarkable.
An MRI of the
right knee revealed a large joint effusion with associated synovitis Small quantities of Rice bodies were visualized layering
within the joint capsule No internal derangement or injury There was a small nonspecific focus of patchy marrow edema
within the medial patella. The patient returned for follow-up shortly after obtaining the MRI of his right knee He was still
experiencing pain and swelling The findings from the MRI were discussed and he was started on Naproxen 250mg BID A
referral to rheumatology was placed to discuss further treatment and workup for his knee pain. The rheumatologist
reviewed the patient's history and MRI findings The differential included Juvenile Idiopathic Arthritis, Lyme disease, or
other autoimmune conditions A CBC, urinalysis, CMP, Rheumatoid factor, ANA, HLA-B27, LDH, Uric acid, Tuberculosis and
Lyme series was collected All results returned back within normal limits.
Final Diagnosis: Juvenile Idiopathic Arthritis, Monoarticular Arthritis of the right knee
Treatment: The patient received an intra-articular steroid injection in the right knee He is also taking Indocin 50mg
twice daily for his pain.
Outcome: The patient will follow-up in 4 weeks to monitor for improvement.
Follow-Up: Due for follow-up in 4 weeks.
Atypical Chest Pain in a NCAA Division 1 Football Player
Authors: Anna L. Waterbrook, MD; Donald Porter, MD
Affiliation: University of Arizona, Tucson, AZ
History: 18 year-old NCAA Division 1 football player presented to the training room with complaints of shortness of
breath and sharp, pleuritic chest and rib pain for the last 2 days. He denies any cough or fever. Pain was insidious in onset
and not associated with exercise. He denies any trauma. He denies any palpitations, syncope or presyncope, nausea,
vomiting, diarrhea or abdominal pain. He has not had any recent surgeries or hospitalizations, and has no significant
family history He does have a history of a right lower lobe pneumonia two years prior for which he was hospitalized, but
had a full recovery. During that hospitalization he had a chest CT which confirmed pneumonia and ruled out pulmonary
embolism He denies use of any medications, supplements, or known medication
Physical Exam: He appeared uncomfortable but did not have any increased work of breathing or respiratory distress.
Vital signs: BP=123/75; Temperature=99.3°F Tympanic; Pulse=93; Respirations=15; Pulse oximetry 97% on room air.
Cardiovascular exam revealed regular rate and rhythm, no murmurs with strong distal pulses. Respiratory exam revealed
clear lung fields with equal breath sounds bilaterally, no retractions, crepitus or deformity. He was speaking in full
sentences. Abdomen was soft, non-tender and non-distended with normal bowel sounds and no masses. Mental status
was clear and he was alert and oriented X 4.
Differential Diagnosis: Pneumonia, pleurisy, costochondritis, rib fracture, pneumothorax, pulmonary embolism,
pericarditis, myocarditis, inherited structural or arrhythmogenic cardiac abnormality, other
Test Results: CXR performed at Student Health was read by radiologist as findings suspicious for left lingular
pneumonia. Repeat CXR in the emergency department (ED) was negative for any acute abnormalities EKG showed an
S1Q3T3 pattern and d-dimer was elevated at 3.25 ug/mL. Troponin I was less than 0.02. Chest CT to evaluate for
pulmonary embolism showed bilateral pulmonary emboli with embolus in the right main pulmonary artery and segmental
extension involving all lobes, left lower lobe and lingular emboli with lingular infarction and evidence of possible mild right
heart strain Bedside echocardiogram noted mild "D sign," with formal echocardiogram showing a mild increase in global
right ventricular size, evidence of mildly elevated right atrial pressure, but no other cardiac abnormalities. Bilateral lower
extremity ultrasound and pelvis MRA found no evidence of blood clots. A complete hypercoagulability work-up was
negative.
Final Diagnosis: Bilateral pulmonary emboli
Treatment: Treatment: He was first evaluated in the training room. Given his discomfort he was sent for a CXR and
treated with over-the-counter medications for pain. After initial CXR results obtained, he was started on appropriate
antibiotics, but returned to the training room later in the afternoon with continued significant pain. Repeat cardiovascular
and lung exams, as well as vital signs were unchanged. Given his amount of discomfort, he was sent to the ED for further
evaluation. Exam in the ED was stable. His pain was treated with narcotics with some relief. After the results of the CT
chest were obtained, he was placed on anticoagulation and admitted to the ICU.
Outcome: He was discharged home on hospital day #3 on Lovenox in improved condition.
Follow-Up: He followed up in the training room the day after hospital discharge and his pain was significantly improved.
He was allowed to work-out with the football team, but was not allowed to participate in any contact activities. He was
seen by a hematologist who recommended lifelong anticoagulation given how large his pulmonary emboli were and the
risk that if he had any future blood clots they would likely be larger and life-threatening. Given this, he is no longer eligible
to participate in football or any other contact sports.
A Dangerous Kickoff
Authors: Thomas McKellar McCloy, MD; Leyen Vu, MD
Affiliation: Providence St. Peter Family Medicine Residency Program, Olympia, WA
History: 15 year old high school football kicker who suffered injury to the left knee during a kickoff at practice. He ran
approximately 10 yards, planted the left foot in preparation to kick the ball with his right foot when he suddenly collapsed
to the ground after loss of extensor control in the left knee. There was immediate, severe pain, swelling and deformity of
the left knee. He was attended to by the athletic trainer, placed in a knee immobilizer and crutches and immediately sent
to the emergency room. No prior history of left knee injury, but states he was having intermittent mild left patellar tendon
pain for 2 weeks preceding the injury.
Physical Exam: General: Alert and oriented to person, place and time Musculoskeletal: Left knee is edematous with
obvious deformity at the patellar tendon and tibial tuberosity. Edema extends from the supra-patellar region down past
the patellar tendon insertion at the tibial tuberosity. There is marked tenderness with palpation and large effusion. Unable
to perform full ligamentous exam due to guarding. Neurovascularly intact. No evidence of compartment syndrome.
Neurological: Normal sensation. Normal muscle tone. Coordination normal.
Differential Diagnosis:
1. Patellar fracture or dislocation
2. Patellar tendon rupture
3. Tibia or fibula fracture
4. Quadriceps tendon rupture or muscle tear
Test Results: X-ray Left Knee, lateral view - Findings: Large tibial tuberosity avulsion fracture that is severely rotated.
Fracture fragment position is perpendicular to the shaft of the tibia. Extensive soft tissue irregularity and swelling adjacent
to the tibial tuberosity and patellar tendon regions. There is a knee effusion. Distal femur, patella and proximal fibula are
intact.
Final Diagnosis: Displaced avulsion fracture of the left tibial tuberosity with significant rotation.
Treatment: Patient received IM dilaudid and phenergan for pain control and to facilitate exam and imaging. Orthopedic
surgeon examined the patient and reviewed imaging and decided to proceed with ORIF the next morning. Longitudinal
incision was made directly over the left tibial tubercle. The fracture fragment was easily seen. They physeal plate was still
open, but the fracture did not extend intra-articularly. No ligamentous or meniscal damage was seen. The fragment was
flipped back into place and pinned down with two 4.0 mm partially threaded screws. These screws were chosen due to
feat of damaging the neurovascular bundle posteriorly, which was actually quite close to the posterior aspect of this. A
full-thickness bite was decided to not be in his best interest due to the location of the neurovascular bundle.
Outcome: The patient tolerated the surgery very well and went home that day. He was non-weight bearing for 4 weeks
after surgery, at which point physical therapy was started. He will follow up 6 weeks after starting physical therapy.
Follow-Up: After a period of non-weight bearing status for 4 weeks, physical therapy progressed to weight bearing in a
locking hinged knee brace. Full activity is anticipated in 12-16 weeks. Plan for return to play next season.
Dyspnea in an Amateur Basketball Player
Authors: Vania Reyes, MD
Affiliation: Saint Mary's Hospital, Waterbury, CT
History: 23 year old male with no significant past medical history presented to the emergency room with worsening
dyspnea on exertion. About six weeks ago he had upper respiratory tract symptoms, including cough and congestion. He
went to a walk in clinic, was given a course of azithromycin, and symptoms improved. About two weeks ago he noted
worsening shortness of breath. Prior to symptom onset, he was able to play a full game of basketball and then he noticed
he would get significantly short of breath after playing for only two minutes. He went to the emergency room and was
given a course of Levaquin and sent home. His symptoms progressively worsened over the next couple of days. On the
day of presentation, he was getting short of breath walking across the room at home.
Physical Exam: Physical examination was notable for a blood pressure of 106/75 and a heart rate of 134. He had jugular
venous distension up to his mandible. He had decreased breath sounds at both bases and upper abdominal tenderness.
He rapidly deteriorated and his blood pressure dropped to 86/50.
Differential Diagnosis: Shock due to Pneumonia, Parapneumonic effusion, Empyema, Myocarditis
Test Results: Initial labs revealed a white blood cell count of 13.9, creatinine 1.4, total bilirubin 2.2, AST 69, ALT 178, and
lactic acid 3.4 A 12-lead ECG showed left atrial enlargement, LVH, and T- wave inversions in V4-V6. Chest x-ray revealed
cardiomegaly and bilateral pleural effusions. He was resuscitated with intravenous fluids and vasopressors and started on
empiric antibiotics His urine output decreased and AST and ALT increased to 371 and 414, respectively. Abdominal
ultrasound was significant for a moderate amount of intraperitoneal fluid, but was otherwise unremarkable. Thoracentesis
of the right effusion revealed a transudative fluid Echocardiogram showed an ejection fraction of 10 percent, a severely
enlarged left ventricle, moderately enlarged left atrium, right atrium and ventricle, and severe mitral and tricuspid
regurgitation.
Final Diagnosis: Cardiogenic Shock due to viral myocarditis
Treatment: He was tapered off norepinephrine and started on milrinone. His urine output improved and hemodynamics
stabilized.
Outcome: Once hemodynamically stable, he was transferred to another hospital with a presumption that he may need a
left ventricular assist device (LVAD) and/or transplant.
Follow-Up: During his hospitalization, he was continued on pressor support. His EF improved and he was discharged
home. He did not require a LVAD. Over the course of about 2 months, his EF continued to improve. He was started on a
beta blocker and ACE inhibitor He was able to gradually return to his usual activity and his EF normalized. An interesting
paradox with myocarditis has been seen in studies, observing that patients with fulminant myocarditis, while more critically
ill at presentation, were more likely to recover left ventricular function than were those with acute myocarditis. McCarthy
et al studied 147 patients considered to have myocarditis from 1984 to 1997 Seven patients in the study received heart
transplantation, none of these patients were in the fulminant group The authors concluded that fulminant myocarditis
was an independent predictor of transplantation-free survival after adjusting for age, hemodynamic variables, and
histopathological findings, and has an excellent long term prognosis.
Hip To Be Healed--A New Treatment Option for Hip Pain
Authors: Thomas W. Plut, DO
Affiliation: Lourdes Medical Associates, Moorestown, NJ
History: A 51 year old active duty army soldier was referred for evaluation of bilateral hip pain of 2 years duration. He
described the atraumatic, insidious onset of bilateral greater trochanteric region pain initially associated with running, but
eventually causing pain with prolonged walking and interfering with sleep. Prior to onset, he had run multiple halfmarathons, but had to discontinue all running due to symptoms. He described a maximal 8/10 severity, bilateral,
symmetrical, generally achy but occasionally sharp, trochanteric pain unaccompanied by mechanical or neurological
symptoms. Pain was increased with running, prolonged walking, and lying on either side. Prior treatment included activity
modification; anti-inflammatory medications; comprehensive PT including core training, Graston technique, and dry
needling; Lidoderm patches, and ultrasound guided corticosteroid injections. None of these interventions provided
significant relief. He was otherwise healthy and had no history of prior major lower limb injuries.
Physical Exam: Physical examination was normal with the exception of bilateral trochanteric tenderness, trochanteric
pain reproduction with hip internal rotation (but no groin pain), and complaints of soreness during hip abductor manual
muscle testing. Specifically, FABER, Stinchfield, and hip impingement testing were negative for groin pain, and he was
neurologically intact and without sciatic or femoral nerve tension signs.
Differential Diagnosis:
1.
2.
3.
4.
5.
Gluteal tendinopathy
Greater trochanteric bursopathy
Hip labral tear
Femoroacetabular impingement
Hip Arthritis
Test Results: Pre-referral non-contrast hip MRI demonstrated bilateral insertional gluteus minimus and medius
tendinosis without superimposed partial thickness tearing, and no evidence of arthritis, labral tear, or femoroacetabular
impingement.
Final Diagnosis: Chronic refractory bilateral gluteal tendinosis
Treatment: Following discussion of treatment options, the patient elected to proceed with sonographically guided
percutaneous ultrasonic tenotomy utilizing the TX-1 device (Tenex Health). The left hip was treated on 10/11/13, using a
total energy time of 3 minutes, 44 seconds. The right hip was treated on 11/8/13, using a total energy time of 3 minutes,
56 seconds.
Outcome: There were no procedural complications and the patient reported minimal post-procedural discomfort, which
was adequately treated with ice therapy. For both hips post-procedure, the patient was instructed to be non-weightbearing for 2 days. He noted marked improvement in pain associated with walking and lying on his side for each hip by
the one week follow up visit. He was instructed on a home stretching program, and allowed to use a stationary bike at a
low resistance setting for 20 minutes. At the 3 week follow up, he was allowed to incorporate low resistance elliptical for
20 minutes and elected procedure for right hip He was advised to gradually increase time and resistance on the elliptical
and stationary bike.
Follow-Up: Though against my advice, at the 6 week follow up visit for the right hip, the patient had run 8 miles with
no pain. He was instructed to further advance all activities as tolerated, including participation in half marathons. Since the
one year follow up, he continues to run half marathons with maximal pain VAS 3/10 and no sleep disturbance. He was very
satisfied with his results and reported that he would have the procedure again or recommend it to a friend or family
member. To my knowledge this is the first report of sonographically guided percutaneous tenotomy utilizing TX1 device
for the the gluteus tendons. When appropriate conservative measures fail, based on this single case, this technique may
be an option that allows for return to activity.
College Soccer Player with Exercise-Induced Lightheadedness
Authors: Jesse Coenen, MD; Ed Reisman, MD
Affiliation: Spokane Sports Medicine Fellowship, Spokane, WA
History: Nineteen year old female Division III college soccer player has experienced exercise-induced headache and
lightheadedness for one year, as well as two episodes of presyncope (pallor, diaphoresis, vision blackout) while standing
still during class The athlete's past medical history was negative for cardiovascular disease but positive for non-Hodgkins
lymphoma in remission There was no family history of exercise intolerance or other cardiac symptoms or risk factors Her
only medication was combined oral contraceptive pill Previous blood tests, including hemogram, thyroid studies, and
basic metabolic panel, were normal Echocardiogram, EKG, spirometry, and brain MRI were also normal Exercise tolerance
test was performed, and revealed mildly decreased exercise tolerance secondary to hypotension causing lightheadedness
Tilt table test was recommended, and midodrine (alpha-1 receptor agonist) was prescribed, 10-15mg daily with exercise
Over the next few months her symptoms improved, and the tilt table test was never completed She returned to campus
for fall semester, and resumed playing college soccer However, the symptoms returned Frequently she had to stop within
ten minutes of playing due to lightheadedness There was no improvement with an increased dose of midodrine.
Physical Exam: The athlete appeared well. Heart rate (HR) 71 and regular Blood pressure (BP) 118/72 Cardiac
auscultation revealed normal S1 and S2 with no murmurs PMI non-displaced 2+ peripheral pulses Lungs clear to
auscultation bilaterally Neuro exam revealed no abnormalities.
Differential Diagnosis: Orthostatic hypotension, dehydration, vasovagal syncope, cardiac arrhythmia, seizure,
hypoglycemia
Test Results: Tilt Table Test: The athlete started without symptoms and was tilted to 70°. The HR slowly increased and at
15 minutes she felt "droopy" with a BP of 104/70 and HR of 95. At 20 minutes she started yawning with the HR at 130, and
at 22 minutes she became very uncomfortable. The HR precipitously dropped to 32 with BP 107/63. The HR then rose
quickly to 113 and then back down to 50. It continued to slow, and there was a precipitous drop in BP to 53/26. There was
a 5 second asystole with syncope. She was placed flat and quickly recovered. Cardiologist interpretation: the test was
positive for syncope associated with Bezold-Jarisch reflex; hemodynamic responses mixed.
Final Diagnosis: Orthostatic syncope/presyncope mediated by Bezold-Jarisch reflex As blood pools in the lower
extremities causing decreased intracardiac volume, the carotid sinus baroreceptors sense hypotension and trigger a
sympathetic response But the left
Treatment: The cardiologist recommended this athlete could safely participate in sports if she would play in an accidentfree environment and rest immediately when having symptoms. However, after a meeting with the team physician and
athletic trainer to discuss risks and her current limited ability to play, she chose to discontinue participation in soccer this
year.
Outcome: There has been no recurrence of symptoms at rest Patient met with a cardiac electrophysiologist, who
agreed with the decision to withdraw from play at this time.
Follow-Up: It has been suggested that young individuals with this condition may "outgrow" it, and the athlete might
consider trying to play soccer again in future seasons She continues to follow-up with her cardiologist.
Case Report: Spontaneous Lateral Thigh Hematoma
Authors: Thomas Hoke, MD; John Hatzenbuehler, MD
Affiliation: Maine Medical Center Sports Medicine, Portland, ME
History: SD, a 53 year old cook, was referred by her PCP to our ultrasound clinic for drainage of a fluctuant mass on left
lateral thigh that was documented to be 8x6x6 cm by a different ultrasonographer. She reported the mass presented
spontaneously 6 weeks prior and initially was accompanied by bruising though that had resolved. She denied any
preceding trauma other than innocently bumping against a table about the time the collection developed. The lateral hip
was painful during the day, but worse at night; she could not sleep on that side due to pain. She denied fevers,
constitutional symptoms, easy bleeding or bruising.
Physical Exam: T: 36.1° BP: 138/78 HR: 106 RR:18 Height:158 cm Weight:74 kg BMI: 30.4 No acute distress. Left leg
with protuberant 15cm fluctuant mass inferior and posterior to the greater trochanter. Left hip with full ROM, no pain with
passive internal or external rotation or FABER. Pain with resisted abduction and external rotation. Exquisite tenderness to
palpation posterior to greater trochanter.Legs with bilateral livido reticularis, no bruising, petechial, or edema, 2+ DPs.
Differential Diagnosis:
Spontaneous hematoma
neoplastic process
coagulopathy
hip abductor/external rotator tear
trochanteric bursitis
abscess
Test Results: CBC: WBC 10.2 HGB 12.2 HCT 38.1 PLT 389 CMP: Electrolytes, LFTs normal. Creatinine 1.34, BUN 21, GFR
41 INR 0.9 PTT 27 Subcutaneous Aspirate: Negative culture, 1152 leukocytes/cmm, 97% lymphocytes. Cytology negative
for malignancy. X-ray Left Hip: Asymmetric sclerosis of the left greater trochanter along with bony resorption superiorly
and laterally. Faintly increased density of the gluteus tendons. MRI Left Hip with and without contrast: 1.2.4 x 5.6 x
8.1cm irregularly shaped non-enhancing fluid collection in the subcutaneous tissues superficial to the gluteus maximum.
No fascial edema. 2.Extensive posterior gluteus medius tendinosis and partial thickness tear. 3.Myotendinous edema
within the gluteus minimus anteriorly extending to some extent distally towards the insertion. Associated sub-gluteal
bursa distension. 4. Small partial proximal vastus lateralis tear. 5. No hip effusion. No bony irregularity.
Final Diagnosis:
Extensive gluteus medius tear
bursitis
loculated lipomatous tissue mass
Treatment: 110 mL of sanguineous fluid was drained under ultrasound guidance. Given spontaneous nature of this
collection and radiologist report of greater trochanter sclerosis and resorption on radiograph, the above MRI was obtained
to rule out neoplasm; it showed extensive gluteus medius and minimus tendinosis and tears, but no bony abnormalities.
She returned one month later with re-accumulation of fluid collection and continued lateral hip pain. Screening blood
work returned normal ruling out infection and coagulopathy. The hematoma was again drained (60mL) and subsequently
injected with 40 mg of depomedrol. She returned three weeks later still with significant gait dysfunction secondary to pain
in the lateral hip. She continued to work long hours in the kitchen. Pain had precluded her from participating in therapy.
Given failure of time, home exercise, aspiration and steroid injection, she was referred to orthopedic surgery for
consideration of surgical management.
Outcome: 14 weeks after initial presentation to our clinic, she underwent open gluteus medius repair with placement of
three suture anchors, a bursectomy, and excision of lipomatous mass. Surgical pathology was negative for malignancy.
Follow-Up: Two weeks postoperatively, she is starting to ambulate more. Gait and pain are improving. She has yet to
return to work.
Chronic Left Knee Instability in an 18-Year-Old Volleyball Player
Authors: Amy Corrigan, DO; David Harsha, MD
Affiliation: St. Vincent's Sport Performance, Indianapolis, IN
History: An 18 yo. Caucasian female presented with chronic, recurring left knee pain and instability after stepping out of
a roller coaster 6 months after her third left ACL reconstruction. Incident resulted in tearing her third ACL graft. She first
tore her ACL on an awkward landing during a volleyball game 1/8/12. She underwent ACL reconstruction using a patellar
tendon autograft on 2/1/12 followed by rehabilitation and return to play less than 6 month later She again tore the L ACL
on 7/12/12 while doing volleyball jumping drills. She underwent ACL reconstruction on 8/8/12 by a second surgeon using
a posterior tibialis allograft She completed rehabilitation and returned to volleyball in April 2013 using an X2K brace On
1/14/14 she slipped and fell at home, tearing her second graft. A third surgeon reconstructed her ACL 1/31/14 using a
bone-patella-bone allograft and she again completed rehabilitation. This time she did not return to any sports before
presenting to our office. She presented to our office for another opinion, expressing a desire to return to sports. PMHX
positive for anemia She has never smoked or used illicit drugs and denies alcohol use FMHX positive for hypertension.
Current medications were iron tabs and Lo Loestrin.
Physical Exam: Afebrile. Alert and oriented. Vital signs within normal limits. Left knee showed a well-healed incisions
over the lateral thigh and anterior tibia. She had positive Lachman and Lelli tests. Range of motion - 3° to 145°. Meniscal
signs negative. Patellofemoral shrug and grind tests negative. Good quad tone and strength.
Differential Diagnosis:
1. Technical failure of ACL reconstruction
2. Biological failure of ACL reconstruction
3. Traumatic failure of ACL reconstruction
4. Failure of ACL due to associated laxity
5. Arthrofibrosis
Test Results:
1.
2.
MRI 8/20/14 - low grade partial tear ACL graft. No evidence of Cyclops lesion as seen on prior exam Punctate
medial and lateral meniscus tears. Small new effusions.
CT left knee 9/16/14 - femoral screw protrusion beyond posterior cortical margin by approximately 9mm Tibial
screw protrusion beyond cortical margin by 3mm anteriorly and 5mm of anterior translation of the tibia relative
femur. Expansion of tibial and femoral tunnels showing 2.5 cm bone defect in tibial tunnel and 16-18 mm defect
in femoral tunnel.
Final Diagnosis: Recurrent ACL tear with 2.5 cm bone defect in tibia and 16-18 mm defect in femur with pain and
instability. Concern for underlying autoimmune disorder causing destruction of multiple allografts.
Treatment:
1.
2.
3.
Started physical therapy October 2014 to correct strength deficits
Bone graft of L femur and tibia planned December 2014 to correct bone defects in anticipation of supporting an
ACL graft in the future. Will plan to draw basic lab pre-op including testing for P acnes that could suggest
infection.
Post-pone ACL reconstruction until bone graft stable.
Outcome: Pending bone graft December 2014 and possible ACL reconstruction.
Follow-Up: Plan CT of left knee 6 months post op from bone graft to establish healing Will need therapy and then
consider ACL reconstruction. At this time is not to return to sport but will continue to work with PT.
Knee Pain in a 14 Year Old Football Player
Authors: Joseph Kennedy, BS; David Soma, MD
Affiliation: Mayo Medical School, Mayo Clinic, Rochester MN
History: A 14 year-old boy with a history of acute lymphocytic leukemia now in remission was at football practice in
August of 2014 participating in a tackling drill when he fell to the ground and landed on his left knee He could not recall
any specific mechanism for the fall but immediately noticed diffuse pain of the knee and associated suprapatellar swelling
He was initially evaluated in the emergency department where plain radiographs and clinical examination were
unremarkable; his symptoms were attributed to mild ligamentous injury He presented for sports medicine evaluation six
weeks following the injury having continued to play football with daily, dull and aching left knee/distal anterior thigh pain
worse with activity and intermittently present at rest He denied any erythema, drainage, fevers, chills, myalgias or
arthralgias elsewhere in his body, or other systemic symptoms.
Physical Exam: Left knee on inspection appeared to have increased fullness over the suprapretellar region without any
erythema or overlying skin/soft tissue abnormality On palpation, the distal quadriceps felt fibrous and firm, yet still
mobile, and was mildly to moderately tender No effusion was present and he was without tenderness over either the
medial or lateral joint lines Extension of the knee was full, however pain limited active and passive flexion to 120° and
130°, respectively. Strength was 5/5 and he was neurovascularly intact throughout the LE bilaterally Negative Lachman,
varus and valgus stress maneuvers, posterior and anterior drawer McMurry maneuver, limited by pain, was without any
frank abnormality No patellar apprehension Mild crepitus noted with lateral greater than medial patellar deviation.
Differential Diagnosis:
Myositis ossificans
Morel-Lavall&eacute;e lesion
Patellar bursitis
Quadriceps tendon injury
Malignant neoplasm or recurrence of leukemia
Patella sleeve injury or patella fracture
Test Results: CBC with differential: Normal. Left Knee US: "In the region of the swelling is a thin area of
hypoechogenicity within the subcutaneous fat measuring 4 cm transversely [right to left] and approximately 6 cm
longitudinally but only 0.3 cm in AP dimensions. This thin sliver of hypoechogenicity lies deep in the subcutaneous fat and
is separate from the underlying quadriceps muscles and tendon. The quads tendon itself is intact with no evidence for
partial tearing. No increased blood flow within the hypoechoic area. No knee joint effusion. The patellar retinacula are
intact." Left Knee MRI: 1. Probable Morel Lavall&eacute;e lesion in the anterior left suprapatellar soft tissues. 2. Cruciate
and collateral ligaments and both menisci are intact.
Final Diagnosis: Morel-Lavall&eacute;e lesion of the left anterior thigh
Treatment: The patient was treated with Tubigrip&reg; stockinette and cryotherapy Consideration was given for both
US-guided drainage and sclerodesis, however the relatively minor size of the fluid accumulation did not warrant this more
invasive therapy.
Outcome: One month after initial diagnosis and treatment, the patient reported moderate resolution of swelling and
complete resolution of his pain.
Follow-Up: Return to normal sporting activity with use of compression stocking to prevent fluid reaccumulation
Consideration given to drainage and sclerodesis if swelling or active range of motion worsen.
A High School Volleyball Player with Proximal Thigh Swelling
Authors: Rachel Chamberlain, MD; Heath Thornton, MD; Chris Miles, MD
Affiliation: Wake Forest University Baptist Health
History: 17 year old female volleyball player initially presented to high school ATC with medial, proximal right thigh
swelling. She noticed it shortly after volleyball season began, she had pain with activity and ATC measured a 5x5 cm area
of swelling ATC was concerned for a hematoma, and treated with stretching and NSAIDS, re-assessing weekly After no
improvement for two weeks patient was evaluated by team physician on sideline of Friday night football game. ROS: mild
pain with playing volleyball; no numbness or tingling, negative for fevers, chills, night sweats, adenopathy, and other areas
of swelling. Her medical history was significant only for migraine headaches and seasonal allergies. She takes occasional
Naproxen and Cetirizine, not on oral contraceptives. Social History: Senior in high school, lives with parents, does not
smoke, use drugs, drink alcohol, is not sexually active. Family History: negative for pediatric cancers, significant for both
grandfathers with prostate cancer and mother with psoriasis.
Physical Exam: General: no distress. Pulm: normal work of breathing. Neuro: Alert, oriented x3. Psych: Normal
mood and affect, insight and judgment appear in tact. Skin: no erythema, rashes, lesions. Musculoskeletal: Right
proximal medial thigh: Right upper thigh obviously larger than left. Somewhat firm, but not indurated, measures 5x6
cm, no obvious fluctuance. Non-painful to palpation, no pitting edema. 2+ pulses. Full ROM of hip and knee, non-painful
ROM. Strength is 5/5 and symmetric, no pain with strength testing. Normal sensation of right lower extremity.
Differential Diagnosis: Hematoma, DVT, synovial or ganglion cyst, abscess/infection, lipoma or hemangioma, soft
tissue sarcoma, osteosarcoma, lymphoma.
Test Results: Soft Tissue Ultrasound: Heterogeneous, solid soft tissue mass in the right thigh measuring 8.5 x 6.7 x 6.4
cm. Findings are most concerning for a soft tissue neoplasm, such as sarcoma. MRI R thigh: Enhancing mass with
aggressive features within the medial compartment of the right thigh. Differential considerations would include a sarcoma.
An alternative possibility would include a nerve sheath tumor, malignant or benign. Pathology: The lesion is a densely
cellular spindle cell neoplasm with oval to elongate nuclei and relatively scant cytoplasm. The mitotic rate is brisk and
there is focal necrosis (overall around 20% necrosis). Immunoperoxidase studies were performed; The morphology of the
tumor is consistent with a synovial sarcoma. Cytogenetic Analysis: Abnormal: Cytogenetic analysis revealed an abnormal
karyotype with the presence of a X;18 translocation and a 5;10 translocation. The t(X;18) is commonly associated with
synovial sarcoma.
Final Diagnosis: Synovial Sarcoma
Treatment: Patient is undergoing neoadjuvant chemotherapy and radiation. Definitive surgery is planned at a later date.
Outcome: Treatment is ongoing. Synovial Sarcoma is an extremely rare cancer found in children, adolescents and young
adults with a mean age of 35. It accounts for only 6% of soft tissue sarcomas, and is found in limbs about 66% of the time,
but can be found anywhere in the body. Poor prognostic predictors include: size &gt; 5 cm, deep seated, inadequate
surgical resection, and local recurrence, patient age &gt; 20 years, monophasic subtype, and mitotic activity of &gt; 10 per
high power field Historically, Synovial Sarcomas have had very poor prognosis, with 5 year survival of 40-75% and 10 year
survival rate of 30-40% However, more recently, with new treatments and combinations of radiation, chemotherapy and
surgery, there is emerging evidence that survival rates are improving.
Follow-Up: Patient is undergoing chemotherapy and radiation at this time, and is not participating in sports. This case
reminds the primary care sports doctor to keep rare but deadly diagnoses in mind when making differentials for extremity
swelling.
Chronic Elbow Pain in a Competitive Strongman
Authors: Lisa Neverman, MD; Mederic Hall, MD
Affiliation: University of Iowa Hospitals and Clinics, Iowa City, IA
History: A 39 year old Caucasian male competitive strongman presented to the sports medicine clinic with chronic right
elbow pain. One year prior, he had experienced sharp pain in the posterior elbow region while performing an overhead
push press. This became progressively worse over the next several months to the point he discontinued training and
competition due to pain. He had been evaluated by multiple providers and failed physical therapy, chiropractic, oral antiinflammatory and analgesic medications and local corticosteroid injection to his region of pain. At time of presentation to
our clinic, he was having pain with simple activities of daily living and was unable to perform any upper body pushing
activities without significant pain.
Physical Exam: Examination of the right elbow demonstrated full range of motion without catching, clicking, or
snapping. There was no tenderness over the medial or lateral epicondyles. Pain was present with palpation of the triceps
insertion at the olecranon. There was no evidence of olecranon bursal swelling, warmth or erythema. Tinel's sign was
negative at the ulnar nerve about the elbow. Pain was exacerbated with resisted elbow extension at the triceps insertion.
Hyperextension test was negative. There was no evidence of valgus or posterolateral rotatory instability. The upper
extremity neurovascular examination was normal.
Differential Diagnosis:
Triceps tendinopathy
Posterior impingement
Stress fracture of the humerus
Chronic joint instability
Osteochondral defect
Osteoarthritis
Olecranon bursitis
Test Results: Radiographs demonstrated an enthesophyte at the olecranon and likely intra-tendinous calcifications in
the distal triceps. No degenerative joint changes. MRI was negative for intra-articular pathology and no suggestion of
stress reaction. There was increased T2 signal in the triceps insertion. Diagnostic ultrasound of the triceps tendon
confirmed intra-tendinous calcifications, cortical irregularity at the olecranon and focal regions of hypoechogenicity
consistent with chronic tendinosis. A small focal partial thickness tear was appreciated.
Final Diagnosis: Chronic triceps tendinosis
Treatment: Patient had failed appropriate conservative treatment and sonographically guided percutaneous tenotomy
was recommended. The TX-1 device (Tenex Health) was used to debrided the degenerative regions of the tendon
including the intra-tendinous calcifications and abrade the olecranon. Total energy time was 3 minutes and 20 seconds.
Patient was given daily range of motion exercises and recommendations to avoid any lifting/pushing over the first 3
weeks.
Outcome: At 3 week follow up patient reported no pain with daily activities and began submaximal strengthening
exercises with physical therapy. By 6 weeks, he had only mild pain at end-range extension and had returned to performing
light weight pushing exercises without pain. He was allowed to progress his strength training as tolerated, but avoiding
ballistic activity. At 3 month follow up patient had fully returned to all lifting activities and was able to painless bench press
300+ lbs and overhead press 200+ lbs. He was allowed to progress back into ballistic activity required for his sport.
Follow-Up: Return to Activity and Follow-Up: At 1 year follow up patient had returned to full competition with no pain or
limitation. He reported extreme satisfaction with the procedure and would recommend it to others. To our knowledge, this
is the first report of percutaneous tenotomy of the triceps tendon in a competitive strongman. While triceps tendinopathy
is a relatively rare condition, it can be debilitating for the competitive strength athlete. When conservative measures fail,
percutaneous debridement should be considered a safe and effective treatment that allows for early return to activity.
Multiple Non-Traumatic Joint Effusions in a Young Female
Authors: Raman Singh, DO; Brian Donohue, DO; Poonam Thaker, MD
Affiliation: Presence Resurrection Medical Center, Chicago, IL
History: 22 year-old Caucasian female presented with a two-week history of a swollen, painful right knee and one-week
of restricted range of motion of her left elbow. She denied recent injury or new activities, but had started increasing the
intensity of aerobic exercises. Her symptoms were worse one week prior to evaluation and had started to improve with ice
and ibuprofen The knee pain worsened with weight bearing and climbing of stairs She denied knee locking, clicking or
painful snapping Concurrently, she also noted restricted range of motion of her left elbow but denied pain, warmth or
recent injury. She denied fevers, weight loss, rashes, or other musculoskeletal problems She had developed intermittent
abdominal cramping with diarrhea over the last six months but no diagnosis was made. She was an active multisport
athlete in high school and college at the Division 3 level including basketball, soccer, cross-country and golf. She denied
serious injuries but recalled recurrent episodes of bilateral knee swelling without injury since high school that
spontaneously resolved within two weeks. These episodes were mildly painful without associated symptoms and did not
limit her sports participation. PMH: high risk HPV and depression. FH: maternal Crohn's disease. SH: heterosexual
relationship, sexually active and used barrier protection, non-smoker. No history of other sexually transmitted infections.
Physical Exam: BP 110/68; Pulse 80; Temp 97.7°F; Resp 20; Ht 5' 7.5"; Wt 144 lb No distress, normal gait. No
abdominal tenderness or rashes. Elbow: No effusion, ecchymosis, erythema, warmth or tenderness bilaterally. ROM:
Right: -5 to 140° and Left: 10 to 120° without pain. Full pronation and supination Strength 5/5 flexion, extension,
supination, pronation, and pinch grip. No laxity with valgus/varus stress Neg Tinel's sign. Knee: Right with 2+ effusion,
significant warmth and tenderness localized to medial/lateral joint line and posteriorly. No ecchymosis or erythema
bilaterally. ROM: Right: 5 to 130° and Left: -5 to 145° Full flexion on right was painful. Strength 5/5 with hip flexion, knee
extension/flexion bilaterally. Neg Lachman, anterior/posterior drawer, valgus/varus stress bilaterally. Neg patellar
apprehension/compression, McMurray's and Apley's compression test bilaterally.
Differential Diagnosis: Joint infection (septic arthritis, Lyme Disease, N. gonorehea)
arthritis, crystal arthropathy, rheumatoid arthritis, reactive arthritis).
synovial osteochondromatosis, synovial hemangioma).
Arthropathy (enteropathic
Synovial disorder (pigmented vilonodular synovitis,
Test Results: Knee/Elbow radiographs: Normal elbow, knee with suprapatellar effusion CBC: WBC 10.8 (81%
neutrophils), CMP w/Ca+: normal ESR 11, CRP 0.23, Uric acid 4.6, ANA/RF/Celiac Panel negative Gonorrhea/Chlamydia
(NAAT), Lyme Ab IgG/IgM WB, Syphilis panel negative Knee Arthrocentesis: cell count: yellow, cloudy, WBC: 5930, RBC:
&lt;10,000, neutrophils: 55, lymphocytes: 38, mono/macro: 7, GS/culture: few leukocytes, no organisms. No bacterial
growth 4 days Crystal analysis: negative. MRI Elbow/Knee: R knee and L elbow showed large joint effusions without
loose bodies or synovial masses. Normal bone marrow signal, joint space and articular surfaces. No fractures, ligamentous
or tendon abnormalities
Final Diagnosis: Seronegative Inflammatory Arthritis (Early Rheumatoid Arthritis)
Treatment: One week after presentation, she developed significant non-traumatic lumbosacral pain with morning
stiffness. Rheumatology was consulted, patient was seen on week three of symptoms and antiinflammatory/corticosteroids were initiated. Gastroenterology was also consulted for evaluation of inflammatory bowel
disease due to recent abdominal symptoms.
Outcome: Currently pending as case is ongoing.
Follow-Up: Physical therapy for ROM with return to activity once inflammatory symptoms are in remission. Frequent
follow-up for common sports related rheumatic conditions: Calcium and Vitamin D deficiency, increased incidence of
bursitis, tendinopathies, enthesitis, avascular necrosis and stress fractures. Plan for Orthopedic evaluation for synovial
disorders if not responding to anti-rheumatic treatment.
Groin Pain in an Adult Hockey Player
Authors: John Nickless, MD; Kaleigh Suhs, DO; Sangili Chandran, MD
Affiliation: Advocate Christ Medical Center - Department of Family Medicine, Oak Lawn, IL
History: A 40 y/o recreational ice hockey goalie presented to clinic with the complaint of left groin pain. The patient
experienced an acute left groin injury during an ice hockey game while making a kick-save. After the initial injury, the
patient continued to have persistent left groin pain that was intermittently worsened with certain movements. The pain
was only temporarily relieved with use of NSAIDs. The patient was referred to physical therapy, and completed a 10 week
course of PT with some improvement in his pain. However, the pain had not completely resolved which prevented him
from returning to play, and was also limiting him in his career as a hardwood floor installer which required frequent
squatting.
Physical Exam: Gen: AOx3, NAD CV: RRR, +S1S2, femoral pulses palpable B/L, peripheral pulses palpable in BLE,
good peripheral perfusion, no edema Abd: soft, non-tender, non-distended, normoactive bowel sounds, no palpable
masses GU: no testicular masses, no testicular TTP, no hernias or inguinal masses noted MS: full ROM of BLE including
adduction, abduction, & flexion of B/L hips, pain was reproduced when patient adducted left thigh against force Neuro:
5/5 strength in BLE including adduction of B/L thighs, hip flexion, & abduction of BLE; normal gait, sensation to light
touch intact in BLE, normal gait, no focal deficits
Differential Diagnosis: 1) Inguinal Hernia 2) Athletic Pubalgia 3) Avascular Necrosis
Test Results: MRI: tear in left adductor longus tendon
Final Diagnosis: Atheltic pubalgia (sports hernia) involving a tear of the adductor longus tendon.
Treatment: 1) PT was performed with mild improvement in pain 2) Laparoscopic repair with mesh was performed
Outcome: Patient tolerated procedure well. He continues to recover & is expected to be able to make a full return to
work as well as athletics without any restrictions.
Follow-Up: Patient's progress will continued to be followed post-operatively.
Knee to the Face
Authors: Christine Luke, DO; Christopher Jenks, MD; Jonathon Schultz, MD; Margaret Gibson, MD
Affiliation: University of Missouri Kansas City Department of Family Medicine and Community Medicine,
Kansas City, MO
History: A 21 year old Division I collegiate soccer player dove for a ball during practice and was kneed in the right eye by
a teammate also going for the ball. The player that was kneed in the face was down on the field for a short period of time
not moving. He was conscious by the time the ATC arrived at his side on the field. He denied neck pain, but did complain
of dizziness, significant headache and nausea. Upon further evaluation by the ATC, he is noted to have deep lacerations at
superiolateral and inferolateral locations of his orbit that were bleeding a fair amount. The athlete was helped to the
training room where a physician performed a thorough head and neck examination. He was diagnosed with a concussion
and the lacerations about the eye were sutured. The patient was to rest and return to the training room for follow up the
next day. The following day, the athlete was re-evaluated in the training room. SCAT 3 Concussion Symptom Score Sheet
at that time had a score 11. He reported vomiting a few times the night before, headache, and feeling tired. He also said
that he sneezed prior to coming in to the training room and it made the area under his eye swell and hurt really bad.
Physical Exam: Gen: No acute distress HEENT: Pupils equal, round, reactive to light. Extraocular movements intact, no
pain with movement. No proptosis. Right eye with periorbital edema and bruising, repaired laceration superiolateral
eyebrow 3 cm and 2 cm laceration inferolateral; no purulence or drainage. Swelling most prominent in the infraorbital
region and that area was most tender to palpation. Lateral subconjunctival hemorrhage. External Auditory Canals clear
bilaterally. Nares clear without discharge bilaterally. No intraoral trauma noted. No malalignment of teeth. Neuro: CN
II-XII grossly intact. Normal gait and tandem gait. Negative Rhomberg. Normal finger to nose and rapid alternating
movement Poor single leg balance No nystagmus, nor dizziness with vertical or horizontal saccades. Normal optokinetic
testing. MSK: Diffusely tender around the orbit; no notable stepoff or crepitus with palpation. Remainder of Exam within
normal limits
Differential Diagnosis:
Simple "Black Eye"
contusion and bruising of soft tissue
Concussion
Closed orbital fracture
Test Results: SCAT 3: Concussion Score Sheet with score 11
CT Face without contrast: Fracture of right orbital floor
with extension of fat into maxillary sinus with no definite ocular muscle entrapment. Facture of medial wall of right orbit.
Right perioral edema. Small air-fluid level in right maxillary sinus.
Final Diagnosis: Medial and Inferior Orbital Floor Fracture after knee to right eye Concussion
Treatment: OMFS was consulted and it was determined that this was a non-surgical fracture. Conservative management
was recommended. Patient was given 7 day course of Augmentin. He was placed on no contact status. The patient was
also instructed to not blow his nose and try to limit sneezing and vomiting.
Outcome: He remained out of competitive play for the remainder of the season, as there were only a few weeks left in
the season.
Follow-Up: OMFS recommended a 6 week restriction of full contact activity with increasing activity each week after the
first week of rest. He progressed back to full activity without further incident or injury. As for his concussion, he was
progressed through a gradual return to play protocol after being asymptomatic one week after the injury and having
ImPACT test results comparable to baseline testing.
All That Impinges is Not FAI
Authors: Joshua Okon, MD; Marc Harwood, MD
Affiliation: Thomas Jefferson University Health, Philadelphia, PA
History: 19 year old pre-professional ballet dancer presents at the request of her athletic trainer to evaluate right hip
pain. Pain has been present for 2 weeks, and is mild in nature, but affects her ability to dance. Her pain occurs mostly with
hip flexion, occasionally with stretching her hip flexor, and is located mostly in the groin, less so laterally and posteriorly.
She denies back pain, numbness, tingling, or weakness. She has no previous history of hip injuries. She denies irregular
menses and denies disordered eating habits. There is no prior history of stress fracture.
Physical Exam: Height: 5'4" Weight: 116 lbs - BMI:19.9 General: Normal athletic appearing female. CV: 2+ bounding
DP/PT pulses, without edema or varicosities Adenopathy: No gross axillary, cervical, or inguinal adenopathy Spine: Full
range of motion in cervical and lumbar spine. No tenderness to palpation of the osseous structures or paravertebral soft
tissues. There is no muscle spasm. Core strength is adequate by functional movement screen. Neurologic: 5/5 muscle
strength in all extremities. +2/4 deep tendon reflexes and they are symmetric and equal.
Sensation/proprioception/cerebellar function are intact. Gait/station are within normal limits. Babinski is down going, and
Hoffman sign is negative. Hips: Left hip demonstrates excellent range of motion with no pain with internal and external
rotation. Right hip demonstrates pain with internal rotation and she has a positive FADIR test both in the seated and
supine positions. External rotation is greater than expected with no pain reproduced.
Flexibility: Beighton score of 7.
Differential Diagnosis: FAI +/- labral tear, femoral neck stress fracture, ischial femoral impingement syndrome, lumbar
radiculopathy, early onset osteoarthritis, gluteal tendonitis, iliopsoas bursitis.
Test Results: X-Ray: Femoral head under coverage with femoral head extrusion index (FHEI) of 27, acetabular
retroversion. MRI: Severe right ischiofemoral impingement with extensive edema within the quadratus femoris, minimal
right iliopsoas strain and minimal right iliopsoas bursitis.
Final Diagnosis: Ischiofemoral Impingement Syndrome
Treatment: Patient took a course of indomethacin and began a physical therapy regimen and improved. After discussion
with surgical colleagues, the patient was also offered an ultrasound guided injection into the quadratus femoris muscle. By
the time of her ultrasound appointment, however, she had improved to the point that she declined the injection.
Outcome: Patient demonstrated excellent improvement with NSAID therapy, physical therapy directed at optimization of
hip mechanics and correction of biomechanical/technical faults.
Follow-Up: Patient began to return to full dancing activities at around 1 month. At 3 month follow up she was still
having some mild discomfort but was able to participate fully in dance. At 6 months after initial evaluation, she reported
no pain in the hip and was dancing pain free while in a home exercise regimen.
Left Leg Pain in an Elite Athlete
Authors: Jeffrey Ijadi, MD
Affiliation: Evergreen Sports Medicine Fellowship Program, Augusta, ME
History: HPI: 51 year old male triathlete with complaint of left leg pain and weakness for 18 years Patient claims
symptoms first began in 1996 during a group bike ride, when his left leg began to cramp, fatigue, and "give out" in his
final few miles. Pain is described as spasming, numbness, squeezing, tightness in left groin, radiating to left calf and foot.
Sensation is worse with endurance activities, increased intensity, and uphill activities, improves with rest, adjustment the
seat position higher during cycling, decreased intensity, and level or downhill activities In 2002, patient stopped
competitive training secondary to bilateral hip severe osteoarthritis Later that year, the patient underwent bilateral hip
replacements with good results of hip pain and above mentioned symptoms In 2003, patient underwent formal physical
therapy and by 2004 patient returned to competition. Patient continued to compete in triathlons and cycling events
through 2014, but left groin and leg symptoms returned with deterioration of performance. After finishing a Half Ironman, patient was unable to simple walk up stairs or hills without symptoms described above. PMH: Bilateral Hip
Osteoarthritis, Insomnia, Paroxysmal atrial flutter Past surgical history: Appendectomy at 15 yrs old. TEE Cardioversion on
7/30/24 & 8/20/14 Meds: none Family History: Father= Rhabdomyosarcoma, Basal cell Ca, HTN, hyperlipidemia
Mom=Basal cell Ca, hyperlipidemia. Social History: Married for 25 years, no children, works as a physical and athletic
trainer, elite athlete, non-smoker, no alcohol use, no IVD use, no caffeine. ROS: CV: Intermittent chest palpitations MSK: +
Left leg pain, sensation of “giving out&rdquo;, weakness, numbness, spasms, squeezing Otherwise negative
Physical Exam: Physical: BP: 121/85, P:47, regular, R: 16, O2Sat: 100% on RA, Weight: 168, BMI:23
Gen: NAD, HEENT:
NC/AT, EOMI, PERRLA Neuro: CN 3-12 intact, normal reflexes, coordination, muscle tone, No focal findings, Lungs CTA
b/l, CV: RRR, No murmurs ABD: Soft, ND/NT, Vascular: Dorsalis pedis & Post tibial pulses 2/3, no edema Skin: warm
and dry MSK: Strength 5/5 in all muscle groups Psych: AAOX3, normal attention span and concentration.
Differential Diagnosis: Rhabdomyolysis, Muscle cramps, Peripheral neuropathy, External Iliac Arteriopathy, Neurogenic
Claudication, Spinal stenosis, Sciatica, Herniated disk, Chronic exertional compartment syndrome, PAD, Osteoarthritis,
Tendonitis, DVT, Varicose veins.
Test Results: Left ABI:0.68 normal 1.0-1.4, borderline 0.91-0.99, abnormal 0.9 or lower.
TEE: Normal Left Ventricular EF
55%, Mild bilateral enlargement. CT Coronary Calcium score: Coronary calcium score equals 64. 0= No identifiable
atherosclerotic plaque, 1-10=Minimal Identifiable plaque, 11-100=Definite but mild plaque, 101-400 Definite moderate
plaque. CT Angio Aorta Run off w/wo contrast: Complete occlusion of the Left external iliac artery with reconstitution of
the common femoral artery.
Final Diagnosis: Left External Iliac Arteriopathy/Endofibrosis
Treatment: Left inguinal ligament release, Dacron patch angioplasty, with endarterectomy.
Outcome: Patient tolerated the procedure well, currently two weeks s/p surgery Patient is able to walk five miles,
incline without symptoms. Patient tolerated the procedure well, currently two weeks s/p surgery Patient is able to walk
five miles without symptoms.
Follow-Up: Patient plans to begin elliptical exercises at three week post op and return to running exercises at one month
post op Patient is determined to participate in half-Ironman in 2015.
Another Terrible Knee?
Authors: Sultan M. Babar, MD; Justin A. Lee, MD
Affiliation: East Carolina University, Greenville, NC
History: 17 year old high school football player with left knee pain. He went to block a kick and another player landed on
his knee, causing a posterior-lateral force on the knee displacing the knee medially. He felt a pop and could not bear
weight immediately. On field evaluation revealed a patellar subluxation that was successfully reduced. He did not return to
the game. He had difficulty bearing weight and was given crutches and placed in knee immobilizer He was seen in clinic 5
days later for further evaluation.
Physical Exam: xam was extremely guarded and limited by pain. ROM: 0 to 90° limited by pain. 1+ Effusion.
Tenderness to palpation was noted medially over MCL, with abundant soft tissue swelling along proximal medial tibial
plateau. Gross laxity was noted on valgus stress test. Patellar apprehension and patellar grind were positive. No laxity
noted on varus stress test Anterior and posterior drawer along with McMurray's tests could not be performed secondary
to pain and severe guarding.
Differential Diagnosis:
Patellar subluxation/dislocation and associated bony contusion
MCL tear
ACL tear
PCL tear
Patellar tendon tear
Fracture of the tibial plateau
Osteochondral fracture
Medial meniscus tear
Test Results: X-ray: Mild soft tissue swelling without underlying fracture or effusion. MRI: 1. Increased signal within
the superior fibers of the PCL, suggesting a high grade sprain/partial tear. 2. Complete tear of the MCL with associated
hematoma. 3. A very small (0.2 cm) partial tear of the superior aspect of the patellar tendon. Small joint effusion with
moderate to severe diffuse subcutaneous edema. 4. Intact medial patellar retinaculum.
Final Diagnosis: Complete MCL tear, high grade strain/partial PCL tear, and patellar subluxation
Treatment: Knee effusion drained. He was placed in knee immobilizer for 3 weeks. Following this, he was placed in a
functional hinged knee brace and referred for formal physical therapy for 3 weeks.
Outcome: About 6 weeks after the injury, athlete was doing well with no pain and full range of motion.
Follow-Up: He was then cleared to do functional return to play He played in game approximately 8 weeks after initial
injury and scored two touchdowns!
Currently participating in the playoffs with no limitations.
Basketball and the Never Ending Leg Swelling
Authors: Megan Bailey, MD
Affiliation: Oregon Health and Science University, Portland, OR
History: A 20-year-old female presents to sports clinic with pain and swelling in the left lower extremity that prevents her
from exercising Her initial injury happened 7 years prior playing high school basketball, when she felt a pop and left knee
pain during the game. Two days after the injury, she started having left lower extremity and knee swelling She did not
initially seek medical care at the time because she thought she sprained her knee and treated with RICE. She noticed that
the lower extremity edema persisted after the knee pain improved Her pain waxed and waned, but started to increase as
her edema increased She was seen about 1 year later in urgent care and had x-rays that were negative for a fracture. The
chronic and persistent swelling and pain prevents her from exercising. She uses a compression sock and elevation to help
with the edema and pain, which initially was helping, but recently has been less effective The swelling of her lower
extremity started to extend to her thigh by the time she presented to clinic. No other specific medical problems. ROS:
Negative for night pain, fevers, weight loss, fatigue, night sweats, or joint pain. PMH: seasonal allergies SH: Lives with
her family, No alcohol, tobacco, or drug use. FM: No one in her family with edema, gout, autoimmune disease, renal
disease, or cancer. Meds: None
Physical Exam: Vitals: BP 102/60, HR 62, RR 12 Gen: Pleasant, well developed, appears stated age, communicates
appropriately CV: RRR w/o murmur, No gallops Lungs: CTAB, no crackles, no wheezing, normal respiratory effort Abd:
Soft, NT/ND, Normal active BS, No HSM Left knee: +pain to palpation of the distal femur, FROM of the left knee, 5/5
strength flexion/extension Dorsalis pedis pulse intact. Neuro: sensation intact except as follows: decreased over the
dorsal aspect of her left foot, left big toe and medial side of the foot Leg circumference measurements: Right foot 3 cm
proximal to the MCP is 20.5 cm girth; left foot 3 cm proximal to the MCP is 22 cm girth. Right calf 3 cm proximal to the
med. malleolus is 20.5 cm girth; left calf 3 cm proximal to the med. malleolus is 24 cm girth. Right calf 20 cm proximal to
the med. malleolus is 31 cm girth; left calf 20 cm proximal to the med. malleolus is 34.5 cm girth. Right calf 10 cm below
knee is 33 cm girth; left calf 10 cm below knee is 35 cm girth. Right thigh 10cm above the knee is 40.5 cm girth; left
thigh 10cm above the knee is 42 cm girth.
Differential Diagnosis: DVT, genetic syndromes (Noonans or McCune Albrights), obstruction, cancer, venous
insufficiency, trauma, infection, primary lymphedema
Test Results: Nuclear Medicine Lymphatics Drainage Mapping: Left lower extremity visually more edematous than the
right Transit in a young individual is expected to be seen along the lymphatic tracts of the iliac chain and
retroperitoneum, and is not seen in this individual.
Final Diagnosis: Lymphedema Precox
Treatment: Referral to the pediatric dermatology vascular anomalies clinic.
sleeve, elevation, decongestive physiotherapy.
Conservative management- compression
Surgical option: Low level laser therapy
Outcome: Lymphedema precox is a primary lymphedema disease, meaning a congenital defect of the lymphatic system.
It accounts for about 75-95% of all cases of primary lymphedema It usually has an onset around puberty, and is 10 times
more likely in females than males. It is usually localized to the foot and calf in a unilateral extremity There is speculation
that estrogen plays a role in the pathogenesis because of the presentation being most common in women going through
puberty.
Follow-Up: Gradual return to activity.
Too Much Pain, No Gain: Chest Pain in a Cross Fit Athlete
Authors: Rony T. Skaria, MD; Kevin Crawford, MD; David S. Edwards, MD
Affiliation: Sports Medicine Fellowship, Texas Tech University Health Sciences Center - Lubbock, TX
History: A 20 year old woman presented with a four month history of left sided chest pain. She is an experienced cross-fit
athlete who trained six days a week for one hour sessions. The pain initially started with overhead workouts, beginning
with soreness over 2-3 weeks and then with a sudden sharp pain experienced during an overhead workout. She was
subsequently unable to do dips, pushups, pull-ups or rows. Pain was exacerbated by activities of daily living such as
reaching overhead, driving, and by deep inspiration. NSAIDS mildly relieved pain. Her pain gradually improved with rest
over two months, and she restarted upper body workouts; however, this exacerbated her pain, so she decided to seek
medical treatment. She denied systemic symptoms, prior fractures or malignancies. She maintained a balanced diet, had
almost daily direct sunlight exposure and took 1200mg calcium and 400 international units of vitamin D daily. Past
medical history was significant for oligomenorrhea, nine cycles in past year. No prior operations. She did not smoke and
had no family history of osteopenia.
Physical Exam: Height: 5’4; Weight: 150 lbs BMI 25.7 RR 12 General: muscular build HEENT, Abdominal,
Cardiovascular : within normal limits Neck: supple, FROM c-spine, no LAD RESP: normal chest wall excursion with equal
chest movement bilaterally without accessory muscle use; pain on left chest wall with deep inspiration Musculoskeletal:
bony tenderness to palpation at mid axillary line at the left 5th rib, worsened with anterior to posterior compression of rib
cage; no palpable mass or hematoma, no bony crepitus; no winging of scapula with wall pushup; pain worsened with left
lateral flexion and rotation of trunk Left shoulder full , pain free range of motion Neuro: sensation intact C5-T1
dermatomes bilaterally Skin: no rashes or vesicles
Differential Diagnosis:
Costal Stress Fracture
Pleurisy
Intercostal muscle strain
Costochondritis
Serratus Anterior/External Oblique Strain
Test Results: AP Chest: Evidence of anterior-lateral 5th rib fracture with some callous formation Shoulder axillary view:
Evidence of left 5th Anterior-lateral rib fracture, with callous formation No other bony abnormalities
Final Diagnosis: Rib Stress Fracture
Treatment: The patient was prescribed a regimen of relative rest, ice and heat for the pain. Home exercise program
including ROM exercises and gentle stretching, strengthening focused on the serratus anterior and external oblique
complex were shown. We educated the athlete on risk factors predisposing to injury; total energy balance and the
importance adequate caloric intake in athletes, especially in the presence of oligomenorrhea. She agreed to decrease
upper body activity, albeit reluctantly.
Outcome: On four week follow up, there were significant improvement of symptoms.
Follow-Up: We recommended no return to upper body weight training until she had pain free range of motion, then
gradual increase in training.
Traumatic Thoracic Disc Herniation in a Pediatric Patient
Authors: Amanda Farrell, DO; Charles Mehlman, DO; Emily Dixon, DO
Affiliation: TriHealth, Cincinnati, OH
History: A previously healthy 13 year old Caucasian male sustained a witnessed injury during a high school football
game. Patient and opponent collided facemask-to-facemask and subsequently fell backwards. No head injury or LOC
occurred. He immediately complained of pain involving the mid and low back. He described the pain as dull and achy
without radiation into the buttocks or lower extremities. Denied paresthesias, nausea/vomiting, weakness, saddle
anesthesias, bowel/bladder incontinence or any prodromal symptoms including fever, night sweats, weight loss, or
malaise. No previous history of back injuries. Patient was sent to the emergency department for further evaluation.
Physical Exam: BP: 137/70; HR: 98; Temp: 98.2°F; Resp: 18; Weight: 72.05 kg Gen: alert, well-developed, wellnourished, NAD HEENT: NC/AT, MMM Lungs: CTAB, no respiratory distress CV: RRR, no murmurs, 2+ pulses
peripherally, brisk cap refill Abdomen: soft, NT/ND Spine/Musculoskeletal: No evidence of deformity, ecchymosis, or
swelling. Normal ROM with side bending and rotation, however limited flexion/extension of T-spine secondary to pain.
Tenderness noted over mid-thoracic spine. Sensation of upper and lower extremities intact and equal. Strength 5/5 in all
muscle groups of upper and lower extremities. Negative SLR. Normal muscle tone. 2+ patellar and achilles DTRs bilaterally.
Neuro: Normal balance and coordination. CN II-XII intact
Differential Diagnosis:
1.
2.
3.
4.
5.
6.
7.
Disc herniation
Compression fracture
Spondylolysis
Spondylolisthesis
Thoracic/lumbar paraspinal muscle strain
Thoracic spine contusion
Slipped vertebral apophysis
Test Results:
1.
2.
Thoracic spine 2 view x-ray - Normal radiographic examination of thoracic spine. No fractures. Normal disc spaces.
Normal alignment.
MRI thoracic spine without contrast - Findings suggesting an acute broad-based disc herniation at the T11/T12
level with complete effacement of the ventral sac and subtle deformity of the ventral cord. The cord remains
normal in signal. There is associated subtle signal in the adjacent T12 vertebra suggesting marrow edema/injury.
Subtle compression fractures involving the T4 and T5 vertebra with likely marrow edema at the T2, T3, and T6
vertebral levels.
Final Diagnosis: Posttraumatic T11-T12 broad-based disc herniation and compression fractures involving T4 and T5.
Treatment: There was no indication for surgical intervention. Athlete was placed in Jewitt back brace for 6 weeks with
activity restriction.
Outcome: At a 6-week follow up visit, patient had no tenderness along TL spine and neurologic exam was entirely
normal. He was then enrolled in PT and permitted to walk/run without the use of the Jewitt brace. A 10-week post-injury
MRI demonstrated stable appearance of T11/T12 disc herniation and resolution of the compression fractures. Back pain
significantly improved.
Follow-Up: One-year follow up MRI again showed stable appearance of T11/T12 disc herniation with similar degree of
impingement on the spinal canal as initially seen. Athlete remained completely asymptomatic. He was referred to
Neurosurgery for recommendations regarding return to play. Decision was made to allow participation in non-contact
sports only. Patient will follow up in 1 year.
I Think I Tore Something: The Pains of CrossFit
Authors: Brittney M. Richardson, MD; Jennifer Daily, MD; Jessica Stumbo, MD; Rebecca Popham, MD
Affiliation: Primary Care Sports Medicine Fellowship, University of Louisville and KentuckyOne Health
History: 40 year old right hand dominant male, who avidly participates in CrossFit, presents to clinic with complaint of
right arm pain and weakness with a suspicion for a possible biceps tear. According to him, two days prior to presentation,
while actively participating in CrossFit training, he began lifting an 800 pound tire as he had many times before. However
this time, when he lifted up on the tire, he felt the sensation of a tear and immediate pain in his right arm. Since then he
has had swelling and pain in the arm and is unable to lift any weight. He also has a hard time lifting his arm against
gravity. He denies previous injury to arm. He has had no previous surgeries. He denies numbness or tingling in his arm
and has noticed no color change or loss of sensation. Past Medical History: Left ankle osteoarthritis, Right lateral
meniscal tear, Tonsillectomy. Medications: None. Family History: Non-contributory.
Physical Exam: General: No acute distress, well appearing and well nourished. Cardiology: Normal rate and rhythm,
normal S1 and S2, no murmurs. 2+ radial and pedal pulses bilaterally. Normal capillary refill of the fingers. No edema or
varicosities noted. Pulmonary: No increased work of breathing or signs of respiratory distress. Clear to auscultation.
Abdomen: Bowel sounds present. Nontender, nondistended with no masses. No hepatomegaly or splenomegaly.
Musculoskeletal: No clubbing or cyanosis. Upper extremity compartments normal. Unable to supinate right arm against
resistance (+Yergason’s test). Palpable defect of distal biceps tendon on right. Visual difference of contour of muscle on
flexion of biceps. Neurovascularly intact. Skin: No rashes or lesions. Warm and dry with normal turgor.
Differential Diagnosis:
Complete distal biceps tendon tear
Partial biceps tendon rupture
Cubital bursitis
Bicipital tendinosis
Entrapment of lateral antebrachial
cutaneous nerve
Elbow dislocation
Radial head fracture
Test Results: MRI Right Elbow without Contrast: Full-thickness tear from distal attachment of biceps tendon with 6.5 cm
proximal retraction of the tendon and mild atrophy of the muscle belly.
Final Diagnosis: Complete Distal Biceps Tendon Tear
Treatment: After getting the report of the MRI, he was called and the injury was explained to him. He was given strict
instructions to restrain from any heavy lifting, pushing, or pulling using his right upper extremity and was encouraged to
continue rest, ice, compression, and anti-inflammatory use. He was then referred to orthopedics for discussion of
nonsurgical versus surgical options for treatment.
Outcome: He is currently awaiting surgery.
Follow-Up: He will be monitored closely after surgery by his orthopedic surgeon, with the expectation of a 8-12 week
recovery. After surgery, patients with similar injuries are generally placed in a hinged elbow brace for immobilization with
active range of motion (AROM) beginning in physical therapy during week 1 and passive range of motion (PROM)
beginning during week 2.
Paraplegic Motorcross Racer with Bilateral Forearm and Hand Pain
Authors: Tobias Gopon, MD; David Petron, MD; Donald Coleman, MD; Stuart Willick, MD; Frank Jackson, DO
Affiliation: University of Utah; Salt Lake City, UT
History: 27 year old T7 complete paraplegic male presented to our institution with 3 years of cramping, weakness, and
swelling of bilateral forearms and hands with motorcross racing. These symptoms were present for a year prior to a
motorcross injury in which he sustained a T7 complete spinal cord injury. He described his pain as burning, 10/10 in
severity, and located bilaterally over the volar aspect of his forearms and hands. He reported bilateral hand weakness only
when he was also experiencing pain and swelling. These symptoms were not associated with numbness, tingling, or
radiating pain from his neck or proximal arm. The symptoms would resolve within 20 minutes of cessation of the
offending activity. The symptoms persisted after the spinal cord injury, as he continued motorcross racing, and were
present to a lesser degree with weight lifting and typing on his computer. His past surgical history included bilateral carpal
tunnel surgery a year prior and a T4-T10 posterior arthrodesis at the time of his spinal cord injury. His past medical history
included hypothyroidism, neurogenic bladder, and chronic neuropathic pain. His medications included: Levothyroxine,
Oxycontin, Adderall, Clonazepam, Oxybutynin, Gabapentin, and Baclofen.
Physical Exam: General: well appearing athletic male in wheelchair. Alert and oriented x 3. MSK: ROM full and without
pain in neck, shoulders, elbows, wrists, or hands. Inspection of bilateral upper limbs showed no abnormalities. At rest,
there was no muscular tenderness in bilateral forearms or hands, and the muscles were not tense. Tinel's sign over the
median and ulnar nerves, Phalen's test, and Spurling's maneuver were negative bilaterally. Capillary refill was normal in all
digits. Neuro: Sensation to light touch in C3 through T2 dermatomes was intact. 5/5 strength in bilateral upper
extremities. 2+ biceps, triceps, and brachioradialis reflexes. Strength and sensation were absent below T7. Skin: No color
or temperature changes were noted in bilateral upper extremities.
Differential Diagnosis: Carpal tunnel syndrome, cubital tunnel syndrome, chronic exertional compartment syndrome
(CECS), referred cervical pathology, chronic tenosynovitis, chronic pain, neuropathic pain, medication side effect, stress
fracture.
Test Results: Compartment pressures of the left superficial dorsal and volar forearm compartments were obtained preand post- exercise. The resulting pressures in mm Hg were as follows: a) Superficial dorsal: pre 18; post 35 b) Volar: pre 20;
post 28. No labs were obtained. No imaging studies were obtained. No electrophysiological studies (i.e. EMG) were
obtained.
Final Diagnosis: Chronic exertional compartment syndrome (CECS)
Treatment: Two weeks after his initial evaluation by a primary care sports medicine provider at our institution, a left volar
forearm fasciotomy was performed. The fascia of both the superficial and deep compartments of the forearm flexor
muscles were released bilaterally via a volar ulnar incision. The fascia of the flexor digitorum superficialis, flexor carpi
ulnaris, and lacertus fibrosis were released in the superficial compartment. In the deep compartment, the fascia of the
flexor pollicis longus and flexor digitorum profundus muscles were released.
Outcome: Following surgery, he was not allowed to perform any upper extremity strength training. One month
postoperatively, he reported having no left sided symptoms and was allowed to return to a gradually progressive upper
extremity strength-training program. At his two month follow up visit, he reported being symptom free in his left forearm
and hand with all activities. At that time, he was released to full activity and motorcross racing. This same process was
repeated one month later for his right forearm with similar results.
Follow-Up: At this time, the patient is approximately one year removed from his initial fasciotomy. He continues to be
symptom free and able to race motorcross as well as lift weights and perform activities of daily living without pain,
swelling, or cramping.
Zero (Exercise) to Football Hero - Not the Best Idea
Authors: Monica Ball-Zondervan, MD; Abigail S. Fletcher, MD
Affiliation: Long Beach Memorial Family Medicine, Long Beach, CA
History: A 16 year old male football player (5'10", 276lbs, BMI 39.6, &gt;99%tile for age) was participating in stretching
exercises and a half mile jog during the first summer practice for his high school JV team when he became confused and
passed out. On regaining consciousness, per trainer account, he was confused and unable to move his arms or legs due to
weakness. He was carried to a shaded area, nonessential clothing was removed by his trainers and EMS was called.
Trainers initiated cooling measures, and EMS brought patient to the ER. Of note, this was his first year of playing football
and he admits his activity over the summer were relatively sedentary.
Physical Exam: On arrival to the ED, he was noted to be altered and agitated. He was febrile to 103°F rectally and he was
given Tylenol suppository. He was noted to be obese and have dry mucous membranes but his exam was otherwise
unremarkable He was given 3L IVF bolus, IV antibiotics for suspected meningitis, and a foley was placed, with finding of
gross hematuria. The athlete was transferred to the Children's hospital for higher level of care.
Differential Diagnosis:
Heat stroke
Rhabdomyolysis
Sepsis
Toxin ingestion
Dehydration
Test Results: Initial labs showed the following: Leukocytosis to 21,000 with 81% neutrophils, otherwise normal blood
count. Creatinine of 2.7 with a BUN of 19, basic chemistry panel was otherwise normal. CKMB was initially 11.5, but
uptrended to 13.6 during ED stay. Troponin 1.67 initially, uptrended to 2.67 CK 703 Lactate 1.5 Utox negative EtOH
negative EKG: ST Elevations consistent with early repolarization. Cardiac ECHO: normal
Final Diagnosis:
Exertional heat stroke
AKI/rhabdomyolysis
troponemia due to AKI
Treatment: By arrival to the Children’s hospital, his temperature was 98.5F and his mental status had returned to
baseline. He was admitted to the pediatric ICU, where he was aggressively hydrated and his cardiac and neurologic status
was closely monitored. Chemistries and cardiac enzymes were followed. Troponin and creatinine down trended with
intravenous hydration, returning to normal range by hospital day 4. CPK peaked at 10,260 on hospital day 2, and down
trended thereafter.
Outcome: The athlete was back to his baseline mental status and his weakness had resolved by time of discharge.
Follow-Up: The young man was recommended not to play until undergoing exercise stress testing and cardiac
evaluation two weeks after discharge. However, he was lost to follow up. He has not returned to football this season.
Right Arm Swelling After Wakeboarding
Authors: James D. King II, MD; James DiStefano, DO
Affiliation: Texas A&amp;M Sports Medicine Fellowship
History: Patient is a 25 year old male who presented initially to an urgent care facility for evaluation of right upper arm
pain. Stated it occurred over the summer when he was wakeboarding and the line became taught and pulled his right arm
excessively. He states he felt a "pop" on the inner aspect of his bicep. Patient had some mild swelling in the right upper
extremity, with no extension into his hands. He was also point tender at the medial muscle belly of the biceps. Patient
was advised to take ibuprofen, try ice therapy, and follow up with Sport's Medicine for further evaluation in the next week.
Patient did not follow up with Sport's Medicine until about three months later with continued symptoms of right upper
extremity swelling, worse after working out his upper extremities.
Physical Exam: Gen: Alert, no acute distress CV: Normal HR and rhythm Pulses: 2+ radial bilaterally with arms down,
0 on right with arm abducted overhead/2+ on left with arm abducted overhead. Paresthesias developed on right upper
extremity &lt;1 min with arm abduction Resp: Normal respiratory effort/pattern Neuro: Normal sensation bilateral upper
extremities Musc: FROM, 5/5 strength with upper extremity flexion, extension, abduction, internal and external rotation.
Tender to palpation in the medial right upper extremity along the biceps muscle at initial presentation, but non-tender at
follow up. Right arm from shoulder to hand is mildly edematous as compared to left.
Differential Diagnosis:
Primary Upper Extremity DVT
Thoracic Outlet Syndrome
Exertional Compartment Syndrome
Arterial Occlusion
Lymphedema
Test Results: Initial xray showed no acute fracture, mass, or dislocation
CT-Angiogram - There was narrowing of the
thoracic outlet on the right with arms up when the distance between the clavicle and the 1st rib are measured, 7 mm on
the right as opposed to 1.7 cm on the left. No definite arterial stenosis or compression is seen, but there may be
significant subclavian vein compression with the arms up. Catheter venogram study needed to further evaluate
compression of subclavian vein.
Final Diagnosis: Thoracic Outlet Syndrome
Treatment: Patient sent for appointment with CT Surgery
Outcome: After meeting with CT Surgery and discussing conservative versus surgical options, the patient decided to start
with physical therapy for Vascular Thoracic Outlet Syndrome. Precautions regarding the possibility of clot formation were
relayed to the patient.
Follow-Up: Patient was allowed to return to full activity with the only change being for him to modify/decreased his
upper extremity weight lifting exercises until resolution, then he may slowly resume full activities without restriction.
Football Player That Can't See
Authors: Frederick Soliman, DO, MS; Jeanne Doperak, DO
Affiliation: UPMC Primary care sports medicine fellowship, Pittsburgh, PA
History: 21 year old division III football player, during kick return sustained a injury to his left eye Athlete reported while
blocking, opponents’ hand enter facemask and scraped his left eye Immediately, he had left eye pain and was unable to
open eye. Athlete reported to team physician left eye pain, blurry vision, along with difficulty making out objects. He
denied any flashing lights, curtain sensation over eye, photophobia, or foreign body sensation in eye On initial exam by
team physician sanguis fluid appreciated in anterior chamber of left eye encompassing about 5% of iris Eye was patched
and player was taking to dressing room for further evaluation. On re-exam after removal of contact lens, sanguis fluid in
anterior chamber increased to 15% of iris. Athlete noted taking Ibuprofen before game.
Physical Exam: Vitals: Afebrile, Hemodynamically stable General: AAOx3, Uncomfortable in appearance HEENT: left
blepharospam, no foreign body identified with eversion of eye lids, sclera injection, anterior chamber with 15% of sanguis
fluid encompassing iris, EOMI intact, pupil not equal left 3 mm, right 2 mm, left pupil sluggish to react to light, visual fields
3 out of 4 intact, visual acuity unable to be assess Skin: no edema or laceration over eye lid or margins MSK: no
deformity of orbit, non-tender to palpation over left zygomatic arch, nasal sputum and frontal lobe
Differential Diagnosis:
1.
2.
3.
4.
5.
6.
7.
8.
9.
10.
Hyphema(traumatic)
Corneal abrasion
Orbital fracture
Traumatic iritis
Dislocated lens
Eye lid laceration
Corneal laceration
Retinal trauma
Foreign body
Blepharospasm
Test Results: Slit lamp exam: no fluorescein dye uptake; Ocular pressures performed by Tono-pen: 27 mm hg
Final Diagnosis: Traumatic Hyphema encompassing 40% of anterior chamber
Treatment: Patient sent to ED for urgent Ophthalmology consult. Medical management with Mydriatic agents, Ocular
steroids, oral steroids along with bed rest for 1 week Daily eye pressures check for next 2-4 days.
Outcome: Hyphema with almost complete resolution 24 hours after injury Ocular pressure returned to baseline with
subsequent exams over the next 24-48 hours No re-occurrence of bleeding. Visual acuity along with visual fields return
to baseline.
Follow-Up: Returned to play in 2 weeks
No Pain, No Gain...Except Chest Pain
Authors: B. Derek Worley, MD; Nina Kukar, MD; Atul Kukar, DO
Affiliation: Lenox Hill Hospital, New York, NY
History: A 34 year old, non-smoker, nulliparous female with no significant past medical history presents to the
emergency room with acute onset of chest pain approximately 30 minutes prior to presentation while dead lifting 200
pounds at a local gym. She had been weight training for the past 6 months in preparation for a competition and had
deadlifted as much as 230 pounds without any adverse effects. The patient described the pain as left sided heaviness, 9/10
intensity, constant, with radiation to the neck and back. Her family history was positive for a myocardial infarction in her
paternal grandfather at the age of 54. She is an exercise enthusiast whose routine consisted of yoga, cardiovascular
training (cycling and swimming), and weightlifting 5-6 days per week for the past several years. She reports taking creatine
daily for several years, as well as vitamins A, B, and D for the past year. She denies smoking, alcohol, or illicit drug use. At
the time of presentation she had a paragard intrauterine device.
Physical Exam: In the emergency department the patient was normotensive with a heart rate of 70. Her physical exam
revealed a well nourished, physically fit young woman, with no abnormal exam findings. The patient weighed 62.6 kg (138
lb).
Differential Diagnosis: Aortic Dissection, Myocardial Infarction, Esophageal Rupture, Pulmonary Embolus, Spontaneous
Pneumomediastinum
Test Results: Initial EKG revealed ST elevations in I and aVL with ST depressions in III and aVF. The initial troponin I,
drawn in the emergency room, was 0.063 ng/ml with a peak troponin of 20.7 ng/ml The initial creatine kinase was 268
U/L, with a peak of 648 U/L Other pertinent negative labs include antinuclear antibody, myeloperoxidase antibody,
proteinase 3 antibody, erythrocyte sedimentation rate, C reactive protein, and quantitative beta HCG ;
Final Diagnosis: Spontaneous Coronary Artery Dissection
Treatment: In the ED, the patient received Aspirin 325mg, Prasugrel 60mg, and Atorvastatin 80mg. She was emergently
taken to the cardiac catheterization lab with a diagnosis of a STEMI. She was found to have a dissection at the proximal
part of the first diagonal (D1) vessel extending distally with small amounts of thrombus noted and TIMI 3 flow. There was
no disease noted in the remainder of the vessels. The patient was treated with aggressive medical therapy, including two
integrillin boluses followed by a drip and a heparin bolus with drip. At the end of the procedure there was a decrease in
the thrombus burden, the chest pain had resolved, and the ST elevations had returned to baseline.
Outcome: An echocardiogram, performed post angiogram, revealed an overall normal ejection fraction with apical
hypokinesis and no significant valvular heart disease Post procedure the patient was monitored in the CCU for 48 hours
with no complications or recurrence of pain. The patient was maintained on an Integrilin drip for 24 hours and a heparin
drip for 48 hours. She was discharged home with Aspirin, Prasugrel, and Atorvastatin.
Follow-Up: She returned to the office four weeks later with no new complaints or physical exam findings. The patient
was cleared to resume her previous workout regimen, minus the deadlifting.
Painful Plie - Case of a 22 Y.O. Dancer with Hip Pain
Authors: Suraj Gathani, MD; Rebecca Popham, DO; S. Grossfeld, MD; Jessica Stumbo, MD; D. Rhoads, MD
Affiliation: University of Louisville Dept. Family and Sports Medicine, Louisville, KY
History: Patient is a 22 y.o female who presents with a five-year history of bilateral antero-lateral hip pain. She teaches
ballet and has aspirations to dance professionally. She is a competitive soccer player when she was younger, but denied
any previous injuries related to that sport. She states that the hip pain is located anteriorly, also somewhat lateral She
notes the pain to be sharp, non-radiating, and worse when doing a grand pli&eacute;. Current treatments have included
chiropractic treatment, and physical therapy. She is currently taking NSAIDs.
Physical Exam: General: Caucasian female, 5'5", 120 lbs. Alert and in no acute distress. Pleasant affect and well
groomed.
HEENT: Unremarkable
Respiratory: CTA B/L, No crackles, No wheeze. No use of accessory muscles are
present.
Cardiovascular: +s1s2, RRR, no signs of vascular congestion.
Neurologic: Reflexes and sensation normal, no
evidence of muscles spasticity.
Neuropsych: Mental status exam reveals that the patient is able to articulate well with
normal ; speech/language, rate, volume and coherence, and demonstrates appropriate judgment and insight Mood and
affect is normal.
Bilateral Hip Exam: External rotation -90°; internal rotation 60° + impingement test Normal turnout of
her hips and patellofemoral joint when standing in first position. No pain to palpation of greater trochanteric region of the
hips. No catching or crepitus with range of motion of hips. No pain with force flexion of the hip while stressing her hip
flexors. Pain when performing grande Plié Negative FABER test.
Differential Diagnosis: Bursitis, Arthritis, Osteonecrosis, Labral tears, Femoral stress fractures, Pelvic stress fractures,
Snapping hip syndrome, Inflammatory disorders, Athletic pubalgia, femoral acetabular impingement.
Test Results: AP Pelvis and Lateral of both Right Hip and Left Hip Radiographs: Reveals femoral acetabular dysplasia
with a pincer-effect involving the acetabulum and the cam-effect involving both femurs, left being greater than right. No
signs of collapse of the femoral heads, no sign of osteolysis or degenerative changes involving the pubic symphysis.
Shallow acetabulum present on both sides. Center of femoral head sits outside of the acetabulum.
Left Hip Arthrogram
MRI: There is a fairly extensive tear of the left acetabular labrum and there is a some impingement morphology at the left
hip.
Right Hip Arthorgram MRI: 1) There is a fairly extensive tear of the right acetabular labrum and some mild
impingement morphology 2) Mild edema is observed in the musculature posterior to the hips bilaterally, predominantly
the quadratus femoris, consistent with muscle strain.
Final Diagnosis: 1) Bilateral femoral acetabular impingement with full thickness articular cartilage loss of the anterior
superior head at the site of the impingement.
2) Bilateral acetabular labral tears.
Treatment: More aggressive interventions such as fluoroscopic guided cortisone injection, arthroscopic osteoectomy,
and labral debridement verses repair were discussed with patient. However, patient chose a more conservative option of
rest from ballet and NSAIDs.
Outcome: Responding well to conservative treatment consisiting of rest from ballet and NSAIDs. Indication for surgery
would be for pain and not to attempt to improve range of motion for ballet. Patient counseled about possibility of
osteoarthritis occurring at an earlier age than its typical age.
Follow-Up: Patient currently "taking a break from ballet". Ballet career is very limited at this point. Patient is to follow up
as needed or if patient chooses to have surgery.
Transient Pain, Paresthesia and Pallor in Collegiate Male Swimmer
Authors: Tad DeWald, MD, MPH; Geraldine Dapul, MD; Paul Withers, MD; Aashish Deshpande, MD; Parmod
Mukhi, MD
Affiliation: Wayne State University
History: A right-hand-dominant 23-year-old male swimmer presented with persistent right shoulder pain, numbness,
pallor, and paresthesia with radiation down the medial aspect of the right upper extremity Patient stated that symptoms
started gradually over a week and would only present during distances over 1,000 meters Symptoms resolved when the
athlete ceased activity.
Physical Exam: 5/5 strength throughout right upper extremity, equal to left Full active range of motion. Sensation intact
bilaterally in all dermatones Positive Roo's test. Equivical Adson's test Patient demonstrated essentially normal physical
exam. However, patient’s symptoms were reproduced with his right arm fully abducted and externally rotated.
Differential Diagnosis:
Thoracic outlet syndrome
Peripheral nerve injury/compression
Impingement syndrome
Brachial plexopathy
Radiculopathy
Venous/Arterial Thrombus
Rotator Cuff pathology
Strain
Sprain
Test Results: Venous and arterial ultrasounds of the right upper extremity were obtained with no evidence of vessel
obstruction or thrombus IR angiogram was then performed and did not show venous or arterial thoracic outlet syndrome.
However, during provocative maneuver of extreme abduction and external rotation patient exhibited compression of the
bilateral axillary arteries and circumflex humeral arteries. MRI of the right shoulder was negative for fat atrophy of teres
minor and infraspinatus mucles. Electrodiagnostic studies did not demonstrate abnormalities in the axillary, ulnar, or
median nerves with normal EMG of corresponding muscles.
Final Diagnosis: Quadrilateral space syndrome
Treatment: Patient was treated conservatively with observation Patient resumed activity and continued to swim
competitively with modification in training duration and regimen, demonstrating improvement in symptoms He continues
to have symptoms when he pushes beyond set training restrictions.
Outcome: He was able to swim competitively during his 3rd and 4th years of NCAA competition Patient elected not to
have surgical intervention as he was able to compete at an elite level despite his symptoms and changes in his training
methods.
Follow-Up: Plan is for patient to be evaluated by a surgeon after the conclusion of his NCAA athletic career for possible
surgical intervention.
Bilateral Groin Pain in a High School Football Player
Authors: Michael Burns, MD
Affiliation: Evergreen Sports Medicine Fellowship, Augusta, ME
History: 17-year-old male football player with 3 weeks increasing bilateral groin pain. The onset of pain occurred during
pre-season training and was insidious and without known trauma. No history of previous groin pain or pelvic injury. No
subjective weakness, loss of sensation, or range of motion. No back pain and no position related increase in pain. Primary
exacerbating factor is running and primary alleviating factor is rest. Mild pain with ambulation and at rest. No current
medication use. No dietary restrictions and no tobacco, alcohol, or drug use. Additional sports participation includes
lacrosse. Patient reported weight training six to seven times a week in addition to twice daily practice with his team. His
weight-training program focused on squats and lunges for lower extremity strength and did not incorporate lateral
movement or focused hip adductor exercises. No significant previous illness or hospitalizations. Family history was positive
for coronary artery disease, hypertension, and diabetes.
Physical Exam: Well-nourished male with BMI of 25.1 (5'7", 160 pounds). At rest, no apparent discomfort. No swelling,
ecchymosis, or evidence of abdominal or inguinal hernia. Tender to palpation over bilateral superior pubic rami. Minimal
tenderness over hip flexors and adductors. 4/5 strength in BL hip abductors and adductors with 5/5 strength in the
remaining upper and lower extremity muscles. Normal sensation and range of motion in lower extremities. Decreased
flexibility in bilateral hamstrings. Negative log roll and FADIR, positive FABER bilaterally, negative SI compression test.
Positive hop test bilaterally. Pain with resisted hip adduction and resisted flexion of the lumbar spine.
Differential Diagnosis:
Pelvic stress fracture(s)
Pelvic stress reaction
Chronic strain of the bilateral hip adductors
Acetabular labral tears
Chronic strain of the bilateral rectus femoris and/or illiopsoas
Congenital hip dysplasia
Synovitis
Inguinal hernia
Sports Hernia
Avascular necrosis of the femoral head
Osteochondritis dessicans of the femoral head
Pubic symphysitis
Test Results: X-ray hips and pelvis: normal alignment, no evidence of femoral acetabular impingement or fracture
MRI
pelvis: increased T2 signal in the bilateral superior and inferior pubic rami surrounding two linear regions of signal loss
consistent with stress fractures of the bilateral superior pubic rami. Serum vitamin D 25-OH: 47ng/mL
Final Diagnosis: Bilateral stress fractures of the superior pubic rami
Bilateral stress reaction of the inferior pubic rami
Treatment: Restricted from sports participation and weight training for six weeks.
Physical therapy to focus on
hamstring flexibility and, after four weeks of relative rest, hip abductor and adductor strength.
Outcome: Resolution of bilateral groin pain at rest and with ambulation. Some mild persistent intermittent pain
associated with running.
Follow-Up: Able to complete a graduated return to play program and return to unrestricted participation 6 weeks
following diagnosis and 9 weeks following onset of symptoms.
An Unusual Case of Chest Pain in a Swimmer
Authors: Clint Turner MD; Michele Kirk MD; Jason Mogonye MD; Josh Sole MD
Affiliation: JPS Sports Medicine, Arlington, TX
History: A 21 year old female presented to clinic with complaints of chest pain, shortness of breath, and a nonproductive cough 2-3 times per day The shortness of breath was exacerbated with exercise and when resting in the
supine position at night She denied congestion, sore throat, runny nose, or fever The pain was sharp and could be
localized over the right anterior ribcage at ribs 7 and 8, from the sternum to the midclavicular line The pain became
increased and radiated to the back while in the horizontal position when swimming laps. It was exacerbated by deep
breaths, direct palpitation, and cough She had a history of controlled asthma on albuterol HFA and had previously been
on Flovent Flovent had been discontinued two years ago due to good asthma control Her symptoms did not improve
with albuterol HFA.
Physical Exam: Lungs: CTAB no wheezing or rales - CV: RRR no m/r/g - Chest: tender over ribs 7-8 with palpitation,
pectus excavatum bilaterally - Abd: BS+ NTND - MSK: Back: no pain with palpation, no muscle wasting on bilateral sides;
Shoulder: no pain with flexion/extension/abduction/adduction/rotation bilaterally; full equal ROM; neg Hawkins, speed,
cross-arm test bilaterally - Neuro: no loss of sensation in extremities
Differential Diagnosis: Costochondritis; Asthma exacerbation; Rib contusion; Intercostal strain; Serratus anterior strain;
Pectus excavatum pain; Pulmonary fibrosis; Pleuritis; Diaphragmatic spasms
Test Results: Chest XR was negative for any acute process or bony abnormalities; Intercostal nerve block of ribs 7 and 8
on the right resulted in no immediate improvement in pain; MRI chest showed pectus excavatum with Haller Index of 5
Final Diagnosis: Pain from pectus excavatum
Treatment: The patient was initially treated for asthma and started on Flovent 44 mcg 2 puffs inhaled BID for 1 week and
withheld from swimming She had no improvement of shortness of breath or cough so Flovent was discontinued She was
then treated for costochondritis and started on ibuprofen 800 mg PO TID for two weeks. During this time, the shortness of
breath resolved spontaneously, but the pain remained. Therefore, a diagnostic and therapeutic intercostal nerve block of
ribs 7 and 8 on the right side was performed in clinic with 1 ml of a combination of Marcaine 0.25% without epinephrine
and dexamethasone 20 mg The patient had numbness of the ribs but continued pain at the sternocostal joint. Tramadol
50 mg PO BID prn was prescribed for the pain Her pain slowly improved with rest but would return with swimming
strokes She was slowly started back to swimming as pain allowed An MRI was performed which found severe pectus
excavatum; therefore she was referred to CT surgery for possible surgical procedures CT surgery recommended
continuing conservative management.
Outcome: The patient has had a slight improvement in chest pain since being withheld from swimming and having
intercostal nerve block She is still having mild chest pain with overhead activities and when performing swimming strokes.
Follow-Up: The patient will slowly be worked back into swimming as tolerated She is starting with kicking drills and
working towards full swim strokes She will be following up in clinic every 2-3 weeks until pain completely resolves If the
pain continues we will consider doing a right sternocostal block under ultrasound or a transverse abdominis plane block.
Facial Trauma and Return-to-Play in a Football Player
Authors: Ravishankar E. Rao, MD
Affiliation: University of Michigan Health System, Ann Arbor, MI
History: 20 year old college football quarterback involved in off-the-field altercation sustains blow from elbow to left
cheek bone on a Sunday night. The following Monday he was seen complaining of swelling and bruising but no
significant pain. He was noted to have mild upward gaze deficit in the left eye. He denied blurry or double vision and
otherwise felt well. Given head trauma, he was evaluated for concern for concussion. After examination and repeat
baseline testing, no concussion was diagnosed. Given severe facial swelling and ecchymosis, as well as tenderness to area
of impact, he was held from practice pending further work-up.
Physical Exam: On physical examination, he was noted to have a subtle upward gaze deficit on the left but otherwise
intact extra-ocular motion bilaterally and negative for diplopia throughout all visual fields. No septal deviation or
intranasal bleeding/discharge noted. Significant infra-orbital and facial swelling present with diffuse tenderness to
palpation of the orbital rim. Otherwise, afebrile with normal vital signs in no acute distress. Normal jaw movement with
associated generalized facial pain on the left. Facial sensation grossly intact and symmetric to light touch.
Differential Diagnosis: - Facial soft tissue contusion - Orbital/Facial fracture with or without entrapment Mandibular injury/fracture
Test Results: CT Maxillofacial w/o Contrast - Mildly displaced fracture through the left zygomatic-maxillary, frontozygomatic, and zygomatic-temporal sutures with component extending through anterior aspect of left maxilla. No
evidence of entrapment of orbital fat or extra-ocular muscles or infra-orbital foramen.
Final Diagnosis: Facial fracture, mildly displaced and depressed, without extra-ocular or orbital fat entrapment. Patient
was referred to otolaryngology and seen 4 days post-injury. By this time, his swelling had improved and ecchymosis
surrounding the orbit became
Treatment: Patient was initially non-contact/non-exertion for the first 2 weeks post injury. He was subsequently
maintained non-contact with allowcance for lifting and endurance training through 6wks post-injury to allow adequate
bone healing prior to donning helmet and returning to unrestricted play.
Outcome: Patient made full recovery with mild cosmetic depression of left facial bones.
Follow-Up: Given injury to region of face which contacts helmet padding, patient was initially restricted from
participation (including practice, lifting, and endurance training) for a period of 2 weeks. He was followed with serial
examinations weekly. Given stable palpation of facial bones, as well as continued improvement in swelling, pain, and
bruising; he was cleared for non-contact/non-helmeted practice with lifting/endurance training. At 6 weeks post-injury,
his fracture remained stable to palpation and he was cleared for full return to play.
Mom! What's on My Finger?
Authors: Aneesh Garg, DO; Jody Ortega, MD
Affiliation: Andrews Sports Medicine and Orthopaedic Center - Birmingham, AL
History: A 9 year-old right-hand dominant third-grade cheerleader with no significant past medical history presents with
a greater than six month history of a small 0.5 x 0.5 cm, mobile, tender, round, bluish-colored mass on the distal portion of
the her palmar middle finger near her DIP joint She does not recall any trauma or inciting event that may have caused the
growth There was no fluctuation of the mass. The only tenderness experienced is when direct pressure is placed on it or
with deep flexion of her finger The rest of her history and review of symptoms was non-contributory.
Physical Exam: Her right hand/wrist exam was normal with the exception of the small 0.5 x 0.5 cm, mobile, non-tender,
round, bluish-colored mass on the distal portion of the her palmar middle finger near her DIP joint Her flexion of both PIP
and DIP was not limited These findings were in comparison to the left hand/wrist exam which was normal She was
neurovascularly intact bilaterally.
Differential Diagnosis:
mucous
synovial
ganglion
epidermoid inclusion cysts
giant cell tumor of the tendon sheath
glomus tumor
ateriovenous malformation
enchondroma
Test Results: Initial AP/Lateral/Oblique views of her fingers did not showed some soft tissue enlargement but no acute
factures, dislocations or bony abnormalities An MRI of her hand with and without contrast did not show any abnormality.
Final Diagnosis: Our final working diagnosis was an arteriovenous malformation given the physical exam findings and
negative imaging work-up.
Treatment: The patient was referred to our hand surgeon for further evaluation and the treatment plan consisted of
surgical excision vs. conservative observation.
Outcome: The patient and mother were still deliberating the course of action.
Follow-Up: She will be allowed to return to her full activities and will follow up on an as needed basis It is important to
keep this diagnosis in mind based on physical exam in the setting of a negative imaging work-up.
A Prescription for Exercise Avoidance
Authors: Bradley Changstrom, MD; Brian Blaker, MD; Javier Valle, MD; Brian Stauffer, MD
Affiliation: Division of Sports Medicine, Department of Family Medicine and Community Health, University of
Minnesota, Minneapolis, MN
History: A 26 year-old recreational female runner with history of ADHD was evaluated after developing chest pain,
dyspnea and palpitations during a 5-mile training run. Her symptoms developed near the end of her run and did not
resolve with rest. After two hours, her chest tightness and palpitations progressed and she presented to the emergency
department. She denied recent illness, fevers, chills, orthopnea, syncope, calf pain, bleeding disorder or edema. She denied
illicit drug use or tobacco use. She admitted to infrequent alcohol use. She uses prescription amphetaminedextroamphetamine occasionally, but none recently. She denied any family history of sudden cardiac death or structural
heart disease.
Physical Exam: Vital Signs: Temperature 98.4° Blood pressure of 140/90, pulse 225, respiratory rate of 16 and oxygen
saturation of 98% on room air. General: Mildly distressed due to chest pain. HEENT: No proptosis or conjunctival pallor.
Neck: Supple, no lymphadenopathy or thyromegaly. Cardiovascular: Tachycardic with regular rhythm. No JVD. No S3
gallop or murmurs. No lower extremity edema. Pulmonary: Clear to auscultation bilaterally. Abdomen: Soft, non-tender.
No hepatosplenomegaly. Skin: No rashes. Neurologic: Normal cranial nerve testing, normal strength, normal reflexes and
normal sensation.
Differential Diagnosis: 1) Hyperthyroidism
2) Electrolyte abnormality 3) Supraventricular tachycardia 4) Exercise
induced bronchospasm 5) Cardiomyopathy 6) Anomalous coronary artery 7) Wolff-parkinson white syndrome 8)
Symptomatic anemia 9) Pulmonary embolism 10) Recreational drug use 11) Pulmonary hypertension
Test Results: Potassium: 3.7. Creatinine: 0.9. Glucose: 120. Magneisum: 1.7. Hgb: 13.0. MCV: 79. Troponins 0.66 then 6.0.
TSH 12.1. free T4: 0.96. Urine toxicology negative. Urine pregnancy negative. Chest radiograph without infiltrates or cardiac
enlargement. D-dimer negative. 1st EKG: Wide complex tachycardia at 225 bpm, left bundle left inferior axis. There was
no resolution of the wide complex tachycardia with three doses of adenosine or with administration of amiodarone. Just
prior to an attempted cardioversion, the patient was given a dose of midazolam and she converted to a normal sinus
rhythm. 2nd EKG: T-wave inversions in V1-V3 and multiform PVCs. Transthoracic echocardiogram: Normal left
ventricular size and function with estimate ejection fraction of &gt;65%. The right ventricle size was normal. Concern for
right ventricular apical hypokinesis. No valvular abnormalities. Cardiac catheterization with coronary angiography:
Angiographically normal coronary arteries. Left ventricular end diastolic pressure of 17 mm Hg. Cardiac MRI: Decreased
right ventricular ejection fraction at 32%. Borderline low left ventricular ejection fraction at 54%. Electrophysiology study:
Several PVC morphologies noted; predominately left bundle axis and superiorly oriented. No inducible tachycardia.
Final Diagnosis: Based off of major criteria of T-wave inversions in V1-V3 and depressed RVEF &lt;40% on cardiac MRI
and minor criteria of sustained ventricular tachycardia with left bundle, left inferior axis, the patient was diagnosed with
arrhythmogenic right ventri
Treatment: The patient was started on metoprolol. She initially refused placement of an automatic implantable cardiac
defibrillator (AICD) due to cosmetic concerns of the device and therefore discharged with a life vest. She was counseled on
discontinuation of exercise as this can advance the disease progression of ARVC/D. Upon follow up, she proceeded with
placement of an AICD.
Outcome: The patient initially elected to discontinue her beta-blocker and go for a run. She subsequently experienced
an appropriate ICD firing due to an episode of ventricular tachycardia. She has since continued her beta blocker. She has
not followed up with recommended genetic testing.
Follow-Up: The patient was instructed to not return to physical activity as exercise increases the risk of ventricular
arrhythmia and worsening of cardiac function in patients with ARVC/D. The patient follows-up intermittently with her
cardiologist.
A Contractor's Rare Thumb Injury
Authors: Danielle DeGiorgio, DO; Bradley Sandella, DO
Affiliation: Christiana Care Health System
History: 56 yo male home contractor, presents to sports medicine clinic with left thumb pain that began while working a
few months prior. Patient reports, while forcefully holding a piece of wood, he developed a cramping sensation in his arm
associated with pain at the proximal aspect of the thumb and inability to move thumb backwards or into extension. He
denies any numbness, tingling or loss of grip strength. The pain is 7/10 over thumb with movement. Patient trialed a
thumb spica brace without improvement of pain or function.
Physical Exam: Left Hand: On observation there is no obvious deformity, edema or atrophy of lumbricles/ interossei
muscles. On palpation, sensation is intact to light touch throughout with a mildly tender nodule at the base of the thumb.
Range of motion and muscle strength testing of the thumb is within normal limits (opposition, abduction, adduction,
flexion), with exception to extension, which is absent. There is a normal pincer grip, power grip and negative "OK" sign
Ulnar and radial stressing of the MCP joint does not reveal any laxity Radial and ulnar pulses are intact.
Differential Diagnosis:
UCL sprain
Avulsion fracture
Dequervain's tenosynovitis
EPL rupture
EPB rupture
Radial neuropathy
Test Results: X Ray: A small ossific or calcific density noted adjacent to the dorsal aspect of the first MCP joint. Minimal
soft tissue swelling is noted in this region. This could represent a small avulsion injury or chondrocalcinosis. Ultrasound
evaluation: Normal visualization of the left distal radius, radio-carpal joint, 1st CMC, MCP joint and 1st proximal phlanx.
There is a hypoechoic area noted along the proximal track of the EPB tendon. There is also marked soft tissue disruption.
MRI Hand/Wrist: Full thickness tearing of the extensor pollicis brevis and extensor pollicis longus tendons. No evidence of
previously visualized ossification.
Final Diagnosis: Rupture of EPL/EPB
Treatment: The treatment options, risks and benefits were discussed at length with the patient He deferred surgical
intervention at this time, due to work schedule. He is currently enrolled in physical therapy to maintain range of motion,
strength and function.
Outcome: Patient is currently able to tolerate his impairment and participate in his activities of daily living and
occupational requirements.
Follow-Up: The patient is fully participating in his activities of daily living and occupation despite EPL and EPB tendon
rupture. He will follow up at the conclusion of his occupational busy season, to reevaluate further management.
Swelling in the Softball Athlete
Authors: Daniel Murphy, MD; Tony Islas, MD; Justin Wright, MD
Affiliation: Texas Tech University Health Science Center. Paul L. Foster School of Medicine, El Paso, TX
History: A 21 year old female, university softball player currently on oral contraceptive medication with a history of
asthma and a family history of uterine and ovarian cancer, presented to the local emergency room for evaluation and
treatment of left axillary tenderness. Patient was treated for a mild case of cellulitis and was discharged home. Patient
subsequently developed left arm swelling one day after receiving treatment at the local ER. Swelling progressively
worsened, resulting in the patient seeking further treatment by the sports medicine physicians at her university three days
later. Patient was sent immediately to another emergency room for further evaluation and treatment.
Physical Exam: Patient had a temperature of 97.4°, heart rate of 79, respiratory rate of 18, oxygen saturation of 97% on
room air, with a blood pressure of 114/58. Patient was alert and oriented with no acute distress. Pupils were PERRL and
extra ocular muscles were intact. Mucous membranes were moist with a supple neck without jugular venous distention.
Lungs were clear to auscultation and symmetric. No wheezes, rhonchi, or rales. Heart demonstrated a regular rate and
rhythm with no obvious murmurs. Abdomen was soft, non-tender with active bowel sounds and no organomegaly. Edema
noted in the left upper extremity in comparison to her right, but no evidence of edema in her lower extremities. Tender
lymphadenopathy noted in the left axilla.
Differential Diagnosis:
Deep venous thrombosis
Infection
Malignancy
Trauma
Anatomical variance/Structural abnormalities
Test Results: Arterial ultrasound of the left upper extremity demonstrated no significant stenosis, but identified a deep
venous thrombosis extending from the left subclavian vein to her brachial vein. Prothrombin time, partial prothrombin
time, and INR were within normal range. Anti-thrombin 3 activity was low at 76.90%. Anti-neutrophilic antibody was
negative. Cardiolipin IgM, IgG, and IgA antibody were within normal range at less than 9 MPL/mL. No Factor V Leiden
mutation was identified. Protein C activity was within normal range at 102%, and Protein S was low at 49% Homocystine
level was low at 4.69 umol/L. Factor II (prothrombin) gene mutation was negative. No Lupus Anti-coagulant detected.
Final Diagnosis: Deep venous thrombosis left upper extremity secondary to oral contraceptive use
Treatment: Anti-coagulation treatment with Xarelto(Rivaraxoban). Referral to a gynecologist to discuss alternative
options for birth control excluding oral contraception. Follow-up with hematology for ovarian and uterine cancer
screening, and determine when anti-coagulation may be discontinued.
Outcome: Pending results from cancer screening due to family history of ovarian and uterine cancer in paternal
grandmother. Patient was initially started on low molecular weight heparin and was subsequently switched to
Xarelto(Rivaraxoban) for long term treatment. Follow-up evaluation with hematology to determine length required for
anti-coagulation treatment.
Follow-Up: Full return to activity is expected with caution as patient is currently on Xarelto(Rivaraxoban). Evaluation by
gynecology for alternative birth control options. Regular follow-up with hematology.
Why are my hands two different colors?
Authors: Eduardo Garza, MD, Texas Tech University Health Sciences Center Family Medicine Residency, El Paso,
Texas Justin M Wright, MD, CAQ-Sports Medicine, Texas Tech University Health Sciences Center Sports
Medicine Fellowship, El Paso, Texas
Affiliation: Texas Tech University Health Sciences Center Family Medicine Residency, El Paso, TX.
History: A 58 year-old male new patient presented to the Sports Medicine Clinic for left shoulder pain The pain started 4
days prior with a gradual onset He described it as intermittent and dull in quality No prior history of trauma to the
affected area. OTC medications provided mild relief. Past medical history includes hypertension and an episode of
shingles He was taking no regular medications The patient works as a truck driver, has a history of alcohol abuse in his
past, and is an ex-smoker.
Physical Exam: Physical examination of the left shoulder revealed full range of motion, mild tenderness to palpation at
the posterior aspect, and 4/5 strength in abduction and external rotation Hawkin’s and Neer’s were negative Given the
patient’s age, rotator cuff tendinopathy was suspected.While discharging the patient, he asked, “why are my hands two
different colors?&rdquo; On further evaluation, he noted an abnormal sensation in his left hand, which was hyperemic
with symmetrical pulses.
Differential Diagnosis:
Rotator Cuff Tendinopathy
Malignancy
Primary upper extremity deep vein
thrombosis
Frozen Shoulder
Test Results: An X-ray and US Venous Doppler ordered for the Left shoulder. X-ray revealed no fractures or skin
abnormalities US Venous Doppler revealed no evidence of thrombosis, but did reveal an ovoid structure, 4.2 x 2.8 cm,
suggesting lymphadenopathy with a malignant etiology. Biopsy confirmed Metastatic Hepatocellular Carcinoma.
Final Diagnosis: Metastatic Hepatocellular carcinoma
Treatment: Started chemotherapy with Nexavar (Sorafenib) then switched to a clinical trial of Erbitux (Cetuximab) and
Avastin (Bevacizumab)
Outcome: MRI of Liver showed small enhancing lesions consistent with multifocal hepatocellular carcinoma Liver is
cirrhotic with borderline splenomegaly. Started chemotherapy with Nexavar (Sorafenib) then switched to a clinical trial of
Erbitux (Cetuximab) and Avastin (Bevacizumab)
Follow-Up: Poor prognosis. TNM Stage 3 16% 5 year survival rate Currently doing well and following up at MD
Anderson.
The Shoulder Pain That Wasn't
Authors: Farah Tejpar, MD; Eric Anish, MD; Jay V. Kalawadia, MD
Affiliation: University of Pittsburgh Medical Center, Pittsburgh, PA
History: During a game, a 20-year-old, right hand dominant, quarterback for a Division I football team sustained a tackle
from his left side, driving his right shoulder into the ground He had immediate pain in the left shoulder and walked off
the field He complained of pain and clicking along the inferolateral border of the left scapula that was worse with
abduction of the shoulder A brief sideline exam determined that he was able to return to the game After two plays, he
was tackled again from the left side He then complained of severe pain in the left scapula and shoulder, and was unable
to return to play.
Physical Exam: On exam, he had no swelling or deformity of the left shoulder girdle He had full range of motion (ROM)
of the left shoulder, but scapulothoracic dysrhythmia was evident He had tenderness to palpation at the sternoclavicular
(SC) joint, in the left supraclavicular fossa, and along the inferolateral border of the scapula Strength testing revealed 4/5
strength with external rotation on the left He had pain at the left SC joint with cross-body adduction. His cervical spine
ROM was full, but lateral bending elicited pain in the posterior shoulder. There was no spinous process tenderness. He
was neurovascularly intact in the left upper extremity with pulses and sensation equal to the right side.
Differential Diagnosis: Scapular body fracture, SC joint sprain, Infraspinatus strain, Acute brachial plexopathy
Test Results: A CT scan of the scapula was ordered on the day of injury which did not reveal scapular pathology, rather a
left anterior non-displaced first rib fracture was identified A chest x-ray was unable to visualize the first rib fracture.
Final Diagnosis: Left Non-Displaced First Rib Fracture
Treatment: Sling-immobilization for one week, followed by gentle ROM exercises of the left shoulder. Over the next two
weeks, he was allowed to progress with non-contact football activity as tolerated.
Outcome: Three weeks post-injury, the athlete's pain had improved significantly and a non-contrast chest CT scan
demonstrated interval callus formation around the fracture site.
Follow-Up: A decision was made to allow the athlete to return to play based on his physical exam and CT scan results
three weeks post-injury. However, the athlete still did not feel comfortable throwing the ball with maximal velocity and
opted to defer his return to competition. He will continue with a modified, non-contact practice schedule, with the hope
that he is able to return to play over the next one to two weeks.
Shoulder Injury in a MMA Fighter
Authors: Brian Fuller, MD; Emily Dixon, DO; Richard Okragly, MD
Affiliation: Trihealth Bethesda Family Practice Primary Care Sports Medicine, Cincinnati, OH
History: 38 year old MMA fighter presents wtih left shoulder pain and weakness for 2 months status post "grappling"
injury in which he was caught in an arm bar with his arm abducted and externally rotated. He is noted to have a history of
a succesfully rehabbed labral tear that is one year old. He also reports difficulty sleeping with frequent awakening
secondary to pain. He has been unable to train/lift since the injury. He has tried to resume his home PT exercises but has
not had relief.
Physical Exam: General Appearance: Muscular, well appearing healthy male. Left Shoulder: Mild atrophy of
supraspinatus and right suprascapular when compared to contralateral side. ROM: Forward Flexion 160, ABduction 150,
External rotation 20, and Internal to T7 Palpation: TTP over bicipital groove. Strength: 4/5 abduction, 4/5 external
rotation, 5/5 internal rotation Special Tests: Positive O'Brien, Hawkin's, Neer's, and Speed's test
Differential Diagnosis:
1.
2.
3.
4.
5.
6.
7.
Rotator Cuff Impingement
Biceps Tendonitis
Repeat Labral Tear
Rotator Cuff Tear
Cervical Nerve Impingement
AC joint Arthitis
Quadralateral Space Syndrome
Test Results: 3 View Xray of Left Shoulder: Mild to Moderate AC joint Arthritic Change, Type 2 Acromion MRI Left
Shoulder: IMPRESSION 1.No evidence of a rotator cuff tear. 2.Development of a large spinoglenoid notch ganglion
cyst2.9 x 2.7 x 3.6 cm Smaller cysts adjacent to the posterior labrum are also again noted, all likely related to the posterior
labral tear. 3.Moderately severe degenerative changes at the acromioclavicular joint
Final Diagnosis: 1. Ganglion Cyst in Spinoglenoid notch compressing suprascapular nerve. 2. AC joint arthritis
Treatment: Cyst was drained under ultrasound guidance. Depomedrol was injected into the cyst space. Resumed home
physical therapy.
Outcome: Immediately following drainage and injection patient reported improved function with no pain and full
strength on exam.
Follow-Up: Patient reports he has resumed lifting activities without difficulty but has yet to return to MMA. Further
follow up is pending at this time.
Not Your Ordinary Ankle Sprain
Authors: Amy McClintock, MD; Nilesh Shah, MD
Affiliation: Summa Health System, Akron, OH
History: Eighteen-year-old female with no significant past medical history referred to the sports medicine office for
evaluation of right ankle pain for one month Initial injury occurred one month prior when patient slipped while running
around a car suffering an eversion type injury She was evaluated in the emergency department on day of injury. Per ED
report, radiographs were negative and she was diagnosed with an ankle sprain She was given crutches and placed in an
ankle brace with instructions to follow up with her PCP. The patient limped into her primary’s office 3 weeks later with
slowly improving, but persistent pain On review, her PCP noted official read of x-rays from ED showed an ankle fracture.
She was instructed to be non-weight-bearing and referred to the sports medicine office The patient was seen one week
later (4 weeks post-injury) She arrived walking on her right foot with slightly antalgic gait Her only complaint was mild
ankle stiffness with plantarflexion and dorsiflexion She denied pain at that time Review of systems was otherwise
negative.
Physical Exam: Vital Signs normal Pleasant female in no acute distress. Right ankle had mildly decreased range of
motion in all directions Mild discomfort to palpation across talar neck Normal neurosensory and vascular exam.
Differential Diagnosis: Ankle sprain, talus fracture, lateral or medial malleolus fracture, Mislabeled initial x-rays
Test Results: Initial ED radiographs available for review showed a Hawkins Type II Talar neck fracture with 15 mm
displacement at the subtalar joint-posterior facet causing near 100% dislocation of the talar body. Repeat X-rays in office
showed shortening of the talar neck with faint fracture line through the neck of the talus extending back diagonally to the
subtalar joint with no significant dislocation Ankle mortise appeared intact. CT ankle confirmed oblique fracture through
talar neck with minimal displacement of the fracture anteriorly-superiorly measuring 1.4 mm. There was no significant
displacement at the subtalar joint-posterior facet Bony bridging was noted across fracture site.
Final Diagnosis: Talar neck fracture-dislocation with apparent self-reduction by ambulation
Treatment: She was made non-weight-bearing at initial sports medicine visit and referred to orthopedic surgeon for
possible surgical repair.
Outcome: The patient cancelled her initial orthopedic appointment and rescheduled 2 weeks later She arrived at
orthopedic visit non-complaint with weight-bearing status She denied any pain at that time and requested note to return
to work. Exam showed patient ambulating with normal gait She lacked about 5 degrees of dorsiflexion and 5-10 degrees
of plantarflexion with mild subtalar stiffness compared to her contralateral ankle She had mild ankle swelling, but no
tenderness. Repeat x-rays showed well maintained alignment with signs of healing There were no signs of avascular
necrosis After lengthy discussion about risks, the orthopedic surgeon agreed to allow her to continue to weight-bear as
tolerated with repeat x-rays in 6 weeks.
Follow-Up: At follow up 3 months after initial injury, patient remained pain free Exam showed symmetric range of
motion to bilateral ankles with no pain or tenderness She was released to follow up as needed with instructions that if any
pain occurs she is to be non-weight-bearing and return for evaluation. One year since last visit and she has yet to call with
complaints of pain.
A Case of Persistent Shortness of Breath
Authors: Christopher P. Varacallo, DO; David Jenkinson, DO; Owen Speer, DO; Kory L. Gill, DO
Affiliation: University of South Carolina Sports Medicine, Columbia, SC
History: 20-year-old female division 1 track athlete developed chest pain, shortness of breath and tachypnea during a
particularly vigorous sprint workout. The shortness of breath progressed and tachypnea could not be controlled on site
and for that reason the decision was made to transfer the athlete to the emergency department for higher acuity care
Previous medical history included ulcerative proctitis for which she was taking mesalamine by mouth daily and gastritis for
which she was taking pantoprazole daily Pertinent procedural history included EGD and colonoscopy four weeks prior to
presentation No pertinent family history. and social history was unremarkable.
Physical Exam: In the Emergency Department, the patient demonstrated tachypnea and tachycardia but otherwise was
hemodynamically stable with normal blood pressure. She was in moderate respiratory distress that resolved with paper
bag ventilation Her lungs were clear with full breath sounds bilaterally Heart was tachycardic without murmur Skin
showed adequate and brisk perfusion Sensation was normal and cranial nerves were intact.
Differential Diagnosis:
Acute exacerbation of asthma
Pneumothorax
Pulmonary embolism
Acute coronary syndrome
Hyperventilation
Anxiety
Test Results: Normal metabolic panel and complete blood count Normal oxygen saturation with pulse oximeter
Noteably, chest x-ray was not done.
Final Diagnosis: Evaluation in the emergency department led to the diagnosis of anxiety attack causing tachypnea
Athlete was given paper bag to breathe into while in the ED which improved her symptoms She was discharged home
and cleared for regular activity.
Treatment: No treatment in the ED or at discharge other than paper bag ventilation.
Outcome: One day later, the athlete boarded a plane and traveled to the Southeastern Conference track meet and
competed in the 400 meter dash at which time she experienced repeated shortness of breath and chest pain that persisted
for 20 minutes post race Her oxygen saturation on site was 80% on room air so she was given supplemental oxygen and
the decision was made to transfer to the emergency department for further evaluation. In the Emergency Department, a
chest x-ray was done which showed a small amount of air underlying the left upper pericardial border and the diagnosis
of pneumomediastinum was made Athlete was subsequently admitted for further monitoring and testing, which included
echocardiogram and gastrografin esophogram, both of which were within normal limits Repeated chest x-ray showed
stable pneumomediastinum and shortness of breath and tachypnea resolved while in the hospital Her vitals remained
stable and she was discharged home Due to her diagnosis she did not fly with the team back to campus, she was advised
to make the return trip via car. The final diagnosis: ; Spontaneous pneumomediastinum due to subpleural bleb rupture
secondary to vigorous exercise
Follow-Up: Outpatient evaluation by cardiothoracic surgery included CT scan which demonstrated air in the
mediastinum She was followed over the next six weeks until pneumomediastinum resolved on chest x-ray She was
witheld from activity during the full six weeks After resolution of the pneumomediastinum, she was cleared for regular
activity She progressed slowly back to full activity but obtained a medical redshirt for the spring track season. Currently
the athlete is participating in full activity and is back to sprinting without residual symptoms.
Not All That Quacks is a Duck, A Knee Pain Diagnostic Quandary
Authors: Catherine R. Mygatt, MD; Purnima Bansal, MD
Affiliation: Greater Lawrence Family Health Center, Lawrence, MA
History: 32 y/o male with Bipolar Disorder presents to his PCP with bilateral knee pain. While playing basketball 7
months prior to presentation, his left leg buckled laterally after a player fell on him. He was seen in the ED and had normal
knee x-rays. He was given a knee immobilizer and told to ice it and follow up with his PCP. He did not follow up due to
lack of insurance. Over the course of 7 months, while his left knee pain persisted, he gradually developed constant, diffuse,
achy 10/10 right knee pain, associated with stiffness and low back pain. It was worse with walking and with cold weather.
No alleviating factors.
Physical Exam: Initial physical exam noted diffusely tender left knee and decreased active and passive range of motion
bilaterally. He had positive varus stress test and McMurray’s test on the left. His spine exam was significant for bilateral
lumbar and paraspinal tenderness with negative leg raise bilaterally. 5/5 strength bilateral lower extremities. He had an
antalgic gait. DTRs were not evaluated. -- An MRI is ordered.-- In the meantime, the patient returns with worsening knee
and back pain not relieved by NSAIDs. He is noted to have an antalgic gait that appeared to normalize when he is denied
narcotic pain medication. His PCP feels that his pain is out of proportion to his physical exam and MRI, so orders
additional labs and refers him to Sports Medicine.
Differential Diagnosis: Lateral meniscal injury, LCL chronic sprain, Chondromalacia patella, Osteoarthritis, Inflammatory
condition, Lyme disease, Factitious pain disorder
Test Results: MRI left knee: Small joint effusion with Grade 2 chondromalacia over medial patellar facet Normal
rheumatoid factor, Lyme titers, ANA, CBC, and ESR.
Final Diagnosis: Working diagnosis: Chondromalacia Patella, possible drug-seeking. -- Case development: The patient
misses his initial sports medicine appointment. At his rescheduled appointment, his chief complaint and HPI are
unchanged except that he repo
Treatment: Patient was referred to neurosurgery emergently, who performed a same-day spinal decompression.
Outcome: One year postoperatively, he continues to have chronic back pain and persistent spasticity in his right lower
extremity that is managed with baclofen and diazepam. He has regained full strength bilaterally, and although
hyperreflexic with 3+ reflexes, his clonus is no longer sustained. The patient can now ambulate without a cane.
Follow-Up: This case highlights the following important principles: 1) Not all knee pain stems from the knee 2) The
extended physical exam is paramount, particularly when pain is out of proportion to the working diagnosis 3) Physicians
must be cautious to not discount complaints from patients with psychiatric comorbidities, whose social interactions may
lead us to mistake an organic disorder for a fictitious one.
A Season Ending Mountain Bike
Authors: Michael Rigby, DO; Sara Neal, MD; Evan Corey, MD
Affiliation: Cone Health Family Medicine Residency, Greensboro, NC
History: A 24 year old male USA Cycling Category 1 Mountain Biker, experienced a "routine" wash out (crash) during a
local race attributed to excessively muddy conditions. Although a seemingly innocent mechanism, he reported direct
impact of his handlebar to his left inguinal region. He experienced immediate and significant pain at the sight of impact
with associated pain and paraesthesia radiating down the left anterior thigh. His discomfort was worsened by active
motion and was relieved by applying direct pressure over the inguinal region. Although able to briefly bear weight, he
required Fire/Rescue evacuation from the course due to his inability to walk due to the pain. Medical history:
unremarkable Medications: none Surgical history: left inguinal hernia repair.
Physical Exam: Vital Signs WNL - mild distress, no extremis. Appropriately alert and interactive. Left Leg Exam: Skin
intact, inguinal region with small amount of non-pulsatile swelling with circular mark in the center of the swelling.
Moderate tenderness. Painful Log Roll, stable pelvic rock. Neurovascular Exam: Bilateral femoral, popliteal, dorsalis pedis
pulses are 2+/4, Sensation is grossly intact.
Differential Diagnosis: Simple contusion; Pelvic fracture; Femur fracture; Femoral cutaneous nerve contusion/injury;
Femoral nerve injury; Traumatic Abdominal Hernia; Femoral artery/vein injury
Test Results: CT of the abdomen and pelvis showed a moderate-sized left groin hematoma with no osseous irregularity
and no evidence of active extravasation. However, there was a suspicious irregularity within the left common femoral
artery (CFA). A subsequent CTA of the pelvis showed a focal dissection involving the left proximal CFA with a nearly
occlusive thrombus in the distal CFA.
Final Diagnosis: "Motor-Scooter Handlebar Syndrome" - blunt traumatic injury of the femoral artery: 1) Left-sided,
incomplete, femoral artery transection status post endarterectomy with vein angioplasty
2) left suprapubic hemat
Treatment: The patient was admitted by vascular surgery and observed overnight. His pain and discomfort improved
while receiving monotherapy aspirin and his neurovascular status remained intact. ;Following discussion of management
options he was discharged post injury day #1 on ASA and Rivaroxaban therapy with activity restrictions.
Outcome: Unfortunately, on post injury day #5 the patient experienced significant worsening in his pain and swelling in
the left suprapubic and scrotal areas. Repeat, emergent CTA showed a significantly enlarging left inguinal hematoma (7.5 x
5.6 x 10.2 cm) and persistent left common femoral artery dissection with intraluminal thrombus. He was taken to the
operating room and the hematoma - attributed to nonspecific venous injury - was evacuated. The common femoral artery
was found to be externally intact. Further investigation revealed complete transection of the tunica intima and media with
resultant flaps which were heaped together causing obstruction of the vessel; there was no thrombus present. ;An
endarterectomy and vein angioplasty was performed. He was discharged on postop day #3 on aspirin monotherapy. At
two-week postop followup there was a small recurrent liquefied hematoma which resolved with needle aspiration
Follow-Up: He was allowed to return to work at 3 weeks status post surgery. The patient began light, non-competitive
cycling at 6 weeks status post surgery. He will be allowed to return to unrestricted cycling at 3-&frac12; months status
post surgery including competition.
Got Water?
Authors: John D. Baldea, MD; Conan V. Chittick, MD
Affiliation: Indiana University Department of Family Medicine, Primary Care Sports Medicine Fellowship,
Indianapolis, IN
History: A 50YO white male basketball referee requested water near the end of a game. After the next play, the referee
collapsed, unconscious and convulsing on the court. A strong carotid pulse was quickly confirmed by the lead author.
The referee regained consciousness, he denied any chest pain, and only reported being thirsty. The referee sat up to drink
some water, became unresponsive, his skin turned pale white and he again lost consciousness. A strong carotid pulse was
confirmed, and he regained consciousness quickly. He added that he had nothing to eat or drink all day due to a busy
schedule He denied any medical conditions, and he took no medications.
Physical Exam: Vital Signs - BP 122/86, Pulse 60, Respirations 18 General - AAO X 3, NAD, sweaty CV - rrr, no m/r/g
2+ radial and carotid pulses bilaterally Resp - clear to auscultation bilaterally Abd - soft, NT, NABS, no HSM, no
tenderness, rebound or guarding Neuro - speaking coherently, could not complete due to arrival of EMS
Differential Diagnosis:
Exercise Associated Collapse
Dehydration
Cardiac Arrhythmia
Myocardial Infarction
Hypoglycemia
Seizure
Neurogenic Syncope
Test Results: An EKG in a local Emergency Department showed sinus bradycardia, ST-elevation in leads II, III, and aVF
with reciprocal changes in the precordial leads He was transported for emergent cardiac catheterization.
Final Diagnosis: Myocardial Infarction
Treatment: Upon catheterization, he was found to have a totally occluded mid-RCA, which was treated with a
thrombectomy, followed by angioplasty and stenting.
Outcome: The referee was discharged 48 hours later, and began cardiac rehabilitation.
Follow-Up: The referee continues to work basketball games, with no further episodes of syncope or other medical
concerns This case demonstrates a highly uncommon and unexpected presentation of a common medical diagnosis.
Traumatic Knee Pain in a Middle School Football Player
Authors: Christopher Urbanek, DO
Affiliation: UPMC St. Margaret Family Medicine Residency, Pittsburgh, PA
History: The patient is an otherwise healthy 13 y/o M complains of R medial knee pain and swelling after being tackled at
football practice the evening prior. He is unsure of the mechanism but does recall one specific play that began the
symptoms. He was evaluated by the athletic trainer on the field and was able to return to play, however with limited
capacity. He denies clicking, catching, popping but continued to have R medial knee pain. After returning home that
evening, he was unable to bear full weight on that extremity and began ambulating with a limp. He developed a large
degree of swelling and endorsed weakness at the knee. He was seen in our office the next morning for further evaluation.
Physical Exam: Visible and palpable effusion. Significant amount of soft tissue swelling along the medial aspect of the
patella with overlying ecchymosis. He is tender to palpation along the medial joint line as well as the MPFL. Able to
perform straight leg raise. Notable pain with full knee extension and flexion greater than 90°. There is increased translation
with anterior drawer and Lachman's compared to his left, however an endpoint is appreciable. Negative posterior drawer.
Stable to valgus and varus stresses. He does have apprehension with lateral patellar translation. Neurovascularlly intact.
Differential Diagnosis:
Patellar dislocation with MPFL tear
Soft tissue hematoma
Pes Anserine bursitis
Distal adductor tendon strain
Closed degloving
Medial meniscal tear
Grade 2 ACL
MCL tear with guarding
Test Results: Knee XR: open endplates but no other bony abnormalities. Knee MRI : fluid collection superficial to
medial patellar retinaculum consistent with closed degloving (Morel-Lavallee).
Final Diagnosis: Closed degloving injury (Morel-Lavallee)
Treatment: Initially placed in brace and on crutches for comfort. After MRI results obtained started gentle PT with
gradual progression to weight bearing and full activity. Held from sport until full swelling subsided and strength and
motion returned - approx. 1 month. Did discuss with family early the possibility of draining lesion under ultrasound but
resolved quickly with conservative treatment so this intervention was not necessary.
Outcome: Complete resolution of symptoms and cleared for full return to sports of choice.
Follow-Up: Fully cleared. Return as needed.
Lethal Leg Pain
Authors: Elizabeth Ann Hill, EAH, DO; Carly E. Day, CED, MD
Affiliation: Cleveland Clinic Foundation, Cleveland, OH
History: A 23 year old male presented to the sports medicine clinic for left leg pain of one month's duration. The pain
started in his left lateral hip and gradually spread to his left lateral knee, with no back involvement. It was a constant 5/10
and crampy in nature. The intensity increased at night, interrupted his sleep, and ultimately brought him in for an
evaluation. He denied trauma, prior injury or constitutional symptoms. He played recreational basketball, but was not able
to play during the week prior due to pain. His most recent occupation involved walking a few miles per day at a retail
store. He was sober after years of polysubstance abuse, including intravenous drugs, with continued tobacco abuse.
Family history was noncontributory.
Physical Exam: He was afebrile and vital signs were within normal limits. He was well appearing and well nourished.
Lower extremity inspection was negative for ecchymosis or swelling. He was markedly tender to palpation along the left
tensor fascia latae, iliotibial band, lateral femoral condyle and fibular head. Range of motion was normal at the hip, knee
and spine. He was neurovascularly intact with normal muscle strength, reflexes and gait. FADIR was positive for mild lateral
hip pain. Ober and FABER tests were negative.
Differential Diagnosis: Initial work-up included lumbar and left femur x-ray images that were negative for bony
pathology. He was given NSAIDS as needed for pain and a home exercise program. He was referred to a sports
chiropractor for soft tissue mobilization. Differential diagnosis for hip and knee pain remained broad, including iliotibial
band syndrome, trochanteric bursitis and less likely osteomyelitis, septic arthritis considering his drug use, labral tear and
malignancy.
Test Results: Less than one month later, he presented to the emergency department with severe low back pain, urinary
frequency and dysuria. CT of his abdomen and pelvis revealed two large soft tissue masses within the left abductor
muscles starting in the groin and extending into the pelvis, displacing the bladder. There was left inferior pubic ramus
bony inflammation without obvious destruction. These findings increased suspicion for osteosarcoma, osteoblastoma,
rhabdomyosarcoma, lymphoma, multiple myeloma and Ewing’s sarcoma. Needle core biopsies of left abductor and pelvis
soft tissue masses were positive for a high grade round cell malignancy. Staging MRI and CT showed bony metastasis to
the cervical and thoracic spine with extension into the epidural space causing mild cord compression. PET correlated with
MRI and CT findings.
Final Diagnosis: Extraosseous Ewing's Sarcoma
Treatment: He underwent chemotherapy and pelvic radiation. He was not a surgical candidate.
Outcome: Extraosseous bone sarcomas, such as Ewing's Sarcoma, account for 3.6% of soft tissue sarcomas (1). Ewing's
Sarcoma most commonly occurs in adolescents and more common in males than females (2). These tumors may present
in a different way than a primary bone tumor, with affected structures determining symptomology. These sarcomas may
be at an advanced stage at time of diagnosis. Constitutional symptoms, such as fevers and night sweats, do not generally
manifest unless metastases are present. Diagnosis is based on immunohistologic findings. Poor prognosis is associated
with extraosseous Ewing's Sarcoma and with metastatic disease at presentation (3).
Follow-Up: He suffered from neutropenic fever, intractable nausea and vomiting, weight loss, pleural effusions and pain
requiring an opioid pain pump. Further imaging showed progression of the malignancy, including metastasis to the lungs,
the skull base and surrounding musculature. He underwent radiation of the brain, chest and lumbar region. He was
hospitalized for dyspnea and transitioned to comfort care. He passed away from respiratory failure 11 months after initial
presentation. This case is a reminder to maintain a broad differential diagnosis when evaluating the athlete with hip and
knee pain.
Acute Knee Pain in a Professional Baseball Player
Authors: Ryan C. Wennell, DO; Mark Lavallee, MD; John Deitch, MD; Chong Choi, ATC; Tyler Noble, DO
Affiliation: Wellspan Sports Medicine
History: 32 year old African American professional baseball player, who was in competition, when he hit the ball and ran
towards first base. As he decelerated to touch first base, he felt his knee buckle and fell face first to the ground. He
proceeded to hop on one foot around the bases as he was unable to bear weight on the affected limb. He was placed in a
compressive Ace wrap and a locked hinged brace. Subsequently transferred to the hospital. Patient is healthy other than
ADHD on Adderall. Denied any numbness or tingling in his legs or other injuries. No loss of consciousness. No tobacco.
No illicit drugs. No history of steroid or antibiotic usage. No surgery to affect leg.
Physical Exam: Inspection: Significant swelling and edema in the right knee compared to the contralateral side.
Palpation: Both his thigh compartments as well as his calf compartments were soft. There was a palpable defect directly
inferior to the patella. No other joint pain. ROM: Inability to extend knee or do straight leg raise. Special Test: ACL, PCL,
LCL, MCL intact with good endpoints. He had 5/5 strength in dorsiflexion, plantar flexion as well as his EHL. He had 2+
dorsalis pedis and posterior tibial pulses and sensation were intact throughout the L3-S1 dermatomes. Unable to
ambulate.
Differential Diagnosis: ACL rupture, quadriceps tendon tear, patellar tendon repair, meniscal tear, collateral ligament
tear, muscular strain (grade I or II), patellar subluxation/dislocation, fracture of the patella, severe patellar tendinosis, acute
sickle cell crisis, gouty arthritis, septic joint.
Test Results: Three radiographs of the right knee do not demonstrate any fracture of dislocation. There is a notable
patellar alta. Contralateral side intrasubstance calcific tendinosis.
Final Diagnosis: Complete rupture of the right patellar tendon from interior pole of the patella. The left knee exhibited
intrasubstance patella tendinosis.
Treatment: Repair of acute right patellar tendon rupture
Outcome: 1) Weeks 0-2 had locking hinged knee brace. Acute inpatient rehabilitation to improve mobility, transfers,
ambulation with crutches, functional ADLs prior to return home. 2) Week 2-6 gentle supervised ROM progression up to
60° of flexion based on intraoperative repair tension. Using Motrin for first 6 weeks for pain. The patient is to wear the
brace locked in extension for ambulation. He can progress toward full range of motion after the 6 week mark. Expect 6-12
months to full recovery.
Follow-Up: The question for debate is in the follow-up... This case entails a common injury with uncommon
presentation. In this case, the baseball athlete was healthy without prodrome complaints when he ruptured his patellar
tendon during eccentric loading as he decelerated towards first base. His sport, as well as, his lack of prodrome complaints
makes this case unique. Given that the patient has tendinosis on the other side the question remains if he requires
therapies given it has now been identified that he is at risk for future injury.
Recurrent Knee Injury in Female Soccer Player
Authors: Kristina A. Johnson, DO; Poonam P. Thaker, MD; Brian J. Donohue, DO
Affiliation: Presence Resurrection Medical Center, Chicago, IL
History: An 18-year-old female division III college soccer player presented to the sports medicine clinic for evaluation of
an acute right knee injury sustained during a soccer game the day prior. Mechanism of this injury was twisting with a
valgus force applied with the knee in flexion. She had immediate pain and swelling of the knee but was able to bear
weight the same day of the injury. On the day of examination, she was no longer able to bear weight on the right lower
extremity. She stated that the pain was different from previous patellar subluxations and concentrated over the medial
aspect of her right knee. At the time of evaluation, she had been ambulating with crutches, taking NSAIDs and using
compression. . She had a history of lateral patellar dislocation and subluxation in the same knee during the past year and
was wearing a Shields type brace to play soccer. She had no other history of known ligamentous or bony injury to this
knee.
Physical Exam: Initial physical examination at sports medicine clinic revealed a 2+ effusion of the right knee with no
overlying skin changes or ecchymosis. Her range of motion was limited from 0-120° both actively and passively in
comparison to full painless active range of motion in the contralateral knee. The patella showed a positive J sign with a
positive apprehension and compression test. She was tender over the medial joint line. McMurray testing was negative.
Lachman testing revealed increased laxity compared to the contralateral knee. She had mild increased laxity and pain with
valgus stress. She was neurovascularly intact.
Differential Diagnosis:
1. ACL tear
2. Medial meniscal tear
3. Recurrent patellar subluxation/dislocation
4. MCL sprain
Test Results: Plain radiographs of right knee including AP/lateral/sunrise/oblique views: negative for fracture
MRI of
right knee: Small knee effusion; Sequela of recent transient lateral patellar dislocation with a comminuted nondisplaced
fracture of the inferior aspect of the medial patellar facet with a contusion of the medial femoral condyle. Medial patellar
retinaculum is intact. No articular cartilage defect.
Final Diagnosis: Recurrent patellar dislocation with comminuted nondisplaced fracture of the medial patellar facet
Treatment:
1. Non-operative conservative management including wearing Shields brace with restricted flexion
2. Closed-chain physical therapy for quad strengthening after pain and effusion improved
Outcome:
1. Continuing with rehabilitation with athletic trainers at university
2. Possible surgical intervention if continued instability
Follow-Up: Pending conclusion of rehab; unable to continue with any sport-specific activity for duration of season
My Back Hurts Too Much To Run
Authors: Jeremy A. Alland, MD; Jeanne M. Doperak, DO
Affiliation: UPMC St. Margaret Primary Care Sports Medicine, Pittsburgh, PA
History: 18-year-old female cross country runner presents with two weeks of progressively worsening right lower
back/sacral pain that began without trauma during a run. She initially had to decrease her mileage, but is currently unable
to run without pain. She has minimal pain at rest, but does intermittently wake up at night due to the pain. She denies
radicular symptoms. She has been doing physical therapy with the athletic trainer, but has not seen any improvement. No
prior history of back problems. She has a history of a tibial stress fracture a year ago while in high school. She also has iron
deficiency anemia treated with iron supplements. She does not take any other medications. She reports a well-balanced
diet without any decreases in caloric intake. She has regular menses.
Physical Exam: Well-appearing adolescent female with normal BMI. Spine with normal alignment and no swelling or
ecchymosis. Tender to palpation along the right sacrum adjacent to the sacroiliac joint, without involvement of the joint.
Flexion, extension and lateral flexion of the spine reveal full range of motion without pain. Negative straight leg raise.
Negative FABIR and FADIR tests. Hip flexion and extension with normal range of motion. Mild pain in sacral region elicited
with hip extension. No pain with hip flexion, adduction, abduction, internal or external rotation. Most severe pain
reproduced with hopping on her right leg.
Differential Diagnosis:
Sacroiliac Joint Dysfunction
Spondylolysis
Stress Fracture
Bone Tumor
Paraspinal Muscle Spasm
Pyelonephritis
Ankylosing Spondylitis
Test Results: Xray of the Pelvis: Normal
MRI of the Pelvis: Non-displaced right sacral stress fracture with an obliquely
oriented fracture line extending from the right sacroiliac joint to the inferomedial right sacral ala. Extensive surrounding
marrow edema is noted. There is a small amount of periosteal edema at the anterior sacral cortex.
CBC, iron studies,
CMP, Vitamin D levels ordered after MRI result returned. Results in process at time of submission.
Final Diagnosis: Right Sacral Stress Fracture
Treatment: Currently ongoing. Patient is restricted from participating in painful activities, including running.
to complete food journal for 1 week and follow up.
Outcome: Pending at time of submission.
Follow-Up: Pending at time of submission. Optimistic for a full return to sport.
Patient
Shouldering the Load of CrossFit: An Unusual Etiology of Shoulder Pain
Authors: Thomas A. Starnes, MD; Bryan R. Prine, MD
Affiliation: University of Florida College of Medicine, Gainesville, FL
History: A twelve-year-old female presented to an outpatient appointment with a chief complaint of delayed swelling
and pain in her right shoulder after CrossFit workouts. Her symptoms had been present for three weeks. Her upper body
exercise regimen included butterfly pullups, handstand pushups, and overhead presses. Her pain was dull in nature, rated
as 5/10 in severity during her exercise regimen, and was alleviated by rest. She had no previous injury or surgery affecting
the right shoulder.
Physical Exam: The patient was in no distress and appeared well-developed and well-nourished. There were no obvious
abnormalities of the right shoulder on inspection. There was tenderness to palpation over the right anterolateral shoulder
in the region of the acromioclavicular joint. Active range of motion was preserved without any difficulty on the part of the
patient. Muscular strength was intact. Special testing maneuvers including apprehension, sulcus, cross-arm, drop arm,
Hawkins, and Neer were negative. The right upper extremity was neurologically intact with intact distal pulses.
Differential Diagnosis: Acromioclavicular joint sprain, traction apophysitis, distal clavicular osteolysis, distal clavicular
fracture, pectoralis minor tendonopathy, biceps tendonitis, labral tear, cervical radiculopathy, neoplasm
Test Results:
1. Anterior-posterior, axillary, and lateral views X-rays of the right shoulder showed an expansile lucent lesion in the distal
right clavicle.
2. MRI of the right shoulder with and without contrast showed a multiloculated expansile cystic lesion within the distal
clavicle.
Final Diagnosis: Aneurysmal bone cyst
Treatment: The patient was referred to orthopedic oncology for further evaluation after the results of the MRI were
obtained. She eventually underwent open biopsy of the right distal clavicle followed by curettage, argon beam
coagulation, and placement of allograft cancellous bone.
Outcome: The patient did well without complications after resection of the aneurysmal bone cyst.
Follow-Up: One month after surgery, the patient was allowed to remove her arm from a sling to begin passive and active
range of motion exercises. She wanted to begin more strenuous exercise but was encouraged to allow more time for
healing. At a six-week follow-up appointment, she was cleared for right upper extremity weightbearing as tolerated and a
slow progression to her previous activity level.
Acute Non-Contact Hip Pain in a Hemophiliac Football Player
Authors: Kevin Miller, MD; Joel Kary, MD
Affiliation: St. Vincent Sports Performance, St. Vincent Family Medicine Residency Program, Indianapolis, IN
History: 18-year-old male high school senior football player with known moderate (4%) Hemophilia A who initially
presented to the ED with sharp, severe, right hip pain after weightlifting (squatting) earlier that day In the ER, he had a
normal hip X-ray and was diagnosed with a hip strain and discharged However, the pain continued to worsen and he
came to the office one week later to be evaluated His pain was worse with lying flat, relieved with sitting upright, and the
pain had progressed to the point of not being able to bear weight on his right leg No fevers, chills, numbness, tingling,
swelling, edema, or skin changes in the leg No history of any bleeding episodes Of note, he states that he had much
lower intensity right hip pain that started during football practice 3 weeks ago Pain slowly resolved and he had resumed
full physical activity about a week before this current injury.
Physical Exam: Gait: unable to bear weight on right leg. Inspection Right hip: No swelling, deformity, or bruising.
AROM: Full Flexion, limited extension and external rotation due to pain. Tenderness along deep hip flexor muscles, no
bony tenderness or tenderness in quadriceps muscles. Weakness with hip flexion, otherwise strength intact. Normal
pulses and sensation.
Differential Diagnosis: Femur or acetabular fracture; Stress reaction in femoral head/neck; Hip labral tear; Legg-CalvePerthes disease; Transient synovitis; Slipped capital femoral epiphysis; Muscle strain (Iliopsoas vs. Rectus femoris);
Muscular hematoma; Hemarthrosis; Septic arthritis
Test Results: MRI revealed a 2.8cm hematoma in the right iliopsoas muscle that tracked down the muscle and right hip
effusion consistent with hemarthrosis.
Final Diagnosis: Iliopsoas strain with hematoma and hip hemarthrosis
Treatment: He was admitted to the hospital by his hematologist for observation, pain control with IV narcotics, and for
infusions of antihemophilic factor (recombinant) He required a total of 2 infusions to return his Factor VIII levels to normal
range in the hospital Discharged from the hospital with close follow-up by the hematologist and required 10 additional
infusions He was prescribed physical therapy and avoidance of any sports activities and weight lifting until pain fully
resolved.
Outcome: Gradual improvement of pain and progression to normal ROM and gait.
Follow-Up: Has not yet returned to full activity or weight training Last evaluated at a 6-week follow-up and still has
tenderness and pain with hip extension and external rotation He is planning on playing basketball this season with the
hope of earning a Division I scholarship.
Ankle Sprain That Didn't Get Better
Authors: Timothy James Von Fange, MD
Affiliation: St. Vincent Sports Performance Indianapolis, IN
History: 50 year old white female with past history significant for only for anemia presents for an inversion ankle injury
that occurred while walking in mid August. The patient had previously been seen about a month prior to her injury for
plantar fascial and midfoot pain with running. Workup at that time was negative for more severe injury and she responded
very well to arch support inserts in her running shoes and was unrestricted at time of her acute injury. She was evaluated
day after injury. She is an active runner although milage had been recently limited due to the mentioned chronic foot issue
She did have a sinusitis/bronchitis in last spring that was treated with steroid pak and nasal steroids.
Physical Exam: Initial evaluation August 18: Marked swelling of ankle and foot. Pain localized mostly to lateral ankle
and 5th MT. ROM globabally limited due to swelling. Two weeks later: Improved pain and swelling and ROM - but still a
lot of pain with weight bearing out of boot. September 18: Markedly improved pain, swelling, ROM. Tolerates full weight
bearing out of boot. Advancing activities out of boot. September 24: Returns due to return of severe pain and moderate
swelling anterior ankle with return to ADLs and minimal attempt to jog. MRI ordered at this time.
Differential Diagnosis:
Occult Fracture
Osteochondral Defect
RSD/CPRS
Test Results: XR Foot and Ankle at initial evaluation - normal ankle XR, small avulsion of base of 5th MT on foot XR MRI
Ankle 10/3/14 Oblique fracture of talus that runs through the posterior subtalar joint. Decreased trabeculation of talus.
Plantar fasciitis. Talar and cuboid bone contusion. (The MRI images of the talus are quite impressive - it is not subtle.) CT
Ankle 10/31/14 Healed talar fracture - still trying to get CD copy of the images
Final Diagnosis: Talar insufficiency fracture extending into the subtalar joint in conjunction with very small 5th MT
avulsion from an inversion injury. It is believed underlying osteopenia / osteoporosis contributed to her condition
although she has not been formally dia
Treatment: She was weightbearing in a short walking boot for the first month after the injury. She could not advance out
of boot after one month and was put back in boot while tests were pending. She ended up spending the better part of 10
weeks mostly in a boot and restricted from activity beyond ADLs. Insurance restrictions made getting imaging a difficult
process and thus treatment timelines were a bit delayed.
Outcome: She is now out of boot tolerating ADLs with no pain
Follow-Up: Plan to pursue osteoporosis workup and mechanical evaluation prior to return to running. Depending on the
actual CT images - will likely have review with foot/ankle ortho to guide an long term prognosis following fracture.
Shoulder Pain and Selective Muscle Atrophy in a 33-Year Old CrossFit Enthusiast
Authors: Michael Saulle, DO; Ethan Rand, MD; C. David Lin, MD
Affiliation: New York- Presbyterian Hospital, University Hospital of Columbia and Cornell, Department of
Rehabilitation Medicine, New York, NY
History: A 33 year-old male CrossFit enthusiast, with a history of a partial right rotator cuff tear presented to the
outpatient office with 3-months of worsening posterior right shoulder pain. He denied any trauma or inciting event. He
described his pain as dull, aching, and 6/10 in severity. Symptoms improved with rest and were exacerbated by CrossFit
exercises (e.g. lowering weights from overhead and kettlebell swings). He denied radiation of the pain, numbness or
paresthesias. He noted weakness with shoulder external rotation. Previous treatments included non-steroidal antiinflammatory medications, muscle relaxants, and a course of physical therapy. All treatments only provided him with
modest improvement of pain and weakness. He reported normal electrodiagnostic testing that was completed two years
prior to this presentation. Prior MRI, from the same time, showed atrophy primarily of the supraspinatus muscle, with
modest wasting of the infraspinatus muscle, suspicious for denervation injury Even with the ongoing pain, he participated
in rigorous exercise regimen five days per week, which he started 10 years ago.
Physical Exam: On inspection, he had well-defined upper extremity musculature, with significant atrophy of the right
infraspinatus muscle and relative hypertrophy of the teres minor muscle. No focal muscular or bone tenderness was
noted. Full active range of motion was noted throughout upper extremities. Neurologically, sensation, strength and
reflexes were normal throughout bilateral upper extremities, except for 4/5 strength with shoulder external rotation.
Differential Diagnosis: Right posterior shoulder pain with isolated atrophy of the infraspinatus muscle was likely due to
either a complete tear of the infraspinatus tendon, or a suprascapular neuropathy Additionally, muscular strain, rotator
tendinopathy, labral tear, space occupying lesions, subacromial impingement syndrome, brachial plexopathy, or cervical
radiculopathy could contribute to his symptoms.
Test Results: Electromyography and nerve conduction studies were consistent with a severe right suprascapular
neuropathy moreso affecting infraspinatus than supraspinatus Non-contrast MRI of right shoulder demonstrated an
enlarged, edematous suprascapular nerve due to entrapment by prominent vessels (likely representing varicies) at the
suprascapular notch; associated denervation atrophy of the infraspinatus, also with associated focal full thickness partial
width tear of supraspinatus tendon; partial thickness tear of infraspinatus tendon; and a labral tear with an associated
paralabral cyst MR arthrogram noted atrophy of supraspinatus and infraspinatus with denervation edema. No evidence of
a paralabral cyst or other mass lesion in the spinoglenoid or suprascapular notches was seen.
Final Diagnosis: Based on the patient’s history, physical exam, relevant imaging and tests, he was diagnosed with
suprascapular neuropathy secondary to vascular compression by suprascapular varicies. He was also found to have
progression of rotator cuff and glenohum
Treatment: Four months following initial presentation, he underwent orthopedic surgery consisting of rotator cuff
debridement, labral repair, and suprascapular ligament release. There were no intra-operative or post-operative
complications. Post-operatively, physical therapy was again initiated.
Outcome: Three months after the operation, he continues with physical therapy twice weekly (reduced due to progress),
as well as working with an athletic trainer in a commercial gym three to four times per week. He reports being relatively
pain free in his shoulder and is regaining strength and muscle bulk appropriately in supraspinatus and infraspinatus
muscles, though he still notes significant muscle atrophy. He hopes to return to his previous activity and exercise level
and resume CrossFit training.
Follow-Up: The patient is scheduled for follow-up at 6 months, both with the surgeon and physiatrist.
The Lateral Femoral Condyle Cartilage Fracture Could be Missed if the Imaging Study
Substitutes the Physical Exam
Authors: Pegah Dehghan, MD; Farshad Adib, MD; Ghazaleh Rahbari, MD
Affiliation: Montefiore Medical Center, Bronx, NY
History: The patient is a 26 year old male who presented with progressive, throbbing pain on the right knee for one
month. The pain started after he fell from the stairs and landed on the right leg. Initially, he was evaluated at a emergency
department for the right knee pain. The imaging study was done which did not show any fracture and dislocation. Patient
was discharged home with diagnosis of knee sprain. The patient states that the pain is sever enough to limit his
ambulation and daily activity. The pain was associated with weakness, instability and swelling of the right knee.
Physical Exam: Examination was notable for tenderness of medial and lateral joint, swelling and limited range of motion
(ROM) of right knee from -5 to 50°. The patient was guarded to anterior and posterior drawer test. We were unable to
perform McMurray test due to pain. The knees were stable on varus and valgus stress test. The Grind test was negative.
Differential Diagnosis:
1.
2.
3.
4.
Medial and lateral Meniscus tear
Cartilage damage
Ligament injury
Patellar dislocation
Test Results: Right knee X-ray did not show any fracture, dislocation or destructive bony lesion. Subsequent right knee
MRI findings were as follows: 1. Contusion of the inferomedial patella corresponding with the impaction fracture injury at
the lateral femoral condyle 2. A large displaced osteochondral fracture involving the central weight-bearing surface of the
lateral femoral condyle with large osteochondral fragment within the medial suprapatellar joint. 3. Fissures at the patellar
articular cartilage 4. High-grade injury of the medial patellofemoral ligament and medial parapatellar retinaculum 5.
Acute patellar dislocation relocation injury
Final Diagnosis: In the follow up visit after MRI of the right knee, the patient stated that he still has the sever pain
accompanied with cracking of the right knee. Examination revealed pain and swelling of the right knee. Also, decreased 5°
extension and 20° flexion on the right knee compared to the other side was noted. These findings suggested patellar
dislocation with lateral femoral condyle cartilage fracture which had been confirmed with MRI .
Treatment: After review of treatment options, he elected arthroscopic right knee surgery with open reduction and
internal fixation of lateral femoral condyle, cartilage fracture lateral release and chondroplasty of the patella and the lateral
tibial plateau. The procedure was performed in April 2014 without complication. Patient tolerated procedure well and
started physical therapy right after surgery.
Outcome: The patient presented without any assistance in the out patient clinic for post operation follow up. The patient
reported significant improvement in pain and ROM of right knee. On physical examination of the right knee, skin is intact,
incisions were well-healed. The patient had no tenderness on palpation of the joint line. The patella was tracking midline.
The right knee had full extension and flexion, comparing to the other side. The knee was stable in all the directions. The
following X-ray of right knee, AP, lateral and patellar view, showed healing of the cartilage damage.
Follow-Up: Currently, the patient is able to do all daily activity independently. He starts his sport activities including
jugging.
A Hit to the Face
Authors: Patrick Jenkins, III, MD; Owen Speer, DO
Affiliation: Palmetto Health-USC Family Medicine Residency, Columbia, SC
History: A 20-year-old female Division I collegiate softball pitcher presented after being struck in the face by a softball
after it was hit back at her in practice. The initial survey revealed an awake and alert student athlete, bloody and painful
mouth, and an upper lip laceration. EMS was called and transported her to the emergency department. She complained of
mouth pain but denied loss of consciousness, headache, focal numbness, weakness, or tingling. Her past history is
significant for two concussions in the past year.
Physical Exam: Vital Signs: temperature 98.6°F, blood pressure 132/68, heart rate 73, respiratory rate 18, oxygen
saturation 100% on room air General Appearance: awake, alert, non-ill appearing Neurologic: alert and oriented x 3,
cranial nerves II-XII intact, strength and sensation intact, no focal deficits Eyes: pupils equal round and reactive to light,
extraocular movements intact HENT: normocephalic, nares clear, oropharynx without erythema or exudates, mucous
membranes moist, laceration through entire thickness involving vermillion border of left upper lift, all teeth present, teeth
9 and 10 displaced inward. Musculoskeletal: cervical spine was non-tender with full range of motion, 5/5 strength
Differential Diagnosis:
Tooth/teeth luxation
Tooth/teeth avulsion
Lip laceration
Facial bone fracture
Concussion
Cervical spine fracture
Intracranial bleed
Test Results: CT Cervical Spine: No acute cervical spine fracture
hemorrhage
CT Head w/o IV Contrast: No acute intracranial
CT Facial Bones w/o IV Contrast: Alveolar process fracture involving teeth 9/10
Final Diagnosis: Alveolar fracture involving teeth 9 and 10 with luxation Full thickness lip laceration Possible
concussion
Treatment: Dentistry was consulted in the Emergency Department for concern of the two displaced teeth. Dentistry
repositioned the teeth with finger pressure and an arch bar was placed. The lip laceration was repaired. The intraoral
portion of the laceration was sutured with chromic gut. The extraoral portion of the laceration was closed with
monofilament suture.
Outcome: The student-athlete was discharged from the emergency department with instructions to follow up with
dentistry in 2-3 days. Concussion evaluation ruled out this diagnosis. The arch bar was removed six weeks later, the
fracture healed, and the teeth remained viable.
Follow-Up: She was subsequently cleared to practice and began doing so three days after the injury. She was required to
wear a face cage while pitching in practice and in competition for the remainder of the season.
Doctor, I Think There's Something Wrong With My Knee Replacement
Authors: Peter H MacArthur, MD; Thomas Fleeter, MD; Thomas Howard, MD; Garry Ho, MD
Affiliation: VCU - Fairfax Family Practice, Fairfax, VA
History: A 77 year old female with a history of a total replacement of her left knee in 2007 presented with a five day
history of progressively worsening left knee pain. She described her pain as anterior, stabbing, and constant. It was
moderate in intensity, worse with ambulation, and improved slightly with rest. There was no history of trauma or other
known mechanism of injury. She denied locking, weakness, instability, swelling, radiation of pain, or fever. A complete
review of systems was otherwise negative. Her past medical history was significant for coronary artery disease, CHF,
hyperlipidemia, GERD, gout, and OA. In addition to her knee replacement, her past surgical history also included coronary
stent placement in 2007 and aortic valve replacement in 2013. Her medications included Bisoprolol Fumarate, Apixaban,
Ranitidine, Nitrostat, Lansoprazole, bASA, Calcium Carbonate-Vitamin D, Rosuvastatin, Furosemide, and Potassium
Chloride. Her family history was non-contributory. She denied alcohol abuse and does not smoke.
Physical Exam: Physical examination revealed an obese female with an antalgic gait. Vital signs were normal. Her left
knee showed no overlying skin changes or appreciable deformity. She had mild swelling and trace effusion. Her alignment
was neutral. There was no tenderness to palpation. Active and passive ranges of motion were both 0-130° and strength
was normal. Her neurovascular exam showed normal sensation, reflexes, and distal pulses. Ligamentous evaluation, special
tests, and the rest of the exam were normal.
Differential Diagnosis: Aseptic loosening of her prosthesis; periprosthetic fracture; arthrofibrosis; osteolysis; bone cyst;
malignancy; osteomyelitis; PVNS; non-malignant tumor; sympathetic dystrophy; septic arthritis; referred pain from the hip
or spine; crystal-induced inflammatory arthropathy.
Test Results: Initial plain radiography of her left knee showed a well-positioned total knee replacement with no evidence
of loosening. An ill-defined radiolucent lesion was seen in the supracondylar femur just proximal to and abutting her
prosthesis. A follow-up CT scan without contrast showed a lytic lesion in the distal femur corresponding to the radiolucent
lesion seen on plain films. ESR and CRP were within normal limits. MRI with contrast revealed a heterogeneously
enhancing, aggressive-appearing lytic lesion in the distal femoral metaphysis extending to the femoral component of the
knee arthroplasty. There was evidence of cortical destruction anterolaterally with minimal extraosseous extension. CT
chest/abdomen/pelvis revealed no evidence of metastatic disease. She was referred to orthopedic oncology and a bone
biopsy was consistent with spindle cell sarcoma and possible leiomyosarcoma.
Final Diagnosis: Histologic examination of the resected tumor showed a fascicular pattern of growth with low grade
spindle cells and evidence of pleomorphism. Smooth muscle markers were negative. Her cancer was categorized as a
grade 2 sarcoma, consistent with malignant f
Treatment: Surgery was performed with radical, wide resection of her distal femur and reconstruction of the knee joint
and femur with segmental endoprosthesis. She did not undergo any radiation or chemotherapy.
Outcome: The patient is recovering from surgery and doing well in the post-operative period. All of her soft tissue and
bone margins were clear and there was no evidence of lymphatic or vascular invasion. The incidence of primary neoplasm
of the bone accounts for 0.2% of cancers. MFH accounts for less than 2% of these primary malignant bone lesions.
Follow-Up: The patient has a good chance for near full recovery. However, recurrence is not uncommon and she will
require regular follow-up for surveillance.
Trick Shots on a Trick Knee
Authors: Daren Molina, MD; Ralph Curtis, MD; Eliot Young, MD
Affiliation: Christus Santa Rosa Primary Care Sports Medicine Fellowship Program, San Antonio, TX
History: A sophomore basketball player experienced a seemingly mild left inversion ankle sprain and was eventually
evaluated in August by her primary care provider a month later due to continued lower leg pain. At that time, the athlete
was tentatively diagnosed with "shin splints" or Medial Tibial Stress Syndrome, as most of her symptoms were of left lower
leg pain. The primary care provider her sent her to physical therapy 3 times a week for one month, and the athlete's pain
seemed to resolve. As basketball workouts intensified several months later, her pain returned and she began to experience
a "popping" sensation on the outside of her knee with occasional tingling down the anterior aspect of her lower leg.
Patient's physical therapist continued to work with her and used a variety of modalities and various taping methods with
only minimal alleviation of symptoms. It was at this time that the athlete then came to see the sports medicine fellow at
her high school's training room for a second opinion.
Physical Exam: Left Knee and lower leg: No effusion or soft tissue swelling, no ecchymosis. Knee Range of Motion
active 0-135 Tender to palpation over fibular head and anterior tibiofibular joint Negative Lachmans, negative anterior and
posterior drawer, negative McMurray's test, no laxity or pain or varus and valgus testing. Mild antero-posterior laxity of
tibiofibular joint with some pain with movement (positive laxity of joint on contralateral leg, but without pain) Strength:
5/5 knee extension and flexion, 5/5 ankle plantarflexion/dorsiflexion/eversion/inversion Sensation over lower leg intact
to light touch.
Differential Diagnosis:
1.
2.
3.
4.
Syndesmotic Injury
Lateral Meniscus Injury
Patellofemoral Pain Syndrome
Proximal Tibiofibular Joint Instability
Test Results: X-ray, Left tibia/fibula: no obvious acute fracture or other osseous abnormalities.
MRI, left knee: mild
reactive fluid around tibiofibular joint, otherwise normal.
Final Diagnosis: Proximal tibiofibular Joint Instability
Treatment: After a review of the literature and consultation with several of our orthopedic surgeons, several options
were discussed. There have been only a handful of cases in the literature of symptomatic proximal tibiofibular joint
instability, and these were treated with surgical fixation. The decision was made to exhaust conservative measures before
considering surgical options. A cortisone injection was placed in the proximal tibiofibular joint and was followed by a
month of rest from sport. Physical therapy was also utilized to help strengthen the muscles of the lower leg.
Outcome: Patient was seen in initial 1 week follow up and still was experiencing some soreness from the injection. After
completion of one month of rest and rehabilitation, she was no longer experiencing pain with light activity and the
decision was made to return her to basketball.
Follow-Up: Patient was allowed to progress as tolerated back into her normal basketball workouts once she was deemed
pain free. She was encouraged to continue lower extremity strength and stretching rehabilitation exercises at home and
under the direction of her high school trainer throughout the remainder of basketball season.
Unusual Cause of Knee Pain in a 16 Year Old Soccer Player
Authors: Ronald Tsao, MD
Affiliation: Midwest Orthopaedics at Rush University Medical Center, Chicago, IL
History: A 16 year old male high school soccer player presents two days after an injury to his right knee. During a game,
he attempted to kick a ball and got his leg caught in between two defenders. He immediately had pain in his knee and fell
to the ground. He thought he could have twisted his knee. He was evaluated by an athletic trainer on the field who
thought he may have sustained an ACL/LCL injury. He was unable to bear weight immediately after the injury. He was
made non-weight bearing on crutches, iced, and elevated the leg over the next 36 hours.
Physical Exam: No ecchymosis. There is a trace knee effusion. Distal thigh circumference on the right is 3 cm larger than
the left. He is diffusely tender to palpation over the right distal thigh. No tenderness over the proximal fibula or tibia. He
lacks 15° of full extension and can flex to 80°. He is tender over the LCL, medial joint line, and medial femoral chondyle.
Lachman test is negative. He has pain with varus and valgus stressing but has firm endpoints.& Posterior drawer and
McMurray tests were limited due to pain and lack of motion. Distal pulses were intact.
Differential Diagnosis:
1.
2.
3.
4.
5.
6.
ACL sprain/tear
LCL sprain/tear
Mensicus tear
Bony contusion of femur
Thigh hematoma
Femur fracture
Test Results: X-ray Right Knee/Thigh -There is slight widening of the lateral femoral physis. Otherwise, the growth plates
appear to be nearly closed. There is a faint linear lucency extending from the mid femoral physis toward the femoral
metaphysis medially. There are no cortical breaks. MRI Right Knee -There is a Salter Harris type 2 fracture of the lateral
aspect of the distal femur with associated subperiosteal hematoma and surrounding hemorrhage and edema within the
soft tissues.
Final Diagnosis: Salter-Harris fracture type II of distal femur
Treatment: He was placed in a long leg cast with non-weight bearing status, on crutches. Repeat radiographs at two
weeks showed no displacement of the fracture.
Outcome: After four weeks, the cast was removed and repeat radiographs demonstrated callus formation at the site of
the fracture. He had minimal tenderness over the distal femur and his thigh swelling had resolved. He was instructed to
continue with his non-weight bearing status but was changed into a drop-lock long brace open from 0-60° to allow for
motion of the joint. Two weeks later, he had improved motion and was pain free. He was advanced to partial weight
bearing and began physical therapy for range of motion and light strengthening exercises. He was slowly advanced to full
jogging and runing over the next four weeks. He then started a graduated return to soccer program and returned to full
soccer activities after 3 weeks.
Follow-Up: Total time to return to play from initiaion of treatment was 13 weeks. He is doing well, playing soccer
without restrictions.
Blunt Abdominal Trauma in a NCAA Division 1 Football Player
Authors: Andrew Saleh, MD, MPH; Valerie Cothran, MD
Affiliation: University of Maryland, Baltimore, MD
History: 20 year-old male Division 1 football wide receiver with known history of lumbar spondyloysis presents with
complaint of LUQ abdominal pain and left sided back pain that began after a road football game. During the third quarter
of the game he had taken a hard hit near the left costovertebral angle while stretching out in attempt to score a
touchdown. He was evaluated on the field by the athletic trainer and orthopedic surgeon and stated to them that he "had
the wind knocked out of him". He was able to return on the next offensive series and completed the game without any
other issues. After the game he began to complain of left sided pain and stated he noticed blood in his urine while
showering. On examination he was noted to have mild left CVA pain without any other abnormalities. He was unable
produce another urine sample and the determination was made to continue to monitor him and reevaluate him after the
bus ride back to the home training room. Upon return he stated that he now had new LUQ abdominal pain and that his
side pain had slightly improved. He was transported to the ER for STAT imaging and further work up.
Physical Exam: Gen: well nourished, alert and oriented, in no apparent distress Neck: No tenderness to palpation, full
range of motion Chest/CV: RRR, no murmurs, negative spring test, negative step off Lungs: Clear to auscultation,
respirations are non-labored, breath sounds equal, symmetrical chest wall expansion, no chest wall tenderness Abdomen:
Nondistended, + LUQ tenderness, no rebound, + guarding, no flank pain, No CVA tenderness
Differential Diagnosis:
1) Renal Contusion/Laceration
2) Splenic Rupture
3) Rib Contusion/Fracture
4) Vertebral Contrusion/Fracture
Test Results: Urinalysis: Cloudy appearance, 3+ blood, 2+ Protein, 2+ protein, 1+ ketone ,Nitrite +, Biliribin neg, Urine
WBC 6-10/hpf, Urine RBC &gt;50/hpf, SG=1.020 Abdominal US (FAST Scan): No evidence of free intraperitoneal fluid
CT Abdomen/Pelvis: Grade 2 left renal laceration. No CT evidence of collecting system injury or pseudoaneurism. Bilateral
Pars defect of L3 vertebral body. CTA: No evidence or renal artery injury
Final Diagnosis: Grade II Renal Laceration
Treatment: Patient admitted to trauma service for observation, IV hydration, monitoring of urine output and pain control
Outcome: Discharged after 48 hours, Athlete remains pain free
Follow-Up: Follow up imaging 2.5 weeks post injury Return to play status to be determined based on sympton
resolution and image findings. Further followup currently pending
Exertional Leg Pain in a Recreational Athlete
Authors: Ryan S. Foster, MD; Joel Shaw, MD; Darrin Bright, MD
Affiliation: Grant Medical Center-Sports Medicine Fellowship, Columbus, OH
History: The patient is a 20 year old female with no significant past medical history who presented to our outpatient
office for right leg pain. She attends a local university and is a recreational runner. During her workouts over the last
several months, she would develop a tightness/pain in the back of her calf which then progresses to complete foot
numbness. Shortly after exercise, she would have complete resolution of her symptoms. Similar symptoms developed at
rest in particular when she had her right leg crossed over her left. Her symptoms were not improving a month ago so she
was seen at the university’s student health center. They were concerned about exertional compartment syndrome so she
was referred to our sports medicine office. Prior to this year, she had had no similar symptoms despite exercising at
roughly the same intensity throughout the year.
Physical Exam: The patient was in no acute distress, and sitting comfortably in her chair. Her gait examination was
unremarkable. She had full range of motion of her right hip, knee, and ankle. Her right lower extremity strength was 5/5
with all tests. She had no pain with any sort of resistance testing. She had normal muscle tone in her calves. Palpation of
bilateral ankles and knees demonstrated no point tenderness. Her PT and DP pulses were 2+ bilaterally. She had full
sensation of the right lower extremity.
Differential Diagnosis:
1. Because her symptoms were mainly present during exercise, exertional compartment syndrome was suspected.
2. Peripheral vascular disease is possible although her young age essentially precludes this diagnosis.
3. A peripheral neuropathy is possible although less likely.
4. Medial tibial stress syndrome and stress fractures would also cause an exertional-type pain.
5. Muscle strains would also be on the differential, albeit less likely.
6. The specific finding of symptom onset with her right leg crossed over her left led us to suspect popliteal artery
entrapment more than other entities.
Test Results: She underwent bilateral lower extremity angiogram demonstrated severe right and moderate-to-severe left
popliteal artery entrapment. Subsequent MRI of the right knee with and without contrast was performed to investigate
her anatomy. It was initially interpreted as an anomalous slip of the medial head gastrocnemius causing slight lateral
deviation of the popliteal artery. However, in discussing with various musculoskeletal radiologists at another facility, it was
felt that the gastrocnemius muscle was actually benign in its appearance and alignment and showed no evidence of a slip.
Hence, there is ongoing debate as to whether her muscle anatomy was the true cause of her entrapment.
Final Diagnosis: Based on the angiogram revealing severe right popliteal artery entrapment which coincided with her
symptoms, it was concluded that her symptoms were from popliteal artery entrapment syndrome. The etiology of the
entrapment, however, remains to be determin
Treatment: The patient underwent vascular surgery at another facility the week prior to the AMSSM case abstract
deadline and the details of her surgical operation had yet to be received. It was known that she underwent bilateral
decompression of the popliteal arteries (despite having only symptoms on the right), but the remaining description of the
surgical findings were not available yet.
Outcome: She was 1 week post-op at the submission of this case abstract. Her full outcome from the surgery was not
known at the time of case submission, although it was known she had an uneventful first few days following the surgery.
Follow-Up: As she is still recovering from surgery, any return to activity decisions were still pending. She has follow up
with our primary care sports medicine office in 2 weeks for further assessment.
Bilateral Knee Pain in an Obese Middle-Aged Female
Authors: Sarah Lucey, MD; Laura Owczarek, MD
Affiliation: Henry Ford Hospital, Detroit, MI
History: 45 year-old female presented to the emergency department with bilateral lower extremity pain. Patient reported
that she was outside running when she suddenly fell and was subsequently unable to stand or ambulate. She localized
pain to anterior knees bilaterally and described it as constant and throbbing in nature. She noted that her knees felt
unstable. Patient denied any previous medical history, medication use, tearing or popping sensation, numbness or
tingling, or previous knee injuries/surgeries. PMH, PSH, Social History, Family History, and ROS were unremarkable.
Physical Exam: Patient was alert, awake, and uncomfortable appearing. She was 5' 10" inches tall and weighed 138 kg
with a BMI of 43. Cardiovascular, respiratory, abdominal and neurologic exams were unremarkable. On examination of
the lower extremities, bilateral knees were moderately swollen with a small abrasion noted over left anterior knee. There
was tenderness to palpation over anterior knees with a palpable defect appreciated at patellar tendons bilaterally. Patient
was unable to perform straight leg raise or active knee extension. No laxity appreciated with varus or valgus stress.
Anterior/posterior drawer and Lachman's tests were negative. Full strength noted of bilateral EHL, FHL, TA, and GS.
Neurovascular exam was intact
Differential Diagnosis:
Fracture - patella, femoral condyle, tibial spine/tuberosity, tibial plateau
Patella/knee dislocation
Quadriceps tendon rupture
Patellar tendon rupture
Meniscal injury
ACL/PCL tear
Test Results: Bilateral knee radiographs: -Patella alta with poor definition of the patellar tendon -No osseous fracture
or dislocation identified Bedside knee ultrasound: -Full thickness tears of the patellar tendon bilaterally Formal knee
ultrasound - Left knee: -Full thickness tear of the patellar tendon from its origin on the patella with hematoma formation
within defect -Defect 2.4 cm -No discrete avulsion fragment -Left quadriceps tendon intact Right knee: -Full thickness
tear of the patellar tendon from its origin on the patella -Defect 3 cm -No discrete avulsion fragment -Right
quadriceps tendon intact
Final Diagnosis: Bilateral patellar tendon rupture
Treatment: Patient was placed in bilateral knee immobilizers and provided with ice and analgesia in the emergency
department. Orthopedics was consulted and patient underwent primary surgical repair the next day, where the patellar
tendon was found to be avulsed from the inferior pole of the patella bilaterally. Patient was placed in long-leg casts and
discharged to an inpatient rehab facility.
Outcome: Following surgical repair, the patient successfully participated in inpatient physical therapy for six weeks and
was discharged with post-operative knee braces and a walker for assistance.
Follow-Up: At 6 months post-operatively, patient was still having persistent pain of anterior knees with movement and
ambulation. Follow-up MRI demonstrated lateral and patellofemoral compartment osteoarthritis bilaterally with intact
repaired patellar tendons. Patient has received bilateral intra-articular steroid injections with excellent relief of
symptoms, and she is currently undergoing outpatient physical therapy for strengthening of bilateral lower extremities.
Patient is to return to previous level of activity as tolerated.
Intermittent Lower Extremity Paresthesias in a 22 Year Old Recreational Athlete
Authors: Carrie Janiski, DO; Robert Norris, MD
Affiliation: Michigan State University Sports Medicine, East Lansing, MI
History: A 22YO healthy female presented to our sports medicine clinic with history of intermittent bilateral lower extremity
paresthesias Active in running and CrossFit after having played softball, volleyball and basketball in high school without significant
trauma, incident or injury. First incidence of these paresthesias was in high school, she awoke with both legs "from the waist down"
feeling tingly but was able to use them and do normal activities She had two more episodes like this while in high school, each time
resolving spontaneously within a few days. She was then asymptomatic until two years ago, while at college, she awoke with another
episode of bilateral leg paresthesias; this time sought care with chiropractor as symptoms did not abate after a couple days She had
subsequently experienced another four episodes since that time, each one involving both legs For these episodes, she was able to
obtain complete resolution of symptoms following a course of chiropractic treatments and, for her most recent resolved episode, a
cupping massage of her back At the time of her initial appointment in our clinic, she had been experiencing paresthesias at the lateral
border and plantar surfaces of both feet for approximately three weeks. In addition to the intermittent lower extremity paresthesias,
she did note occasional ulnar paresthesias bilaterally since beginning her job at a credit union, none lasting more than a day and
without weakness or discomfort. She did have one episode of headache in the preceding month for which she sought care at an
emergency department for the "worst headache I've ever had" She denied other personal or family history of neurologic or
autoimmune disease. She had been unable to identify any triggers for these episodes among her activities, specific motions, exertion,
illness, stress, trauma, diet, environment, sexual exposure, drug use, excessive alcohol use or travel She completed two courses of
physical therapy for piriformis syndrome in the past, which she stated was not helpful and she did not continue with any targeted home
exercise program or modalities.
Physical Exam: On physical examination, no deformity was noted with normal posture and gait, bilateral knees with patellofemoral
crepitus and mildly increase Q angle She was noted to have a left short leg. No clubbing, cyanosis, effusion or edema was noted of the
lower extremities with normal full range of motion of all joints She was neurologically intact with no focal deficits, facial droop, antalgic
or ataxic gait. Her cranial nerves, myotomes, dermatomes and reflexes were intact and equal She demonstrated no upper motor neuron
signs, chorea, tremor or cog-wheeling with intact coordination.
Differential Diagnosis:
Piriformis syndrome or other peripheral sciatic nerve impingement
hereditary neuropathy with liability to pressure palsy
multiple sclerosis or other demyelinating conditions
lumbosacral radiculopathy due to disc herniation
spinal stenosis
spondylosis or mass
conversion or somatization disorder
Lyme disease
vasculitis
anemia
vitamin deficiency
hypothyroidism
Test Results: Laboratory studies done within the previous year were reviewed and included a normal CBC, CMP, TSH, iron studies and
vitamin D level. An EMG of the lower extremities was ordered and demonstrated no evidence of radiculopathy An MRI of the brain was
ordered and revealed "too numerous to count" lesions within the brain highly suggestive of a demyelinating process such as multiple
sclerosis.
Final Diagnosis: Multiple Sclerosis
Treatment: The patient returned to our clinic for a discussion of her test results and working diagnosis She was referred to a
Neurologist for further management.
Outcome: The patient is currently scheduled to establish care with a Neurologist.
Follow-Up: She has resumed her usual activities and was encouraged to continue with a regular exercise regimen.
The Ankle Fracture That Did Not Hurt
Authors: Robert Areson, MD; Rich Sisson, MD
Affiliation: Exempla Saint Joseph Department of Family Medicine, Denver, CO
History: Patient is a 11 year old male who presents to fracture clinic for a reported right ankle fracture. The patient
injured his ankle approximately 3 days prior while playing at school. He recounts running and accidentally stepping on the
foot of another student and inverting his right ankle. He went to his PCP on the same day where Xrays revealed an usual
appearance of the distal medial malleolus. Due to concern for fracture, the patient was placed in a non-weight bearing
posterior leg splint and referred to the fracture clinic for further evaluation. Since that time he has been feeling well, with
pain well controlled with minimal Advil as needed, and is able to partially weight bear on his right foot with minimal
discomfort.
Physical Exam: After removal of the posterior splint, ankle exam reveals no effusion, edema, or gross deformity He has
minor stiffness of the right ankle due to immobilization but otherwise he demonstrates normal dorsiflexion, plantar
flexion, inversion, and eversion There is no tenderness over the lateral malleolus or base of the 5th metatarsal There is
very mild focal tenderness over the lateral ankle and ATFL but none over the medial malleolus Patient is able to ambulate
essentially normally with regards to gait and stride.
Differential Diagnosis:
Fracture of medial malleolus
Sprain of the anterior talofibular ligament
Physeal injury or Salter-Harris fracture
Accessory ossification center (os subtibiale)
Test Results: Initial plain films of the right ankle: Skeletally immature patient Fragmentation involving the medial
malleolar epiphysis which may be a normal variant, however fracture cannot be excluded Mild widening of the lateral
malleolus physis also identified Soft tissue swelling is suggested medially Recommend correlation with site of injury and
follow up radiographs to exclude Salter-Harris type III fracture Salter-Harris type I fracture involving the lateral malleolus
also not excluded. Weight bearing films on follow up (3 days later) of bilateral ankles: Similar appearance of
contralateral medial malleolus with an accessory ossification center distal to the medial malleolus consistent with bilateral
os subtibiale. Bilateral ankles otherwise appear normal and symmetric without acute injury.
Final Diagnosis: Os subtibiale. Right ankle sprain.
Treatment: Patient was treated with lace up ankle brace for 7-14 days for comfort and support with weight bearing as
tolerated Early mobilization of the ankle was encouraged and he was referred to physical therapy for range of motion,
proprioception, and strengthening exercises for treatment of the ankle sprain.
Outcome: The patient recovered well, already demonstrating significant clinical improvement at the time of this visit He
made a full return to his normal activities without limitation.
Follow-Up: The patient had good clinical improvement with no long term sequelae from this injury He walked out of the
clinic and shortly returned to his previous level of activity This case illustrates the importance of recognizing normal bony
variants in the setting of pediatric growth and development to make the correct clinical diagnosis This case also
underscores the importance of obtaining bilateral images for comparison when the radiographic findings are not
consistent with the clinical presentation.
Freshman College Soccer Player with Bilateral Shin Pain
Authors: Brandon Haugh, MD; Mark Lavallee, MD
Affiliation: Wellspan York Hospital, York, PA
History: 18 year old freshman women's college soccer player with 3 week history of bilateral shin pains. Over summer she
was running 3 miles a day, seven days a week preparing for college soccer. The season began with two a days involving
30+ miles of running daily. The pain is located over the anterior shins and the anteriolateral aspect of both legs. Initially
the pain was only present after extended running. Then progressed to pain early into running and up to 10 minutes after
rest. She now has pain after 2 minutes of activity and can only go for 10-15 minutes before needing to stop due to pain.
Her legs and calves feel very tight during the pain. She denies numbness or tingling, as well as, pallor, color changes or
temperature change in her feet. She had similar pains in high school a year ago which resolved after four weeks of
inactivity. PMH: Cyst in left tibia bone discovered by doctor one year prior MEDS: None ALL: None
Physical Exam: General: Well-developed, well-nourished, muscular female. 5ft 2in
HEENT: Normocephalic,
atraumatic, PERRLA, EOMI Neck: Supple, no thyromegally, Respiratory: Nonlabored Musculoskeletal: Full range of
motion at hips, knees, and ankles. 5/5 strength and normal sensation in bilateral lower extremities. No sensory deficits. 2+
dorsalis pedis, posterior tibial and popliteal artery pulses. Some increased turgor over the anteriolateral aspect of both
shins and tenderness over bilateral shins at the mid and middle third of the tibia. Fibular rock test was negative. There is
general tenderness over musculature. Exercised patient on treadmill in Training room, she went about 8 min and started to
develop pain, tingling. Encouraged to go another 2 minute and examined her. She had no skin palor, normal two-point
discrimination, definite tight, increased turgor in all compartments, and palpable but sluggish posterior tib and dorsalis
pulses. Cap refill &lt; 2 seconds.
Differential Diagnosis:
Medial tibial stress syndrome (bilaterally)
Exertional compartment syndrome (bilaterally)
Stress fracture, Left Proximal Tibia
Bone Lesion of Unknown Nature, Left Proximal Tibia
Popliteal Artery Entrapment
Test Results: AP and Lateral Radiograph of Left Shin - Bubbly Lesion in the proximal tibia, which is probably a
nonossifying fibroma (5.1 x 3.8 x 3.0 cm) very close to posterior periostium. Otherwise no definite acute abnormality.
(When compared to XR done one year ago the same exact lesion was present and there were no changes from prior
imaging) AP and Lateral Radiograph of Right Shin - No significant finding MRI Left Tib Fib w/wo contrast no
evidence of stress fractures. Juxtacortical lesion posteriorly in the proximal tibial shaft which is probably a fibrous cortical
defect. Small amount of marrow edema along the anteriolateral and upper border of the bone lesion involving the
proximal tibial metaphysis and extending along the metaphyseal/epiphyseal junction raising the question of underlying
occult fracture.
Final Diagnosis: Left knee non-ossifying fibroma, stable
related
Pathologic fracture around non-ossifying fibroma, stress
Bilateral lower extremity pain consistent with exertional compartment syndrome.
Treatment: Two basic option: conservative vs. surgery to clean out and pack the NOF with bone cement. She opted for
conservative treatment. Because he could walk on the pathologic fracture, treatment of the exertional compartment
syndrome was relegated to "deconditioning" treatment.
Outcome: Initially she was able to play through the majority of a game with pain only at the very end. Her symptoms are
worsening. She currently has pain half way through the second half of the game.
Follow-Up: Post season plan for exertional compartment testing at rest and with exercise. If positive, then to undergo 4
compartment surgical release.
Further treatment information pending.
A New Differential Diagnosis and Treatment to Consider for Lateral Leg Pain in Runners
Authors: Katherine Victoria Yao, MD; Carolyn E. Thompson, DO; Christopher Visco, MD
Affiliation: New York Presbyterian Hospital of Columbia and Cornell, New York, NY
History: A 36-year-old male presents with left lateral leg, ankle, and foot pain His pain began two months prior to being
seen in the office, without any inciting event He experiences intermittent burning pain, tingling sensation, and numbness
to the left lateral leg, ankle, and foot The pain radiates from the left lateral leg to the dorsal foot His symptoms are worse
with "most activities": walking, exercising, and sitting with his legs crossed in the figure-four position The pain particularly
is worse after running 2-3 miles and has restricted his ability to run His symptoms are relieved with rest and lying down.
The patient has no other associated symptoms and he denies any back pain nor has any radiating pain in the leg above
the knee.
Physical Exam: Lower extremities exam shows asymmetry of the soft tissue approximately 10 centimeters above the left
lateral malleolus in the region of the lateral compartment, appearing atrophic when compared to the right side Pulses are
intact in the bilateral dorsalis pedis Alignment of the lower extremities is normal without generalized ligamentous laxity
He has normal flexibility in the lower limbs except for tight gastrocnemius muscles bilaterally. Neurologic exam reveals a
non-antalgic gait. Bilateral patellar and achilles stretch reflexes are 2+ His strength is 5/5 in all myotomes in bilateral
lower extremities Sensation to light touch and pin-prick is intact in all dermatomes of the lower limbs except in the left
lateral lower leg and dorsal foot Straight leg raise test is negative Tinel's sign is negative over the fibular nerve at the
fibular head, but positive at the superficial fibular nerve near the area of atrophy on the left lateral leg.
Differential Diagnosis:
Common fibular nerve entrapment
Superficial fibular nerve entrapment
Lateral compartment syndrome
Peripheral vascular disease and vascular insufficiency
Deep vein thrombosis
Fibula fracture
Lateral malleolus fracture
Test Results: XRAY showed normal soft tissue and normal anatomy of the left tibia, fibula, talus, calcaneus, and tarsal
bones without fractures. EMG of the lower extremities showed normal findings including normal fibular nerve studies.
Ultrasound of the left superficial fibular nerve showed an aberrant path and asymmetry when compared to the right side
The left distal superficial fibular nerve was found to be deep to and embedded in the deep fascial layer of the lateral
compartment when compared to the right where it remained superficial to the deep fascia There was less subcutaneous
fat found in the painful area.
Final Diagnosis: Tethered distal left superficial fibular nerve and entrapment in the deep fascia causing superficial fibular
neuropathy.
Treatment: The patient received an ultrasound-guided superficial fibular nerve block with two-percent lidocaine After
assessing immediate results, the patient was treated with needling of the lateral compartment deep fascia and mechanical
dis-entrapment of the superficial fibular nerve under ultrasound guidance.
Outcome: The patient had immediate relief of pain after receiving the lidocaine block to the left lateral superficial fibular
nerve He had continued relief and resolution of symptoms following the ultrasound-guided needling and mechanical disentrapment of the superficial fibular nerve.
Follow-Up: The patient had complete resolution of his symptoms following his treatment He was able to return to his
regular running and exercise activities in the next several weeks. This case illustrates the importance of considering
fascial entrapment and tethering of peripheral nerves as a potential cause for neuropathic symptoms Ultrasound guided
mechanical dis-entrapment of the nerve in this case allowed the patient to avoid invasive neurosurgical interventions
which are usually required for similar cases.
19 YO Male Undergoing a Heavy Lifting Routine with Insidious Left Forearm Pain
Authors: James Franz, DO; Aaron Lear, MD
Affiliation: Akron General Medical Center, Akron, OH
History: A 19 year old healthy male college student presents with insidious left forearm pain. The pain has been present
over the past 2 months and is localized to the medial aspect of his left forearm. He denies any trauma or injury to the
forearm Upper body lifting aggravates his symptoms, which linger the remainder of the day after each routine. He notes
improvement of the pain with rest. He has not taken any medication for the pain. The patient reports he began an
aggressive workout program approximately 5 months prior in which he accelerated from a lifestyle of relative inactivity to
a regimen of predominately upper body, 6-7 day per week workouts. He expresses concern about taking time off from
lifting that he will lose the progress he has made in his physique.
Physical Exam: Appearance of the bilateral arms reveals no visual abnormalities, ecchymoses or edema. There is full
active range of motion in the left arm with no limitation. Palpation reveals a tender, palpable mass present at the distal
third of the left ulna. Tenderness noted with palpation of the dorsal and volar musculature over the left ulna. No pain
reported with resisted flexion or extension of the elbow. Moderate pain elicited with resisted pronation and supination of
the left forearm. Moderate pain elicited with resisted wrist flexion and mild pain with resisted wrist extension. No
tenderness with grip strength. Muscle strength is 5/5 and equal bilaterally in the upper extremities. No sensory deficits
noted in the upper extremities.
Differential Diagnosis:
Extensor Carpi Ulnaris Tendiopathy
Ulnar Stress Fracture
Intersection Syndrome
Biceps Tendonitis
Medial Epicondylitis
Test Results: Xray: 2 views of the left forearm show no fracture or dislocation. Periosteal reaction at the middle to distal
third section on the medial portion of the ulna consistent with the mass palpated on exam. MRI of the left forearm with
and without contrast: Focal periosteal thickening, edema and enhancement along the volar aspect of the mid ulnar shaft
correlating to skin marker placed at the palpable abnormality. This extends over a cranial caudal length of approximately
15mm. Findings most consistent with a grade 3 stress response. No distinct fracture line. Large amount of edema is seen
throughout the ulnar shaft.
Final Diagnosis: Left ulnar stress fracture
Treatment: The patient was instructed to take a minimum of 2 weeks off from upper body lifting. At that time, if his pain
was resolved, he was allowed to return to light exercise involving the upper extremities 2 times weekly for 2 weeks and for
up to 30 minutes daily on his lifting days. He was to avoid heavy lifting completely over this time. Follow-up was
scheduled for 4 weeks.
Outcome: The patient took 16 days total rest from lifting and exercise. He then returned to a lighter, modified upper
extremity lifting regimen and completed 3 workouts prior to follow-up. At 4 week follow-up he had reported almost
complete resolution of the pain except for a mild ache after one of his workouts. His exam had improved, as there was no
longer any tenderness or appreciable palpable mass within his left forearm. He reported no pain with resisted arm
movements as previously noted.
Follow-Up: The patient was instructed to limit his upper extremity workouts to 2 times per week over the next 2-3 weeks
and to use lighter weights compared to his pre-injury level of lifting. He was to continue on a graduated lifting program in
return to full activity.
Neck Injury - Football
Authors: Drew Duerson, MD
Affiliation: Nationwide Children's Hospital, Columbus, OH
History: Patient is a 21 year old male, college senior football player that was evaluated on the playing field during a
game after sustaining a neck injury. While playing wide receiver and attempting to block for the running back he went
helmet first into the defender resulting in an axial load and hyperextension neck injury. He was found prone on the field
and unable to move his arms or legs. He reported no feeling from the neck down to his feet. He denied any loss of
consciousness. Using cervical spine precautions, he was log rolled and immobilized on a spine board. His face mask was
removed and helmet stabilized. He was then taken via EMS to the nearest emergency department.
Physical Exam: He was alert and oriented to person, place, and time. No cervical midline tenderness. He demonstrated
0/5 strength in his upper and lower extremities with no grip strength and inability to dorsi- or plantarflex his feet.
Sensation to light touch was absent in his hands and lower legs.
Differential Diagnosis:
1.
Cervical Spine Fracture +/- Dislocation
2.
3.
4.
5.
Transient Quadriplegia
Burner
Cervical Sprain/Strain
Spear Tackler's Spine
Test Results: In the ED, labs were obtained which included a normal CBC with differential, comprehensive metabolic
panel, coagulation panel, and blood alcohol level Imaging studies were unremarkable and included a head CT without
contrast; CTA of the neck with contrast; CT and MRI of the cervical, thoracic, and lumbar spine; and AP view of the chest
and pelvis No evidence of any acute spine abnormalities was identified Normal spinal canal diameters were calculated
Chronic bilateral spondylolysis of the L5 pedicles were visualized.
Final Diagnosis: Transient Quadriplegia
Treatment: Neurosurgery was consulted in the ED upon presentation. Approximately three hours after the initial injury,
while obtaining his spinal MRI, he began to regain sensation in his upper extremities followed shortly after by his lower
extremities. His strength improved as well. After his work-up was completed, he was admitted overnight to the trauma
service. After a short observation period he was discharged home with a cervical collar to be worn for comfort. He weaned
out of this over the next couple days as his neck pain resolved. He was restricted from all contact sports until further
follow-up with Neurosurgery.
Outcome: Three days after his discharge from the hospital, he again reported transient numbness and tingling in his
hands and feet. In result, he returned to the ED but was discharged home without repeat work-up given his symptoms
quickly resolved. He had no further symptoms after this visit. He followed-up with Neurosurgery approximately two weeks
later.
Follow-Up: At his Neurosurgery follow-up it was decided to restrict him from returning to football for the remainder of
the season. He was cleared for all non-contact sports. Currently, he is attempting to obtain a medical redshirt in hopes to
play football next season as a fifth year senior.
An Atypical Case of Bilateral Exertional Leg Pain
Authors: Jeremy L. Riehm, DO; Barry K. Diduch, MD; Delmas J. Bolin, MD, PhD
Affiliation: Unverisity of Virginia
History: An 18-year-old NCAA Division I swimmer presented to the athletic training room with a complaint of bilateral
exertional calf pain. Approximately one year prior, she had developed similar symptoms which workup included initial
negative compartment syndrome testing. However, repeat compartment pressures revealed elevated superficial posterior
and lateral compartment pressures and an MRI established a diagnosis of bilateral popletial artery entrapment syndrome.
Therefore, she underwent fasciotomy of these compartments and bilateral functional entrapment release. The athlete
described her current pain as a deep "brain freeze" and endorsed an achy, burning sensation in her calves during
swimming. She had no weakness or foot drop and denied any paresthesias into her foot. After a period of conservative
treatment and activity modification, her pain continued.
Physical Exam: Gait was without antalgia. Mild genu recurvatum was noted. Well healed surgical scars were present
without evidence of herniation. Calves were soft and non-tender and no swelling or erythema was noted. She had normal
ankle range of motion and was neurovascularly intact. There was a negative straight leg raise.
Differential Diagnosis:
1.
2.
3.
4.
5.
6.
Peripheral neuropathy
Lumbar radiculopathy
Reoccurring exertional compartment syndrome
Reoccurring popletieal artery entrapment syndrome
Fascia herniation
Metabolic disturbance
Test Results: EMG/NCS: Negative MRI Angiogram: negative for popliteal artery entrapment syndrome B12: 496 (211946) Folate: 13.3 (&gt;3.0) TSH: 2.15 (0.45-4.5)
deep posterior compartments
Compartment pressure testing: Elevated values in bilateral anterior and
Final Diagnosis: Bilateral chronic exertional compartment syndrome of bilateral anterior and deep posterior
compartments.
Treatment:
1. Continued conservative measures including modified training
2. Medical redshirt for the season
3. Fasciotomy planned (December 2014)
Outcome: Fasciotomy planned (December 2014)
Follow-Up: Standard post-surgical return to play is planned. Usual follow-up will occur. Will have an updated status
before April 2015.
Cross Country Runner with Lateral Foot Pain
Authors: Darren Presley, MD; Gregory Rowdon, MD; John Baldea, MD
Affiliation: Indiana University - Purdue University at Indianapolis, Indianapolis, IN
History: 21 year old, Caucasian female, NCAA division 1 cross country runner presented with six days of L-medial arch and dorsal lateral
midfoot pain with running. No injury reported. Patient states at about the two mile mark of a run had to alter her gait secondary to pain.
Had been undergoing icing and iontophoresis in the athletic training room and able to increase to four miles without altering gait.
When pain caused gait alteration, she had been converting to cross training. She tried wearing inserts and her symptoms worsened.
Patient reports similar symptoms about six months previous which resolved spontaneously. Patient returned to clinic eight days after
initial presentation. Had been cross training over this period and was progressing back to running when pain worsened and now
localized to dorsal lateral midfoot.Has not been able to run secondary to limping with pain. Past medical history is significant for iron
deficiency anemia treated with ferrous sulfate. Reports regular monthly menses with no aberrations and takes a combined oral
contraceptive pill. Denies nicotine use and reports occasional alcohol use.
Physical Exam: Initial exam revealed no edema, erythema, ecchymosis or deformity. Mild pes planus was noted with weight bearing.
Patient had no tenderness to palpation over the lateral malleolus, styloid of fifth metatarsal, fourth or fifth metatarsal, navicular,
calcaneous, cuboid or cuneiforms. Sensation was intact. Dorsiflexion to 20° and plantarflexion to 50° without pain. Strength testing was
5/5 for dorsiflexion, plantarflexion, inversion and eversion. Repeat exam eight days later revealed a normal inspection, range of motion
and strength, however, now the patient had tenderness at the junction of the cuboid and the fourth and fifth metatarsals. Patient did
not experience increased pain with torque applied across the midfoot.
Differential Diagnosis:
Peroneus brevis tendonitis
Peroneus longus tendonitis
Avulsion of the fifth metatarsal styloid process
Sprain of the lateral midfoot ligaments
Stress fracture of the lateral metatarsals
Stress fracture of cuboid
Stress fracture of the cuneiforms
Test Results: X-ray of left foot including weight bearing and one contralateral view revealed no acute fracture or malalignment was
seen and joint spaces are well preserved. MRI of the foot was then obtained and revealed an intense osseous stress reaction within the
cuboid, centered about a tiny focus of subarticular decreased signal intensity at the cuboid fourth metatarsal articulation. Surrounding
soft tissue edema dorsolateral to the midfoot including edema in the extensor digitorum brevis musculature. Remainder of exam
normal. A non-contrast CT of the foot with coronal and sagittal reformatted images was ordered revealing a stress-type, incomplete
fracture through the anteromedial cuboid which appears to extend into the inferior fourth metatarsal joint space. There is some
increased density in the anterior cuboid consistent with callus formation related to a healing response.
Final Diagnosis: Cuboid Stress Fracture
Treatment: There are no clear guidelines to direct care due to the rarity of the injury Thus, in consultation with an orthopedic surgeon,
the patient was placed in a walking boot and made non-weight bearing for a six week period. After this period she will be allowed to
weight bear and progress out of the boot over two weeks. During this time she will be allowed to swim in the deep end with no flip
turns or weight bearing on the bottom of the pool.
Outcome: Patient is currently undergoing treatment.
Follow-Up: Plan for follow up CT of the foot after the patient has completed the 8 week treatment plan. If the CT results are favorable
she will be allowed to slowly return to competitive running. We are also considering a gait analysis upon her return to competition.
A Stressful Scenario: Bilateral Thigh Pain in a DIII Distance Runner
Authors: Nicholas Edwards, MD; Christopher Hogrefe, MD
Affiliation: Department of Emergency Medicine, University of Iowa Hospitals and Clinics, Iowa City, IA
History: AM is a 19 year old female with a history of medial tibial stress syndrome who presented to a Sports Medicine Clinic with three
months of worsening bilateral thigh pain. She is a track and cross-country athlete at a local Division III college. The pain originated in
the right quadriceps; however, left thigh pain subsequently developed. Her left thigh pain was now greater than that on the right. This
throbbing pain was localized to the medial aspect of the proximal thighs, and was mild to moderate in severity. The pain was
exacerbated by sitting, standing up, and ambulation; however, was most severe with running and using the elliptical machine. She
reported a growing hindrance to her running regimen, as well as sleeping difficulty. There was no recent trauma to her thighs. AM had
been performing hip strengthening exercises, as well as utilizing ice and NSAIDS, without symptomatic relief. AM had gone nine
months without a menstrual period, however reported irregular menses at baseline. She was previously on oral contraceptives, however
had stopped taking these. She denied fluctuations in her weight. She denied fever, erythema, ecchymosis, swelling, numbness, tingling,
weakness, clicking, locking, or instability of her lower extremities.
Physical Exam: Vitals: BP 115/59; Pulse 66; BMI 19.85 kg/m2 Musculoskeletal: No erythema, ecchymosis, swelling, or atrophy of
bilateral thighs on inspection. Tender to palpation along the medial aspects of both femurs without focal tenderness. There was pain
with fulcrum testing of both thighs. She had full range of motion of bilateral hips, and full flexion and extension of the bilateral knees.
No pain with passive or resisted range of motion. 5/5 strength in the bilateral lower extremities, with intact sensation, and 2+ dorsalis
pedis/posterior tibial pulses. Bilateral thigh pain reported with ambulation, left &gt;right. FABER, FADIR, and resisted logroll maneuvers
were negative bilaterally.
Differential Diagnosis:
Femoral stress fracture
Quadriceps strain
Hip adductor strain
Iliopsoas strain
Avascular necrosis of femoral head
Hip labral tear
Pubic symphysis dysfunction
Osteitis pubis
Pelvic fracture
Lumbar radiculopathy
Sports hernia
Malignancy
Lyme disease
Test Results: Both Femur AP/Lateral Plain Film: Periosteal thickening on the medial surface of bilateral femurs, consistent with stress
reaction. MRI Left Femur: Stress fracture involving the mid-medial and posterior left femur.
Final Diagnosis: Left femur stress fracture. Concomitant right femur stress fracture suspected given appearance on plain films.
Treatment:
Utilize crutches until able to ambulate pain-free, at which point she could ambulate as tolerated. No running while symptomatic.
Aqua therapy, as tolerated
Avoid NSAIDs. Start Vitamin D/calcium, and promote caloric intake.
Follow-up in 4-6 weeks.
Outcome: 3-week follow-up: Off of crutches. Ambulating and swimming without pain. 5-month return: AM had resumed running
without thigh pain. She had left knee pain and was diagnosed with IT band syndrome; which improved with a corticosteroid injection.
2014 Cross-Country: AM set the school record in the 6K. She finished second at the conference meet.
Follow-Up: Follow up at 3 weeks and 5 months
A Limp That Would Take Your Breath Away
Authors: Katerina A. Backus, MD
Affiliation: Florida Hospital for Children Department of Pediatric Orthopaedics, Orlando, FL
History: A 27 month old presented to pediatric orthopaedic clinic with a one month history of altered gait in the right
lower extremity. Right knee swelling was noted at the onset, but has since resolved. His gait recently deteriorated to the
point where he preferred to crawl rather than walk. A few weeks prior to the onset of altered gait, he fell off the couch and
required staples in his head. He had been evaluated by his PCP and who ordered labs, which were reportedly
unremarkable. Past Medical History: None Past Surgical History: None Social History: Lives part-time with mother and
part-time with father. Family History: Hypertension, stroke, diabetes, heart disease and cancer. There is no family history
of rheumatoid arthritis. Review of Systems was positive for poor appetite and recent URI symptoms, including
rhinorrhea, cough, and 102°F fever over the last few days.
Physical Exam: VS Wt 12.3 kg, Ht 61 cm, Temp 98.8°F, P 137, bp 113/53, RR 36, 97% 02 Sat Alert and cooperative.
Tachypneic Increased WOB. Abdominal breathing. Extremities warm and well perfused. No erythema, ecchymosis, or rash.
Full painless passive ROM of the hips, knees and ankles bilaterally No knee or ankle effusions Fussy with palpation of
right knee and femur Circumducted gait, with limited active hip flexion, and intoeing in right lower extremity.
Differential Diagnosis:
Fracture
Nonaccidental Trauma
Septic joint
Toxic synovitis
Discitis
Tumor
Leukemia
Test Results: Radiographs of pelvis and entire right lower extremity were unremarkable. Wbc 8.7/Hbg 13.3/Hct 35.3/plts
265 61% L, 18% M, 1% E, 0% B, 19 bands CMP positives: K 3.1, Co2 17, glucose 382, phos 3.3, AST 49. LDH 431 Chest XR:
Posterior mediastinal mass with left posterior 7-9 rib erosions and widening of the right paravertebral stripe. Left pleural
effusion. Opacified lower two-thirds of the left hemithorax. Mediastinal shift to the right. Chest CT: Large left paraspinal
mass filling a large portion of the left chest cavity measuring up to 8.6 x 7.6 x 10.9 cm. This lesion is partially calcified and
invading the central canal with severe central canal stenosis at T8 and T9. There is associated rib and neural foraminal
erosion. Small left-sided effusion. Compressive atelectasis involving a large portion of the left lower lobe and lingular
segment. Rightward shift of mediastinal structures with no vascular compromise identified. MRI of Thoracic spine NEURAL FORAMINA: There is a T8-centered mass lesion invading the left extradural epidural and widening the left neural
foramen extending into the chest cavity measuring approximately 9 x 9 x 11 cm, with compression of the cord against the
left posterior wall of the vertebral body. Urine VMA creatinine 10. Emergency T7-T9 epidural tumorectomy and spinal
decompression was performed. Tumor pathology demonstrated ganglioneuroblastoma.
Final Diagnosis: T7-T9 epidural ganglioneuroblastoma
Treatment: Emergency T7-T9 epidural tumorectomy and spinal decompression was performed. He was subsequently
enrolled in a chemotherapy protocol for ganglioneuroblastoma.
Outcome: His gait significantly improved within 24 hours of epidural tumor removal He was running in the hospital the
day after surgery.
Follow-Up: His gait has normalized. He is currently receiving chemotherapy.
An Intercollegiate Basketball Rebound Causes More Than Just a Cervical Hyperextension Injury
Authors: David Cormier, DO, DPT; Barry Jordan, MD, MPH
Affiliation: New York Presbyterian/Columbia & Cornell Medical Centers, New York, NY
History: A 20 year-old previously healthy intercollegiate basketball player was referred to the hospital with severe
headache after sustaining a direct blow to the chin during a rebound at a basketball game. The player described hitting his
chin against another player's head thus causing his head and neck to "snap back". The athlete was removed from play but
due to worsening headache, he was subsequently referred to an outside hospital for evaluation. A CT scan was normal and
the patient was discharged home and instructed to take Ibuprofen. The next day he experienced the acute onset of right
sided weakness and the inability to speak.
Physical Exam: On admission to the hospital his neurological exam was notable for a global aphasia and right
hemiplegia.
Differential Diagnosis:
intracranial hemorrhage
cerebral contusion
internal carotid artery dissection
vertebral artery dissection
cervical spine injury
Test Results: The initial CT scan of the head read as normal. A follow up MRI demonstrated a left middle cerebral artery
(MCA) infarction and possible C2 fracture. CT angiogram of the head and neck demonstrated total occlusion of proximal
left internal carotid artery and occlusion of the M1 segment of the left MCA. An EEG demonstrated continuous left
hemispheric slowing and attenuation suggestive of underlying focal dysfunction.
Final Diagnosis: Traumatic cervical hyperextension injury resulting in left carotid artery dissection and embolic left
MCA infarction.
Treatment: The patient was not treated with tPA secondary to time elapse. Neurological ICU course was complicated
by increased intracranial pressure and the patient was subsequently briefly intubated and sedated with eventual successful
extubation. He was started on aspirin daily for anticoagulation and weaned off of anti-epileptic drugs and able to
participate in bed-side physical therapy.
Outcome: Patient was able to transfer sit to stand with minimal to moderate assist and ambulated one step forward
and back in sets of five with two therapists. He was able to volitionally produce vocalization consistently upon controlled
exhalation. With his left hand, he was able to copy his first name with model. He then was able to write his first name
independently without model.
Follow-Up: After initial post-acute course of rehabilitation he was ambulating with a quad cane, contact guard assist
200 feet. He was able to produce short 2-3 word sentences. At 6 month phone follow up he is ambulating independently
around the community, completing stairs independently, speaking full sentences clearly but slowed, and will attempt
online college courses this winter. Continued deficits with right hand grip, and receiving botox injections every four
months to the right upper extremity.
Case Series: The New Face of Disordered Eating in Athletes
Authors: Jennifer Prins, MD, MPH; Jeanne Doperak, DO
Affiliation: Western Psychiatric Institute and Clinic, Pittsburgh, PA
History: Athlete 1 is a 20 year-old runner assessed in the training room by medical staff and found to have fractured hip,
the latest in a series of stress fractures she has sustained. Athlete 2 is a 17-year-old runner, recently diagnosed with a
stress fracture in her femur Close to a year ago her primary care physician recommended she see a nutritionist due to
concerns for low weight and amenorrhea She gained 20lbs with the recommendations of the nutritionist during a time
when she was recovering from a muscle injury and not running However, when she returned to running she did not
continue with the dietitians recommendations, again dropped to her previous weight, and amenorrhea resumed She
appears to be eating a healthy diet, noting she strictly avoids processed foods as they trigger migraines.
Physical Exam: Athlete 1 is 96.1% of IBW, denies any current or past eating disorder behaviors including binging,
purging, or laxative use, and appears to be eating a fairly healthy diet Although her diet appears normal at first glance, it
is not sufficient to sustain fuel the large amount of running she completely weekly, and she endorses current amenorrhea.
Athlete 2 is 89% of IBW, denies any current or past eating disorder behaviors including binging, purging, or laxative use,
denies fear of weight gain and reports her ideal body weight is "any weight that allows me to be healthy and run".
Amenorrhea continues to be an intermittent issue. Other than current stress fractures, physical exam unremarkable.
Differential Diagnosis:
Unspecified Eating Disorder
Malnutrition due to absorption issues such as celiac disease
Endocrine disorder resulting in accelerated metabolism
Test Results: Compete metabolic panel is unremarkable. Results negative for celiac disease or other digestive disorders.
DEXA scan currently not concerning for osteoporosis.
Final Diagnosis: Orthorexia is an increasingly studied phenomena among athletes characterized by overvalued ideas
regarding eating only "healthy foods", often resulting in strict self-imposed diets, and negative physical and performance
consequences due to malnutrition an
Treatment: Consultation with a nutritionist to increase caloric intake to match energy expenditure from running.
Outcome: In spite of being counseled to increase their caloric intake by a dietitian and making some small and
temporary changes, both athletes do not appear to making substantial changes particularly in addressing the
recommendation of adding more fat to their diet, and gaining enough weight to resume menstruation.
Follow-Up: On follow-up both athletes continue to struggle with stress fractures and amenorrhea These two cases
illustrate athletes who deny traditional eating disorder behavior, appear to have only the subtlest symptoms of disordered
eating, and yet both appear unable to maintain a weight necessary to avoid the female athlete triad, struggling with
amenorrhea and have been sidelined by multiple and serious stress fractures These cases highlight the need for greater
investigation and understanding of orthorexia and disordered eating among athletes, which, although not appearing with
classic eating disorder symptoms, can still result in devastating physical consequences and injuries.
Left Foot Numbness in a High School Athlete
Authors: Ashlee LaFontaine, MD; David Soma, MD
Affiliation: Mayo Clinic, Rochester, MN
History: Patient is a 17-year-old male athlete who presented with chief complaints of left foot coldness and numbness with activity.
Several months before presentation, the patient first noticed numbness and coolness of his left foot predominantly while running. He
plays several sports with a large amount of running, including basketball, cross-country, and track. Symptoms began early in exercise,
became persistent, and gradually faded without any specific intervention. Occasionally, symptoms occurred with cold exposure. The area
of numbness was localized over the medial forefoot primarily involving his first, second, and third toes extending proximally almost to
the calcaneal region. During these episodes, he would also occasionally experience cramping and a feeling of discomfort extending up
to the lowest portion of his posterior calf. Other associated complaints included a yellow-white discoloration of the skin with these
changes. Patient's symptoms occurred with at least moderate intensity activity only and resolved with rest. His sports performance and
daily activities were overall unaffected; however, he would often have difficulty falling asleep due to achiness in his leg at night. He
denied pain, previous occurrences, trauma, tobacco use, symptoms in the right leg, gait changes, or weakness during or after activity. Of
interest, his father is a former smoker and recently diagnosed with symptomatic anomalous origin of the right coronary artery from the
left sinus of Valsalva.
Physical Exam: Musculoskeletal: Inspection reveals normal coloration with symmetric muscle bulk and tone. No changes in skin or
hair. No tenderness to palpation. Legs warm to touch with left foot slightly cooler than right. Full and symmetric active and passive ROM
of knees and ankles. Strength 5/5 of lower extremities bilaterally. Normal toe and heel walking. Sensation intact to light touch. Right
dorsalis pedis (DP) and posterior tibial (PT) pulses were 2+. Left DP and PT pulses were absent. Popliteal were difficult to palpate but
seemed asymmetric greater on the right than left. Otherwise, normal.
Differential Diagnosis:
1.
2.
3.
4.
5.
6.
7.
8.
Popliteal artery entrapment syndrome
Exertional compartment syndrome
Tarsal tunnel syndrome
Popliteal Cystic Adventitial Disease
Early onset peripheral artery disease/Buerger's Disease
Trauma
Raynaud's Phenomenon
Vasculitis
Test Results: 1. Left foot & ankle (standing, 3 views): No acute fracture, malalignment, or other abnormalities. 2. Lower extremity
Doppler: Moderate left infrapopliteal level occlusive arterial disease with diminished ABI in left lower extremity pre- and post-exercise.
3. CTA (Abdomen, pelvis, lower extremity): Left popliteal artery entrapment by the thin portion of the medial head of the gastrocnemius
muscle without evidence of arteritis. Otherwise, normal. 4. Bilateral MRI/MRA lower extremity: The left gastrocnemius muscle medial
head inserts more laterally than normal. The tendon travels between the popliteal artery and vein causing occlusion at the knee joint
level. The right popliteal artery shows significant narrowing with plantar flexion, minimal narrowing with dorsiflexion, and is widely
patent in neutral without definite evidence of anatomical compression.
Final Diagnosis: Type III popliteal artery entrapment
Treatment: 1. Referral to Vascular Surgery (surgery was approximately 6 weeks after presentation): Left popliteal artery release and
reconstruction with right great saphenous vein. 2. Daily baby aspirin until follow-up ultrasound excludes any evidence of intimal
hyperplasia ;
Outcome: 1. Patient tolerated the procedure well without significant complication. 2. Doppler studies 4 months after surgery showed
normalized ABI at rest and after exercise. Left DT and PT pulses became easily palpable.
Follow-Up: 1. At follow-up vascular surgery appointment (4 months post-op), almost all symptoms had resolved. Patient had a rapid
return to full activity without restriction. He ran longer distances with only mild residual numbness in his proximal right medial calf. 2.
Follow-up popliteal ultrasound will determine length of aspirin treatment.
Lateral Knee Pain in a College Track Athlete
Authors: Jason Foster, MD
Affiliation: Promedica Sports Care, Toledo, OH
History: 20 yr old division 1 college track athlete presented 8/27/14 at training room clinic with left lateral knee pain for
the past 3 months She described an injury which occurred in May during the triple jump competition when planting: her
left knee gave out laterally and she was unable to finish the meet due to pain and instability. Her athletic trainer's
assessment was an LCL injury and the athlete responded well to conservative treatment in the training room. She was able
to compete the following week Over the next 3 months she took time off of running, continued a home exercise program,
and gradually returned to running 4 weeks prior to presentation In clinic she complained of aching lateral knee pain
associated with feeling weak which worsened at the end of her runs She denied swelling, giving out, mechanical
symptoms.
Physical Exam: left knee: no ecchymosis, swelling or effusion noted. There was tenderness over the lateral joint line
and lateral femoral condyle. AROM wnl, strength 4/5 with extension and 5/5 with flexion. Mild pain with varus stress but
no increased laxity compared to right. Negative lachman, mcmurray, valgus stress. Dial test revealed 5-10°; of increased ER
at 30° flexion when compared bilaterally.
Differential Diagnosis: LCL sprain, IT band syndrome, lateral meniscal injury, posterolateral corner injury, biceps
femoris tendinopathy
Test Results: MRI of the left knee showed minor focal edema just superficial to the LCL, small baker's cyst, minor fluid
along pes anserine tendons, and minor edema along the periphery of the popliteus muscle. Upon further viewing of MR
images, a small cystic structure was identified along the lateral meniscus, consistent with a meniscal cyst. This was
discussed with radiology who agreed and addended the report.
Final Diagnosis: left knee lateral meniscal cyst
Treatment: We discussed options including surgical debridement and ultrasound-guided injection. We agreed to
proceed with ultrasound guided injection. The cyst was identified under ultrasound, a 25g needle was used to inject 80 mg
depomedrol and 3 ml 1% lidocaine. The needle was used to puncture the cyst 10-15 times to aid in cyst breakdown. She
was instructed no activity for 2 days, then gradual return to run. 2 weeks later she noted improvement but noted
intermittent sharp pain with runs. She was prescribed topical pain cream made at a local compounding pharmacy
consisting of ketoprofen, colchicine 1 week after her f/u in the office she complained of sharp posterior knee pain and
"locking". exam revealed hamstring spasm and weakness. She was treated with stretching and iontophoresis.
Outcome: 4 weeks after her injection she was pain free and tolerating workouts. She continues routine stretching and
treatment with ATC.
Follow-Up: At the time of case submission, the athlete has been asymptomatic for 6 weeks and is doing well with her
workouts. She will follow up as needed.
Failed Drug Test Leads to an Unexpected Finding in a NCAA Baseball Player
Authors: Bradford Mitchell, MD; Kirtida Patel, MD
Affiliation: Duke Sports Medicine, Durham, NC
History: A 21 year old NCAA Division I baseball pitcher was randomly selected for an October drug screening during the
off-season in between his sophomore and junior years The NCAA soon thereafter notified the school's head athletic
trainer that the player had failed his drug screen due to testing positive for hCG. The team's primary care team physician
was notified and had the patient come in to discuss these findings. At this appointment the patient endorsed taking
creatine, amino acid, and caffeine supplements in the past, but he denied current use of any performance enhancing
agent. The patient denied any current health concerns, and a review of systems was unrevealing other than approximately
1-2 months of bilateral areolar tenderness, without galactorrhea The patient denied a history of any medical problems,
and his family history was only positive for inflammatory bowel disease in one first degree relative.
Physical Exam: The patient was afebrile with normal vital signs His physical exam was normal except for mild firmness
and tenderness of bilateral areolae as well as slight enlargement of his left testicle without appreciable mass or
tenderness.
Differential Diagnosis: 1. Inappropriate use of NCAA banned substance
2. Germ cell tumor
3. HCG producing
pituitary adenoma
Test Results: CMP - normal; LFTs - normal; TFTs - normal; LDH - normal; hCG - 189 (nl 0 - 3); Prolactin - 24 (nl 4-15);
AFP -65.3 (nl 0-8.3) Testicular Ultrasound: normal epididymides, normal right testicle, left testicle with punctate areas of
microlithiasis as well as moderately coarse calcifications in the central portion of the testicle. No definite mass is associated
with these findings and these areas may represent the suquela of prior infection, trauma, or possibly areas of dystrophic
calcifications The testicle otherwise demonstrates a homogeneous echotexture. Arterial and venous waveforms are
identified as well as color filling of vessels. CT Chest/Abdomen/Pelvis with contrast: significant for enlarged infrarenal left
periaortic lymph nodes&hellip; may represent metastasis.
Final Diagnosis: Testicular Cancer
Treatment: A radical orchiectomy with lymph node dissection was performed, and pathology confirmed the diagnosis
of Mixed Germ Cell Tumor. Pathology revealed tumor consisted of seminoma (50%) and embryonal carcinoma (50%). Post
operatively, the patient received 4 cycles of BEP (Bleomycin, Etoposide, and Cisplatin).
Outcome: An AJCC staging of T2 (m), NX, MX was given post-surgery. The patient is currently being followed with serial
labs and CTs every 3 months. All subsequent testing to date is consistent with remission.
Follow-Up: The patient returned to school in the summer, approximately 9 months after his initial diagnosis. At that
time he was fully cleared by his oncologist and the team physician to return to play. The patient has progressed through
conditioning and strengthening and has participated fully in baseball activities in the fall.
A New Kind of Runner's High
Authors: Christopher Ledford, MD; Mark E. Lavallee, MD
Affiliation: Uniformed Services University, Bethesda, MD
History: 25 year old male runner, presented to the medical tent after completing his first marathon in 3hr 57min with
complaints of nausea and vomiting with dizziness, unsteadiness and paresthesias He related training only partially for the
race with a long run of 20 miles in the preceding weeks He reported beginning to feel nauseous just after mile 10 He
reported several bouts of, non-bloody, non-bilious emesis while on the course He walked through and drank either water
or a sports drink at every hydration station while on the course He voided once during the race. He presented to medical
tent Alpha just beyond the finish line for evaluation During intake, he vomited three times
Physical Exam: Initial Vitals: 118/70, HR98, temp (rectal probe) 102.3°F-repeated to 100.8°F with rectal thermistor (left in
place). In general, he was a pale Caucasian male, oriented to person/place/time and situation. He was sweaty in
appearance with rapid speech. CV: RRR, no M/R/G; Pulm: CTAB Abd: active bowel sounds in all four quadrants, soft and
nontender to palpation in all four quadrants Extremities: no edema seen in hands Repeat vitals 45 min after initial vitals:
110/70, HR 90, Temp 99.8°(rectal thermistor)
Differential Diagnosis: hyponatremia, heat injury, dehydration, acute viral gastritis (evolving gastroenteritis), metabolic
syndrome, exercise associated collapse, medication side-effect/overdose/toxicity
Test Results: Stat chemistry: Na:142/ K: 4.3; Cl: 104; TCO2: 19; Glu: 87; BUN 24, Cr: 1.7, An Gap: 25
Final Diagnosis: ADDITIONAL HISTORY: Upon further questioning, he related a history of attention deficit hyperactivity
disorder. He had taken his usual dose of dextroamphetamine/amphetamine (Adderall XR) 25mg that morning but had
taken his usual evening dose of dex
Treatment: Supportive care was provided during his time in the medical tent Repeat vital signs were within normal
limits. He felt better approximately 45min after presentation and he tolerated 250ml of water orally without further emesis
No IV fluids were administered He ambulated without difficulty and was discharged from the medical tent.
Outcome: He departed the medical tent approximately 55minutes after presentation. His symptoms were significantly
improved. In telephone follow up three days after the event he reported that his symptoms had entirely resolved by the
day after the marathon and he drove himself home without any incident and has returned to work with no notable
sequalea. This case is important because prescription stimulants, particularly those used in treatment of ADHD, may be
viewed as performance enhancing for endurance events. Though they have been demonstrated to increase power and
endurance in short-term controlled trials, their use in long term endurance events is poorly understood. Amateur athletes
competing in events that are not governed by an agency that tests for banned substances (such as WADA or NCAA), are
at risk of side effects from mis-use of prescription medications. Physicians that prescribe stimulant medication should
carefully counsel patients about the potential for stimulant toxicity in endurance events.
Follow-Up: This runner was advised to not use extra doses of medication for his ADHD when he was going to participate
in endurance events. Additionally, he was given carbon copy of his medical evaluation document of treatment that was
rendered in the medical tent and advised to follow up with his primary care physician upon return to his home.
Elbow Pain in a College Football Player: More than Just the Funny Bone
Authors: Owen Speer, DO; Siraj Abdullah, DO; Jeffrey Guy, MD
Affiliation: Department of Family & Preventive Medicine, University of South Carolina; Columbia, SC
History: A twenty-year-old Division I collegiate football player experienced acute onset left elbow pain at practice. He
was playing defensive end in a full contact drill during the first week of preseason. The injury occurred when the athlete
reached out his left arm to make a tackle and was hit from behind while coming to the ground. He was immediately
unable to move his left arm due to pain and ran to the sideline holding this arm close to his body. The pain was severe,
sharp in nature, and radiated up and down his arm. He denied any numbness or tingling in his left hand or fingers, and
denied any shoulder pain or difficulty moving his shoulder.
Physical Exam: The athlete presented awake and alert in moderate distress. He was muscular and fit in appearance.
The left elbow was slightly deformed with a marked sulcus posteriorly, just proximal to the olecranon process. The elbow
was positioned at approximately 45 degrees of flexion and his left forearm was in a neutral position with his thumb
pointed up. Severe pain occurred with more than 10° of motion was possible in flexion, extension, supination, or
pronation. There was no crepitus to palpation or with motion. The elbow was diffusely tender. Radial and ulnar pulses at
the wrist were 2+ bilaterally. Sensation was intact to light touch in his hand and fingers. No bruising, bleeding, or skin
breaks were noted. Except for the posterior deformity, no muscular defects were appreciated in upper arm or forearm. The
right side was unaffected.
Differential Diagnosis:
1.
2.
3.
4.
Elbow dislocation
Elbow fracture
Tricep rupture
Ulnar nerve neuropathy
Test Results: None
Final Diagnosis: Posterior elbow dislocation
Treatment: Within 1 minute of presentation, the athlete was placed prone on the sideline training table with his left arm
hanging off the table. His upper arm was abducted to 45° and stabilized on the table surface. The elbow was extended
slowly as a downward and distractive force was applied to the lower arm. With a palpable relaxation of the surrounding
tissues, relocation of the ulna in the humeral trochlea was appreciated. A posterior splint was applied, with a sling, for
comfort.
Outcome: Reduced left posterior elbow dislocation, likely not fully dislocated based on the ease of relocation
Postreduction fluoroscopy was negative for acute fracture. X-rays performed later that evening were negative for acute
fractures.
Follow-Up: The principle treatments were non-steroidal anti-inflammatory drugs and aggressive physical therapy. Early
and gentle range of motion exercises were implemented within the first week. Transition to a hinged elbow brace took
place after 2 weeks. The athlete returned to full contact practice 6 weeks following the injury. He successfully regained all
of his range of motion, strength, and sport-specific functionality. He played football the remainder of the season in the
hinged elbow brace without any complications.
Professional Soccer Athlete with a Tender Ankle Nodule
Authors: Kenneth Lee, MD; Justin A. Classie, MD
Affiliation: New York Hospital Queens, Weill Cornell Medical College, Flushing, NY
History: A 30 year old male professional soccer player, with history of multiple ankle sprains, left anterior cruciate
ligament repair (2001), right anterior cruciate ligament repair (2005), left knee arthroscopy for loose body, notchplasty,
and microfracture (2013), presented with right lateral ankle pain. Date of injury was July 9th, 2014 during a practice game.
He was running full speed, made an acute cut inward, suddenly rolling his right ankle. He was able to bear partial weight,
but unable to continue play. Lateral ankle pain was 8/10, nonradiating, worsened with any movement, especially ankle
inversion, and improved with rest and ice.
Physical Exam: On initial exam, inspection revealed a swollen ankle, and diffuse tenderness to palpation of the lateral
ankle. The patient had full range of motion of both ankles. Strength exam was significant for right ankle eversion 4+/5,
otherwise the strength was 5/5 throughout. Sensory was intact, reflexes were 2+, and dorsalis pedis as well as the
posterior tibialis pulses were intact bilaterally throughout. Ankle special tests were unremarkable, including anterior
drawer test, squeeze test, varus/valgus stress testing of the metatarsophalangeal joints, fracture test, and inversion talar tilt
test.
Differential Diagnosis: ankle sprain from a ligament tear, syndesmotic sprain, bone fracture, peroneus tendon
tear/rupture, peroneus tendon subluxation, ankle impingement syndrome, subtalar joint sprain, and other tendon injuries
such as involvement of ankle flexor hallucis longus, tibialis posterior, or achilles tendon.
Test Results: Ankle X-rays unstressed and stressed showed no fracture or significant ligament instability.
Final Diagnosis: Ankle sprain
Treatment: The athlete was initially put in a lace-up ankle brace for three days, given non-steroidal anti-inflammatory
medications for pain control, and treated with rest, ice, compression, and elevation (RICE) protocol as well as physical
therapy treatments given by the athletic trainer for two weeks.
Outcome: After the above treatment, pain was significantly improved, eversion strength returned to 5/5 on the right
ankle, and he was able to return to play in two weeks.
Follow-Up: The athlete returned to play after two weeks of treatment. At three weeks follow-up during training room
day, he was complaining of persistent lateral ankle pain. On this exam, a tender nodule was palpated at the inferior aspect
of the lateral malleolus, which was not palpated on initial exam. The rest of the exam was unremarkable. At this point,
magnetic resonance imaging of the right lower extremity was done on the athlete, which showed a full-thickness tear of
the peroneus longus tendon with retraction to the level of the calcaneus. Despite the athlete's significant tendon injury, it
was not impairing his ability to compete at the professional level, and a plan was made to consider surgical correction
once the season was over. The athlete continued working with the trainer, received RICE protocol, and took non-steroidal
anti-inflammatory drugs as needed for pain. Now that the season is over, he will go see a foot and ankle orthopedic
specialist for possible surgical correction of his peroneus longus tendon rupture.
An Unusual Cause of Knee Pain Complicated by Pregnancy
Authors: Kari Sears, MD; Linda Mansfield, MD
Affiliation: South Bend - Nortre Dame Sports Medicine Fellowship, South Bend IN
History: The patient is a 25yo female who presented to clinic initially in July, 2013 with right lateral knee pain radiating to
the hip The pain had been progressively worsening over 2 months She was diagnosed with IT band syndrome and given
home exercises and Naprosyn She returned 1m later with persistent pain, new buttock pain, and a positive straight leg
raise She was referred at that time for formal PT Two months later, she continued to have radiating pain, now with back
pain radiating to buttock and right thigh and knee, paraspinal muscle spasms, and difficulty walking due to pain. Since the
previous visit, the pt found out she was pregnant, and was about 15w at the time of visit. An MRI of the lumbar spine was
ordered, but the pt returned 2 months later and had not had the MRI because she was afraid of the effects on the baby
She was still having radiating leg pain, and at that time an US guided piriformis injection was performed The pt returned 5
days later with resolution of her back and buttock pain, but continued severe right lateral knee/thigh pain Her pain was
worst at night She had a 9lb weight loss since the beginning of her pregnancy, which she said was from diminished
appetite due to pain She denied numbness or tingling, fevers, chills, or other systemic symptoms
Physical Exam: Physical exam at that time was 6 months after her initial presentation. She had tenderness over the right
distal IT band at the femoral condyle and about 3-4cm proximally. She had normal ROM of the right knee and hip without
worsening pain. Ober test (+). Straight leg raise (-).
Differential Diagnosis: IT band syndrome, femoral stress fracture, bone tumor, and muscle strain
Test Results: X-Rays of the knee and femur showed cortical thickening of the lateral femoral shaft MRI of the right thigh
showed cortical thickening with a small nidus and mild bone marrow edema consistent with an osteoid osteoma of the
femoral shaft.
Final Diagnosis: Right femoral shaft osteoid osteoma
Treatment: The patient was 29w pregnant at the time of diagnosis, which significantly complicated her treatment.
Possible treatment options were discussed at length, including continued narcotic use for pain control, NSAID use to be
discontinued no later than 34w gestation, and radiofrequency ablation by interventional radiology using CT guidance.
Each treatment option had significant concerns related to the pts pregnancy. After extensive discussion with high risk OB,
IR, and the pt, we initially started her on NSAIDs with excellent pain relief, and elected to proceed with radioablation.
Outcome: She underwent successful radio frequency ablation at 32 3/7w with near complete resolution of her pain
postoperatively. Unfortunately, about 3w after the procedure, she developed a deep soft tissue MRSA infection that was
unresponsive to the limited selection of appropriate outpatient abx, and required inpatient management for IV abx and
open surgical debridement. Afteer a complicated recovery course, she was discharged home for the remained of her
pregnancy, and went on to deliver a healthy full term baby boy.
Follow-Up: She has undergone physical therapy to regain quad strength and minimize pain from scar tissue, and has
returned to daily activities. She still has limitations at work due to weakness with prolonged standing, and has not yet
returned to physical activity.
Scapula Winging in a Recreational Weightlifter
Authors: Nathan Cardoos, MD; John Herbert Stevenson, MD; Lee Mancini, MD
Affiliation: University of Massachusetts Memorial Medical Center, Worcester, MA
History: A 19yo right-hand dominant college student, landscaper and recreational weightlifter presented to his primary
care office with a history of "powerful dull pain" in his right upper back and shoulder that woke him from sleep. This pain
lasted five hours and was followed by subjective soreness, decreased range of motion (ROM) and strength. The patient
denied any recent or past trauma or injury. Personal medical history, surgical history and family history were
unremarkable. Prior to the onset of pain, his general physical activity level had been significantly decreased due to an
infectious mononucleosis diagnosis two weeks prior to the visit, for which he was seen in the emergency department and
received IV and oral steroids. His PCP noted pain with abduction beyond 90 degrees and mildly decreased internal
rotation, diagnosed him with a possible rotator cuff strain, and referred him to sports medicine where he was seen the
next day. At his initial visit with sports medicine, he was diagnosed with shoulder impingement syndrome and latissimus
dorsi strain and referred to physical therapy. He returned 8 weeks later, having not done any physical therapy, with report
of resolved pain and a few weeks of new "bulging in the back of his shoulder", along with difficulty performing overhead
press or bench press due to reduced ROM. He was noted to have scapula winging on exam and decreased active
abduction. An MRI was obtained at this time. On follow-up three weeks later, history was unchanged and physical exam
was as below.
Physical Exam: Right shoulder with no skin changes, edema or erythema. Trapezius muscles symmetric with normal
tone. Moderate winging of the right scapula, which is exacerbated with wall push-up No tenderness to palpation over the
entirety of the shoulder Active abduction of the right shoulder 130°. Forward flexion 160°. External and internal rotation
normal. Normal ROM of the left shoulder. Biceps, triceps, deltoids and grip strength 5/5 Empty can 5/5 with no pain
elicited. External rotation and internal rotation both 4+/5 Neer's, Hawkin's, Speed's, Cross arm and O'Brien's tests all
negative bilaterally. Normal bilateral radial pulses with brisk capillary refill. Sensation intact to light touch in the bilateral
upper extremities. Neck: Full ROM, Spurling's test negative.
Differential Diagnosis: Traumatic injury to long thoracic nerve (serratus anterior paralysis), spinal accessory nerve
(trapezius), or dorsal scapular nerve (rhomboids) due to blunt trauma, repetitive weightlifting or landscaping, or prolonged
abduction of the arm while sleeping. Non-traumatic neurologic injury including viral illness (e.g. infectious
mononucleosis, influenza, tonsillitis-bronchitis), allergic-drug reaction, drug overdose, toxic exposure (e.g. herbicides,
tetanus), C7 radiculopathy, and aortic coarctation. Brachial plexus injury Neuralgic amyotrophy/Parsonage-Turner
Syndrome Fascioscapulohumeral muscular dystrophy Shoulder instability
Test Results: Monospot test positive. X-rays right shoulder: Glenohumeral and acromioclavicular joints congruent. No
fracture, osteoarthritis or soft tissue calcification. Normal shoulder. MRI right shoulder: Slight infraspinatus tendinosis. No
discrete rotator cuff tendon tear. Inferolateral tilt of the acromion could contribute towards impingement. Mild subdeltoid
bursitis.
Final Diagnosis: Serratus anterior paralysis due to mononeuropathy of long thoracic nerve after infectious
mononucleosis
Treatment: Referred to physical therapy for periscapular strengthening, increasing rotator cuff strength and ROM,
thoracic spine mobility. The patient returned to college out of state.
Outcome: On follow-up by phone 10 weeks later, the patient reported completing 7 sessions of physical therapy and
compliance with home exercises. He noted subjective improvement of approximately 65% with regard to strength and
ROM. He noted that the winged scapula was still present but appeared less prominent.
Follow-Up: The patient replaced his normal weightlifting with physical therapy exercises. He will follow a functional
progression back to weightlifting once his rotator cuff and periscapular strength is symmetric.
A Rare Diagnosis and Repeated Knee Injury in a Young Athlete
Authors: Anthony Kohlenberg, MD; Ketan Mody, MD
Affiliation: Adventist LaGrange Memorial Hospital Family Medicine Residency, LaGrange IL. Elite Sports
Medicine Institute, Westmont IL.
History: A 12-year-old male presents complaining of right-sided anterior knee pain. Pain has been reportedly increasing
over several months during participation in basketball and baseball. He also describes pain with kneeling on hard surfaces.
4 weeks prior to exam, the patient was prescribed physical therapy for patellafemoral syndrome. At that time, x-ray of
right knee was negative for fracture, however the right patella measured 3.5 x 2.8cm, noticeably smaller than the 3.8 x
3.6cm left side. Past medical history is significant for an intrauterine brain injury known as periventricular leukomalacia
(PVL). A pediatric neurologist diagnosed the condition at age 6 when the patient exhibited right leg weakness, right
toenail hypoplasia, and left-handedness on exam. Diagnosis was confirmed with brain MRI.
Physical Exam: General: The patient is a well-developed, left-handed male with normal body habitus for stated age.
Inspection: The right leg has less muscle mass than the left. The right calf circumference measures 29.0cm, the left
measures 31.5cm. The right leg measures 1cm shorter in length. Active ROM: Right leg ROM is from 0° to 125° of
flexion. There is anterior knee pain with complete extension. Right foot ROM is from 40° in plantarflexion to only 10° in
dorsiflexion. Left foot plantarflexion reaches 20°. Palpation: There is 2+ tenderness of the inferior patella on the bone, no
tenderness of the medial or lateral peripatellar borders. There is no patellar apprehension and no tenderness on the joint
line, MCL, or LCL. Strength: Right leg flexion/extension and foot plantarflexion/dorsiflexion are 4+/5. Respective left
sided strength is 5/5. Special Testing: Right leg ligamentous stability tests are unremarkable. Neurologic: Bilateral
patellar and Achilles reflexes are 2+/4. Gait: Trace right-sided foot drop and intoeing.
Differential Diagnosis: Patellafemoral Syndrome, Patellar Tendinitis, Sinding-Larsen-Johansson Syndrome, Patellar
Contusion, Infrapatellar Fat Pad Syndrome, Bipartite Patella, Patellar Fracture
Test Results: MRI right knee: Complete, undisplaced fracture off the inferior pole of the right patella
Final Diagnosis: Stress fracture of the right patella
Treatment: The patient was placed in a knee brace to restrict full flexion and remained non-weight bearing for 2 weeks.
For the next 4 weeks he was limited to walking/ADLs while wearing his protective brace. After 6 combined weeks of rest
the patient was pain-free, and resumed activities as tolerated without bracing.
Outcome: 6 weeks after resolution of symptoms, the patient returned with right knee pain. Symptoms began after
sprinting out of the batter's box during a baseball game. X-ray found a new complete, undisplaced fracture of the right
inferior patella.;
Follow-Up: The patient was once again placed in a protective brace and removed from activities until pain-free. After 4
weeks of rest, he gradually returned to activities as tolerated while completing physical therapy. Stress fractures of the
patella are uncommon in athletes. They typically arise in normal bone that has been exposed to repetitive, excessive stress
(fatigue). Less commonly, they can result from abnormal bone structure or density exposed to relatively normal loading
(insufficiency). Most cases of insufficiency are secondary to an iatrogenic insult.
Complete undisplaced fractures
typically heal with non-operative treatment. Generally, patients can be expected to make a full recovery and return to preinjury levels of activity. This patient deviates from the normal course by experiencing a repeat injury. The patient's right leg
sequelae secondary to PVL could be a source of bone insufficiency, and his 7-day-a-week activity level in multiple sports
could fatigue the patella. Both of these factors may have predisposed further injury. Although now pain-free, if he fails
conservative management, the patient may require prolonged immobilization or surgery.
An Overlooked Cause of Hip Pain
Authors: Jillian E Sylvester, MD; Jesse DeLuca, DO
Affiliation: Fort Belvoir Community Hospital, Fort Belvoir, VA
History: 33 yo female presenting with progressive left hip and back pain while training for a marathon. She is an avid
runner, now 10 months postpartum. Reports running until 37 weeks gestation, resumed marathon training 3 weeks
postpartum. Insidious onset of pain over 1-2 months; endorses acutely worsened anterior hip and buttocks pain following
a half marathon two weeks ago. Treated with 1 week rest and NSAIDS. Pain returned with running, 9/10 in severity, now
extending to lateral hip and lower back. She is unable to run 1 mile without severe pain. Endorses 6/10 pain with walking,
10/10 running. Patient is otherwise healthy, medical history non-contributory. Reports breastfeeding 6 months
postpartum. Medications include prenatal vitamins while breastfeeding. Menses returned 1 month ago.
Physical Exam: Antalgic gait, right hip unremarkable Bilateral hips have normal appearance Left hip TTP over SI and
greater trochanter Passive range of motion (ROM) demonstrates full hip flexion, extension, adduction, abduction, and
internal rotation. Cannot assess external rotation secondary to pain and guarding Active ROM positive for straight leg
raise to 45°, non-painful in hip 5/5 strength, neurovascularly intact Positive FABER, negative FADIR.
Differential Diagnosis: Stress Fracture; Iliopsoas Bursitis vs. Tendinopathy; Osteoarthritis; Acetabular Labral Tear
Test Results: MRI Hip: stress changes of proximal left femur diaphysis involved in inferior portion of lesser trochanter
Bone Scan: Grade 1 stress fracture at posteromedial aspect of left proximal femoral shaft at lesser trochanter Stress
reactive changes most prominently involving the bilateral femoral shafts, right knee, bilateral tibias, right midfoot
Vitamin D, Calcium, PTH: normal
Final Diagnosis: Stress Fracture of left proximal femoral shaft at lesser trochanter; multiple stress reactive changes
Treatment: Given severe pain and radiology revealing stress fracture and multiple sites of stress reactive changes,
recommended non-weight bearing until pain improved, followed by physical therapy.
Outcome: Undergoing physical therapy, deferred marathon
Follow-Up: Pregnancy and postpartum lactation place increased calcium demands upon women in order to provide
necessary calcium to the developing fetus and neonate. Pregnant women lose 50mg-250mg per day during their second
and third trimesters; nursing mothers transfer 250mg-350mg of calcium daily through breastmilk However, prenatal
vitamins do not contain sufficient calcium to match these losses The result is a transient decrease in Bone Mineral Density
(BMD), some studies documenting 3% losses in BMD at the hip and sacrum Some studies do not document normalization
of BMD until 19 months postpartum Pathologic fractures secondary to pregnancy- and lactation-induced osteoporosis
are rare but well documented in the literature. This baseline "bone deficit" may predispose postpartum women to stress
fractures when returning to athletic activity following delivery. The benefits of exercise in pregnancy and postpartum
period are established in literature; in pregnancy, studies show that women who exercise lose less BMD as compared to
non-exercising pregnant women Postpartum, women who exercise are shown to have improved glucose control, lower
BMIs, and increased HDL levels as compared to non-exercisers, regardless of exercise intensity. Since 2002, several
governing bodies have released guidelines regarding exercise in pregnancy and the postpartum period. However, these
guidelines focus on the immediate, 6-week postpartum period The physiologic and morphologic effects of pregnancy
extend well beyond 6 weeks; for instance, elevations in cardiac output as well as decreases in BMD have been shown to
persist in women 1 year postpartum. Providers should be cognizant of the continued impact of pregnancy upon
postpartum women, including continued calcium losses through breastfeeding This perspective is valuable in helping
shape "exercise prescriptions" for women as they return to athletics postpartum Providers should have a higher degree of
suspicion for stress fractures in pregnant and postpartum women presenting with musculoskeletal pain.
12 Year Old Gymnast with Chronic Wrist Pain
Authors: Adam Fletcher, MD; Gregory Garrison, MD
Affiliation: Mayo Clinic, Rochester, MN
History: A 12 year old female gymnast presented to her primary care clinic with pain in her left wrist which has been
persistent over the past year. She had sustained an injury of the left wrist 18 months prior with hyperextension under
pronation while performing a cartwheel. She recalls a popping sensation and immediate pain with swelling in the ulnar
region. She denied numbness and tingling. Plain films were negative for fracture, and she was treated conservatively with
activity restriction, compression brace, ice, and acetaminophen. She continued to experience persistent pain in the area
two weeks later, and imaging was repeated which was again negative for fracture. She attempted to return to her normal
activities including gymnastics, horseback riding, and soccer, but would have recurrent, intermittent, sharp pain after lifting
heavy objects or attempting cartwheels and handstands Additionally, she would experience a chronic dull, aching pain at
the distal ulna with a popping or clicking sensation during pronation and supination.
Physical Exam: The general appearance of her left wrist was normal without edema, erythema or lesions. No positive or
negative ulnar variance. She maintained full active range of motion. Pain was not reproduced by passive range of motion.
Palpation of the wrist revealed tenderness to the dorsum of the distal ulna. Her wrist was otherwise nontender including
no tenderness on the radial side, anatomical snuff box, ulnar snuff box, or along the extensor carpi ulnaris tendon. She was
neurologically intact without decreased sensation to light touch throughout the left hand. Her grip strength as well as
flexion and extension of wrist and fingers against resistance were 5/5. She had a normal capillary refill. Provocative testing
included a negative scaphoid instability test and negative Finkelstein's maneuver. Piano key test was positive; with a
pronated wrist, she had approximately 1 cm of translation of the distal ulna in the palmar direction associated with pain.
Similar testing on the right was negative.
Differential Diagnosis: - Recurrent simple wrist sprain - Occult fracture - Chronic distal radioulnar joint instability Triangular fibrocartilage complex disruption - Midcarpal or lunotriquetral instability - Ulnar impaction syndrome Extensor carpi ulnaris subluxation - Juvenile idiopathic arthritis
Test Results: Plain films including AP, lateral, and oblique views reveal no fractures. There is no widening of the distal
radioulnar joint space or obvious dislocations or subluxation. Further imaging is pending.
Final Diagnosis: Probable chronic distal radioulnar joint instability
Treatment: She was encouraged to continue conservative measures and avoid exacerbating activities. She was referred
to orthopedic hand specialists for further evaluation and treatment.
Outcome: Patient evaluated by orthopedic hand clinic with persistent concern for chronic ligamental injury. She has
abstained from gymnastics for several weeks without improvement of pain.
Follow-Up: Evaluation is continuing to take place. She is scheduled for a wrist MRI with follow up shortly after the
additional imaging.
A 21 Year-Old Elite Level, Female Track Sprinter 4 Year History of Unexplained Abdominal Pain
Exacerbated by Training
Authors: Mark McEleney MD; Stephen Simons, MD; Kevin McAward, MD
Affiliation: Saint Joseph Regional Medical Center Family Medicine Residency, Mishwaka, IN
History: The patient was initially evaluated by a sports medicine physician following a 400 meter track event during
which performance was impaired by abdominal pain This athlete was currently undergoing evaluation and observation for
chronic, relapsing and worsening episodes of pain The condition spanned 4 years beginning with post exercise vomiting,
then progressing to post prandial peri-umbilical pain and eventually predominantly exercise related abdominal pain. She
previously received an extensive gastrointestinal work-up including upper GI with esophogram, gastric emptying study
and laboratory tests She carried a provisional diagnosis of gastritis for which she was treated with H2-Blockers Despite
this extensive evaluation and symptoms continued to persist.
Physical Exam: Vital signs within normal limits. Patient was well developed and well nourished with a BMI of 19.3.
Weight had fluctuated 10-12 lbs. over 4 years. At the post race exam, despite the appearance of significant pain, the
abdomen was soft without tenderness, rebound or guarding. No masses appreciated. Remainder of physical exam was
normal.
Differential Diagnosis:
Gastritis
Peptic ulcer disease
Inflammatory bowel disease
Mesenteric ischemia
Median arcuate ligament syndrome
Pancreatitis
Cholelithiasis/cholecystitis
Functional abdominal pain syndrome
Test Results: CBC, CMP, Lipase wnl H Pylori IgG: negative Gastric emptying study: normal (7/2011) Glucose
hydrogen breath test: negative (1/2014) Upper GI with small bowel follow through: unremarkable. (7/2011) Abdominal
ultrasound: two right renal non-obstructive calculi. (7/2011) Doppler studies of the celiac artery: increased flow velocity.
CT angiography of abdomen (exam performed with inspiration): severe stenosis at the origin of the celiac axis with poststenotic dilation. Superior mesenteric vein (SMV) noted to wrap around Superior mesenteric artery (SMA). Possible
intestinal malrotation. (2/2014)
Final Diagnosis: Exercise associated abdominal angina due to Median Arcuate Ligament Syndrome. The patient's
presentation and diagnostic findings indicate the possibility of vascular compromise as a source of exercise induced
abdominal pain.
Treatment: Diagnostic laparoscopy with laparoscopic release of the median arcuate ligament and concomitant
neurolysis of the celiac nerve plexus The diagnostic laparoscopy showed compression of the celiac artery by the median
arcuate ligament with no evidence of other pathology within the abdominal cavity. Post release intraoperative celiac artery
duplex exam revealed normalization of celiac artery flow velocities ensuring adequate decompression.
Outcome: 3 weeks postop the patient described complete resolution of symptoms with moderate exercise though she
did still experience infrequent vomiting in the post-prandial period and occasional pain that was less severe than
previously.
Follow-Up: Patient returned to moderate intensity sport specific training by 3 weeks after surgery. Return to activity will
continue with gradual increase in sports specific training.
Lateral Foot Pain in a Division 1 Football Player
Authors: Siatta B. Dunbar, DO; Elizabeth D. Evans, DO
Affiliation: Rutgers University - Robert Wood Johnson University Hospital
History: A freshman defensive lineman reported 4 days of increased left lateral foot pain during his pre-participation
physical exam. He described the pain as being "dull and over the outside of his left foot" Four days prior, he was running
stadium bleachers, which is a new activity for him, and he denied any twisting injury, fall or missed step He further denies
any swelling, warmth, redness, bruising, instability, decreased strength, weakness, numbness or tingling Prior to this
episode, he noted intermittent pain over the lateral foot for 4 weeks that worsened during running and physical activity
There was no pain with walking. Prior to the onset of symptoms 4 weeks ago he denied any change in activity, running
volume, running surface, change in footwear, use of orthotics or dietary modifications. He has no previous history of stress
injury or fracture.
Physical Exam: ROS: No night pain, fevers, chills, unexplained weight loss PHX: No prior foot injuries, chronic medical
illness or supplement use Height: 77.5' Weight: 341 lbs BMI:39.9 Gen: large athletic male, steady gait without a limp
Left foot/ankle: Normal appearing arch with weight bearing, normal alignment to lower extremity and no hindfoot
valgus/varus No swelling or ecchymosis over the lateral foot. Tenderness over calcaneocuboid joint and along the body
of the cuboid. Minimally tender over plantar aspect of the cuboid. Tenderness over peroneal tendon with pain graded as
6/10 with resisted eversion. No tenderness over ATFL, CFL and syndesmosis. No tenderness at base of 5th metatarsal,
medial aspect of ankle or hind foot. Full non-painful active range of motion in dorsi- and plantar flexion, inversion and
eversion. Negative talar tilt, anterior drawer and squeeze. No signs of anterior or posterior impingement. Negative hop
and tunning fork Neurovascularly intact
Differential Diagnosis:
Peroneal Tendinopathy/Strain
Midfoot Sprain
Cuboid Occult Fracture
Cuboid Stress Fracture
Cuboid Syndrome
Os Peroneum
Tumor (benign vs. malignant)
Test Results: Left foot Xray (7/14/14): Negative for fracture. Left foot MRI (7/16/14): There is diffuse marrow edema
involving the cuboid seen on T1, T2 and STIR Small transverse fracture line within the lateral mid cuboid seen on T1.
Vitamin D (7/17/14): 18 Calcium (7/17/14): 9.5
Final Diagnosis: Cuboid stress fracture
Treatment: Walking boot with no activity for 4 weeks. Obtained Vitamin D and Calcium levels. Non-impact cross-training
started at 6 weeks. Evaluated his body mechanics and was fitted for custom orthotics. Repeat xrays ordered at 12 weeks.
Outcome: Redshirted his freshman year. Continues to have pain with activity at 12 weeks
Follow-Up: Given the athletes continued pain he has been unable to return to activity. He continues to be closely
followed with repeat imaging pending at time of submission.
Uncommon Course of a Common Injury
Authors: Benjamin Petty, MD; John Hyden, MD; Nahum Beard, MD
Affiliation: University of Tennessee St. Francis/Campbell Clinic Primary Care Sports Medicine Fellowship
History: A 21 year old female rifle athlete suffered a concussive event on 10/26/2013 after falling and hitting her head in
her dorm room She did not have loss of consciousness, but had headache, nausea, irritability, and dizziness. She had a
complicated and long recovery, and complained of various somatic and affective (anxiety) symptoms. She was also seen
by a Neuropsychologist in February 2014 and was started on Escitalopram to treat her associated symptoms She
presented to the athletic training room on 3/20/2014 with worsening of her post concussive syndrome after hitting her
head on a car door. She was seen in the Emergency Department that day with symptoms of dizziness, headache, nausea,
vomiting, anxiety, irritability, and fatigue. She was given pain medication, phenergan, and anti-inflamatory medication. The
athlete was instructed not to travel and to avoid training activities. She also reports that she is trying to lose weight and is
on a modified vegan diet.
Physical Exam: Vitals: 5'4"; 155 lbs BP 129/71 Pulse 64 General: NAD; HEENT: normocephalic atraumatic, PERRLA, EOMI,
no septal deviation, no neck pain; Musculoskeletal/Neuro: Cranial Nerves 2-12 are intact, full AROM, strength 5/5 in both
upper and lower extremities, no focal neurologic deficits, no tremor, smooth pursuit testing and saccades testing
reproduced headache and dizziness without significant nausea.
Differential Diagnosis:






Severe/Moderate Traumatic Brain Injury
Depression
Post Traumatic Stress Disorder
Post-Concussive Syndrome
Arnold-Chiari Malformation
Vitamin Deficiency
Test Results: CT of Brain - normal MRI of Brain W/WO - normal CBC, CMP, Sed rate, Folate, TSH - normal B12 265pg/mL
Final Diagnosis: Post-Concussive Disorder complicated by B12 deficiency
Treatment: The patient was treated with Escitalopram, Mirtazepine, Vitamin B12 supplement, and Fish Oil. The patient
was also instructed to increase dietary B12 intake.
Outcome: The athlete had a significant improvement in her mood after 3 days of B12 supplementation. She improved
both socially and academically.
Follow-Up: She is doing well and is back competing on the rifle squad. She continues to be asymptomatic.
Persistent Knee Pain in the College Female Athlete
Authors: Bryce Meredith, DO; Wade Rankin, DO; Kelly Evans-Rankin, MD
Affiliation: University of Kentucky, Lexington, KY
History: An 18 yo African American female presented with a two month history of left knee pain Patient was playing
basketball and "came down funny on her left knee". She stated she had to come out of the game at that point and was
unable to continue playing She was seen by the athletic trainer at her college and the trainer was concerned about a
meniscal tear. Patient stated the majority of her pain was on the anterior medial aspect of the knee with some aching in
the posterior aspect of the knee. She stated that running made the pain worse, and NSAIDs did not relieve her pain, and
she has been using Tylenol for pain relief. Before presenting to the office, she had been going to the trainer on a daily
basis, where she received therapy including electrical stimulation. Patient denied previous injury to the knee.
Physical Exam: Examination of the knee showed no erythema, ecchymosis, or effusion She had full passive rom. Active
rom was limited secondary to pain. She had diffuse TTP, most notably at the anteromedial aspect and at the medial
femoral condyle She was stable with varus and valgus testing at 0° and 30°. She had a positive Patellar grind test, but
negative apprehension test. She had a negative Apley's Grind test, Bounce Home test, medial or lateral McMurry's test,
Anterior Drawer, dial test, Lachman, Pivot Shift, and Posterior Drawer. Posterior tibial and dorsalis pedis pulses were intact
Sensation was intact to light touch She demonstrated an antalgic gait.
Differential Diagnosis:









ACL Tear
Meniscal Tear
Osteochondral lesion
Patellofemoral Pain Syndrome
Patellar Tendonitis
Patellar Subluxation
Meniscal tear
Infection
Malignancy
Test Results: A knee xray showed joint effusion in the suprapatellar bursa. No fracture Joint spaces appear preserved.
A noncontrast MRI of the lower extremity identified left knee medial femoral condyle bone marrow edema with focal cyst
like change. Within the bone edema is a linear signal abnormality, which may represent a tiny subchondral fracture. Also
identified on the MRI was medial patellar facet cartilage fibrillation. A second MRI was performed 5 months following the
initial MRI and identified improving medial femoral condyle bone marrow edema and cartilage findings consistent with
early osteoarthrosis.
Final Diagnosis: Left knee medial femoral condyle subchondral fracture.
Treatment: Patient was non-weight bearing with crutches for 2 weeks. At 2-week follow up, patient reported
compliance with non-weight bearing status on crutches. Her pain had improved some but was still bothering her at night.
Continued PT with her trainer doing electrical stimulation and ice treatments. At 4-week follow up patient's pain was 5060% better, with occasional pain. She continued working with her school's athletic trainer At this point she was still
getting electrical stimulation and ice treatments and was also completing quad strengthening.
Outcome: At 2 month follow up, she was progressing her return to play, and was able to do squats, leg raises and
jogging, but she was still having some pain Repeat MRI due to continued pain and was evaluated by Ortho regarding
abnormal cartilage signal; MRI indicated cartilage findings consistent with early osteoarthrosis. Ortho told pt/parent he
had concerns for ACL tear based on exam and history and wanted to perform knee arthoscopy to repair the ACL and to
visualize the cartilage. Arthroscopy completed with anterior compartment debridement synovectomy and pad
debridement. No ACL pathology noted on direct visualization via arthroscopy.
Follow-Up: At 6 week post-op follow up patient was doing well, continuing PT and was close to returning to playing
basketball
Sacrococcygeal Pain in an Adolescent Soccer Player
Authors: Scott Simpson, MD; Devyani Hunt, MD
Affiliation: Washington University, St. Louis, MO
History: A 15 year old female developed coccydynia after participating in 8 soccer games over two days. Onset was
atraumatic; she had not fallen or collided with another player. She had difficulty walking due to pain following the final
game. Six days prior to the onset of her pain, she reported a fever and was treated with amoxicillin/clavulanate for otitis
media. She also had an inflamed area in her breast tissue with concern for infection and was switched to clindamycin to
cover both potential infections. She saw her pediatrician the day after symptom onset. Labs and x-rays obtained at that
time were unremarkable. The pain became progressively worse over the next several days and she was seen by an
orthopedic nurse practitioner. She reported continued pain walking, a decreased sitting tolerance and constitutional
symptoms including fever and chills, nausea and headaches. An MRI was obtained the following day demonstrating a left
ovarian hemorrhagic cyst and no abnormality of the coccyx. She saw her pediatrician and started oral contraceptive pills. A
follow up pelvic ultrasound demonstrated resolution of the ovarian cyst. A gynecology consult revealed no tenderness
over her ovary and felt that her pain was unlikely related to the previous hemorrhagic cyst. She was then referred to a
sports medicine specialist. She was seen by one of the authors 12 days after the onset of her pain. At that time, she
reported severe progressive sacrococcygeal pain of 10/10 intensity. The pain was exacerbated by coughing, sitting and
transitional motion when rising from a seated position. She had no associated pelvic pain or lumbar pain but did endorse
headaches, neck pain, and nausea. Her physical examination revealed tenderness over the coccyx. There was no associated
lumbar or pelvic tenderness. There was no pain with lumbar or hip range of motion. Her neurologic examination was
normal. The previous radiographs and MRI were reviewed and demonstrated an abrupt 90° anterior angle of her coccyx
between the C1 and C2 segments. A subtle increased signal on the T1 images was noted at the C1 segment. A limited
three phase bone scan was obtained, which demonstrated increased uptake in the coccyx consistent with fracture.
Physical Exam: 9/15/2014: Severe point tenderness over the sacrococcygeal region
9/24/2014: Ongoing tenderness to
palpation over the sacrococcygeal region. Normal neurologic and cervical spine examination.
Differential Diagnosis: Coccygeal fracture, referred pain from coccygeal cyst, infection (abscess or osteomyelitis), pain
related to atypical angulation of coccyx, pelvic floor muscle pain secondary to guarding from a hemorrhagic ovarian cyst
Test Results: 1. X-rays sacrum and coccyx 9/15/2014: No evidence of fracture, anterior angulation of the coccyx 2. CBC
with differential and ESR 9/15/2014: Normal (ESR 10) 3. ESR 9/24/2014: 5 4. MRI pelvis 9/19/2014: Anterior angulation
between the first and second coccygeal segments of nearly 90°, no bone marrow or periosteal edema, slight T1
hyperintensity of the first coccygeal segment, 3.8 x 2.8 x 2.7 cm left ovarian lesion felt to represent a hemorrhagic cyst 5.
Limited three phase bone scan of the sacrum, pelvis and hips 9/29/2014: Focus of increased update in the distal coccyx
consistent with stress fracture
Final Diagnosis: Coccygeal stress fracture
Treatment: Relative rest, coccyx wedge cushion, pain medications as needed, calcium and vitamin D, physical therapy
with a pelvic floor specialist once her pain improves
Outcome: Six weeks after symptom onset, her pain had improved 50% but she was experiencing intermittent abnormal
movement and popping in the region of her coccyx (she described the sensation of something shifting).
Follow-Up: She will gradually return to sports participation as her pain resolves. She will follow up 8 weeks from her last
visit.
Right Arm Pain and Weakness in a Collegiate Football Player
Authors: Jason L. Eggers, MD; Peter Seidenberg, MD
Affiliation: Penn State Primary Care Sports Medicine Fellowship State College
History: A 20 yo collegiate football player experienced the immediate onset of right upper extremity pain and weakness
after making a tackle. During the tackle, the right shoulder pad was depressed by the opposing player's helmet. There was
no loss of consciousness or symptoms suggestive of concussion. He was able to walk off the field under his own power.
He denied any neck pain. He complained of diffuse pain, numbness, and paresthesias involving the right upper extremity
as well as inability to move his right arm.
Physical Exam: No appreciable motor function was noted in the right upper extremity. No sensation to light touch was
present throughout the right upper extremity. He had normal sensory exam and strength of the left upper extremity and
bilateral lower extremities. Right radial pulse was palpable. Full cervical ROM was present in all planes.
Differential Diagnosis:
Cervical Fracture/Dislocation
Brachial Plexus Injury
Central Cord Syndrome
Test Results: Cervical xrays and CT were negative for fracture or dislocation. MRI of brachial plexus had findings of
edema around upper trunk. Lumbar MRI for new onset LBP revealed blood in the thecal sac. Brain MRI/MRA was negative
for hemorrhage. Thoracic MRI was negative. Cervical MRI with possible evidence of right C6 nerve root avulsion. EMG at 4
weeks interpreted as a diffuse brachial plexus injury. EMG at 12 weeks showed no evidence of reinnervation. CT
Myelogram at 12 weeks was suggestive of C7, C8 nerve root avulsions.
Final Diagnosis: Traumatic diffuse brachial plexus injury
Treatment: Patient was admitted the day of injury and both neurology and neurosurgery were consulted. He was treated
with a short course of oral steroids, analgesics, and placed in a sling and a wrist extension brace. During his initial 3 day
hospitalization, he recovered slight function of his finger flexors, wrist flexors, shoulder retractors and protractors.
Sensation also partially returned in the distribution of the C5, C7, C8, and T1 dermatomes. After discharge, he continued
with physical therapy directed toward maintaining upper extremity ROM and increasing strength. One week after
discharge he developed the onset of low back and buttock pain, requiring readmission for finding of blood in the thecal
sac. He was again treated with a course of oral steroids and analgesics over a 3 day hospitalization. After discharge he
resumed physical therapy for his right upper extremity.He achieved some degree of finger and thumb flexion, wrist flexion,
hand intrinsic function, shoulder elevation, protraction and retraction. At 13 weeks, surgical exploration with
electrodiagnostic testing revealed nerve root avulsions at C6, 7, and 8 as well as damage to the C5 nerve root. Using a
sural nerve graft, the damaged portion of the C5 root to suprascapular nerve and axillary nerve was repaired. Intercostal
motor nerve transfer of the 4th, 5th, and 6th intercostal nerves to the motor branch of the musculocutaneous nerve,
including the brachialis and biceps branch was completed. Sensory intercostal neurotization with transfer of the 4th, 5th,
and 6th intercostal nerves to the median nerve was performed. The spinal accessory nerve was transferred to the triceps.
At approximately 9 months, tendon transfer surgery to allow wrist and finger extension was performed.
Outcome: Surgical expectations were largely met and there were no complications.
Follow-Up: The athlete was unable to return to football activities. He regained additional limited function of his right
upper extremity including some active shoulder abduction and flexion, elbow flexion and extension. Tendon transfer
restored some limited finger and wrist extension. Sensation function in the palmar aspect of the hand improved.
Twenty-One Year Old Male Football Defensive Lineman with Right Dorsal Midfoot Pain
Authors: Brett W. Martindale, MD; Stephen D. Scharmann, MD
Affiliation: McKay-Dee Hospital Family Medicine Residency, Ogden, UT
History: The patient first developed vague right midfoot pain during Spring football practice. A walking boot was
empirically placed for several weeks by the athletic training staff, and the pain resolved over the summer. Asymptomatic,
he initiated pre-season practice in late summer; however, his foot pain recurred. He was evaluated by a podiatrist, found
to have no swelling or ecchymosis, but to be tender in the navicular-cuneiform and 3rd and 4th tarsal-metatarsal joints.
Radiographs were negative, and he was given a diagnosis of "capsulitis" of these joints. Treatment included a
corticosteroid injection into both joints He was able to resume practice without pain, but he presented one week later to
the sports medicine clinic having felt a painful pop in the midfoot while pushing off. He stated this pain was different than
what he had discussed with the podiatrist He could not push off without pain, but could bear weight. His exam was
nonspecific with tenderness diffusely over the dorsal midfoot. Radiographs were again obtained, showing pes planus, but
no fracture or other abnormalities. He was treated conservatively with ibuprofen and ice. Prior to the season's first game
one week later, he received another steroid injection (3 weeks removed from the first injection) in the midfoot region by a
team physician. Following this second injection, he was able to play in the game without pain, but the pain recurred
following the game. He presented to the sports clinic 5 days later ambulating, but again complaining of continued pain
across the dorsal midfoot. At this point he was referred for advanced imaging.
Physical Exam: Right foot with full range of motion, no significant swelling, no bruising. Tenderness to palpation over
the midfoot, specifically over the dorsal navicular-cuneiform joint with pain increased with passive midfoot supination,
resisted plantar flexion and axial load to the 1st metacarpal. Anterior drawer and talar tilt of the ankle are negative, and
the neurovascular exam is unremarkable.
Differential Diagnosis: Lisfranc ligament sprain, capsulitis of the tarsal-navicular joint, tendinopathy, tarsal or
metatarsal stress fracture, pes planus with chronic midfoot sprain.
Test Results: MRI: Complete vertical fracture with 4.5 mm of separation involving the lateral one third of the tarsalnavicular with associated osseous edema. Adjacent bone bruising is identified in the mid and lateral cuneiform without
other fracture seen. Mild bone bruising in the distal talus. Intact lisfranc ligaments. CT: Confirmed MRI findings.
Final Diagnosis: Complete tarsal-navicular fracture, likely preceded by occult stress fracture.
Treatment: Referral to orthopedic surgery. He underwent open reduction with internal fixation of the tarsal-navicular
bone requiring bone graft harvested from right calcaneus. He was casted for six weeks post-operatively. Comment by the
orthopedist suggested that the intraoperative appearance of the fracture site exhibited evidence of chronic callous and
sclerotic bony changes of long-standing duration.
Outcome: At 7 weeks post-operatively, he remains in a non-weight-bearing boot and will not begin weight-bearing with
physical therapy until 8 weeks post-operative.
Follow-Up: He continues to follow-up with the orthopedist and is healing well. However, the extent of his recovery
remains in question. His ability to return to play is uncertain at this time. A return to his previous level of football ability is
highly questionable.
Chronic Achilles Tendinosis: The Modern Achilles Heel?
Authors: Kashan Rizvi MD, MPH Donald Heitman MD
Affiliation: Hoboken University Medical Center, Hoboken, NJ
History: 54 year old female with 2 year history of left Achilles pain. The patient was diagnosed with left Achilles
tendinosis. MRI done at that time showed fusiform thickening of Achilles tendon consistent with tendinosis. In addition
there was an intrasubstance tear of Achilles tendon with focus of fluid signal abnormality measuring 1.1 cm in
craniocaudal diminesion x 2mm in transverse x approximately 2mm in AP dimension. The patient did physical therapy for
past year, 2 times a week with little to no improvement. At time of initial visit patient reported pain to worse when
walking on inclined treadmill. In addition to physical therapy patient was using nightime splinting modalities and NSAIDs.
Patient also began using wedges in shoes to help alleviate pain. Despite above treatments, pain remains 7/10.
Physical Exam: Left ankle: fusiform swelling of the Achilles tendon, roughly 5 cm proximal to insertion. No lateral,
anterior or medial ankle tenderness to palpation. Tenderness over fusiform swellingbut no tenderness distally at Achilles
insertion or at the retrocalcaneal bursa. Full range of motion compared to R ankle. Dorsiflexion 5-10°. Plantar flexion to
55°;. Inversion and eversion 20°and 25° respectively. Patient able to perform single and double leg heel raise but with
minimal discomfort
Differential Diagnosis:
Achilles bursitis
Ankle osteoarthritis
gastrocsoleus strain/rupture
Haglund deformity
Test Results: Xray left ankle: no calcifications of achilles tendon, fusiform swelling present, no signs of Haglund
deformity. No subtalar arthrosis or ankle arthrosis or OCD lesions. MRI left ankle: Severe tendinosis, interval decrease in
size of tiny intrasubstance tear of the Achilles tendon at level of tendinosis. Tenosynovitis of posterior tibial tendon.
Final Diagnosis: Severe achilles tendinosis, small intrasubstance tear of Achilles tendon
Treatment: 2 cc lidocaine injection, followed by 5 cc of PRP administered under sterile conditions in multiple approaches
both within and surrounding the Tendon. Patient placed in cam walker boot and partial weightbearing with crutches. 3
days post injection patient started on gentle stretching. 2 weeks post injection started on strengthening. Physical therapy
3x week.
Outcome: 4 weeks status post injection, patient noticed 75% improvement.
Follow-Up: Patient was able to return to activities such as Zumba, and higher level cardio which she had not been able
to do for past 2 years since getting ankle pain. Patient to continue physical therapy as needed.
Hiding in Plain Sight: An Uncommon Cause of Posterior Elbow Pain in an Adolescent Pitcher
Authors: Robert Donlan, DO; Michael W. Moser, MD; Kevin R. Vincent, MD, PhD, FACSM; Jason L. Zaremski,
MD
Affiliation: Department of Community Health and Family Medicine, University of Florida, Gainesville, FL
History: A 15 year-old male multi-sport athlete presented to our clinic with complaints of acute left elbow pain after
throwing a pitch during a baseball game and feeling a painful "pop" in his elbow. He denied any swelling, bruising,
weakness, or paresthesias. It only hurt with throwing activities. He had been seen previously in our clinic 6 months prior
with medial apophysitis in the same elbow. An MR arthrogram at the time revealed an intact ulnar collateral ligament and
he returned to play without issue following a short-course of physical therapy.
Physical Exam: He presented with no visible edema or ecchymosis over the left elbow. His active and passive range of
motion at the elbow was full in all planes without pain. Varus and valgus stress testing of the elbow, including the milking
maneuver, were negative for laxity or pain. He was neurovascularly intact throughout the left arm. He had significant
tenderness over the left posterior elbow along the olecranon and to a lesser degree over the medial epicondyle.
Differential Diagnosis:
1. Valgus Extension Overload Syndrome
2. Osteochondritis Dessicans of the Elbow
3. Medial Apophysitis
4. Avulsion Fracture of the Medial Epicondyle
5. Ulnar Collateral Ligament Injury
6. Salter Harris Fracture of the Olecranon
Test Results: Radiographs were initially obtained of the left elbow and read as normal. However, radiographs of the
asymptomatic right elbow were also obtained and showed a much smaller physis (1.1mm) compared to the left (2.1mm).
Subsequent MRI evaluation of the left elbow revealed increased edema surrounding the olecranon physis.
Final Diagnosis: Non-Union Fracture of the Olecranon Physis
Treatment: He was instructed to cease all throwing/hitting activities and heavy weight-bearing through the left arm for
3 months. A bone stimulator, in conjunction with a non-throwing PT regimen, was also utilized to promote and speed
healing. He was pain-free following this three-month period. A repeat MRI showed a significant decrease in peri-physeal
edema. He then began physical therapy with a progressive throwing program over a subsequent 8-week period. However,
he had a return of posterior elbow pain while throwing under 50% of his maximum velocity during his program. Repeat
radiographs at this time revealed a persistent open olecranon physis posteriorly.
Outcome: After discussion with our surgical colleagues, the patient underwent an open reduction and internal fixation
of the left olecranon physis due to persistent non-union eight months after initial injury. The patient underwent this
surgery and his post-surgical rehabilitation without complications.
Follow-Up: We presented a rarely reported case of non-union of the olecranon physis in a pitcher. It is imperative to
understand that the pitching motion creates tremendous forces across a joint and thus could produce malunion or nonunion of an osseous injury. If non-union does occur, surgical management is indicated given the sport specific demands
on the elbow during pitching. Understanding the biomechanics of overhead throwing can assist and tailor both nonoperative and operative treatment, as well as rehabilitation programs, to allow pitchers to successfully return to play
without set-backs.
L3 If I Know
Authors: Rhett R. Frei, DO; Brent S. Rich, MD
Affiliation: Utah Valley Orthopaedics and Sports Medicine, Provo, UT
History: 14 year old male presents with an acute low back injury that occurred approximately ten days ago He is on a
freshman football team and states when he went up for a catch, he extended in the air and felt an immediate pain in his
lower back He took the next ten days off from football but then when he returned to play yesterday, he was tackled from
behind and had an exacerbation of his lower back pain Most of the pain is in the left side of the lower back and he
describes it as sharp He denies bowel or bladder incontinence He denies numbness or tingling in the legs and has no
previous history of back complaints He has been icing and resting his back for treatment No previous past medical or
surgical history No drug use Otherwise, he is a healthy 14 year old male He is wondering what our opinion is on when
he can return to play.
Physical Exam: VITALS: Temperature 37°C, blood pressure 117/77, pulse 89, O2 sat 100% room air. GENERAL: well
developed, well nourished, in no acute distress Interactive on exam. NECK: supple, full active range of motion.
MUSCULOSKELETAL: Mild scoliosis in lumbar spine, no other visible deformities Patient reports significant lumbar pain
with active extension There is mild pain with right side bending and rotation Thoracic and lumbar spines are non-tender
in the midline Moderate left lumbar para-spinal muscle fullness and tenderness to palpation in the area Appropriate
strength in resisted hip flexion, knee flexion and extension, and hip abduction and adduction bilaterally. NEUROLOGIC:
Alert and oriented. Gait is appropriate. Sensation is intact to light touch bilateral lower extremities. Normal and symmetric
deep tendon reflexes in the patellar and achilles tendons. Negative straight leg raise bilaterally. SKIN: warm and dry. No
ecchymosis in the flanks.
Differential Diagnosis:
Acute low back strain
lumbar scoliosis
spondylosis
spondylolisthesis
Test Results: 5 views of the lumbar spine: The L3 vertebral body on the AP and lateral views has a somewhat irregular
appearance suggesting a congenital malformation Spina bifida occulta appears to be present at L5 CT lumbar spine
without contrast: No evidence of acute fracture Developmental anomaly of the L3 vertebral body/butterfly vertebral body
Incomplete fusion of the posterior elements of L5.
Final Diagnosis: 1. L3 butterfly vertebra which is a rare congenital spinal anomaly It results from the failure of fusion of
the lateral halves of the vertebral body because of persistent notochordal tissue between them. 2. Lumbar muscle strain
Treatment: Physical therapy with an emphasis on core strengthening exercises He was also prescribed Diclofenac to
take as needed for acute pain relief We recommended a flak jacket that he should wear while playing football.
Outcome: We felt like the incidental finding of the L3 butterfly vertebra should not limit him in progressing back to full
contact sport It is a congenital anomaly and is most often asymptomatic for patients.
Follow-Up: Gradual return to full contact sports as tolerated Follow up in 2-3 weeks.
Bridging the Divide - Early Repolarization Versus Coronary Disease
Authors: Ariel Nassim, DO; Tamara R. Brodsky, MD
Affiliation: Mount Sinai Beth Israel, Icahn School of Medicine at Mount Sinai, New York, NY
History: A 25 year old male with no significant past medical history presented to the emergency department (ED) for
chest pain of 1.5 weeks duration. Pain was described as intermittent, sharp, and exacerbated by exertion, associated with
fatigue and shortness of breath. Work-up was negative with EKG showing normal sinus rhythm with LVH and early
repolarization. He was discharged from the emergency department with recommendation to follow up with his primary
care physician. Five days later he was seen by his PMD and found to have new-onset elevated blood pressure of 164/84.
Labs were sent for secondary causes of hypertension, and plan to follow up two weeks later. However, later that day,
patient went to the ED for near syncope while exercising. While doing his weight lifting routine he had fatigue,
lightheadedness, and substernal non-radiating chest tightness. His symptoms resolved with rest, however with each set of
weight lifting, he noticed similar symptoms. Upon arrival to the ED his symptoms had resolved. He denied any history of
syncope but endorsed a family history significant for both parents dying of cardiac events at young age, of which the
exact etiology was unknown. He denied any recent alcohol or drug use, denied smoking cigarettes, recent travel, animal
exposure, or sick contacts. He denied any high-risk sexual behaviors.
Physical Exam: BP - 164/86 HR - 77 Temp - 97.8° RR - 12 and saturating 100% on room air.
Comfortable appearing,
in no acute distress No rashes Cardiac exam revealed a regular rate and rhythm, normal S1, S2, no murmurs, rubs, or
gallops Lung exam unremarkable No cyanosis or edema noted on extremity examination Neurologically intact
without focal deficits
Differential Diagnosis:
1. Acute Coronary Syndrome
2. Pericarditis
3. Myocarditis
4. Hypertrophic Obstructive Cardiomyopathy
5. Mitral Valve Prolapse 6. Cardiac arrhythmia (Brugada/Wolff-Parkinson White/Long QT syndrome)
7. Pulmonary Embolism
8. Aortic Dissection
9. Anxiety Disorder
Test Results: 1. EKG on admission showed normal sinus rhythm @ 66bpm, with large voltage in precordial leads,
consistent with LVH, and up sloping ST segment elevations in V2-5, which was initially interpreted as early repolarization
2. CBC, CMP, TSH, CRP, Coags, plasma metanephrines, HIV, d-dimer, Urine toxicology, and urinalysis were within normal
limits 3. Troponin peaked at 0.289ng/mL, CK was 595U/L 4. Cholesterol panel: Total - 144mg/dL LDL - 51mg/dL HDL 67mg/dL Triglycerides - 131mg/dL 5. Trans-Thoracic Echocardiography (TTE): normal LV systolic function, no valvular or
structural abnormality noted. 6. Cardiac Catheterization: long segment of systolic compression in the mid-Left Anterior
Descending artery as well as 30% stenosis in the ostium of the 1st marginal branch of the left circumflex artery.
Final Diagnosis: Myocardial bridging with systolic compression of long segment of mid-LAD
Treatment: Anti-platelet (Aspirin 81mg daily), statin (Atorvastatin 20mg daily), and calcium channel blocker (Diltiazem
120mg daily)
Outcome: Patient now asymptomatic without further episodes of chest pain or lightheadedness
Follow-Up: Patient was advised to refrain from exercise until follow up with cardiology 2-4 weeks after discharge.
Don't Let It Under Your Skin: Foot Rash
Authors: Kacie Cassady DO; Gerardo Vazquez, MD; Arthur Islas, M.; Justin Wright, MD
Affiliation: Texas Tech University Health Sciences Center Paul L Foster School of Medicine, El Paso, TX
History: 21 year old female collegiate basketball athlete from the dessert southwest presents with a pruritic rash on the
dorsum of her left foot for one week. She states the rash is more pruritic at night and has spread from between her toes to
the dorsum of her foot. She had tried an over the counter steroid cream without improvement. She denies having similar
rash in the past. She denies any sick contacts. Upon further questioning, patient states the rash has become more
pruritic over the past week. Initial onset was one month prior after a basketball tournament in Puerto Rico. Patient admits
to walking barefoot on the beach while in Puerto Rico.
Physical Exam: Left foot: raised, erythematous skin tracking with serpentine appearance between 2nd and 3rd toes,
extending proximally 5 cm onto the dorsum of the foot. The area is non-tender to palpation. There is no erythema
extending proximally up the leg. The remainder of examination is unremarkable.
Differential Diagnosis:
Tinea pedis
Scabies
Contact Dermatitis
Superficial thrombophlebitis
parasite infection
Creeping Hair
Test Results: Visual inspection provided positive identification
Final Diagnosis: Cuteaneous Larva Migrans
Treatment: The patient was prescribed Albendazole. However, due to cost, prescription was changed to Ivermectin.
Outcome: The rash improved after treatment with Ivermectin.
Follow-Up: Patient was re-examined one week after completion of treatment. The skin tracking was diminished and
pruritus resolved. Patient was educated on disease transmission. She most likely became infected with the larva during her
barefoot walk on beach that was contaminated with animal feces. We advised patient to wear sandals when walking on
the beach. She was cleared to return to collegiate basketball.
Unusual Weakness After Ankle Sprain in High School Athlete
Authors: Steven M. Albrechta, MD; Clinton A. Hartz, MD
Affiliation: The Ohio State University Department of Family Medicine, Division of Sports Medicine, Columbus,
OH
History: 16 year old male football player who presented for evaluation of left ankle and foot pain. He had inversion injury
2 weeks ago during his football game. Initial presentation was to a local urgent care. He was told he had a "fracture and
nerve damage" subsequently placed in AFO walking boot which made his pain and numbness worse. He then presented
to our clinic for further evaluation and management. He reports some numbness along dorsum of foot. Pain has since
continued and progressively worsened especially with prolonged walking. He reports numbness, aching and burning pain
that is worse at night. He also had some bruising along lateral ankle and foot which has now resolved. His leg pain is
constant and worsens with activity. He now reports the inability to lift his great toe and has worsening numbness. When
he scratches his lateral leg, he experiences foot numbness and tingling.
Physical Exam: Inspection revealed no bruising, mild swelling over anterior lateral ankle. Palpation reveals tenderness
over peroneal muscle belly and tendons and over the ATFL. Left ankle has 0° of dorsiflexion and 30° of plantar flexion. He
has 4+/5 strength with resisted ankle plantar flexion, dorsiflexion, and eversion 5-/5 inversion. The anterior drawer test
and Thompson's test are negative. There was no crepitus with motion of his ankle. There is no muscular atrophy. Pulses
are present and symmetric. Skin turgor is good. Great toe exam reveals inability to extend with full flexion and decrease in
sensation to light touch.
Differential Diagnosis:
Acute Compartment Syndrome of anterior compartment
fibula head fracture
acute ankle sprain
peroneal tenosynovitis
nerve impingement secondary to hematoma
sarcoma
Test Results: Negative tibia/fibular x-ray. Negative left foot x-ray. Negative left knee x-ray. Unremarkable limited
diagnostic ultrasound of EHL tendon and lateral ankle. MRI of left tibia an irregular contoured oval structure which courses
along the medial aspect of the peroneal tendon. It originates near the fibular head and courses inferiorly along the length
of the peroneal tendon to approximately the distal fibula shaft. It measures approximately 1.8 x 0.7 x 18.7 cm. EMG shows
injury to deep peroneal nerve in calf distal to extensor digitorum longus and proximal to EHL, axonal injury with some
portions of nerve still intact.
Final Diagnosis: Deep peroneal nerve impingement secondary to hematoma after ankle inversion.
Treatment: Start formal PT for strengthening ankle and foot, referral to Ortho foot and ankle for possible surgical
decompression, start massage therapy, activity modification with rest until pain free with ambulation once pain free with
ambulation cleared to start return to football with lace up ankle bracing.
Outcome: Good with slow improvement of great toe strength and improved sensation.
Follow-Up: Held from sport due to pain with ambulation and worsening numbness. Started PT for massage and
strengthening. He was allowed to return to sport with pain as a guide and lace-up ankle brace for support after clearance
from Orthopedic Foot and Ankle.
Bilateral Lower Extremity Numbness/Weakness - Marathon Runner
Authors: McKennan Thurston, MD; David Soma, MD
Affiliation: Mayo School of Graduate Medical Education, Rochester, MN
History: A 19-year-old female marathon runner presented to a sports medicine physician with tightness in her lower
calves as well as gradual onset of numbness and weakness in her feet with progressive running. Symptoms worsened
while training for a half-marathon. In high school, intermittent pain with running was present, but since then she has been
running longer distances, which has resulted in numbness, pain and weakness in her feet. At a comfortable pace, onset of
pain and tightness in the lower posterior calves began 10 to 15 minutes into her run. As she reached the 2-3 mile mark,
these symptoms worsened, and she would develop numbness in her feet. The foot numbness would force her to stop
running. She did some gentle stretching to partially alleviate the symptoms. The numbness seemed to be localized to the
forefoot and toes, but she felt it more on the dorsum of the feet. The left leg symptoms were worse than the right. She
denied any feelings of cold sensation or skin color changes.
Physical Exam: Bilateral lower extremities have normal appearance with no significant atrophy. Shaven legs, but no signs
of any decreased vasculature. On palpation, she is nontender over the posterior, anterior, and lateral compartments of the
calf. No tenderness over the anterior or anteromedial tibia. No tenderness over the fibula. Full symmetric ROM of the
ankles bilaterally. Pulses well palpated at the level of the femoral, popliteal, anterior tibial and posterior tibial arteries.
Dorsiflexion and plantarflexion of the ankle does not reproduce any of her symptoms. Strength is 5/5 with flexion and
extension of the ankles.
Differential Diagnosis:
1. Chronic exertional compartment syndrome
2. Functional popliteal artery entrapment syndrome
3. Stress reaction/fracture
4. Peripheral neuropathy
5. Medial tibial stress syndrome
Test Results: X-RAY Both tibias and fibulas negative. No radiographic evidence of stress reaction. Tiny benign bone
island in the proximal right tibia anteriorly COMPARTMENT PRESSURE Patient exercised for 48 minutes at a speed of 6.5
mph and an incline of 1°. Pre-exercise pressure Superficial posterior: Left 16, right 14. Deep posterior: Left 9, right 15.
Lateral: Left 7, right 7. 1-minute post-exertion Superficial posterior: Left 15, right 15. Deep posterior: Left 23, right 22.
Lateral: Left 8, right 1. 5-minute post-exercise Superficial posterior: Left 10, right 14. Deep posterior: Left 9, right 13.
Lateral: Left 12, right 12. MRI (REST AND ISOMETRIC RESISTANCE) There is complete occlusion of the popliteal arteries at
and below the knee bilaterally with resisted plantarflexion on the FIESTA imaging. This is seen comparing (neutral) resisted
plantar flexion). The popliteal veins appear attenuated with plantarflexion. </blockquote> MRI (STATIC) No abnormal
course of the popliteal artery. No muscular slips or fibrous bands are apparent. No accessory muscle. No abnormal signal
in the visualized muscles or bones. Cruciate and collateral ligaments are intact. Menisci are intact. Articular cartilage is
intact.
Final Diagnosis: Functional popliteal artery entrapment
Treatment: Patient referred to vascular surgeon. Performed wide resection of the gastrocnemius fascia in the upper left
leg. Limited fasciotomy of the superficial posterior compartment. Release of the musculotendinous bands of the soleus
muscle which compressed the neurovascular bundle. Release of the interosseous membrane. Resection of the plantaris
muscle and its tendon.
Outcome: An intraoperative duplex scan showed the distal popliteal artery, anterior tibial artery, and the tibioperoneal
trunk to all be widely patent. Diameter of the distal popliteal artery was about 3.8 to 4 mm. She maintained normal blood
flow velocities. There was no change in the caliber of the artery with forced plantar or dorsiflexion.
Follow-Up: Patient instructed on crutches post-operatively and did develop slight neuropathic pain well treated with
Neurontin. Anticipate a gradual return to running over 6-8 weeks.
Don't Throw This Breaking• Ball: A Pitcher with Acute Arm Pain
Authors: Jeffrey C. Lai, MD; Michael A. Pahl, MD
Affiliation: Long Beach Memorial Primary Care Sports Medicine, Long Beach, CA
History: A 29-year-old, right-handed baseball player (Ht 6'3" | Wt 210 lbs | BMI 26.2 kg/m2) was pitching his "best game
of the season" for his recreational hardball league. His right arm "felt great" through 6 innings as he notched 10 strikeouts.
In the 7th inning, his arm started to "feel dead," but he continued pitching through the sensation and was able to maintain
his usual 85 mph fastball against the first few batters. Subjectively, he reported throwing at 95% effort. Then, on a twostrike count with a runner in scoring position, he "gave it a little extra" on the next fastball. An audible "crack"
accompanied a painful "pop" in his right arm as he delivered the fastball. He fell to the ground in pain, unable to move his
right arm. His teammates noted a visible deformity over his arm, with excruciating pain at any attempt to move his
shoulder or elbow. Past medical, surgical, and social history: the athlete was a former college-level pitcher and currently
works part-time as a private pitching coach. He pitches with a three-quarter (3/4) arm slot. He is in good health without
any significant medical or surgical history. His fracture history includes a facial bone fracture and a thumb fracture during
his youth, both from being hit by a baseball. He is a non-smoker and denies IV drug use.
Physical Exam: He presented to the orthopaedic sports clinic a day after being evaluated at an outside emergency room
where he was given pain killers and placed in a posterior long arm splint. The right arm was swollen and tender around the
distal humerus and elbow. The skin was intact. Elbow ROM was limited due to pain. There was 2+ pitting edema of the
hand. The radial and ulnar pulses were strong; hand and forearm sensations were intact to light touch; wrist extension was
strong.
Differential Diagnosis:
Ball-thrower's fracture of the humerus
Anterior shoulder dislocation
Humeral stress fracture
Holstein-Lewis humerus fracture with radial nerve palsy
Test Results: AP and lateral radiographs showed a comminuted spiral fracture of the distal one-third humeral shaft with
large butterfly fragments.
Final Diagnosis: Ball-thrower's fracture of the humerus
Treatment: The athlete elected open reduction and internal fixation to minimize recovery time. Three days after his initial
injury, he underwent ORIF using plate and screw fixation. Exploration of the radial nerve showed no signs of injury or
contusion. The post-operative course was unremarkable.
Outcome: At 1 week post-op, the sling was removed. The arm was swollen and elbow ROM was uncomfortable, however
there was minimal weakness on triceps extension. Wrist and finger extension remained strong, and gentle ROM was
encouraged. By 2 weeks post-op, he had returned to work at his desk job. At 1 month post-op, shoulder ROM was full,
and elbow ROM was 5-135°. Pain was minimal, and he was ordered for physical therapy. Instead of formal PT, he
performed home Jobe exercises daily, initially without weights, and then progressing to holding a baseball. By 3 months
post-op, he was able to throw a ball 90 feet without pain.
Follow-Up: At the time of this writing, he is 3.5 months post-op. He is recommended for formal physical therapy to
increase his shoulder and elbow strength following a program such as the Throwers Ten. He can also begin an interval
throwing program. He has reservations about throwing a ball at maximal effort, so if he decides to return to competitive
pitching, we will recommend consultation with a sports psychologist.
Nontraumatic Left Ankle Pain in Sedentary Young Male
Authors: Sam Hwu, MD; Jessica Knapp, DO; David Wing, MD
Affiliation: Tufts University Family Medicine Residency at Cambridge Health Alliance, Malden, MA
History: 19 yo sedentary male who initially presented on January 2012 with left foot and ankle pain for 3 years He did
not recall any injury or trauma He states that the pain is intermittent and mostly occurs when he is walking At times he
feels that his foot and ankle are unstable and has a sensation of "giving out". He denies numbness or tingling of his left
foot.
Physical Exam: Patient is weight-bearing on left foot but with slight antalgic gait There is no deformity, swelling,
ecchymosis, or asymmetry noted on inspection His left foot has mild tenderness to palpation over ATFL and anteroinferior
to medial malleolus No tenderness to palpation of base of fifth metatarsal or navicular bone. Negative Tinel’s sign of
tarsal tunnel and negative squeeze test He has full range of motion with slight pain on inversion and eversion He has full
strength of the ankle and is neurovascularly intact distally.
Differential Diagnosis:
1. Anterior talofibular ligament strain
2. Deltoid ligament strain
3. Reflex Sympathetic Dystrophy
4. Stress fracture
5. Posterior tibial tendinopathy
Test Results: XR left ankle Bony defect of the medial talar dome consistent with an osteochondritis. No apparent acute
bony injury. Joint spaces are preserved.</blockquote> MRI left ankle Osteochondral lesion in posterior medial talus
measuring 5.4 x 15mm with surrounding T2 hyperintensity in the underlying bone suggesting underlying
instability</blockquote> Fluid and inflammation around tibialis anterior tendon.</blockquote> Moderate bone marrow
edema in distal cuboid and mild marrow edema in proximal fifth metatarsal. No fracture line.</blockquote> No tibiotalar
or subtalar effusion. The sinus tarsi and tarsal tunnel are unremarkable.</blockquote>
Final Diagnosis: Osteochondritis Dissecans of the talus
Treatment: 1. After XR showing OCD, MRI was ordered for further investigation. 2. The patient did not get the MRI
initially but returned for follow-up 10 months later due to worsening pain at which time an MRI was obtained. 3. NSAIDs
and Tylenol for relief of pain. 4. He continued with persistent pain and the decision was made by the orthopedist to do an
arthroscopy of his left ankle with retrograde microfracture drilling of the talar dome.
Outcome: One week post-operatively, he was transitioned to partial weight-bearing with crutches. One month postoperatively he was continued on crutches due to persistent pain and instructed to use a bike for cardiovascular
conditioning. He was then lost to follow-up and did not do physical therapy. Over the course of his diagnosis, he had
multiple ED visits for left ankle pain with XRs showing persistent OCD lesion of talus.
Follow-Up: He returned for follow-up with orthopedic surgery approximately 2 years after initial surgery with worsening
left ankle pain and instability for a re-evaluation. XR showed persistent osteochondral lesion of talar dome without new
periosteal reaction. He was referred to a tertiary care medical center for possible revision cartilage procedure.
Sequela of Auricular Trauma After Fall From Golf Cart
Authors: Scott Annett, MD; Michelle Wilson, MD; Irfan Asif, MD
Affiliation: Greenville Health System/Steadman Hawkins Clinic of the Carolinas, Greenville, SC
History: 15yo female with a history of ADHD initially presented with headache and left sided otalgia following a head
injury Five days earlier, she had fallen from a golf cart during a football game and struck the left side of her head on
concrete. She was unconscious for approximately five minutes and vomited several times She was taken to a nearby ER,
where Head CT and cervical spine X rays were negative for intracranial abnormalities, including bleeds or fractures.
Evaluation by Audiology and ENT demonstrated a left sided hemotympanum She was started on
ciprofloxacin/dexamethasone otic and it was recommended that she undergo a CT of her temporal bones, however, the
patient's family declined this study. She initially complained of nausea, vomiting, and dizziness in the ER, but these
symptoms quickly resolved She did, however, continue to complain of a headache, hearing difficulty, fatigue, low energy,
irritability, difficulty with memory, and insomnia when she was first evaluated in the sports medicine clinic. At close
interval follow up four days later, she reported that she had an inability to smile on the left and diminished taste on that
side as well These symptoms had developed soon after her last appointment.
Physical Exam: BMI 18.8 BP 108/71 HR 82
CN 2-12 grossly intact with exception of 7th nerve (ipsilateral mouth
drooping, smoothing of the brow, and flattening of the nasolabial fold); PERRLA, strength of UE and LE's B is 5/5 and
equal, sensation to light touch intact, nl reflexes, nl tandem gait, negative Romberg, nl cerebellar testing to include RAM's,
finger/nose pointing, and heel/shin
Differential Diagnosis:
1.
2.
3.
4.
5.
Ramsay Hunt Syndrome (herpes zoster oticus)
Viral Meningitis
Facial Nerve Paralysis associated with Temporal Bone Trauma
Concussion
Chronic Otitis Media
Test Results: Head CT- sphenoid sinusitis, negative CT of the Brain otherwise Cervical XR- 3 views, mid cervical
scoliosis concave on the left; negative for acute fractures L Elbow XR, 3 views- negative for acute fracture L Hand XR, 3
views- negative for acute fracture ER Labs- WBC 24.3K, Hgb 13.0, Hct 37.3, Plts 372 Na 139, K 3.3, Cl 102, Bicarb 27, BUN
10, Cr 0.8, Glucose 120, Ca 8.9
Final Diagnosis: Delayed Onset Facial Nerve Paralysis in setting of Concussion and Acute Traumatic Head Injury with
Hemotympanum
Treatment: Pt was diagnosed with a concussion in the setting of her head trauma and was kept out of activity and
school as well as counseled on cognitive rest Her methylphenidate was held Her symptom severity score was 28 utilizing
a SCAT3 She was placed on a 10 day taper of prednisone for her facial nerve palsy.
Outcome: Her facial nerve palsy and concussive symptoms resolved over the next week.
Follow-Up: She was cleared to return to school ten days after her initial injury She had follow up with Audiology and
ENT, and they noted that her hemotympanum had resolved and cleared her for all activity She was doing well at school
without symptoms, so her methylphenidate was restarted and she was cleared to begin a graduated return to play
protocol. Facial nerve palsy following an acute head injury is easily explained, but the mechanism for delayed facial nerve
paralysis has not been well documented It may be related to an expanding hematoma that compresses the facial nerve
and causes ischemic changes Early recognition and treatment is essential, especially for PCSM physicians who are seeing
an increasingly high volume of head injuries in their sports medicine clinics.
Atypical Foot Pain in a Female College Soccer Player
Authors: Alison S. Lee, DO; Bill Dexter, MD
Affiliation: Maine Medical Center, Portland, ME
History: Female college soccer player with 2 months of right medial heel pain. She recalls having a mild inversion ankle
sprain and noticed medial pain develop after she started taping her ankle The athletic trainers thought she was
developing an Achilles tendinitis, so rehabilitation was initiated Despite local treatments her pain worsened as the season
progressed. Her pain was primarily worse with running She denied numbness or tingling She had some posterior ankle
swelling At this point her exam seemed consistent with retrocalcaneal bursitis, and initially she was treated with local and
systemic anti-inflammatory therapy, rehab exercises, and taping of her medial arch Her pain continued so a diagnostic
ultrasound-guided lidocaine injection to the retrocalcaneal bursa was performed She experienced a slight reduction in
pain after this injection suggesting a retrocalcaneal bursits was contributing to her pain A cortisone injection under
ultrasound guidance {ultrasound images to be included} was subsequently performed Her pain, however, quickly
returned, and she was unable to play soccer for longer than 20 minutes and began having pain with standing and walking
Her past medical history was significant for right lower extremity exertional compartment syndrome 3 years ago, which
subsequently led to fasciotomy.
Physical Exam: Examination revealed mild fullness in the posterior ankle There was tenderness of the proximal
calcaneus, medial greater than lateral She had tenderness along the ankle joint line There was mild tenderness of the
distal Achilles tendon but none at its insertion. She had mild tenderness in the area of the retrocalcaneal bursa. Active and
passive ROM was restricted in dorsiflexion by 10° compared to the left side Otherwise she had full ROM She had pain
with active supination and forced plantar flexion There was no ligament laxity Her neurovascular exam was intact.
Differential Diagnosis: Persistent retrocalcaneal bursitis; Poor mechanics/ankle instability after incomplete ankle
rehabilitation; Achilles tendinopathy; Nerve entrapment; Flexor hallucis longus tendinopathy; Calcaneal stress fracture;
Talar dome osteochondral defect; Plantar fasciitis; Posterior tibial tendon dysfunction
Test Results: X-RAY of calcaneus {x-ray image to be included} was normal MRI of ankle {MRI images to be included}
revealed 1.3 cm cystic structure within calcaneal body with adjacent edema. Moderate ankle joint effusion. Mild Achilles
tendinosis. Mild retrocalcaneal bursitis.
Final Diagnosis: Calcaneal cyst with adjacent edema concerning for increased risk of fracture; Mild Achilles tendinosis;
Mild retrocalcaneal bursitis
Treatment: Patient was instructed to rest completely She was put into a short-leg boot and instructed to be non
weight-bearing.
Outcome: She currently is non weight-bearing Once she is pain-free at rest, she can attempt weight-bearing while
wearing the boot She will continue to progress in a step-wise fashion, being pain free at each step.
Follow-Up: Her soccer season is over She will not return to full weight-bearing activity until she is pain-free, and after
repeat MRI at 4-6 weeks demonstrates resolved edema surrounding the cyst Given the size and location of the cyst it is
unclear if further more invasive treatments are necessary It is still unclear why there was an ankle effusion Though these
cysts are common, they uncommonly are symptomatic It is not clear why she developed symptoms, but her ankle injury
preceding the onset of pain may have caused a change in her mechanics, possibly precipitating her symptoms. If she
remains symptomatic despite rest, or if pain returns in the future, surgical intervention may be necessary.
Controversial Return to Play in an Epileptic Rower
Authors:
Affiliation: University of Iowa, Iowa City IA
History: 18-year old female division 1 rower presented for her entry PPE in August. She reported a history of being
diagnosed with epilepsy (complex partial seizures with generalization) diagnosed in 2008 at age 13. Since then she had
had 3 seizures the last of which occurred in the locker room after a swim practice in 2010. She was cleared for full
participation in rowing. Subsequently she presented with a breakthrough seizure during practice in September. This was a
complex partial seizure lasting 60 seconds. She had a sense that a seizure was going to happen and alerted her trainer and
was escorted into the training room. She had her typical post ictal symptoms of headache and fatigue. She had 2 two
additional witnessed seizures over the next two weeks. These all occurred on dry land during high end interval work. She
had one additional suspected seizure in her sleep when she woke with headache having bit her tongue.
Physical Exam: General: Well appearing 18-year old female, A&Ox3, no acute distress Eyes: EOMI, PERRLA, sclera not
injected. Full visual field with confrontation Heart: RRR, clear S1/S2, no murmurs Lungs: CTA-bilaterally Neuro: CN II-XI
intact, face symmetric. Reflexes at C5-C7, L4, L5, S1 2/4 bilaterally. Normal finger-to-nose. Normal rapid alternating hand
movements. Normal gait. Negative Romberg. Normal tone and power throughout upper and lower extremities with
muscletesting. Sensation intact to light touch bilaterally at C5-T1 and L2-S1.
Differential Diagnosis:
Breakthrough seizure
Pseudo seizure
Conversion disorder
Hyperventilation
Test Results: EEG-normal
Final Diagnosis: Breakthrough Seizures
Treatment: Neurology Consulted - Lamotrigine increased to 200 mg BID
Outcome: Neurology consult, increased Lamotrigine to 150 mg in the morning in addition to 200 mg dose in the
evening and provided Midazolam intranasal rescue medicine. After one week, neurology increased medication to 200 mg
twice daily. The patient had two more seizures while her medication was ramping up, but none since. She had an EEG
which was normal.
Follow-Up: A discussion was had with the patient involving neurology and our sports medicine team regarding
allowing her to continue rowing. Given her predictable auras and strong desire to row, the patient elected to continue
dry land practicing. The patient understood the risks of having a seizure while in the boat on the water. It is controversial
to allow a patient with epilepsy to participate a sport such as rowing given the possibility of drowning. It is even more
controversial to allow a person with uncontrolled epilepsy to continue in a sport such as rowing. The sports medicine team
will allow her to progress back toward full participation as long as her seizures return to a well-controlled state. Things
taken into consideration in this case: 1. The patients feet are securely strapped into the boat. 2. The large boat in which
she was rowing is deep and not easily capsized. 3. She is not alone in the water. 4. She has predicable auras. 5. The
patient and her family were counselled and were in favor of her continuing to row with full knowledge of the risks. 6.
Teammates will need to be educated and comfortable with her in the same boat and having a potential seizure.
Headache in a Rower
Authors: Amrit Bhardwaj, MD, MS; Andrew Reisman, MD, ATC; David Webner, MD; Kevin DuPrey, DO
Affiliation: Crozer-Keystone Health System, Springfield, PA
History: An 18-year-old female collegiate rower presented to the University's sports medicine clinic with new onset of
headaches and diplopia. She underwent wisdom teeth extraction three weeks prior, and originally attributed headaches to
her surgery. Her post-extraction course was also complicated by a syncopal episode. She noted a distant history of
migraine headaches. She reported resolution of post-surgical headaches before returning to college, however, her
headaches returned two days prior to presentation during crew practice and she developed diplopia for the first time.
Physical Exam: Eye examination revealed no discharge, PERRL, EOMI, and fundi were normal. Convergence and tracking
were normal, with no nystagmus.
Differential Diagnosis: 1) Viral Labyrinthitis 2) Migraine Headaches 3) Concussion 4) Intracranial Space Occupying
Mass 5) Idiopathic Intracranial Hypertension (Pseudotumor Cerebri) 6) Meningitis
Test Results: At scheduled follow up 2 days later in sports medicine office, she was still symptomatic with headaches,
diplopia and had developed mild tinnitus Fundoscopic exam revealed optic disc edema She was referred to
ophthalmology the same day for urgent evaluation Visual acuity was 20/30 OD and 20/20 OS. Dilated fundoscopic exam
revealed bilateral optic disc swelling. She was sent to the ED for further evaluation and imaging. In ED, a brain MRI/MRV
revealed prominent optic discs bilaterally, with no other abnormalities LP revealed an opening pressure of 530 mmH2O
(normal &lt; 250 mmH2O). CSF analysis was within normal limits: clear, 8 RBCs, 2 WBCs (no PMNs, 12 monocytes, 78
lymphocytes), glucose 52, protein 27.
Final Diagnosis: Based on history and physical examination, the working diagnosis was idiopathic intracranial
hypertension (pseudotumor cerebri). This was attributed to the minocycline that she was prescribed for her acne.
Treatment: During the LP, CSF was drained to decrease pressure to 200 mmH2O. She had improvement of her
papilledema after the therapeutic LP. She was started on acetazolamide 500mg twice daily and her minocycline was
discontinued.
Outcome: Ten days after her ED visit, she saw a neuro-ophthalmologist for consultation. He ordered a Head CTA/V,
which was normal. On examination, she had 20/20 OD and 20/20 OS, but noted to have 3+ papilledema bilaterally and her
acetazolamide was increased to 2500 mg/day to maximize medical treatment prior to possible optic nerve sheath
fenestration surgery She saw several doctors in the following 2 months, including neuro-ophthalmology and
neurosurgery. She was given prism glasses for diplopia and was noted to have an enlargening blind spot. Cerebral MR
venography, revealed a transverse venous sinus stenosis possibly preventing proper drainage of CSF, and was deemed a
candidate for transverse venous sinus stenting or VP shunt placement The patient chose medical management and her
acetazolamide was further increased to 3,000 mg/day. She underwent two additional therapeutic LPs at 2 months and 4
months after diagnosis.
Follow-Up: While on high dose acetazolamide, she developed severe exercise intolerance. She noted shortness of breath
with walking, was unable to participate in crew, play saxophone in marching band, or tolerate even 1-2 minutes of
exercise. These symptoms were attributed to the metabolic acidosis caused by acetazolamide. Five months after diagnosis,
her vision abnormalities and blind spot had completely resolved. Over the course of 6 months her acetalozamide was
tapered. Once acetalozamide was discontinued, she began exercising again and was able to tolerate 5 minutes of running.
However, one month after discontinuing acetazolamide her headaches and tinnitus returned. Her ophthalmologist noted
normal appearance of her optic nerves, but a recurrence of her visual field blind spot. She was restarted on 500mg of
acetazolamide twice daily. Currently, she is still unable to return to activity due to exercise intolerance.
Upper Extremity Paresthesia in 16 Year Old Female Semi-Competitive High School Swimmer
Authors: Joseen A. Bryant, MD; Aaron Lee, DO; Danielle Bass, MD
Affiliation: MacNeal Hospital Sports Medicine, Berwyn, IL
History: A 16YO R-hand dominant female swimmer arrived to training room with complaint of progressive pain in Rupper extremity for one month. No known injury nor increase in training intensity. Onset was sudden with
numbness/tingling over medial aspect of right arm down to 5th digit of right hand. Maximum level of discomfort was
8/10, most notably when swimming and right arm in full forward flexion/abduction of swimming stroke. Noted occasional
weakness in her right upper extremity but attributed this to discomfort. Decreasing the frequency of her training swims
and massage over her hand produced mild improvements in abnormal sensation. She denied any discoloration of her arm
or other exacerbating factors. This athlete had been swimming for her high school team for years without any previous
history of upper extremity injury nor similar symptoms. Review of symptoms beyond musculoskeletal complaints was
noncontributory.
Physical Exam: Athlete in no acute distress with no asymmetry, deformity, discoloration nor atrophy of upper
extremities noted. Positive painful arc, drop arm, Hawkin's, Neer's and Jobe's, Spurling's (left rotation and left lateral bend)
maneuvers with pain and paresthesia down medial aspect of right upper extremity to ulnar portion of 4th digit and
radial/ventral/ulnar portions of 5th digit. Radial pulses intact (2+) and symmetric. Decreased sensation over medial aspect
of right upper extremity (compared to left) with 1+ biceps deep tendon reflex compared to the contralateral 2+ biceps
tendon reflex. Follow-up in Sports Medicine Clinic (after 1 week of complete rest from overhead activity): BMI 32.6
(98%tile; Ht 162.56cm, Wt 86.18kg). No acute distress. Full range of motion at cervical spine. Noted spasm over right
paravertebral musculature with palpation near base of neck eliciting paresthesia down right medial upper extremity.
Again, sensation to light touch decreased over medial aspect of right upper extremity down to 5th digit. Biceps tendon
reflexes symmetric. Adson's and Roos maneuvers negative, but Spurling test again positive with L-chin rotation and Llateral bend of cervical spine. Upon inspection of upper back/shoulder/chest, no asymmetry nor atrophy noted. Negative
painful arc, drop arm, Hawkin's, Neer's and Jobe's tests. However, increase in R-scapular fatigue/winging (in comparison to
left) with repetitive shoulder shrugs in varying directions. Strength normal throughout. Negative Tinel's sign at the ulnar
tunnel. No detected atrophy or weakness in hand grip or digit adduction/abduction on exam. Radial pulses were 2+ and
symmetric.
Differential Diagnosis: Right Thoracic Outlet Syndrome (neurogenic v venous); Right Brachioplexopathy; Right Ulnar
neuropathy
Test Results: Multiple views of cervical spine and R-shoulder obtained and assessed as within normal limits. MRI without
contrast of cervical spine/neck soft tissues (inclusive of brachial plexus level) assessed as within normal limits with noted Rinternal jugular vein enlargement (approximately 4x size of left) was deemed a normal variant by radiology. Also noted
was the incidental finding of low lying cerebellar tonsils which may be consistent with Chiari malformation type I. However,
this is not believed to be major contributor to this athlete's symptoms as the constellation of symptoms for Chiari (severe
headaches, neck pain, unsteady gait, poor hand coordination, vision/speech abnormalities, etc.) are not consistent with
this athlete's case.
Final Diagnosis: 16yo Right Hand Dominant Swimmer with possible Venous v Neurogenic Thoracic Outlet Syndrome
secondary to right scapular dyskinesia resulting in scalenus anticus with inferior trunk of right brachial plexus (or C8 nerve
root) compression.
Treatment: To date, rest from overhead activity and targeted physical therapy.
Outcome: Improvement of symptoms after 2wks of physical therapy. Further management pending and dependent upon
symptom improvement after continued targeted physical therapy.
Follow-Up: Yet to be determined. If plateau of improvement occurs, there will be consideration for electromyography.
Genetic Shoulder Pain?
Authors: Emily Martin, MD
Affiliation: Northwestern Medicine, Chicago, IL
History: A 29-year-old healthy man presents to the sports medicine clinic with bilateral shoulder pain. He states the pain
has been present for at least 10 years, but has gotten progressively worse over the past 3 years. Both of his shoulders
reportedly "pop in and out" but he has never fully dislocated them. This sensation has started to occur more frequently
over the past few years and he also reports decreased grip strength on the right. He has been able to continue exercising
and is active in CrossFit, which he thinks helps, but his limitations are increasing, particularly with overhead activities and
weight lifting. He reports no previous traumatic injury or fall related to the pain and has no neck pain or numbness and
tingling down his arms. He has no past medical history but takes Propecia, a daily multivitamin, Osteo Biflex Joint Shield
and omega-3 fatty acids every day. He works as an architect and does not drink alcohol or smoke cigarettes. His family
history is unremarkable and he has no allergies to medicine.
Physical Exam: He is a well-appearing man in no acute distress. His smile is transverse, mouth is weak and eyelid closure
is weak. He can barely whistle. No cervical spine tenderness, negative Spurlings. He has a stooped posture with shoulders
sloping forward (R &gt; L). Scapulae are medially and anteriorly rotated with scapular winging at rest. Superior angle of
bilateral scapulae are protruding posterior to clavicle and are non-tender to palpation. Both shoulders have full and equal
ROM, however his shoulders sublux and reduce during full abduction. This improves with scapular stabilization. The
strength of his rotator cuff muscles is 5/5 bilaterally. He is unable to flex his right shoulder when internally rotated (Neers).
No sulcus sign and negative apprehension/relocation sign bilaterally. Deltoids, biceps, triceps demonstrate 5/5 strength
but trapezius is high riding due to weak rhomberg and serratus anterior. Wrist flexion/extension and finger extension
strength is 5/5. Lower extremity strength is 5/5 with 2+ DTRs. Normal sensory and circulatory exam.
Differential Diagnosis:
Multidirectional shoulder instability
Scapulothoracic dysfunction
Muscular Dystrophy
Brachial plexopathy
Long thoracic nerve palsy
Test Results: Bilateral Shoulder XR: No acromioclavicular joint arthritis, well-maintained glenohumeral joint, scapulae are
medially and anteriorly rotated so glenoid is anterior facing (R &gt; L). No fractures or dislocations. EMG/NCV: Normal;
Genetic testing for FSHD deletion mutation: Positive
Final Diagnosis: Facioscapulohumeral Muscular Distrophy (FSHD)
Treatment: Supportive care: occupational therapy, physical therapy, aerobic exercise, NSAIDs prn Neurology follow up
in Muscular Dystrophy Association (MDA) Clinic; Genetic counseling
Outcome: Patient is doing well in occupational and physical therapy. He is still able to do CrossFit and other exercises.
He plans to go to genetic counseling and will follow up in muscular dystrophy clinic.
Follow-Up: MDA clinic follow up. Return to activity as tolerated.
I Have a Lump in My Neck
Authors: Wazim Buksh, MD, MPh; Dean Padavan, MD; Moneef Hauter, MD; Ramon Julian Pesigan, MD
Affiliation: Atlantic Health Systems, Department of Sports Medicine, Morristown, NJ
History: A 41 year-old Caucasian woman presents with a three month history of left sternoclavicular (SC) joint pain and
swelling. She states the pain is non-radiating and a 6-7 out of 10 in intensity. She denied any history of trauma, fever or
chills. Interestingly, in 2007, she presented with a similar complaint where a work-up, which included x-rays and an MRI,
revealed Osteitis Condensans, a rare condition that affects the medial end of the clavicle. At that time she was treated with
NSAIDS and improved over several months. She felt as if her current symptoms were very similar to her episode in 2007.
She denies any rashes, recent infections, drug abuse, or travel history. Her pain was worsened with overhead movements
and improved with rest. She was then started on an NSAID and physical therapy. The patient was instructed to follow up
in six weeks but returned to the clinic in three months, stating that she had no improvement whatsoever. She also
admitted to a subjective fever of 103°F upon follow up.
Physical Exam: Vitals: T: 37.6; HR: 86; RR: 14; BP: 127/62; HT: 5'10", WT: 220 lbs.; BMI: 34.4 General: Well appearing,
overweight woman in NAD Neurological: AAO x 3, CN II-XII grossly intact; Sensation intact; Strength 5/5 in deltoid,
biceps, triceps and wrist extensors. Reflexes 2+ and symmetric HEENT: EOMI, PERRLA, Anicteric Sclera Lymph: No
lymphadenopathy CVS: S1S2+, RRR, NO M/R/G/B/JVD LUNGS: CTA No W/R/R VASCULAR: No bruising or ecchymoses.
+ pulses B/L SKIN: No Rash or Jaundice MSK: + tenderness approximately 1 ince from left SC Joint and over the SC
joint. No warmth/erythema. +Prominence over SC joint. ROM of shoulder was WNL. SPECIALTY EXAM: +pain on
crossover portion of O'Brien's Test.
Differential Diagnosis:
SC joint separation
SC joint synovitis
SC joint Arthrosis
Osteitis Condensans
Osteomyelitis
Sternoclavicular hyperostosis
Bone cysts
Friedrich's Syndrome
Osteochondroma
Test Results: -ESR: 25 -Vit D: 27.5 -CBC showed a WBC count of 10 with a differential of 50% neutrophils, Hgb of 13,
Hct of 45 and a platelet count of 200. -All other Labs wnl Needle Aspiration Anerobic Cultures: Proprionibacterium Acnes
<br
Final Diagnosis: SC joint infection vs. Osteitis Condensans
Treatment: The patient was started on IV daptomycin for four weeks. This was done after consultation with our
colleagues from Infectious disease.
Outcome: After two weeks of treatment with IV antibiotics, the patient still reported no improvement of her symptoms.
She will continue antibiotic therapy for two more weeks.
Follow-Up: If the patient reports no improvement in her symptoms, she will be referred to cardiothoracic surgery for a
bone biopsy of the affected area.
Hip and Lower Abdominal Pain in a Professional Soccer Player
Authors: Kris A. Homb, MD; Lawrence W. Frank, MD
Affiliation: Spine and Sports Physiatrists, Elmhurst, IL
History: A 27 year old otherwise healthy male professional soccer player presents with a seven month history of right
lower abdominal and hip pain The patient's symptoms developed gradually without a specific injury He complained of
intermittent aching and stabbing pain in his deep right lower abdomen aggravated by running for 30-60 minutes His pain
was nonradiating and an 8/10 on a numeric pain intensity scale He denied numbness and tingling in his bilateral lower
extremities Previous treatments included rest, ice, stretching, and chiropractic care, all with temporary improvement His
past medical history was significant for a right sports hernia repair and right knee arthroscopy, seven and five years ago,
respectively. Prior to our visit, he was seen by a general surgeon who ruled out a recurrent sports hernia.'
Physical Exam: Vitals signs were within normal limits The patient was a well-developed and nourished male Abdominal
exam revealed a point of maximal tenderness deep to the right lateral edge of rectus abdominis muscle 3-4 inches
superior to its pelvic insertion There was no palpable mass, abdominal wall defect, or rebound tenderness Lumbar spine
exam revealed incidental right low back pain with extension Right hip exam revealed rectus femoris and iliopsoas
contractures, but negative FABERS, FADIR, and grind tests Hip range of motion was normal. Muscle strength testing was
5/5 throughout his bilateral lower extremities. Right iliopsoas stretching and manual muscle testing reproduced his pain.
Differential Diagnosis: Differential diagnosis consisted of recurrent sports hernia, spigelian hernia, iliopsoas strain,
iliopsoas tendinitis, external oblique abdominal strain, iliopectineal bursitis and pelvic/hip stress fracture.
Test Results: MRI of the abdomen revealed normal but prominent psoas musculature encompassing approximately
two thirds of the abdominal cavity volume without tears, tendinopathy, or other abnormalities. Limited views of the
lumbar spine did not show any obvious stenosis or disc herniations. MRI of pelvis revealed an unremarkable hip, no hip
fluid accumulation, no hip tendinopathy, no psoas tear, no evidence of stress fractures. Diagnostic ultrasound revealed
no evidence of anterior labral tear, subluxing or snapping psoas tendon, femoral head deformity, distal iliopsoas tear, or
abdominal wall defects adjacent to the rectus abdominis.
Final Diagnosis: His working diagnosis was proximal psoas strain with secondary facet mediated back pain related to
altered spinal biomechanics.
Treatment: A right iliopsoas trigger point injection was performed just inferior to the inguinal ligament with modest
improvement, however, he was unable to return to full sports participation. Due to persistent pain, it was determined to
perform repeat trigger point injections of the proximal intra-abdominal portion of the right psoas just anterior to the L3,
L4, and L5 transverse processes with fluoroscopic guidance We concurrently treated his facet mediated symptoms by
performing right medial branch blocks at L3, L4, and L5.
Outcome: After the injections, the training staff noted marked improvement in hip flexor length with symptom relief, and
the patient was able to return to full sports activity within 10 days.
Follow-Up: He continued to report 100% improvement of symptoms when seen in follow up 1.25 years later.
Conclusion: The psoas muscle may be a primary pain generator for athletes with unexplained lower abdominal and groin
pain and should be included in the differential diagnosis As the vast majority of the muscle is intra-abdominal, and the
standard injection approach to the muscle is in the most distal portion, proximal psoas trigger point injections with
fluoroscopic guidance may be useful in the management of psoas muscle pain.
Interesting Preparticipation Exam in an Incoming NCAA Volleyball Player
Authors: Stuart Lisle, MD; Christopher McGrew, MD; Shane Cass, DO
Affiliation: University of New Mexico Primary Care Sports Medicine Fellowship, Albuquerque, NM
History: An 18-year-old female volleyball player presented for her incoming preparticipation exam She noted that 8
days prior to the exam, she had actually sustained a head injury while playing in a club team game She was hit in the
head with a ball and immediately did not feel right, but continued to play and actually sustained an injury to her right foot
later in the game She felt like she rolled over on her foot in a plantar flexion and abduction motion, and noted pain,
swelling and difficulty bearing weight She did not recall a specific pop As for her head injury, several symptoms were
noted including balance problems, photophobia, and headache She filled out a standardized symptom checklist and
noted 22 total symptoms with a severity score of 84 She denied any prior concussion or foot problem.
Physical Exam: Her complete neurologic exam was normal aside from her balance, which was not performed due to the
foot pain Her right foot did not appear to have any swelling or bruising; she did have tenderness to palpation over the
medial tarsometatarsal joint and pain with flexion and abduction. She walked with a mild limp Otherwise, her full physical
exam was normal.
Differential Diagnosis: -Concussion, subdural hematoma, structural abnormality of the brain, depression
-Lisfranc
injury, midfoot sprain, metatarsal fracture, posterior tibialis tendon dysfunction, navicular or cuboid injury
Test Results: Standing films were obtained of the bilateral feet, which appeared to show slight widening at the Lisfranc
joint on the affected side The official radiology report was normal An MRI showed a nondisplaced stress fracture of the
proximal second metatarsal and marrow contusion of the base of the second metatarsal at its articulation with the lateral
cuneiform. A structural MRI was performed of her brain as part of a research study This showed an abnormality within
the right frontal subcortical area and a follow up MRI with contrast then showed a small, oval area of hyperintensity in the
right frontal subcortical white matter without mass effect or abnormal enhancement.
Final Diagnosis: 1. Concussion 2. Second Metatarsal Stress Fracture 3. Brain Lesion
Treatment: 1. She was held out of classes until her symptoms were not worsened by mental function Her symptoms
resolved over the course of about 3 weeks from the date of the initial injury and then a modified return to play protocol
was performed due to her foot injury. 2. She was initially placed in a cast boot for six weeks Subsequently, she was
allowed to progressively return to full activity. 3. A Pediatric Neuro-Oncologist saw her for the abnormal MRI.
Outcome: 1. She returned to full activity without problems.
2. She had some soreness and a click in her midfoot about
three months after the initial injury This was felt to be due to some articular damage at the time of the injury, but that the
fracture itself was healed. 3. The Neuro-Oncologist felt the lesion was most likely secondary to the head injury and not a
neoplasm.
Follow-Up: 1. She returned to activity as noted earlier.
2. She slowly returned to full physical activity. If the soreness
and clicking persists after the season, likely a repeat MRI will be recommended. 3. A repeat MRI in 6 months was
recommended.
A Trivial Blunt Trauma Injury in an Athlete
Authors: Ricardo Garcia, DO
Affiliation: Texas A&amp;M Family Medicine Residency Bryan, TX
History: 17 yo football player presented to the ER for evaluation after sustaining blunt trauma to the perineal scrotal
region during football practice. Initial injury was obtained after a player running in front of him fell hitched against the
ground with foot in the air and struck him in the scrotal perineal region. He thereafter, was found to have blood in the
meatus with spontaneous discharge prompting him to present to the Emergency Room with coaching staff. A urinalysis
demonstrated gross hematuria per the ER physician when spontaneously voiding. He denied sensation of incomplete void,
fever, or significant discomfort by the time he arrived to the hospital. Scrotal ultrasound, cystogram, and retrograde
urethrogram were immediately performed.
Physical Exam: Gen: Alert, anxious, in mild distress while on fluoroscopy table.
CV: Regular rate and rhythm. Resp:
Normal respiratory effort/pattern, Clear to auscultation Bilaterally. Abdomen: Soft, nontender, nondistended. No gross
suprapubic distention; No evidence of ecchymosis of the abdominal or perineal region appreciated. Genitourinary: Blood
at the Meatus. Penile shaft without ecchymosis or swelling. Testes are palpated within the scrotal sac bilaterally and there
is no gross evidence of testicular trauma, no intratesticular mass. There is minimal tenderness of the testes without scrotal
edema, ecchymosis or induration. Perineal inspection reveals some tenderness elicited at the level of the bulbous urethra;
however, there is no gross evidence of ecchymosis or hematoma in this region. Musc: Full ROM elicited with 5/5
strength in upper and lower extremities. No cyanosis, clubbing, or edema. Neuro: No gross focal deficits appreciated.
Differential Diagnosis:
Pelvic/Pubic Rami Fracture
Penile shaft/Urethral/Bladder/Prostate injury
Pelvic Hematoma
Urogenital vasculature injury
Test Results: Scrotal Ultrasound, Bedside Flexible Cystoscopy/Cystogram, and Retrograde Urethrogram performed
revealing contrast leakage and extravasation at the level of the base of the bulbous urethra.
Final Diagnosis: Partial Anterior Bulbous Urethral Laceration with associated gross hematuria secondary to blunt
trauma.
Treatment: Prophylactic antibiotics, placement of Indwelling Foley and Suprapubic Catheter for 4-6 weeks.
Outcome: Patient sustained a partial bulbous urethral laceration secondary to blunt trauma while at football practice that
was treated conservatively with indwelling Foley and suprapubic catheter (vs surgery). Retrograde Urethrogram and
Cystogram for repeat staging 5 weeks later revealed normal healing over the mucosal irregularity seen at the bulbous
urethra. No evidence of residual stenosis or tear was seen with normal demonstration of voiding without pain.
Follow-Up: Patient returned to full activity and play one week later, approximately 6 weeks after initial injury. No further
complications. Follow up as needed.
Just A Little Knee Pain...
Authors: Kelly Ryan, DO; Valerie Cothran, MD
Affiliation: University of Maryland Primary Care Sports Medicine Fellowship
History: Patient is a 12 year old male who presents with several weeks of left knee pain and intermittent swelling He
initially denies any specific injury but later reveals an incident where he landed on that knee during football. He describes
instability, popping , and catching He had difficulty with full range of motion until swelling improved HIs symptoms were
worse with activity and improved with rest. Treatment by family and PCP included ice, rest and knee sleeve, which together
improved his pain and he was able to return to football He was unable to describe where his pain was initially but signals
over the patella.
Physical Exam: Patient is a well appearing male in NAD; He is alert and oriented. He has moderate effusion but no
erythema; Left Knee: He is non tender over the entire knee; FROM 0°-130° extension to flexion Intermittent clunk noted
with active extension;; Patellar tracking midline; Patellar mobility: 2 quadrants medial, and l2 quadrants lateral without
pain; Stability intact in all planes including Anterior/Posterior drawer and varus and valgus stress; McMurray without click
or pain bilaterally; Gait is normal with trace quad avoidance; Hip symmetric painless ROM
Differential Diagnosis:
Bone contusion
Patellar Fracture
Chondromalacia
Osteochondritis dissecans
Irregular presentation of meniscal pathology
Test Results: Xray - Originally read as normal; Open growth plates MRI - 13x3x11mm Osteochondral defect involving
the medial articular surface of the patella with circumferential linear high T2 signal surrounding the lesion and adjacent
patellar marrow edema - unstable osteochondral defect involving the medial articular surface of the patella
Final Diagnosis: Osteochondritis Dissecans
Treatment: At initial visit patient was restricted from sports; Brace was continued; MRI ordered. Once MRI confirmed
Grade III OCD lesion he was given crutches and NWB. He was evaluated by ortho and had reduction and fixation of OCD
lesion
Outcome: Patient doing well and put in extension brace. Patient scheduled for follow up.
Follow-Up: RTP will be determine after completion of full post-operative protocol
Acute Paresthesias in a High School Football Player
Authors: Will Paulson, MD; Dave Olson, MD; Shane Maxwell, DO
Affiliation: Broadway Family Medicine Clinic - University of Minnesota, Minneapolis, MN
History: AB is a 17-year-old otherwise healthy male who presented to clinic five days after sustaining a right shoulder
injury in a high school football game. The patient, a linebacker, attempted to tackle a running back by diving, leading with
his right shoulder and impacting his opponent's thigh. He immediately felt the onset of numbness and weakness in his
right arm. The patient sat out for a couple minutes at which point the feeling returned to his arm and he completed the
game with minimal pain in his shoulder. Since that time he has experienced intermittent right shoulder numbness and
pain associated with mild weakness.
Physical Exam: Inspection: no misalignment, atrophy, or other abnormalities.
Palpation: generalized tenderness to
palpation over deltoid without specific localization. No palpable abnormalities in clavicle and acromioclavicular joint.
Range of motion: full passive and active range of motion of arm. Some pain with range of motion testing at full abduction.
Strength: 5/5 as tested in right upper extremity. Special testing: Spurling's, Yergason's, and apprehension negative.
Neer's and Hawkins' positive. Neurologic: Subjective paresthesias in proximal upper biceps with full arm abduction. No
sensory deficits. 2/4 deep tendon reflexes in right upper extremity.
Differential Diagnosis: Acute brachial plexus injury (stinger/burner), impingement syndrome, thoracic outlet syndrome,
cervical radicular process, skeletal fracture, muscular strain, and less likely neoplasm and medial brachial fascial
compartment syndrome. Guiding our differential are the exam findings consistent with neuropathy in the C5 and C6
nerve roots that form the superior trunk, which is the most commonly injured area in acute brachial plexus injuries.
According to Seddon's classification this injury was most consistent with a neuropraxia (type I), which involves
demyelination and recovery in minutes to six weeks.
Test Results: We decided to hold off on imaging at this visit. There were no concerning signs or symptoms of central
nervous system process or skeletal injury that would warrant immediate CNS imaging or radiography. In the case of
stingers, it is recommended that electrodiagnostic testing (i.e. electromyography) be pursued only if there is at least three
weeks of persistent symptoms (Kuhlman). Magnetic resonance neurography of the brachial plexus is not indicated unless
EMG and spine MRI are inconclusive (Dynamed). One small study showed that ultrasound might be more sensitive and as
specific as MRI for detection of nerve pathology in patients with suspected brachial plexopathy or mononeuropathies
(Neurology 2013).
Final Diagnosis: Acute Brachial Plexus Injury
Treatment: We recommended initiating rehab exercises. Rehabilitation should be in a stepwise process. First, restoring
pain free mobility, second, concentric and eccentric strengthening at different speeds and directions, and third, sportspecific therapy (Thein).
Outcome: The patient had mild symptoms that persisted only for three days after this visit. Following that, he gradually
increased his activity and had full, painless range of motion and full strength within one week. He completed the rest of
the season without any further exacerbations.
Follow-Up: We recommended a return to play 1-2 weeks after clinic visit. He was held out of a game two days after the
clinic visit, however he had full game participation by athletic trainer clearance fourteen days following the injury (nine
days after the clinic visit). It is recommended that return to play only be granted in contact sports once the athlete is
asymptomatic and has full painless range of motion (Kepler). At the clinic visit he was told to follow up in one week,
however he did not due to the resolution of symptoms.
Hip Pain in a Collegiate Cross Country Athlete
Authors: Ryan A. Sprouse, MD; Joseph H. Armen, DO; Philip S. Perdue Jr., MD
Affiliation: Brody School of Medicine, East Carolina University, Greenville, NC
History: A 22 year old Caucasian female Division I cross country athlete presented to clinic for an evaluation of hip pain. During the six
weeks prior to presentation, gradually developed R-hip and groin pain. Pain radiated to her R-sacroiliac region, was sharp and
intermittent, and associated with activity. Pain was present with running, other impact activities and activities of daily living including
walking. Denied traumatic injury. The athlete often ran greater than fifty miles weekly. Over the three weeks before presentation, she
ceased running and was performing cross training and low impact activity that became pain free. She had taken NSAIDs for pain relief
which helped slightly. Denied clicking, catching or locking of her R-hip joint. Denied bowel or bladder dysfunction, lower extremity
neurologic symptoms, low back pain and genitourinary symptoms. History of stress fractures of the right third metatarsal and R-fibula,
irregular menses and vitamin D deficiency. Her family and social histories were unremarkable.
Physical Exam: Vitals: weight 116 pounds, height 61 inches, BMI 21.2, blood pressure 113/68, heart rate 77. General: Healthy
appearing Caucasian female. Musculoskeletal: Normal walking gait. Right hip with normal range of motion and strength. Leg length
equal. Groin pain with resisted hip adduction and anterior hip pain with resisted hip flexion. No tenderness to palpation of the pubic
symphysis, hip adductors, or hip flexors. Tenderness to palpation over the right sacroiliac joint. FABER and FADIR caused a mild, poorly
localized hip girdle pain. Log roll negative. Lumbar spine exam unremarkable. Neurologic: Normal lower extremity exam.
Differential Diagnosis:
Femoroacetabular impingement
Hip labral tear
Femoral head avascular necrosis
Femoral neck stress fracture
Osteitis pubis
Pubic bone stress fracture
Hernia
Adductor strain
Obturator neuropathy
Ilioinguinal neuropathy
Sacroiliac joint dysfunction
Sacral stress fracture
Lumbar spine pathology
Test Results: MRI: Right sided sacral alar stress fracture. Pubic rami and pubic body stress reactive changes. DEXA: Z scores: lumbar
spine -1.7. femoral neck -0.9. total hip -0.1. Labs: Vitamin D 36 ng/ml. Potassium 5.4 mmol/l. Total calcium 10.4 mg/dl. Phos 3.7
mg/dl, TSH 1.6 uIU/ml. T4 6.2 ug/dl. PTH 25 pgm/ml. Estimated caloric intake / requirement: intake 1,700 kcals/day; requirement 1,8001,900 kcals/day
Final Diagnosis: Right sided sacral alar stress fracture. Low bone mineral density.
Treatment: The athlete's treatment plan followed a multifaceted approach. She is in the process of being treated with twice daily
vitamin D 2,000U and calcium carbonate 500mg supplementation for eight weeks. Weight and nutrition have been cautiously
monitored by our consulting nutritionist with the goal of creating a positive energy balance. Clinical psychologist was consulted for
evaluation of her perseveration on exercise. Athlete was restricted to a period of six weeks of pain free, low impact activity before slowly
returning to running activity.
Outcome: The athlete has been able to maintain a positive energy balance as she has gained five pounds since the onset of her hip
pain. Continues to follow up with clinical psychology once weekly for cognitive behavioral therapy. Currently pain free and has started a
gradual return to running activity.
Follow-Up: After undergoing a period of six weeks of pain free, low impact cross training, the athlete has progressed to three times
weekly light running on alternating days at 25% of her preinjury total weekly mileage and fifty percent of her typical preinjury pace.
Progression to complete return to running activity will continue to be granted slowly at about 10% weekly increments if the athlete
remains pain free, maintains a positive energy balance including stable body weight, and continues to be compliant with cognitive
behavioral therapy.
Twist and Shout: Knee Pain in a Kayaker
Authors: Handel A. Jones, MD; Jonathan A. Becker, MD; Rebecca E. Popham, MD
Affiliation: University of Louisville and KentckyOne Health Sports Medicine Fellowship; Louisville, KY
History: A 49-year-old female injured her right leg on a kayak trip. The pain occurred after she twisted her leg in attempt
to free her foot from being stuck in the mud of a marsh She continued to have constant right shin pain, right lateral knee
pain and diminished of range of motion for approximately 2 months. Her pain was exacerbated with weight bearing,
especially going up stairs. She received minimal relief with anti-inflammatory medication and hot tub soaks
Approximately three months after the initial injury, the patient had another event where she slid and hyper-extended her
right leg This event exacerbated her symptoms prompting a sports medicine evaluation. She denied fever, chills and
weight loss. PMH: Hyperlipidemia, Breast cyst, Gestational diabetes Medications: Aspirin 81 mg daily Family Hx: CAD;
Cancer of unknown origin
Physical Exam: General: Alert, cooperative, no distress HEENT: Normocephalic, atraumatic, sclerae anicteric
Cardiovascular: Regular Rate and Rhythm Lungs: Breathing non-labored on room air. Musculoskeletal: Upper extremities
and left lower extremity components were normal Right lower extremity demonstrates tenderness over tibial tubercle.
Knee range of motion is restricted at 75° of flexion Neurovascularity intact. Pain elicited at lateral aspect of joint space
upon lateral rotation of knee while standing on right leg flexed at 20° (+ Thessaly's Test). Skin: Normal turgor, warm and
dry.
Differential Diagnosis:
Lateral Meniscus Tear
Lateral Collateral Ligament Tear
Anterior Cruciate Ligament Tear
Popliteal Cyst
Giant Cell Tumor of Bone
Multiple Myeloma
Osteoarthritis
Tibial Fracture
Test Results: MRI Right Leg without Contrast: Large bone lesion within the proximal tibia eccentric towards the lateral
side of the proximal tibia measuring 5.1 x 4.1 x 3.6 cm (craniocaudal x transverse x AP) CT of Right Leg without Contrast:
Large well-defined lytic lesion eccentrically located in the central and lateral proximal tibia, measuring 4.9 x 4.0 x 3.6 cm
(craniocaudal x transverse x AP) There is cortical thinning seen with no definitive bony expansion Cortical breakthrough is
seen 2.3 cm distal to the joint line level.
Needle Biopsy Right Proximal Tibia Bone Lesion: Multiple fragments of giant
cell rich lesion with background of monomorphic cells showing a focal spindled morphology The giant cells show
abundant nucleoli and are evenly dispersed No definite osteoid observed.
Final Diagnosis: Giant Cell Tumor of the Bone, Grade II
Treatment: Upon receiving the final reports from imaging and pathology, patient was referred to orthopedic oncology
After explanation of both surgical and non-surgical options, the decision was made to treat the tumor with denosumab.
This treatment plan warranted non weight-bearing status until repeat radiographs show mineralization of tumor.
Outcome: Patient initially received denosumab injections at Days 1, 8 and 15 of treatment course She is now currently
on an injection regimen occurring every 4 weeks The plan is to reassess tumor size in February 2015, as that would be 6
months of scheduled therapy.
Follow-Up: She will be monitored by orthopedic oncology throughout treatment regimen with an expectation of
treatment to be completed in 6 - 9 months During and after treatment, special attention needs to be paid attention to
calcium levels as denosumab has been shown to cause hypocalcemia.
Acute Bilateral Elbow Pain
Authors: Ben J. Tseng, MD
Affiliation: Advocate Lutheran General Hospital, Park Ridge, IL
History: LB is a 36 year-old man with no significant past medical history who presented for evaluation of acute bilateral
elbow pain. He was doing handstands for his kids when he heard a "rip", then felt sudden pain and swelling in both
elbows, followed by a fall. Since then, he has noted persistent posterior elbow pain and swelling. He has been using ice
and ibuprofen with some improvement in the pain Past medical history was unremarkable. He denied any prior
medications and was only on prn ibuprofen. He denied any history of anabolic steroid use or prior corticosteroid
injections. He works as a technician for a phone company. He exercises regularly, mainly weightlifting.
Physical Exam: Notable for tenderness to palpation at the olecranons bilaterally, with marked soft tissue swelling in the
upper arm proximal to the olecranons. Passive ROM was 10-130 degrees in flexion and extension bilaterally, with normal
ROM with pronation and supination. 3/5 strength with elbow extension bilaterally, with severe elbow pain elicited with
resisted elbow extension bilaterally.
Differential Diagnosis: 1. Triceps tendon rupture 2. Olecranon fracture 3. Radial head fracture 4. Distal biceps
tendon rupture 5. Elbow dislocation
Test Results: Plain films were unremarkable. MRI of the right elbow showed high grade partial thickness tear of the
distal triceps tendon at the myotendinous junction with a tendon gap of 1.8 cm, extending into the muscle belly with a
moderate fluid collection. MRI of the left elbow showed a high grade partial thickness tear of the distal triceps tendon
involving the lateral portion of the triceps tendonwith a tendon gap of 1.6 cm.
Final Diagnosis: Bilateral partial thickness triceps tendon rupture
Treatment: The patient was referred for consultation with an orthopedic surgeon, who recommended surgical tendon
repair, which the patient declined.
Outcome: Pain and swelling improved with conservative management. The patient regained enough strength to
perform all ADL's without limitations, but had to modify his weight lifting routine.
Follow-Up: The patient was lost to orthopedic surgery follow-up. On follow-up with his primary care physician, he was
performing all occupational duties without any limitations, and was able to resume weight lifting with some modifications.
The Tooth of the Matter: Atypical Hip Pain in a Marathon Runner
Authors: Irvin Sulapas, MD; David S. Edwards MD; Jennifer J. Mitchell MD
Affiliation: Department of Family & Community Medicine, Baylor College of Medicine, Houston, TX
History: A 29-year-old white female presented to clinic with a complaint of chronic left hip and groin pain. An avid
runner, she had completed 9 marathons. Onset of pain occurred with running at 2 weeks post partum, 1-year prior (G3P3).
The pain was intermittent, but recurred after running about 1.5 miles. Only running exacerbated it, and she denied pain
while doing other activities like yoga or barre class. Valsalva did not exacerbate pain. She denied any trauma, or any
radiating pain down to the leg, lateral hip or back. She denied locking or catching of the hip. Her description of the pain
was that of a deep sensation, which resolved in a half day of withdrawal from running. She had no bowel or bladder
symptoms. Her only medication was an oral contraceptive.
Physical Exam: BMI 20.8; Gen: No acute distress; HEENT: Normocephalic; Neck: Supple; CVS: RRR; Resp: Non-labored
breathing; Abd: No organomegaly, soft, non-tender, non-distended; MSK: Normal gait; No abdominal pain with sit-up
motion from supine to seated position Hip - negative supine log roll, 5/5 strength on extension and flexion without pain,
Negative FABER/FADIR, Non TTP to groin area, non TTP to greater trochanter area, no pelvic tilt appreciated, symmetrical
SI joints without TTP, negative hop test bilaterally, 5/5 strength on hip adduction/abduction without pain Back - Non TTP
on spinous processes, negative stork test, full flexion, extension, rotation, and lateral bending with no pain Lower
extremities - no leg length discrepancy or atrophy noted
Differential Diagnosis:
Athletic pubalgia
Hip flexor strain
Labral tear
Osteitis pubis
Stress fracture of the femoral neck or pelvis
Test Results: X-ray of left hip and pelvis - bony abnormality on left ischiopubic ramus, 2 mm asymmetry of the pubic
symphysis noted on flamingo stress view, 5 mm radio-opaque densities in the mid-pelvis and left adnexa; MRI Hip/Pelvis No evidence of labral tear or osteitis pubis, mature teratoma (5.8 x 4.8 x 7.1 cm) noted on left adnexa with several teeth
present.
Final Diagnosis: Mature teratoma of left adnexa
Treatment: After long discussion with the patient, a referral to a gynecologist was done for removal of the teratoma. In
the interim, she was advised to stop running due to concern for potential rupture of the cystic component of the lesion.
Outcome: Two weeks later, she had the teratoma removed laparoscopically by her gynecologist.
Follow-Up: She is now asymptomatic. She resumed running a week after the procedure. Four months later, she was able
to complete a half marathon in 1:55 without any hip pain.
Mid-Foot Swelling in a Female Collegiate Soccer Player
Authors: Christopher Renjlian, MD, MBE; Arsh Dhanota, MD; Ashley Mueller, MS, ATC
Affiliation: The Children's Hospital of Philadelphia
History: A 20 year-old NCAA Division I female soccer player presents for evaluation of right foot pain localized to the first
metatarsal-phalangeal (MTP) joint. She is transferring care from her home town. Her symptoms began 3 months ago,
after she pivoted on a plantar-flexed foot during a soccer match. MRI done one month ago showed sclerosis and minimal
edema of the medial hallux sesamoid bone, suggesting sesamoiditis or avascular necrosis. She has continued to play
soccer, and has managed her pain with rest, ice, NSAIDs and a CAM boot that she wears when off the field. Her chief
complaint today is continued pain over the first MTP joint. During the physical exam, she is incidentally noted to also have
swelling and mild tenderness over the dorsomedial aspect of the right mid-foot. On further questioning, she reports that
she was “cleated” over the mid-foot several times during a soccer match 3 weeks prior to today’s evaluation. The mid-foot
injury has not been the patient’s primary concern and has not limited her from play.
Physical Exam: Examination of the right foot revelas swelling and mild tenderness over medial aspect of the midfoot.
There is no pain with a squeeze of the mid-foot, but there is mi ld pain with foot pronation and abduction. In addition,
there is continued tenderness over the first MTP joint, accompanied by pain and limited range of motion with extension of
the first toe.
Differential Diagnosis: The differential diagnosis for the new finding of mid-foot swelling in this high level athlete
included: - traumatic fracture of the navicular bone - stress fracture of the navicular bone - traumatic fracture of the
proximal portion of the first and second metatarsals - stress fracture of the proximal portion of the first and second
metatarsals - sprain of the lisfranc ligament
Test Results: X-ray performed on the day of the visit showed no acute fracture or bony displacement.
Despite the lack
of findings on the x-ray, there was a high index of suspicion for a lisfranc ligament injury based on the location of swelling.
Given the significant morbidity associated with lisfranc injuries, a more intensive diagnostic pathway was chosen. A
repeat MRI was ordered. The study showed continued sclerosis and marrow edema of the medial sesamoid. New findings
were consistent with a stress fracture at the base of the second metatarsal and a grade 2 sprain of the lisfranc ligament.
Following the MRI diagnosis of a lisfranc sprain, a weight-bearing CT scan was ordered to evaluate for instability under
conditions of functional stress. CT showed anatomic alignment of the lisfranc joint with no evidence of instability.
Final Diagnosis: Stress fracture of the base of the second metatarsal. Sprain of the lisfranc ligament without joint
instability.
Treatment: Consultation with a foot and ankle specialist was sought. Given the stability of the joint demonstrated by CT,
non-surgical treatment was chosen. The patient was placed in a short-leg cast and made non-weight bearing for three
weeks.
Outcome: Follow-up assessment, including removal of the cast, a physical exam and repeat radiographs, was planned at
the conclusion of the three week period.
Follow-Up: If she has stable radiographic findings and clinical improvement in mid-foot swelling and tenderness at that
time then she will be transitioned to a CAM boot for two to three months and begin a course of physical therapy for full
rehabilitation.
18 Y/O Female with Convulsions After Getting Hit in Head by Volleyball
Authors: Sagir Bera, DO, MPH, MS, ATC, CSCS
Affiliation: University of Arizona Sports Medicine Fellowship, Tucson, AZ
History: 18-year-old patient with history of depression, ADHD, migraine with aura, and two previously diagnosed
concussions. Patient was warming-up for volleyball tournament when she was hit on the top of the forehead by a
volleyball. She immediately began feeling headache, photophobia, phonophobia, weakness, nausea, and dizziness. As
patient was being escorted back to the training room she dropped to the floor and began having tonic-clonic movements,
with eyes remaining open. Rizatriptan was administered with resolution of convulsions within twenty minutes. Patient had
no post-ictal state and remembered entirety of events She was immediately symptom free and remained symptom free.
Patient had history of uncomplicated concussion at the age of 13. Prior to this patient had no medical history of ADHD,
depression, migraine headaches, or learning disability. No family history of medical problems. Second concussion was on
03/2014 after getting hit on side of head by volleyball. Had months of difficulty concentrating, fatigue, sleep disturbance,
irritability, and depressed mood. In addition, had intermittent headaches lasting for 2-3 days, with associated symptoms of
photophobia and phonophobia. Approximately two months after initial injury, patient was allowed return to limited
activity. On first day back she rode a stationary bike for eight minutes, then began feeling headache, dizziness, and blurry
vision. She immediately went to floor and began having generalized twitches without loss of consciousness. Upon further
evaluation by neurology it was determined symptoms were due to migraine headaches with aura. She was started on
treatment for depression (celexa), ADHD (vyvanse), and migraine headaches (rizatriptan). Eventually returned to full activity
four months later after resolution of most symptoms, return of post-exertion ImPACT at baseline, and unremarkable exam.
Physical Exam: Unremarkable general exam. Neurologic exam unremarkable, as well, including normal mental status,
cognition, cranial nerves, motor strength, reflexes, coordination, gait, balance, and sensory exam. Flattened affect on exam.
Differential Diagnosis:
Tonic-clonic seizure
Post traumatic seizure
Psychogenic seizure
Concussion
Second impact syndrome
Post-concussion syndrome Migraine with aura
Test Results: CBC, CMP, UA, urine toxicology = negative. MRI brain w/ and w/o contrast = negative. EEG = negative.
Final Diagnosis: Psychogenic non-epileptic seizure with history of migraine headaches
Treatment: Given rizatriptan with resolution of seizures in fifteen minutes. Held out from physical activity until further
evaluation. Referred to neuropsychiatry, including follow-up with neurology. Celexa and vyvanse were continued.
Outcome: Re-evaluated by neurology, as well as neuropsychiatry, confirming underlying medical history of migraines
and acute psychogenic seizure. Symptoms were not thought to be due to new/repeat concussion. Rather, the symptoms
were thought to be due to patient fearing return of concussion-like syndrome after being hit in head and experiencing
mild headache. She was continued on treatment for underlying ADHD and depression. In addition, rizatriptan should be
taken as required for migraine headaches.
Follow-Up: Patient allowed return to full physical activity following standard return-to-play protocol. Optimize medical
treatment for underlying migraine headaches with rizatriptan, as well as management of depression and ADHD symptoms.
As of this time, she has not returned to physical activity, but plans to do so soon. Follow-up with sports medicine and
neurology in 6 months.
Knee Pain in a Morbidly Obese Young Adult Male
Authors: Oren M. Rodriguez, MD MPH; Michael E. Joyce, MD
Affiliation: University of Connecticut, Storrs, CT
History: A 21 year-old morbidly obese male was evaluated for insidious onset of right knee pain over the past year. He
first noticed the pain following an afternoon of outdoor chores, but denied accidents or trauma. The following day he
experienced knee stiffness and difficulty fully extending his knee. The pain was deep to the patella, non-radiating and
worse with excessive walking and knee flexion. His pain was mildly improved with NSAIDs. He denied swelling, mechanical
symptoms and instability. He was actively pursuing weight loss and was able to use a stationary bicycle without knee pain.
Physical Exam: On inspection, the patient walked without a limp, yet had a substantial varus deformity of the right knee
and a modest varus deformity of the left knee. There was moderate pain to palpation at the medial jointline and mild pain
at the lateral jointline, pes anserine, and posterior joint capsule. The patient's active range of motion arc was from was
from 15° to 105°. The patient had pain laterally with McMurray's test, without any click. All stability tests were normal. The
patella tracked without tilt or shift, yet the patellar grind test was positive.
Differential Diagnosis: 1) Knee osteoarthritis 2) Patellofemoral syndrome 3) Meniscal cartilage tear
Test Results: Right knee radiographs: 1) Mild osteoarthritis of the medial compartment. 2) No osteoarthritic change in
the patellofemoral joint 3) Deformity consistent with Blount's disease showing 26° of high tibial varus and a
corresponding 15° of distal femoral varus deformity </blockquote>
Final Diagnosis: Right knee pain secondary to Blount's disease with chronic malalignment, exacerbated by morbid
obesity, which worsened in late childhood
Treatment: 1) Valgus custom unloading brace and physical therapy. 2) Recommendation of weight loss. 3)
Consideration of high tibial osteotomy of the knee to preserve knee function following weight loss
Outcome: The patient's symptoms were manageable with conservative treatment. He was doing well with physical
therapy and weight loss efforts.
Follow-Up: The patient was encouraged to continue low-impact physical activity, including stationary bicycling. Aquatic
exercise was added to his physical therapy regimen. The patient anticipated meeting with a nutritionist as part of his
weight loss efforts. He will follow-up when he reaches his goal weight or if his symptoms worsen.
A Collegiate Football Player with Medial Knee Pain and a Past Repair
Authors: Kentaro Onishi, DO; Jacob L. Sellon, MD
Affiliation: Mayo Clinic, Rochester, MN
History: A 20-year old collegiate wide receiver with a history of right medial meniscus repair twelve months ago
presented with medial knee pain. He initially did well post-operatively but then experienced an insidious onset of
atraumatic right medial knee pain during off-season training. During pre-season football practice, the pain was impairing
his cutting movements. He denied knee swelling, mechanical, or neurologic symptoms Initial training room examination
had revealed a palpable mass at the medial joint line suspicious for a parameniscal cyst.
Physical Exam: General: Fit-appearing college age male. Gait: Nonantalgic. Skin: Small non-tender mass with
rubbery consistency over medial joint line. Musculoskeletal: Right knee had full range of motion without exacerbation of
pain in extension or flexion. Palpation revealed no effusion and focal tenderness over the mid medial joint line, just
inferior to a nontender palpable rubbery mass. McMurray test provoked medial knee pain without a palpable click.
Cruciate and collateral ligament stress maneuvers were normal. Neurologic: normal sensation and no Tinel's sign about
the right medial knee.
Differential Diagnosis:
Parameniscal cyst
Medial meniscus recurrent tear
Distal semimembranosus
tendinopathy/bursopathy</ li>
Saphenous mononeuropathy
Medial collateral ligament (MCL) bursopathy
Test Results: Focused diagnostic ultrasound of right medial knee revealed a hypoechoic, nontender, nonvascular, mobile
subcutaneous mass (9 x 8 x 3 mm, CC x AP x ML) on the surface of the superficial MCL Suture was visualized extending
deep from the mass, through the superficial and deep MCL, and into the peripheral medial meniscus. Varus/valgus stress
revealed suture tension and associated dynamic medial meniscus extrusion. The MCL was otherwise intact.
Final Diagnosis: MCL bursopathy secondary aberrant medial meniscus repair suture
Treatment: We discussed options, including corticosteroid injection and surgical consultation to consider suture removal
Given the patient's desire to complete his last college football season and his lack of mechanical symptoms, we felt an
injection was the best option to facilitate his return to play goals. With aberrant meniscal suture spanning the MCL,
correlative tenderness to sonopalpation, and no signs of additional intra-articular pathology, we felt an MCL bursa
anesthetic/corticosteroid injection would best target his symptoms and confirm the pain generator. Ultrasound-guidance
was used to precisely deliver a 1 mL mixture of 0.5 mL of 6 mg/mL betamethasone (3 mg) and 0.5 mL of 1% lidocaine into
the MCL bursa, using a 27-gauge, 1.25 inch needle.
Outcome: The patient had excellent pain relief both immediately after the procedure during the anesthetic phase and
subsequently over the next several days with onset of the corticosteroid effect.
Follow-Up: The patient returned to unrestricted football practice in one week, and he played the remainder of the
season with no pain and his team won the state championship.
You Hit Like a Girl
Authors: Krystle Jones, MD; Jason Mogonye, MD
Affiliation: John Peter Smith Sports Medicine Fellowship
History: 17 year old healthy right handed female presented to clinic with throbbing, 6/10 right sided ulnar wrist pain and
swelling that started after she punched a wall 1 week prior She was seen initially at the ED, was diagnosed with a fracture,
and was placed in a temporary short arm splint and instructed to follow-up with orthopedics She continued to have pain
despite immobilization but denied any neurovascular symptoms.
Physical Exam: NAD, healthy well developed teenage female Right hand and wrist exam: TTP over the dorsum of the
wrist just distal to ulna TTP over 4th and 5th metacarpals, both dorsally and ventrally Mild edema over ulnar aspect of
wrist extending to the dorsal aspect of the 4th and 5th metacarpals Ecchymosis covering majority of palm Limited ROM
with wrist flexion/extension and radial/ulnar deviation due to pain FROM with flexion and extension at MCP, PIP, and
DIP joints of all fingers Grip strength 4/5 in the right hand compared to 5/5 on the left Sensation to light touch intact
Radial pulse +2 Capillary refill &lt; 2 seconds</blockquote>
Differential Diagnosis: 1. 5th metatarsal fracture 2. Hamate fracture 3. Metacarpal/carpal contusion 4. TFCC injury
5. Flexor/extensor carpi ulnaris tendon injury 6. Pisiform fracture 7. Ulnar artery thrombosis 8. Ulnar nerve neuropathy
Test Results: Xray right wrist and hand: Linear fracture at the ulnar aspect of the hamate body No evidence of
metacarpal fracture CT right wrist: Comminuted displaced fracture of the hamate body Intra-articular fractures at the
base of the 4th and 5th metacarpals
Non-displaced fracture of hook of hamate
Final Diagnosis: 1. Comminuted displaced fracture of the body of the hamate 2. Closed fracture dislocation of CMC
joint at ring and small finger 3. Non-displaced hook of hamate fracture
Treatment: 1. Immobilization in short arm cast based on xray findings of hamate fracture 2. CT scan obtained to
further evaluate hamate fracture and assess for possible displacement 3. Ortho hand consulted regarding CT findings.
Recommendation for non-operative management. 4. Pain control with Ibuprofen.
Outcome: Patient is currently immobilized in a short arm cast and being managed non-operatively.
Follow-Up: 1. Patient seen weekly for FU 2. No contact sports or activities high risk for re-injury x 12 weeks.
Atypical Wrist Pain
Authors: Nicholas Moore, MD; Giselle Aerni, MD; Tessa Balach, MD
Affiliation: University of Connecticut Primary Care Sports Medicine Fellowship, Hartford, CT
History: 31 year old male with a 2 month history of insidious onset left wrist volar mass He describes a sensation of
catching/clicking with full finger flexion and has some associated pain with this movement He has pain in the morning,
but not during the day He denies swelling or numbness No prior history of wrist injury or trauma but he does repetitively
use the wrist working as a saw operator.
Physical Exam: On inspection, there are no overlying skin changes There is no palpable mass on the hand while the
fingers are extended However, when he moves into full finger flexion, a palpable click is felt beneath the carpal tunnel and
a small mass appears just ulnar to the palmaris longus at the level of the wrist crease The mass again disappears as he
moves into extension The mass is oval shaped, measuring roughly 1cm x 2cm It is non-tender to palpation, mobile, nonfluctuant, and feels contiguous with the underlying tendon He has full wrist flexion and extension His strength is 5/5
throughout Tinel, Phalen and Durkan’s tests are negative.
Differential Diagnosis: 1. Benign tumor of the median nerve: ganglion cyst, fibrolipomatous hamartoma, traumatic
neuroma 2 Malignant tumors: giant cell tumor, neurofibroma, schwannoma 3 Hemangioma 4. Calcific Tendinosis
Test Results: Diagnostic ultrasound of the left wrist: -Hyperechoic mass identified on short and long axis, appearing
underneath the flexor retinaculum, seemingly from the flexor digitorum profundus or superficialis. -The mass is cylindrical,
measuring 2.09cm long with a 0.73cm diameter at its widest point. -Mild surrounding hypoechoic change. -On dynamic
ultrasound, the mass can be clearly visualized moving beneath the retinaculum as the fingers are flexed, reproducing
physical exam findings.</blockquote> X-ray of the left wrist: -Negative.</blockquote> MRI of left wrist with and without
contrast: -9 x 8 x 18mm ovoid well-defined mass arising from distal carpal tunnel, lying anterior to the flexor digitorum
profundus tendon at the level of the 3rd metacarpal base. -High signal fatty component within the mass and nodular to
linear areas of intermediate signal. -Mild surrounding peripheral capsular enhancement. -Normal bone marrow signal.
-Probable fibrolipomatous hamartoma, soft tissue hemangioma unlikely.</blockquote>
Final Diagnosis: Volar wrist mass, probable Fibrolipomatous Hamartoma
Treatment: Initial conservative treatment with a splint for pain relief and full work participation.
Outcome: Surgical excision of left volar wrist mass due to persisting symptoms.
Follow-Up: Functional rehabilitation for pain free range of motion, normal strength and return to full activity.
Right Knee Pain in a Female Softball Player
Authors: Jason Read, MD; Arthur Islas, MD, MPH; Justin Wright, MD
Affiliation: Texas Tech University Health Sciences Center Sports Medicine Fellowship El Paso, TX
History: A 15 year old female softball player presents to sports clinic with worsening right knee pain. She noted
intermittent pain for the last 2 years with worsening pain, locking, and instability symptoms for the last 6 months. The pain
was worse with running Patient also noted intermittent bruising and swelling along her right pes anserine area for the last
year. Her pain was relieved with rest and mildly improved with NSAIDs. She noted no acute injury over the previous two
years. She denied any ankle, hip or back pain. The pain was located along the right knee medial joint line and pes anserine
area. Review of systems was negative.
Physical Exam: Examination of her lower extremities revealed a normal foot, ankle and hip exam bilaterally. Gait exam
was also normal with mild pain noted along the medial joint line of her right knee with ambulation. Mild swelling and
tenderness to palpation was noted along her right pes anserine area. There was no ecchymosis or erythema. ;She had
normal knee range of motion with mild medial joint line pain on full right knee flexion. She had a negative Lachmans,
McMurray, Thessaly and Apley tests. There was no ligament laxity with valgus or varus testing. Anterior and posterior
drawer tests were also negative.
Differential Diagnosis:
ACL Tear
Meniscus Tear
Pes Anserine Bursitis
Osteochondritis dissecans
Test Results: X-rays of the right knee were obtained prior to initial clinic visit showing multiple osteochondromas MRI
results of the right knee revealed: no acute abnormality,the ligaments, tendons and menisci were normal, multiple
osteochondromas were seen at the medial distal metaphysis of the femur and posterior aspect of the proximal tibial
metaphysis. Dynamic Utltrasound also performed in clinic was concerning for the pes anserine tendons of the right leg
catching on the osteochondroma located on the patient's proximal tibia which could explain the brusing and swelling of
the pes anserine area and locking symptoms patient had been experiencing.
Final Diagnosis: Hereditary Multiple exostoses with catching of the pes anserine tendons
Treatment: Patient continued to use NSAIDs and ice for pain, however due to continued pain, patient was referred to
pediatric orthopedic specialist for evaluation for possible surgery.
Outcome: Due to continued pain, patient was referred to pediatric orthopedic specialist.
Follow-Up: Full return to activity is expected.
A Chronic Knee Problem Worsened by Wrestling
Authors: Chad Johnson, DO
Affiliation: Utah Valley Sports Medicine Fellowship, Provo, Utah
History: 18 year old male with history of a swelling injury about his right knee when wrestling 2 years prior to
presentation. The injury was treated like a prepatellar bursitis, including a steroid injection in to the area. Afterward he
developed "stretchy skin" in this area. On the day he presented to the clinic, he was at wrestling practice and went in for a
takedown, which resulted in tearing open of the skin overlying the knee
Physical Exam: Large open curvilinear laceration of the right knee superior to the patella, and measuring 15 cm.
Underlying muscle was visualized and intact. Minimal subcutaneous fat present. Large medial recess distal to the
laceration and not involving the joint.
Differential Diagnosis: Large laceration resulting from: fat necrosis, subcutaneous hematoma, morel-lavallee lesion,
previous hemangioma
Test Results: Serial ultrasound exams revealed large fluid collection which gradually resolved. Fluid was sent for analysis
and was found to be serosanguinous fluid without any concerning pathology.
Final Diagnosis: Chronic Morel-Lavallee lesion with traumatic laceration.
Treatment: Multilayer closure with copius irrigation at presentation. This was followed with percutaneous draining on
seven separate occasions under ultrasound guidance. Sclerodesis with doxycycline 100 mg in 5 cc of 1% lidocaine on
three separate occasions was performed as well.
Outcome: Marked improvement of epidermal laxity, well healed laceration, effusions resolved. Full function of the knee
preserved. Adhesion of skin to underlying tissues visualized on MSK ultrasound.
Follow-Up: After 3 months, he returned to wrestling with a knee pad and is doing well. He is following up regularly in
the university training room and has minimal persisting symptoms.
13 Year Old Male with Right Knee Pain
Authors: BJ Balcik, MD; Holly McNulty, MD
Affiliation: University of Arizona Sports Medicine Fellowship, Tucson, AZ
History: This is a 13 year old male presenting to sports medicine clinic with right knee pain of one month duration.
Patient reports he was inadvertently struck on the lateral aspect of the knee with a golf club one month prior to
presentation. He denies any twisting mechanism or fall onto the knee at the time of injury. Pain is mostly over the
anterolateral aspect of the knee but at times is posterior. The pain is described as sharp and worsened with both flexion
and extension. However, he denies any worsening of pain with weight bearing. He denies any sensation of locking or
instability but does state it does feel weak. He denies previous knee problems or surgeries. Of note, the patient was seen
by his PCP 3 days after the injury and x-rays of the knee were done at that time. The radiographs were negative for
fracture but did show soft tissue swelling and effusion. He was started on a course of naproxen which he states does help
the pain. He has also been wearing a knee sleeve which does seem to help his symptoms.
Physical Exam: In general, he is in no acute distress. He has significant difficulty moving from the chair to the exam
table. Inspection of the right knee shows significant generalized edema. There is no redness, bruising or abrasions.
Palpation revealed diffuse generalized tenderness to palpation of the anterior knee There is a moderate effusion present.
Range of motion of the knee is significantly limited with flexion to 45° and extension to 10°. There is no joint laxity with
varus or valgus stress. Swelling and patient guarding made further ligamentous and meniscal exams difficult so they were
not done.
Differential Diagnosis: Fracture, ligamentous injury, meniscal injury, other musculoskeletal pathology.
Test Results: X-rays of the right knee showed periosteal reaction and mottled, permeative appearance of the internal
bone architecture of the distal right femoral metaphysis, concerning for bone tumor. Differential considerations include
Ewing sarcoma, osteosarcoma or infection/osteomyelitis. Review of initial x-rays obtained 3 days after the injury showed
an acute fracture of the distal metaphysis. This fracture was healing on subsequent x-rays. MRI was recommended for
further evaluation. MRI of the right knee and femur showed osteomyelitis of the mid right femoral diaphysis extending
inferiorly to the epiphysis with a large subperiosteal abscess and multiple smaller interosseous abscesses with associated
myositis.
Final Diagnosis: Osteomyelitis of the right femur with septic knee.
Treatment: The patient was admitted to the hospital and underwent arthrotomy of the right knee, incision and drainage
of the subperiosteal abscess and curettage of distal femoral osteomyelitis. He was started on intravenous Cefazolin 1.5 g
q6 hours which he was on for the duration of his hospital stay.
Outcome: The patient spent a total of 5 days admitted to the hospital. He remained afebrile and was without
leukocytosis throughout the course of his stay. Intraoperative cultures were positive for methicillin sensitive
staphylococcus aureus. A central venous catheter was placed for prolonged course of IV Cefazolin He was discharged
from the hospital on non-weight bearing status. He continued IV Cefazolin for 4 weeks and then was transitioned to PO
Keflex for an additional 2 months of treatment.
Follow-Up: Patient was discharged from the hospital non-weight bearing but had gradual return to full weight bearing
status over a course of 6 weeks. He started physical therapy 2 months after discharge from the hospital.
It Takes Toes to Tango - Persistent Chronic Foot Pain in a Professional Dancer
Authors: Asad R. Siddiqi, DO
Affiliation: New York University Langone Medical Center - Rusk Rehabilitation, New York, NY
History: A 28 year old professional ballet dancer presents with a complaint of painful left foot and left second toe pain for greater than
two years. She reports no specific inciting event or trauma, but she does note the sensation of a "rock" under her second toe,
particularly in the demipointe position. Pain is also worse with any activity which involves the foot rolling over the left second metatarsal
head. She denies pain in full pointe positions. There is no pain during running or jogging in sneakers, however she notes that her left
second toe "curls up" after prolonged walking or activity. She has decreased her dancing significantly in the past two years, and is
currently taking only one class per week. As a result, she reports deconditioning of her legs with dance-specific activities. She does
report improvement of her foot pain with acupuncture two months prior to presentation. Her last x-rays of the foot were in 2012, which
she reports were "normal". She had previously been evaluated by two orthopedic surgeons, and had been referred for MRI but was
unable to obtain financial authorization to have them performed.
Physical Exam: Patient is a well-developed, well-nourished female sitting comfortably on examination table in no acute distress.
Bilateral gastrocnemius/soleus tightness was appreciated on passive stretch of calves. Examination of the feet were notable for
significant hallux valgus bilaterally (L&gt;R). Thick callus was noted on plantar surface underlying 2nd/3rd MTP bilaterally, symmetric
appearing. Reducible hammering of the left second digit is appreciated. There was mild tenderness to palpation of the plantar surface
of the left second MTP joint. Drawer test of the left second MTP revealed approximately 50% dorsal excursion of proximal phalanx on
the second metatarsal, compared to 20% on the right. Full ROM of all 10 digits, with left second digit passive extension beyond
functional limits without frank dislocation. No pain with resisted toe flexion bilaterally. Painless Mulder's click was appreciated between
the left 4th and 5th digits with squeeze test, otherwise squeezing of the met heads does not elicit pain bilaterally. Gait is intact, not
antalgic. There is pain when she rolls up onto the ball of her left foot. There is full strength and sensation in the bilateral lower
extremities. Tinel sign at the medial ankle is negative.
Differential Diagnosis:
MTP subluxation
MTP synovitis
Morton's neuroma
Freiberg disease
Metatarsal stress fracture
Sesamoiditis
Metatarsalgia
Flexor tendinitis of the foot
Tarsal tunnel syndrome
Test Results: Weight-bearing X-ray of the left foot demonstrates no acute fracture or dislocation. Hallux valgus appreciated (angle 30
degrees). Widening noted of the intermetatarsal space between 1st and 2nd MTPs (1-2 IM angle 15°). Short first ray noted with mild
DJD of the sesamoids. Other joint spaces preserved.
Final Diagnosis: 1) Second MTP joint instability with subluxation, likely chronic and related to repetitive 2nd digit extension stress 2)
Second MTP joint synovitis 3) Bilateral hallux valgus
Treatment: 1) Pontoon taping with &frac14;" podiatric wool felt applied to the first and third metatarsal heads in office to off-load
2nd MTP joint. Recommended patient to procure and apply when dancing 2) Physical therapy aimed at strengthening toe flexors and
ankle stabilizers, stretching of the gastrocnemius/soleus complex, dance specific functional retraining, and proprioceptive training 3)
Recommend stiff-soled/rocker bottom sneakers for walking and running.
Outcome: With application of pontoon taping, patient demonstrated demipointe with improved pain and increased stability.
Follow-Up: Patient left the office with a plan to begin physical therapy and gradually increase her dance activity as tolerated with
pontoon taping. She has not yet followed up in office at this time.
Prolonged Recovery After Radial Head Fracture
Authors: Natasha Desai, MD; Arsh Dhanota, MD; George Russel Huffman, MD
Affiliation: Children's Hospital of Philadelphia-University of Pennsylvania Sports Medicine Fellowship; Division
of Sports Medicine in the Department of Orthopedics, Philadelphia, PA
History: BZ is a 30 year old male recreational soccer player with no significant past medical history, who presents to the
Sports Medicine office with prolonged left elbow pain and stiffness after a radial head fracture. Four weeks prior, the
patient was playing soccer and he fell on an outstretched arm and hyperextended his elbow. He had immediate elbow
pain and swelling. He was seen in the emergency department (ED) on the day of injury. In the ED, a 2-view X-ray of his
elbow demonstrated a large joint effusion and a minimally displaced, intra-articular radial head fracture. He had no
shoulder or wrist injury. BZ was placed in a long arm posterior splint and sling and was instructed to follow up with Sports
Medicine. He has been seen in the office at 1 week, 2 weeks, and now 4 weeks post-injury. The patient was taken out of
the splint at week 1 and kept in the sling as needed for comfort. He was started with range of motion (ROM) exercises at
week 2 in attempt to decrease elbow stiffness. At the 4 week mark, BZ reports that the pain is much improved, now only
has minimal pain with movement, but reports persistent stiffness despite early ROM. No reported weakness or numbness.
Remainder of review of systems is unremarkable.
Physical Exam: Left Elbow: INSPECTION- Mild swelling and mild effusion noted. No deformity or ecchymosis. Skin
intact. PALPATION- Mild tenderness over the radial head. Otherwise no boney or tendon tenderness. No crepitus. ROMExtension to Flexion 5°to 90° feels stiff and painful. Pronation 0°-70°, and Supination 0°-80°. MUSCLE STRENGTH- Elbow
Flexion- 5/5, Elbow Extension- 5/5, Pronation- 5/5, Supination- 5/5 NEUROVASCULAR- Sensation intact and equal over
bilateral upper extremities. Radial pulse 2+ with brisk capillary refill. </blockquote> SPECIAL TESTS- Varus and Valgus
stress negative. Tineal sign at cubital tunnel negative. Tennis elbow and golfer's elbow negative.
Differential Diagnosis:
Radial Head Fracture
Radial Head Subluxation/dislocation
MCL or LCL ligament injury
Lateral Condyle fracture
Capitellum Cartilage Injury
Capitellum fracture
Test Results: INITIAL X-RAY: Large joint effusion, minimally displaced fracture of the radial head that extends to the
articular surface. (Very small fragment noted in the antecubital fossa- not noted by radiologist on initial read) REPEAT XRAY (4 weeks post injury): Small joint effusion, healing radial fracture, previous fragment not visualized. MRI (4 weeks
post injury): Intra-articular fracture of the radial head with mild displacement. Fracture of the anterior capitellum with a
displaced intra-articular osseous fragment. Post traumatic contusion of the medial and lateral epicondyles and the
olecranon process. UCL and RCL intact.
Final Diagnosis: Occult coronal shear fracture of the capitellum (Grade 1) with intra-articular radial head fracture (Mason
type II)
Treatment: The patient was referred to Orthopedic Sports Medicine. He underwent open fixation of a non-union
capitellar fracture with an autograft, LCL repair and capsule release.
Outcome: In the post-operative exam under anesthesia, the patient was able to get to full ROM.
Follow-Up: At 3 weeks post-op, BZ is continuing to recover, still with pain and swelling. He is in PT and on a home
exercise program to regain full ROM. He has yet to return to full activity.
Headache and Diplopia in a Weight Lifter
Authors: Peter L. Hoth, MD
Affiliation: UI Sports Medicine - University of Iowa, Iowa City, IA
History: A 33 year-old weight lifter presented for evaluation of intermittent episodes of diplopia in the mornings for the
past year. The episodes would last about 30 minutes and then spontaneously resolve. The diplopia was present daily for
about one week, and would then resolve, with episodes recurring about once every 2 months. The episodes were not
related to illness or exertion. He also complained of a sharp headache above his left ear as well as a dull headache in the
left occipital region. The headache and the diplopia usually did not coincide. For the majority of the episodes he had no
aura prior to headache onset. The headache would frequently last about 2 hours and improved with ibuprofen. The
headaches were present one to two times per month, and he never experienced the headache during exercise. He did not
have any associated nausea, vomiting, fevers, chills, earache or sore throat. His past medical history was negative for any
chronic medical problems and he had no regular medications. He worked in construction and did do vigorous resistance
training 4 to 5 days per week. He quit smoking cigarettes 6 years ago and drank alcohol, a few beers per week. His family
history was significant for hypercholesterolemia, arthritis, coronary artery disease and hemorrhagic stroke secondary to
ruptured intracranial aneurysm.
Physical Exam: Vital Signs: BP 116/70, Pulse 80, Respiratory Rate 20, Temp 97.5°F, BMI 29.8 kg/m2 General: Alert, well
appearing, muscular build Head: Normocephalic, atraumatic Eyes: Conjunctivae and corneas clear. Pupils equal, round
and reactive to light. EOM full without nystagums. Fundi benign with sharp discs. Throat: Mouth moist, no erythema,
lesions or exudates Neck: Supple, no adenopathy Thyroid without enlargement, tenderness or nodules. No carotid
bruits. Lungs: Clear to auscultation Heart: Regular rate and rhythm, no murmur Extremities: Normal, no edema
Neurological: Cranial Nerves II-XII intact. Normal strength, sensation and reflexes throughout
Differential Diagnosis:
Migraine headache
Intracranial mass or tumor
Cerebral aneurysm
Myasthenia gravis
Transient ischemia attack
Giant cell arteritis
Test Results: Fasting Lipid Panel: Total Cholesterol - 177 Triglycerides - 169 HDL - 35 LDL - 108 Fasting glucose 90 Creatinine - 1.3 BUN - 17 Hgb - 15.6 Hct - 44.5 MRA head, with and without contrast showed normal carotid
arteries without focal stenosis There was a 3 mm left paraclinoid aneurysm directed superomedially Bilateral anterior and
middle cerebral arteries are normal in caliber without focal stenosis or aneurysm The vertebral, basilar and posterior
cerebral arteries are normal in caliber without focal stenosis or aneurysm.
Final Diagnosis: 3 mm left paraclinoid aneurysm
Treatment: Referral to neurosurgery for further evaluation and treatment.
He was restricted from resistance training
until his follow up with neurosurgery. Neurosurgical options for treatment included observation and follow up with serial
imaging studies, pipeline stenting or microsurgical clipping. Patient elected to proceed with pipeline stenting.
Outcome: After pipeline stenting he had some increased headache and symptoms for 1 to 2 weeks, however symptoms
then completely resolved.
Follow-Up: Six weeks post treatment his symptoms resolved and he had returned to resistance training without adverse
effect.
An Uncommon Cause of Forearm Pain
Authors: Kirsten Oliversen, MD; Aaron Lee, DO
Affiliation: MacNeal Hospital, Berwyn, IL
History: The patient was a 36 year-old woman, right-hand dominant, who initially presented with burning left forearm
and hand pain for 9 days She had no trauma or other injury She noted generalized swelling over the wrist and forearm
She reported numbness/ paresthesias over the forearm distal to the elbow, but not including the hand She reported that
her arm felt heavy The dorsal aspect of the left hand was painful, but volar side felt normal She reported dropping items
with her left hand the previous week She noted left- sided neck pain 1 day prior to presentation She denied any similar
previous issues 1 month prior to this presentation, she had been seen by her primary for viral conjunctivitis, followed by a
non-specific rash consistent with a viral exanthema. She had no fevers or other systemic signs of infection at that time.
Physical Exam: Left hand: Inspection: no erythema, asymmetry, ecchymoses, effusion, or atrophy. Slight swelling
around wrist. Palpation: nontender. Range of Motion: full. Strength: normal except unable to flex 1st IP joint. Special
Tests: Tinel sign negative (carpal tunnel) and Phalen maneuver negative (carpal tunnel) Able to oppose left thumb Unable
to perform pincer on left Unable to flex thumb at the IP joint. Neck: full range of motion without pain No neck
tenderness Negative Spurling's Negative Adson's Left elbow: Unable to sublux ulnar nerve, and Tinel's over the ulnar
nerve is negative.
Differential Diagnosis: 1. Cervical radiculopathy 2. Forearm strain 3. Medial epicondylosis 4. Anterior interosseus
nerve impingement 5. Anterior interosseus neuritis
Test Results: X-rays of the left arm and hand were negative for fracture or dislocation Distal arm edema was seen.
Final Diagnosis: Idiopathic anterior interosseus neuritis
Treatment: She was prescribed oral prednisone 20mg daily for 5 days She was sent to occupational therapy for custom
brace and improving dexterity.
Outcome: Treatment is ongoing. She reported improvement in left forearm discomfort after the oral steroids, but no
improvement in flexion of the left first interphalangeal joint She was sent for electromyography for further delineation of
neurologic deficit.
Follow-Up: The patient is able to perform her work duties and daily activities She will continue regular activity using her
splint, and she will continue to regain use of the non-dominant thumb by working with occupational therapy She will
continue to follow-up every 4 weeks during her treatment course.
Bilateral Wrist Effusions in Healthy Young Male
Authors: Stacey L Dault, DO; Ricardo Colberg, MD
Affiliation: American Sports Medicine Institute and Andrews Sports Medicine and Orthopaedic Center,
Birmingham, AL
History: A 22-year-old male presented to office for evaluation of bilateral wrist pain of two weeks duration. Pain was
described as moderate-to-severe sharp, constant pain when moving hands, fingers or wrists and associated with stiffness,
swelling and significant weakness in hands Onset was sudden with no known injury, but did occur after a significant
amount of manual labor at his work as a general cleaner. He had previously been evaluated in the emergency department
for right wrist pain with x-ray which showed no bony abnormalities Patient was started on naproxen BID His pain
worsened; he was seen at urgent care and diagnosed with tendonitis, changed to diclofenac Symptoms continued to
worsen Right hand developed extensor lag in wrist and hand Pain and swelling developed in left wrist. He followed up
with a primary care provider who then referred patient for evaluation in sports medicine clinic. Denied fevers, chills,
weight loss or symptoms in other body systems No significant past medical history Denied alcohol, tobacco, IV drug
abuse Reported light marijuana use. On initial presentation to sports medicine clinic, patient was treated with a single
dose of 10 mg dexamethasone IM followed by five days of prednisone 40 mg PO while initial laboratory workup was
completed Patient reported improvement in symptoms while on steroid burst but quick recurrence of symptoms upon
completion Return visit exam was unchanged Joint aspiration preformed on follow-up visit.
Physical Exam: Well developed, well-nourished male in no acute distress Awake, alert, oriented Bilateral wrist exam
with mild erythema and warmth to touch Moderate soft tissue swelling in distal forearm and wrist, predominately in volar
compartment with associated severe tenderness to palpation Wrists held in flexion for comfort Severely limited range of
motion with 30° of flexion and difficulty obtaining extension Poor muscle tone in bilateral wrists secondary to pain with
strength 2/4 in all planes on right; 4/5 on left Sensation and pulses intact and equal bilaterally Capillary refill intact
Negative carpal compression test, Phalen's sign and Tinel's sign.
Differential Diagnosis: Inflammatory arthropathy Crystal arthropathy Pyogenic Rheumatic Systemic lupus
erythematosus Severe tenosynovitis/carpal tunnel Mixed connective tissue disease Kienbock's disease Carpotarsal
osteochondromatosis
Test Results: Bilateral wrist x-ray showed severe soft-tissue swelling throughout distal forearm and wrist; no fractures,
no dislocations, no mal-alignment, normal bone density. CBC with WBC 10.8, Hb 13.0, Platelets 506, Neutrophils 78%,
Lymphocytes 14% CMP WNL except Total protein 8.6, ALT 73, Alkaline Phosphatase 245 Anti-CCP 6, CRP 17.40, ESR
&gt;100 ANA negative, RF negative Synovial fluid analysis: no crystals; Nucleated cells 37375, Neutrophils 89%,
Monocytes 7%, Lymphocytes 4%, RBC 9385, markedly decreased viscosity Synovial fluid gram stain: many WBC, no
organisms Synovial fluid culture: Light growth N. gonorrhoeae, beta-lactamase negative Urine GC/CT: (+) Gonorrhea, (-)
Chlamydia
Final Diagnosis: Gonococcal arthritis
Treatment: Patient was referred back to primary care for appropriate treatment of gonorrhea urethritis and arthritis and
was treated with Ceftriaxone 1 gram IM q 24 hours x 7 days and empiric Azithromycin 2 grams PO x 1 for possible coinfection with Chlamydia trachomatis Partner was also tested and treated.
Outcome: Full course of antibiotics completed with significant improvement in wrist effusions and pain Patient was seen
for weekly follow up with slow improvement in range of motion and strength.
Follow-Up: After completion of antibiotics, patient was referred for physical therapy He was allowed to return to activity
and work as tolerated. Despite recommendation for further follow-up, patient did not return to office for re-evaluation.
Post-Concussion Symptoms or Something Deeper Still?
Authors: Ty Clayton Jones, MD; Wade R. Gaal, MD; William Aaron Rosenberg, MD
Affiliation: The University of Nevada School of Medicine Sports Medicine Fellowship in Las Vegas, Las Vegas,
NV
History: An 18 year old student athlete with a past medical history significant for seven prior concussions -- the last two
years ago -- was elbowed in the right cheek during a basketball game. She passed the SCAT3 courtside and returned to
play. She presented to the athletic training room clinic three days later with complaints of frontotemporal headache,
nausea, dizziness, photosensitivity, phonosensitivity, feeling slowed down, irritable, and drowsy. On exam, she was tender
to palpation over the right inferior orbit and had worsening headache and dizziness with vestibular testing ImPACT
testing demonstrated a total symptom score of 15 and a deficit in reaction time (a decrease from the 92nd percentile at
baseline to the 25th percentile) and a decrease in her cognitive efficiency index from 0.37 to 0.27. She was sent for CT of
her orbit which demonstrated a subtle right maxillary sinus fracture at the anterior wall without displacement, for which
she was cleared by ENT over the course of the week. Her concussion symptoms resolved by post-injury day 8 Once
asymptomatic, ImPACT was repeated and demonstrated a return to baseline. She was returned to sport per a graduated
return-to-play protocol without complications. She returned to the athletic training room clinic eight months later with
complaints two weeks of constant throbbing bilateral frontotemporal headaches (occasionally awakening her from sleep)
and difficulty focusing in class.
Physical Exam: Headache and dizziness reproduced with vertical gaze and fixed gaze testing with head movement.
Normal heel-toe walk, negative romberg, normal fundiscopic exam, normal symmetric strength testing in upper and lower
extremities, CNII-XII grossly intact.
Differential Diagnosis: Prolonged post-concussive syndrome secondary to traction damage, space-occupying
intracranial lesions, vascular pathology, and migraines.
Test Results: MRI of the brain without contrast demonstrated a probable arterial loop arising from an M2 branch of the
right middle cerebral artery and right anterior temporal region with a rounded contour mimicking a 9 mm aneurysm An
MRA was ordered to further characterize the lesion. This study demonstrated high-grade stenosis of the proximal and mid
left transverse sinus consistent with venous sinus occlusive disease, with distal flow within the left sigmoid sinus and
proximal left internal jugular vein.
Final Diagnosis: Left transverse sinus thrombosis.
Treatment: Neurosurgery was consulted. Of note, the patient had no personal or family history of clotting disorder, was
not taking oral contraceptive pills, and was a non-smoker. The neurosurgeon deduced that her symptoms were secondary
to dehydration and she was started on daily ASA 81 for four weeks.
Outcome: Her symptoms resolved over the course of two weeks while on ASA 81 therapy.
Follow-Up: A four-week follow up MRA demonstrated resolution of the thrombosis with no residual stenosis. Following
this normal study, she underwent a graduated return-to-play protocol without complication and has been asymptomatic
since.
Leg Pain, More Than Just a Strain
Authors: Jimmy Gonzales, MD; Laura Schnettgoecke, ATC, LAT; J. Field Scovell, MD; Jennifer Mitchell, MD,
FAAFP
Affiliation: Texas Tech University Sports Medicine Fellowship; Lubbuck, TX
History: 20 yo Division 1 female soccer athlete with several days of left groin pain during spring soccer season
Symptoms increased upon return from road trip with 8 hour bus ride. Pain was constant, sharp, and severe with radiation
down her left leg and calf cramping and not relieved by acetaminophen or ibuprofen Exam three days after onset of pain
revealed no obvious musculoskeletal source, but did show mild left leg swelling. No history of direct trauma and no
respiratory complaints. PMHx: Dysmenorrhea, constipation PSHx: None Family/Social histories: non-contributory Meds:
Oral contraceptive 0.3/30 PO daily, started 4 months prior
Physical Exam: VS: Ht 5" 4', Wt 139lbs, HR 81, BP 138/80 Gen: Athletic, female, NAD Exam was benign including
normal right leg and peripheral pulses with the exeption of LLE swelling in thigh and calf, mild hip pain with ROM
Differential Diagnosis: Deep venous thromboembolism Cellulitis Superficial thrombophlebitis Venous valvular
insufficiency Lymphedema Popliteal cyst Ovarian mass impacting venous return Muscular injury (Quadriceps,
Hamstrings, Calf)
Test Results: CBC: WBC - 9.8, Hgb - 11.7, Hct - 35.2, Plt - 212, Normal differential Normal labs: CMP, Lipid panel, B-HCG
Hypercoaguability evaluation negative including: Protein S Activity, Protein S Antigen, Free Protein S Antigen,
Antithrombin III, Prothrombin mutation, Factor V Leiden Doppler u/s LLE: Thrombus in left common femoral vein and
greater saphenous vein with occlusion. Left LE venogram: occlusive thrombus with stenosis from origin of left common
iliac vein through popliteal vein 2 months post therapy, Doppler u/s LLE: No evidence of thrombus; normal blood flow
Final Diagnosis: Left common iliac vein thrombosis with compression from right iliac artery; May-Thurner Syndrome
Treatment: Patient admitted after ultrasound revealed extensive left leg DVT. Initially given 15mg rivaroxaban orally
and OCP's were discontinued Venogram was performed and underwent mechanical thrombectomy with tissue
plasminogen activator (tPA) infusion along with ballooning of common iliac stenosis. Repeat venogram the next day
showed no residual thrombus within the LLE, and only 10% narrowing of left common iliac vein. Given marked
improvement and patient's young age, no stent was placed Anticoagulation was initiated with enoxaparin and warfarin,
but after discussion with patient and family was transitioned to rivaroxaban at discharge.
Outcome: 12 week f/u ultrasound showed no evidence of thrombus and normal flow She completed 3 months of
anticoagulation and has returned to activity for her senior year of competition She was advised to wear TED hose and
ambulate frequently during travel.
Follow-Up: May-Thurner Syndrome related DVT accounts for only 2-3% of all lower limb DVTs Contributing factors in
this case include OCP use, recent travel and possible dehydration Initial thrombolysis was used to attempt to minimize
residual thrombus and thus minimize future complications Stenting was not performed given virtual resolution of
thrombus with minimal stenosis in a young, competitive athlete. IVC filter was not placed given 2012 Society of Vascular
Surgery guidelines not recommending one with adequate catheter directed thrombolysis. The patient gradually
increased her activity level and was allowed aerobic and strength conditioning only with no contact drills while on
rivaroxaban. Given the athlete's uncomplicated course, satisfactory follow-up Doppler study, and this being her senior year
of competition, the decision was made to discontinue anticoagulation at three months The patient has been compliant
with no further OCP use and ambulation during travel with TED hose She has had one subsequent episode of left upper
leg pain and swelling early in the season, which warranted repeat Doppler ultrasound, which did not reveal new thrombus.
Heel Pain in a Young Soccer Player
Authors: Caitlyn Mooney, MD; Susannah Briskin, MD
Affiliation: University Hospitals Case Medical Center/ Rainbow Babies and Children's Hospital
History: 11-year-old male soccer player who presents with four months of bilateral ankle pain. He was previously on a
national soccer team and ran about 4-5 miles daily. Seen by multiple providers and diagnosed with Sever’s disease. Tried
heel cups, heel lifts, orthotics, a walking boot, night braces, and stretching exercises. Has used scheduled motrin and aleve.
Pain continues to worsen. His activities have been limited to swimming and walking for two months. A week prior to
presentation he had a day when he was unable to ambulate. Had history of swelling around the Achilles but in the past
week also noted right sided posterior lateral ankle swelling. He denies any known injury. ROS: positive for knee pain
started when in walking boot; rash-diagnosed with contact dermatitis of elbows a week earlier and treated with steroids;
new onset morning stiffness (one hour); fatigue and headache with negative work up 3 months prior Past Medical
History: remote hand fracture, foot fracture
Physical Exam: General: Thin male.
Left ankle: Notable for normal appearance, tenderness at the insertion of the
Achilles on the calcaneus. Full range of motion and strength. Right ankle: Notable for swelling anterior, lateral, posterior
ankle. No erythema, no warmth. Tenderness at the insertion of the Achilles on the calcaneus and ATFL. Full strength,
ankle dorsiflexion less than five degrees short of neutral. Pain with squeezing heels bilaterally. Gait: Toe walkinghesitant to place heels on ground. Remainder of exam within normal limits with exception of rash of bilateral upper
extremities.
Differential Diagnosis: 1. Sever's Disease 2. Juvenile Idiopathic Arthritis 3. Brodie's Abscess 4. Lyme Disease 5. Septic
Arthritis 6. Salter Harris Fracture 7. Ankle Sprain 8. Stress Fracture 9. Reactive Arthritis 10. Lupus 11. Multifocal
Osteomyelitis 12. Malignancy
Test Results: Right Ankle X-ray: accessory area of growth over the medial malleolus, soft tissue swelling.
CRP: 0.115
ESR: 20 Lyme titer: negative ANA: + (1:80) Rheumatoid factor: 1.0 HLA B27 antigen screen: + Right Ankle MRI:
Synovitis, joint effusion involving ankle and subtalar joints without marrow edema or cortical erosion. Small amount of
fluid in retrocalcaneal bursa.
Final Diagnosis: Juvenile Idiopathic Arthritis - Enthesitis Related Arthritis Category
Treatment: Sports medicine clinic prescribed physical therapy which resulted in some symptomatic relief. Labs ordered
and when came back concerning for JIA referred to pediatric rheumatology. Pediatric rheumatology started him on
meloxicam which resulted in decreased pain after three weeks, but he had persistent swelling thus sulfasalazine was added
to regimen. Cam boot re-prescribed due to persistent swelling for several weeks.
Outcome: By two-month follow up he felt that he was 100% back to normal, able to play around house with minimal
pain. Able to participate in camp. A week after starting sulfasalazine, his swelling and morning stiffness improved to less
than five minutes. Sulfasalazine resulted in &lt;1 hour of headache multiple times a week, occasional upset stomach. At
time of his two-month follow up his ESR normalized to 7. First eye exam performed and was without evidence of
inflammation.
Follow-Up: At two-month follow up, after resolution of effusion and pain, he was allowed to begin gradual return to
activity. After attempting to return to soccer, he had intermittent bilateral heel pain only with activity. He restarted
physical therapy which resulted in some improvement He is to continue gradual return and refrain from playing in cleats
until pain free with regular shoes Holding off on return to competitive soccer temporarily. Rheumatology and
ophthalmology follow frequently to monitor disease and he remains on meloxicam and sulfasalazine.
Knee Injury in a High School Football Player
Authors: Emueje Ophori, MD; Kendra McCamey, MD; James Borchers, MD
Affiliation: The Ohio State University Wexner Medical Center
History: 17 year-old male, who sustained lateral impact to his right knee after being tackled during football. Reportedly
felt a pop, knee collapsed inward, and was unable to immediately bear weight. Upon initial side line evaluation, knee pain
was severe, diffuse, throbbing, 8/10 intensity. There was no swelling. Pertinent past history included a right fibular fracture
five years ago in a non-sports related accident and reports of persistent catching and intermittent instability of the right
knee six months prior to this injury.
Physical Exam: Well-developed male, in obvious discomfort, alert and oriented. No effusion or ecchymosis of the right
knee. There was a positive sag sign. Point tenderness noted over the medial joint line, medial tibial plateau, and lateral
tibial plateau. Severe pain with full extension, flexion beyond 30°, and circumduction. Positive medial McMurray's,
Posterior drawer, and valgus laxity. Anterior drawer, Lachman, and varus laxity tests negative. The patient was reexamined
3 days later in sports medicine clinic. He had 3+ effusion and severe pain, making comprehensive knee examination
challenging. Findings consistent with sideline evaluation with the exception of Lachman test, which was equivocal.
Differential Diagnosis: 1)PCL tear 2)MCL tear 3)Meniscal tear 4)ACL tear 5) Patellar contusion 6)Patellar
subluxation 7)Tibial plateau fracture
Test Results: X-rays unremarkable. MRI showed posterior cruciate ligament tear, grade III medial collateral ligament tear,
posterior horn root tear of the medial meniscus with extrusion, osseous contusions of the lateral femoral condyle and
lateral tibial plateau, and joint effusion.
Final Diagnosis: Acute grade II PCL peel off tear, grade III MCL tear with avulsion from tibia, and meniscal root tear.
Treatment: Right knee arthroscopy with medial meniscus repair, lateral tibial plateau chondroplasty, posterior cruciate
ligament repair, and open medial collateral ligament reconstruction/augmentation with Achilles allograft.
Outcome: No significant postoperative complications. Physical therapy initiated after one week.
Follow-Up: He was non-weight bearing for 6 weeks, allowed 0°-45° ROM from 2-4 weeks postop, and 45°-90° ROM
from 4-6 weeks. He was placed in a hinge brace locked in extension and advised RICE for control of swelling. At 6 weeks
postop he was doing well with plans to transition to PCL brace. He has not been cleared to return to sports. This a case of
an acute right grade II PCL tear, grade III MCL tear with avulsion from tibia, and meniscal root tear, sustained by a high
school football player after a valgus force to a fixed right knee. One of the findings that makes this case unique is there
was no injury to the anterior cruciate ligament, which often occurs during contact-induced valgus loading when the knee
is fixed Furthermore, medial ligament tears are most commonly isolated after a valgus load to the knee in a flexed
position. Patients typically report localized pain and swelling along medial knee structures and associated instability in
cases of grade III tears of the MCL Intraoperative examination also indicated a peel-off tear of the posterior cruciate
ligament, which often requires a specific arthroscopic assisted technique for fixation. Stability of the knee depends on
dynamic and static forces, often making the multi-ligament injured knee a challenge to diagnose Both positional and
anatomical classification methods assist in diagnosis These injuries are usually sustained during high impact activities such
as motor vehicle accidents and competitive sports. Detailed history and physical examination is essential in determining
the extent of the injury and developing a definitive treatment plan. Immediate or delayed surgical intervention must be
individualized. Surgical candidates should be deemed optimal to complete postoperative rehabilitation with goals of
improving function and stability Surgical intervention and a comprehensive rehabilitation program provide a favorable
prognosis.
Sports Preparticipation Screening Detects Cardiac Disease in an Asymptomatic Preadolescent
Athlete
Authors: Jeremy L. Coleman, MD; James B. Robinson, MD
Affiliation: University of Alabama, Sports Medicine Fellowship, Tuscaloosa, AL
History: A 13 year-old African-American male was evaluated during a large, station-based Preparticipation Physical
Evaluation (PPE). He is a talented, high-intensity athlete that reported occasional high blood pressure readings and an
innocent flow murmur that was appreciated on his annual exam two years prior; otherwise, he is healthy. He specifically
denied chest pain (CP), shortness of breath (SOB), palpitations, lightheadedness, syncope, or a family history of
hypertrophic cardiomyopathy or premature sudden cardiac death. A screening EKG showed sinus rhythm, prominent
precordial voltage, "dome-shaped" convex ST segment elevations in the anterior precordial leads (V1-V3), and diffuse T
wave inversions (most notably in the inferolateral leads).
Physical Exam: The patient was normotensive (BP 126/82, height &gt; 90th %), non-tachycardic, and afebrile. Lungs
were clear to auscultation bilaterally. Cardiac examination revealed a regular rate and rhythm with a grade III/VI
crescendo-decrescendo systolic murmur heard best at the left upper sternal border that increased with valsava. No JVD or
carotid bruits were present and distal pulses were 2+ bilaterally. No clubbing, cyanosis, or edema was noted in his
extremitites.
Differential Diagnosis: 1. Pediatric Hypertrophic Cardiomyopathy (HCM) due to a sacrcomeric defect 2. Athlete's
Heart 3. Hypertensive Left Ventricular Hypertrophy (LVH) 4. Phenocopy of HCM (ie. Fabry's Disease)
Test Results: ECHO: - Concentric LVH (maximal wall thickness of 1.6 cm) with speckled appearance of the myocardium
- Hyperdynamic global systolic function (EF &gt;75%) - No LV outflow obstruction or anterior motion of the mitral
valve</blockquote> STRESS ECHO: - Standard Bruce Protocol: Achieved predicted maximal heart rate without CP or SOB
- Hypertensive response to exercise (baseline BP 152/73, HR 71; peak BP 220/60, HR 84); widespread repolarization
abnormalities, and dynamic LV outflow tract obstruction</blockquote> CARDIAC MRI: - Abnormal LV myocardial mass at
125 g/m2 (47-87 g/m2) - Concentric LV thickening with slightly more prominent apical involvement (1.6 cm in anterior
basilar septum and basilar inferolateral wall). - No abnormal uptake of gadolinium</blockquote> RENAL ARTERY
ULTRASOUND: No renal Artery stenosis detected CMP, CBC, TSH/T4, Anti-Streptolysin O antibodies: wnl
Final Diagnosis: Hypertrophic Cardiomyopathy (sarcomeric defect vs phenocopy of HCM) in a Preadolescent
Treatment: - Referred to pediatric cardiologist and instructed to avoid strenuous activity - After 3 months of rest, the
repeat EKG and ECHO were unchanged - Patient was given a home AED and instructed to avoid competitive sports Patient has started formal counseling to help cope with likely diagnosis and prognosis
Outcome: Holter monitoring and genetic testing are pending.
Follow-Up: Referred to pediatric cardiologist and started formal counseling.
Concerning Bony Lesion in a Soccer Player
Authors: Jitesh Umarvadia, MD; Garry W.K. Ho, MD; Thomas M. Howard, MD
Affiliation: VCU Fairfax Family Practice Sports Medicine Fellowship Program, Fairfax, VA
History: 35 year old male recreational soccer player presented to the sports medicine clinic with 5 weeks of left lower leg
pain and swelling. He reported acute onset of pain while running the day after a soccer game. He denied any acute injury
or trauma. The pain was located in the distal anteromedial lower leg over the tibia. He described the pain as dull and achy,
worse with ambulation and causing him to limp. He denied bruising but noted erythema over the left distal tibia. He also
reported focal swelling which worsened throughout the day. The application of ice at home did not alter his symptoms. He
denied fevers, chills, numbness, tingling, or loss of strength His past medical and surgical history was significant for left
ACL reconstruction in 2010. He was otherwise healthy.
Physical Exam: On presentation, he was afebrile and vital signs were stable. Focused physical examination revealed soft
tissue edema with erythema in the anteromedial distal tibia. There was tenderness to palpation with warmth and woody
edema over the area. ROM was normal at both the knee and ankle. Muscle strength testing was normal. The squeeze test
of the distal tibia-fibula elicited pain. He had poor single leg balance secondary to pain in the affected leg. The distal
neurovascular exam was normal.
Differential Diagnosis: myositis ossificans cellulitis DVT or superficial thrombophlebitis osteomyelitis anterior
tibialis strain medial tibial stress syndrome tibial stress fracture syndesmosis injury osteoid osteomalacia primary
neoplasm
Test Results: Left Tibia/Fibular AP & Lateral Radiographs - no evidence of fracture; subtle cortical erosions and soft
tissue calcifications overlying the medial distal bony tibial cortex; multiple transverse radiolucent lines anteriorly through
tibial cortex. MSK Ultrasound of Distal Tibia - cortical irregularity and erosions of distal tibia; overlying mixed echogenic
soft tissue edema; significant neovascularity and hyperemia on Power Doppler over cortical erosions. MRI Left Lower
Extremity With & Without Contrast - moderate to marked periosteal reaction along left distal tibial diaphysis with
associated marrow edema; small subtle incomplete fracture line along the medial tibial cortex; no discrete lytic or blastic
bone lesion seen. CT Left Tibia Fibula Without Contrast - subtle curvilinear lucencies in medial and posteromedial cortex
of left distal tibial shaft with nonaggressive overlying periosteal bone formation; no obvious underlying lytic or blastic
lesion.
Final Diagnosis: Distal tibial stress fracture
Treatment: He was immobilized, placed in a cam walker boot, and restricted from activity.
Outcome: He has tolerated the cam walker boot well and has noted significant improvement in his pain and swelling.
This case is of interest because initially there was concern for primary bony neoplasm given radiographic and sonographic
findings. However, MRI and CT findings pointed to tibial stress fracture.
Follow-Up: He was referred to an orthopedic oncologist for their opinion given findings on imaging and exam. We are
awaiting their consultation. He will be re-evaluated with MSK Ultrasound at his follow-up visit to assess for any
sonographic changes and/or improvement of previous findings. We will obtain a repeat CT scan in 6-8 weeks to document
radiological healing. Thereafter, he will begin physical therapy and progressive return to activity.
Can't Keep Your Eye on the Ball - Blurry Vision in a Collegiate Volleyball Player
Authors: Christine Harris-Spinks, MD; Stephanie Chu, DO; Sourav Poddar, MD
Affiliation: University of Colorado, Denver, CO
History: 19 y.o. F collegiate volleyball player with a hx of 2 prior concussions was struck under chin by a volleyball on a
rebound from a spike. She was not expecting the impact and it forced her head posteriorly. She complains of headache,
neck pain, nausea, dizziness and emotional lability. She was diagnosed with a concussion by the ATC, held from play, and
placed on cognitive and physical rest. Initially she reported 8/22 concussion symptoms with a total symptom score of
20/132. Four days later, the patient had improvement in symptom scores and return to baseline on Standardized
Assessment of Concussion (SAC). Additional neuropsychological testing demonstrated return to baseline parameters.
However, as she had tried to return to class and cognitive/visual effort, she reported blurry/double vision and trouble
focusing on objects in left lateral visual field.
Physical Exam: Neck exam was within normal limits without any tenderness to palpation. Modified BESS test found 0
errors on double-leg stance, 1 error on single-leg stance, and 1 error on tandem stance. Her upper limb coordination
exam was 1/1. Delayed recall 4/5 (at baseline). IMPACT Test returned to baseline. Eye examination: 20/20 vision in both
eyes on eye chart. Convergence: reports double vision when object is 10cm from nasal bridge. Vertical gaze: intact
superior and inferior gaze with both eyes. Lateral gaze: she was able to gaze fully to the right with both eyes. The right eye
was able to achieve full gaze to the left. However, in the left eye there was a 6mm lag in terminal lateral gaze. On visual
field testing, she had reduced ability to see items in left peripheral visual field compared to the right.
Differential Diagnosis: - Orbit fracture/Trauma
- Myasthenia gravis - Multiple sclerosis - Thyroid eye disease
- Stroke - CN VI palsy due to concussion - CN VI palsy due to Vasculopathic etiology (dm, HTN) - CN VI palsy due
to increased intracranial pressure (neoplasm, mass, bleed) - Post viral CN VI palsy
Test Results: - 9 days after concussion, IMPACT computer testing was better than baseline. - Brain MRI without
contrast: normal - Discussion with a neuro ophthalmologist: likely post concussive CN VI palsy, no need to be formally
seen by neuro ophthalmology if no worsening symptoms or no improvement in 6-8 weeks. Should be followed regularly
to evaluate EOM and convergence.
Final Diagnosis: Cranial nerve VI/left lateral rectus palsy after a concussion.
Treatment: - Reassurance and avoiding practice until convergence and lateral gaze returned to normal. - After return
of normal convergence and lateral gaze, went through vestibular rehab with her ATC.
Outcome: - 4 weeks after the concussive blow her convergence returned to normal - 5 weeks after the concussive
blow, she was able to achieve full lateral gaze in the left eye
Follow-Up: - After return of full lateral gaze, she restarted practice including hitting live balls and reported no problems
with tracking or reaction time. - 5.5 weeks after the concussive blow, she played in a collegiate volleyball game without
issues. She had good reaction time and did not report any problems with vision or following the ball. She played well and
continues to play for the team.
Ocular Trauma with a Reversible Pupillary Defect
Authors: Andrea Kussman, MD; Joshua Scott, MD
Affiliation: UCLA Family Medicine, Santa Monica, CA
History: A 22 year old male soccer player was struck by an elbow in the right eye during the last 90 seconds of a soccer
game He felt immediate pain around the right eye, and noted blurry vision with decreased visual acuity but no diplopia
The patient presented to the physician on the sidelines within 5 minutes of the injury
Physical Exam: No acute distress, well-nourished well-developed male Soft tissue swelling noted around the right eye,
with small superficial abrasion inferior to the eye Patient was able to open eye fully, and extra-ocular movements were
intact Right eye vision was poor on gross assessment of vision, but patient was able to identify large shapes Right pupil
was noted to be oval in shape (picture available) Left pupil was within normal limits
Differential Diagnosis: Iris sphincter tear open globe injury traumatic optic neuropathy
Test Results: The patient was taken to the emergency room where ocular exam was performed by the emergency room
physician, and CT head was obtained CT showed no evidence of orbital fracture, and was within normal limits By the time
the CT scan had been obtained, the right pupil was still larger than the left, but less oval in shape (picture available) Vision
had also improved dramatically. The patient was seen by an ophthalmologist the following day, and detailed ophthalmic
exam revealed a mildly irregular pupil, a micro-hyphema, commotio retinae, and traumatic posterior vitreous separation.
Final Diagnosis: Iris sphincter tear complicated by micro-hyphema, commotio retinae, and traumatic posterior vitreous
separation.
Treatment: The patient was followed weekly by an ophthalmologist with serial exams He was told to refrain from
practicing for a minimum of 3 weeks, and to avoid any activity such as lifting for 1-2 weeks There is a risk that with any
trauma or Valsalva, the hyphema could rebleed and become significantly larger This would be a surgical emergency due
to risk of eye staining It was important to rest for the commotio retinae because if the retina was bruised again, edema
could reaccumulate and result in a scotoma This could be catastrophic to vision if the scotoma happened to form over
the macula The posterior vitreous separation also carried an increased risk of traumatic retinal detachment, so rest was
recommended for this as well The patient was given strict return precautions for any floaters or flashers Iris sphincter
tears typically resolve within 6-8 weeks independently, however some cases may require surgical repair after 3-4 months if
no improvement is noted.
Outcome: The patient's vision rapidly returned to normal His pupil slowly regained its normal shape and size over the
next 3 weeks There were no new scotomas or hyphemas No retinal detachment developed.
Follow-Up: The patient did not play for 3 weeks After that time, he returned to competitive level practices and games
He was followed on a weekly basis by the ophthalmologist, and after return to play he was followed by the team physician.
Recalcitrant Wrist Pain in Patient Using an Aromatase Inhibitor
Authors: David Woznica, MD; Monica Rho MD
Affiliation: Rehabilitation Institute of Chicago and Northwestern Memorial Hospital, Chicago, IL
History: 55 year old woman with history significant for right cervical radiculitis, breast cancer and left sided wrist tendon
stripping surgery for DeQuervain's tenosynovitis presented for evaluation of right wrist pain. At her initial diagnosis of
breast cancer in 2001 she was started on tamoxifen and thereafter developed multiple joint arthralgias, eventually
becoming maintained on celecoxib. In 1994 she had developed bilateral DeQuervain's tenosynovitis due to childcare; this
improved on the right with steroid injection but required tendon stripping on the left with full resolution in 1995. She
presented for a right wrist pain of three months duration which began after gardening. This was initially self treated with
anti-inflammatory dose ibuprofen as well as nocturnal and eventual daytime wrist splinting. This burning, stabbing and
stiff pain improved with neutral positioning, brace usage, and worsened withcryotherapy.
Physical Exam: Alert and oriented, appropriate, well nourished, no distress. Normal cardiovascular and pulmonary
exam. Neurologic exam notable for symmetric 2+ reflexes and 5/5 strength in the bilateral upper extremities except for
the left biceps (5-/5). Right wrist exhibited tenderness to palpation of the 1st dorsal compartment, full range of motion,
and a positive Finkelstein's test.
Differential Diagnosis: 1: Right sided DeQuervain's tenosynovitis 2: Cervical radiculitis 3: Aromatase inhibitor
associated osteoporotic stress fracture
Test Results: Musculoskeletal Ultrasound: Focal tenosynovits around extensor pollicis brevis (EPB) tendon Wrist MRI:
Extensor pollicis brevis and abductor pollicis longus tenosynovitis
Final Diagnosis: Right wrist DeQuervain's tenosynovitis
Treatment: Inital course of eccentric activity provided no relief. Second intervention involved ultrasound guided steroid
injection to first dorsal compartment. Following failure of steroid injection, ultrasound guided needle tenotomy to APL and
EPB was performed. Due to insufficient response to tenotomy, nitroglycerin patches were prescribed for off-label use to
the painful area.
Outcome: Initial conservative measure failure was noted, however the patient began to develop improved symptoms
after two weeks of nitroglycerin patch usage. Total time from onset of symptoms to improvement was approximately
seven months.
Follow-Up: At time of case submission patient with improving symptoms however ongoing degree of pain limiting her
full activities of daily living.
An Unusual Cause of Chest Wall Pain
Authors: Chandra Maloney, MD; Jordan Chun, MD
Affiliation: Group Health Cooperative Family Medicine Residency, Seattle, WA
History: A 39-year-old male shipyard laborer presents to clinic with a complaint of right chest wall pain for approximately
three months. He reports the discomfort began acutely while weightlifting (shoulder shrugs), which is his main form of
exercise. He states he had been doing very heavy powerlifting in attempt to bulk up, and was focusing on cable fly
exercises to strengthen his pectoral muscles. He also states his work at the shipyard includes throwing heavy rope
repetitively. He describes the pain as a severe, sharp tearing sensation over the right side of his sternum and along his
clavicle. Moving his right arm in all directions causes discomfort, and the pain has interfered with his ability to weightlift
and perform his work duties. The patient denies history of blunt trauma to the chest. He reports activity modification has
not led to resolution of the pain, although he has continued to work and perform limited weightlifting.
Physical Exam: Vital signs: pulse 66, height 6 ft, weight 216 lbs CV: regular rate and rhythm MSK - R chest wall: Inspection: No visible defect of the chest wall or right shoulder. No ecchymosis, soft tissue swelling, erythema, or lesion. Palpation: Diffuse tenderness to palpation over the right chest wall and sternum near the origin of the pectoralis major, as
well as the manubrium. No palpable defect. -ROM: Active ROM and passive ROM of the right shoulder and chest are full,
but patient reports pain with maximal extension, forward flexion, and abduction of his right shoulder. -Strength: 5/5
strength in right upper extremity, although pt reports reproduction of pain with resisted shoulder adduction. -Neuro:
Distally neurovascularly intact.
Differential Diagnosis: Partial tear of the pectoralis major sternoclavicular arthritis sternal stress reaction/fracture
rib fracture costochondiritis bone tumor
Test Results: MRI right chest: There is no evidence of tear in the right pectoralis muscle. There is abnormal bone
marrow edema in the right side of the upper manubrium immediately below the sternoclavicular joint close to the first
costal manubrial junction, which may be due to a nondisplaced fracture or bone contusion. Bone scan (ordered at 6week follow-up): There is mild increased uptake in the delayed phase along the right side of the manubrium and
sternoclavicular joints, suggestive of possible arthritis.
Final Diagnosis: Probable manubrial stress fracture - Given the patient's history of acute onset of pain while
weightlifting, physical exam finding of right sternal tenderness to palpation, and the superior specificity and anatomic
localization of MRI findings compa
Treatment: The patient's initial treatment plan included modifying his activity to pain-free only and gentle shoulder
range of motion exercises. A work letter for light duty was provided.
Outcome: At the patient's six-week follow-up appointment, he had reaggravated his symptoms extending his shoulder
at work and lifting his dog at home. A bone scan was ordered to further clarify the pathology and stage of healing for
work and exercise planning (see results above). The patient's case was subsequently discussed with two thoracic surgeons,
who reported there are no good surgical options, and recommended conservative care.
Follow-Up: The patient was advised to follow prolonged (six months or more) rest and activity modification to not
provoke the area and allow it to heal as best as possible. The patient has not yet been contacted for follow up.
A Pain in the Gut
Authors: Christopher R. Pruitt, CRP, MD
Affiliation: O'Connor Hospital Sports Medicine Fellowship Program, San Jose, CA
History: Chief complaint: Abdominal pain
History of Present Illness: 29 year old previously healthy male, presenting
with 4 days of progressive abdominal pain The pain started shortly after a soccer game, in which patient recalls being hit
in the mid-abdomen with a soccer ball No other trauma, no loss of consciousness, he continued to play and finished the
game Shortly afterwards he noticed some generalized abdominal pain, achy in nature and worse with palpation Over the
next few days the pain continued, but became more intense and localized to the right lower quadrant Patient denied any
nausea or vomiting, no dysuria or hematuria, but did endorse anorexia, non-bloody diarrhea and subjective fever for the
past day No sick contacts, no dietary changes, no history of inflammatory bowel disease.
Physical Exam: Vital Signs: HR 80, BP 150/85, RR 18, SpO2 100% on RA, Temp 98.4, Wt 74.8 kg General: 29 yo male,
non-toxic in appearance, resting on stretcher bed in moderate distress HEENT: Normocephalic, atraumatic, PERRL, EOMi,
dry mucus membranes, oropharynx clear Neck: supple, full ROM, no meningismus Cardiovascular: regular rate and
rhythm, no murmurs or gallops Respiratory: Lungs clear to auscultation bilaterally Abdomen: no distention, mild diffuse
tenderness, but markedly greater in right lower quadrant, + rebound tenderness Back: non-tender, no CVA pain
Extremities: warm and well perfused Skin: no rashes
Differential Diagnosis: Appendicitis mesenteric adenitis pancreatitis inflammatory bowel disease bowel or
mesenteric contusion/hematoma liver laceration cholecystitis perforated viscous ascending colon diverticulitis
Test Results: CBC: WBC 8.0, hemoglobin 14.6, hematocrit 41.5, platelets 201 CMP: Sodium 143, potassium 4.3, chloride
99, bicarbonate 28, BUN 19, Cr 1.26, glucose 76, calcium 9.8, AST 24, ALT 29, alkaline phosphatase 95, bilirubin 1.7, protein
9.1, albumin 5.1 UA: Spec gravity 1.032, ketones 3+, no blood or leukocytes, no nitrites CT Abdomen/Pelvis with
contrast Impression: Nearly 5cm hyperdense and heterogeneous spherical mass at medial aspect of ascending colon,
likely representing hematoma. Moderate amount complex, free intraperitoneal fluid consistent with hemoperitoneum.
Appendix not reliably visualized, although several gas bubbles in the vicinity of the tip of the cecum may represent
portions of appendix No inflammatory changes in the vicinity noted Appendicitis cannot be excluded, but is unlikely in
the context of the above findings. Liver, spleen, adrenals, gallbladder, kidneys and pancreas unremarkable.
Final Diagnosis: Proximal colon hematoma with hemoperitoneum
Treatment: Patient received two doses of morphine 4mg in the ED, one dose of zofran 8mg, and 1L IV fluids in the ED
Surgical consult was placed with recommendations for observation and serial abdominal examinations Repeat hematocrit
q6 hours Patient to remain NPO overnight and continue with IV fluids.
Patient was admitted to the family medicine
service overnight Serial abdominal examinations were performed with gradual improvement in symptoms. ;Pain was
controlled with tylenol 650mg prn.
Outcome: Repeat laboratory analysis showed a drop in hemoglobin to 12.4 (from 14.6 on arrival), thought to be
dilutional in nature given his dehydration on presentation and treatment with IV fluids. Repeat hemoglobin the evening of
discharge was 12.1 Patient’s vital signs remained within normal limits, and he was transitioned to a PO diet without issue
Patient required no further narcotic analgesia or antiemetics He was discharged home with scheduled follow up with in
the next two days.
Follow-Up: Patient reported gradual improvement in abdominal pain over the next week, and diarrhea resolved shortly
after discharge He was withheld from strenuous activity for the next month, followed by a gradual return to full contact
No notable complications were reported No surgical follow up was required.
College Football Player with Decreased Pulses After Non-Contact Knee Injury
Authors: Michelle Napoli, MD; David Petron, MD; R. Judd Robins, MD
Affiliation: University of Utah; Salt Lake City, UT
History: During a game, a 20-year-old football player stumbled while running up field and extended his right leg to
regain balance. His foot planted awkwardly with his weight distributed over the leg, causing a hyperextension injury of the
knee He was immobilized on the field and evaluated by team physicians. Due to pain severity there was difficulty
obtaining a satisfactory history or physical exam. Plain films were obtained in the locker room with no evidence of fracture.
Due to significance of pain, it was decided that the player would not travel home that evening on his planned flight A few
hours after the injury, pain increased and the patient was taken to the emergency department for further evaluation Pain
was in the right knee, 8/10, and &lsquo;cramping’. He denied sensory changes or motor weakness in the right leg.
Additional History: No past medical history. Two unspecified surgeries on his left foot. No medications.
Physical Exam: General: Alert and oriented x 3. In moderate distress. MSK: Right lower extremity: Large right knee
effusion, inability to range the knee secondary to pain. Calf is soft and compressible. The anterior and lateral
compartments are soft and compressible. 5/5 strength with ankle dorsiflexion, plantar flexion and great toe extension.
Palpable and dopplerable posterior tibial pulse on the right. Nonpalpable or dopplerable dorsalis pedis pulse. Much
stronger posterior tibial and dorsalis pedis pulse on the left that are both dopplerable and palpable. Neuro: Sensation
intact to light touch in all lower leg nerve distributions. Skin: Right foot slightly colder than left. No breaks in the skin
noted. Ankle Brachial Index: 0.96
Differential Diagnosis: Compartment syndrome popliteal entrapment syndrome thrombosis arterial injury
ligamentous injury fracture
Test Results: On initial labs, creatinine was found to be 1.93 Other labs were non-significant. Plain films of the right
knee, tibia and fibula did not reveal any fractures. Due to elevated creatinine a CTA was deferred in favor of a non-contrast
MRI with an initial read of no popliteal or other arterial injury. Grade 3 ACL tear with edema pattern consistent with
anterior dislocation relocation was demonstrated.
Final Diagnosis: Popliteal arterial occlusion secondary to multiligamentous knee injury involving complete tear of the
ACL, LCL, popliteal tendon tear, and biceps femoris avulsion.
Treatment: Vascular surgery was consulted due to clinical concern for vascular injury despite first read of MRI without
evidence of popliteal artery injury Seven hours after initial evaluation, dorsalis pedis pulse was lost and the right foot
became cooler to touch. ABI's decreased to 0.87. The patient was taken to the OR for a right lower extremity angiography
due to increased concern for popliteal artery injury. Angiography revealed acute occlusion at the mid popliteal artery with
reconstitution of the proximal tibial arteries via fairly good collaterals. The patient underwent a right distal popliteal
artery to junction of anterior tibial artery and tibioperoneal trunk artery interposition bypass. Prophylactic fasciotomies
were not performed.
Outcome: MRI addendum read as: 1. Slow flow or clot in the popliteal artery distal to the branching of the geniculate
artery. 2. Complete ACL tear. Intact PCL. 3. Severe posterolateral corner injury, including biceps femoris avulsion from the
tibia with at least 3 cm retraction, complete LCL tear. 4. Partial MCL tear. 5. Edema pattern consistent with anterior
dislocation-relocation. The patient is currently two days status-post popliteal artery bypass in stable condition in the SICU.
He is awaiting multi-stage ligamentous repair. More information will be available as the case evolves. Had the patient
traveled home that evening, he would have likely suffered morbidity to his leg due to vascular compromise.
Follow-Up: Approximate 12-month period before this patient will be cleared to return to sport.
The Case of a High School Athlete with a Twist
Authors: Michael P. Hagen Jr. MD
Affiliation: Methodist Willowbrook Sports Medicine Fellowship, Houston, TX
History: A 17 year old male right handed quarterback with no known past medical history suffered a twisting injury to his
right hand and wrist by getting his hand caught in a face mask. The injury occurred the night prior to presenting to injury
clinic. The athlete was previously evaluated on the sideline and placed in a splint. Prior to the injury, he was a high
functioning quarterback and baseball pitcher. On presentation to the injury clinic, he had swelling of his right hand and
complained of pain in his right middle finger and right wrist. He denied deformity, stiffness, weakness, numbness or
tingling.
Physical Exam: Right upper extremity examination: Inspection: mild swelling of the radial aspect of the wrist, near the
anatomic snuff box, and swelling of the PIP joint of the 3rd finger Palpation: tenderness of the 3rd middle phalanx, 3rd
PIP joint, 3rd metacarpal, radial styloid process, dorsal scaphoid and volar scaphoid Range of motion: limited flexion and
extension of the 3rd PIP joint and decreased active range of motion of the wrist secondary to pain Strength: decreased
grip strength of the hand secondary to pain and normal strength of the wrist
Differential Diagnosis:
metacarpal fracture
phalanx fracture
phalanx sprain
tendon injury
distal radius fracture
scaphoid fracture
other carpal bone fracture
Test Results: Hand x-rays taken at the patient’s initial clinic visit showed a fracture of the proximal right middle phalanx.
A scaphotrapezial abnormality with no definite fracture was also seen. To further evaluate this abnormality, a CT of the
right upper extremity was obtained. This study showed a scaphotrapezial coalition with alteration of the alignment
between the scaphoid and lunate. This finding raised the possibility of dorsal intercalated segment instability and no
fracture was noted. Follow-up wrist x-rays showed bilateral congenital scaphotrapezial fusion.
Final Diagnosis: The athlete was diagnosed with a proximal right 3rd phalanx fracture and bilateral congenital
scaphotrapezial fusion.
Treatment: At the patient’s initial injury clinic visit he was placed in a thumb spica cast to treat the unusual
appearance of the scaphoid. The athlete was placed in a thumb spica cast until further evaluation could be obtained. He
was also treated with rest, ice, compression and elevation of the right upper extremity as well as with PRN NSAIDs. After
ruling out a scaphoid fracture, his proximal right 3rd phalanx fracture was treated with buddy taping.
Outcome: The athlete was released to return to play once a scaphoid fracture was ruled out.
Follow-Up: At the athlete's initial clinic visit, he was instructed to return to the clinic in 1 week for reevaluation and
repeat x-rays. When the patient followed up, he was diagnosed with a congenital scaphotrapezial fusion. He was released
to return to play once a scaphoid fracture was ruled out.
Persistent Foot Pain in a Long Distance Runner
Authors: khawaja Asim Siddique MD; Willian Bennett MD
Affiliation: Hofstra-North Shore LIJ health system Glen Cove Hospital, Glen Cove,New York, NY
History: 45 year old Caucasian male with no significant past medical history presented to a sports medicine clinic with
the complaint of persistent right foot pain that has progressed over a period of four weeks to such an extent that he is
barely able to walk without pain. Denied any trauma or injury to his foot. He used to be a long distant runner and currently
participating in 10K/5K runs. The patient has been using over the counter analgesics with minimal relief. Pain was
localized to the lateral and medial aspect of his mid and hind foot. Non-radiating, 9 out of 10 in intensity, constant dull
pain that aggravated by walking and mildly relieved with rest.
Physical Exam: Physical Exam showed no obvious deformity apart from callus underneath the first metatarsal head,
point tenderness at the base of fifth metatarsal as well on the medial aspect of mid foot. Range of motion was limited due
to pain with maximal intensity of pain during inversion and eversion of foot. Gait was antalgic due to pain.
Differential Diagnosis: Ligament injury Tenosynovitis Peroneus longus tendon rupture Os peroneus fracture Stress
fracture
Test Results: X ray of Right foot with 3 views showed accessory ossification center posterior Medial to the cuboid. There
was mild osteoarthritis involving the Metatarsophalangeal and Inter phalangeal joint of the first toe. The remainder of the
foot was grossly intact. We decided to do MRI while putting him on non-weight bearing. MRI showed: Full-thickness
tear of the peroneus longus tendon just as it enters the cuboid tunnel along the plantar/lateral aspect of the
calcaneocuboid cuboid joint and with superimposed fracture of associated os-perineum in this region. The base of the
fifth metatarsal appears normal without evidence of fracture of the distal peroneus brevis attachment to the base of the
fifth metatarsal.
Final Diagnosis: Full-thickness tear of the peroneus longus tendon just as it enters the cuboid tunnel along the
plantar/lateral aspect of the calcaneocuboid cuboid joint and with superimposed fracture of associated os-perineum
Treatment: Patient was referred to the foot surgeon for the open repair of the torn tendon. Surgery was successful
without any complications
Outcome: Follow up in 6 weeks showed good improvement.
He was able to walk with mild pain.
Follow-Up: Full return to sporting activity was achieved in 2 months.
Get A Grip
Authors: Chanida K. Supavong, DO; Khalilah Clarke, MD; Nicholas DeFauw, DO
Affiliation: John Peter Smith Primary Care Sports Medicine Fellowship, Arlington, TX
History: A 14 year old Caucasion female volleball player and cheerleader presented to Sports Medicine Clinic
complaining of a small bump near the base of her right middle finger. She first noticed the mass about 6 months prior to
her visit to the clinic. She denied any known inciting trauma to the finger, although she had jammed her fingers many
times in the past playing volleyball. The mass increased in size over several months and was painful to direct pressure and
gripping objects. She denied triggering of finger, sensory deficits and weakness of hand. She worried that the bump will
interfere with cheerleading. X-ray of the hand at 8 weeks prior to evaluation had shown no bony lesion or growth.
Physical Exam: - Remarkable for a smooth, well circumscribed, mobile 7 mm mass located at the base of her right
middle finger, about 1 centimeter distal to the MCP joint - 5/5 grip strength - Intact sensation - Full range of motion of
all joints - Mild tenderness to palpation of mass
Differential Diagnosis: - Tendon sheath g

Similar documents

Evaluating and Treating Sports Injuries in Young Athletes

Evaluating and Treating Sports Injuries in Young Athletes Gentle Palpation: with focus on bony vs. soft tissue structures; crepitus; step-off’s; bony mass; growth plates ($1,000,000 exam tool: one finger to localize tenderness) ROM: flexion, extension, ab...

More information

ICL #1 - ISAKOS

ICL #1 - ISAKOS • fibular styloid (posterior division) and anterior medial downslope of styloid (anterior division) • important static stabilizer of external rotation C. Mid-Third Lateral Capsular Ligament 1. Seco...

More information